Вы находитесь на странице: 1из 215

version: 1.

CHAPTER

1 FUNCTIONS AND LIMITS

Animation 1.1: Function Machine


Source and credit: eLearn.Punjab
1. Quadratic Equations eLearn.Punjab 1. Quadratic Equations eLearn.Punjab
1. Functions and Limits eLearn.Punjab 1. Functions and Limits eLearn.Punjab

1.1.3 Notation and Value of a Function


1.1 INTRODUCTION
If a variable y depends on a variable x in such a way that each value of x determines
Functions are important tools by which we describe the real world in mathematical
exactly one value of y, then we say that “y is a function of x”.
terms. They are used to explain the relationship between variable quantities and hence play
Swiss mathematician Euler (1707-1783) invented a symbolic way to write the statement
a central role in the study of calculus.
“y is a function of x” as y = f(x) , which is read as “y is equal to f of x”.

1.1.1 Concept of Function


Note: Functions are often denoted by the letters such as f, g, h , F, G, H and so on.

The term function was recognized by a German Mathematician Leibniz (1646 - 1716)
A function can be thought as a
to describe the dependence of one quantity on another. The following examples illustrates
computing machine f that takes an input x,
how this term is used:
operates on it in some way, and produces
exactly one output f(x). This output f(x) is
(i) The area “A” of a square depends on one of its sides “x” by the formula A = x2, so
called the value of f at x or image of x under
we say that A is a function of x.
The volume “ V ” of a sphere depends on its radius “r” by the formula V = 4 pr3, so
f. The output f(x) is denoted by a single
(ii)
3 letter, say y, and we write y = f(x).
we say that V is a function of r.

The variable x is called the independent variable of f, and the variable y is called
A function is a rule or correspondence, relating two sets in such a way that each
the dependent variable of f. For now onward we shall only consider the function in
element in the irst set corresponds to one and only one element in the second set.
which the variables are real numbers and we say that f is a real valued function of real
Thus in, (i) above, a square of a given side has only one area.
numbers.
And in, (ii) above, a sphere of a given radius has only one volume.

Given f(x) = x3 - 2x2 + 4x - 1, ind


Now we have a formal deinition:
Example 1:
(i) f(0) (ii) f(1) (iii) f(-2) (iv) f(1 + x) (v) f(1/x), x ≠ 0
1.1.2 Deinition (Function - Domain - Range)

Solution: f(x) = x3 - 2x2 + 4x - 1


f(0) = 0 - 0 + 0 - 1 = - 1
A Function f from a set X to a set Y is a rule or a correspondence that assigns to each
(i)
f(1) = (1)3 - 2(1)2 + 4(1) - 1 = 1 - 2 + 4 -1 = 2
element x in X a unique element y in Y. The set X is called the domain of f.
(i)
f(-2) = (- 2)3 - 2 (-2 )2 + 4 (-2) - 1 = - 8 - 8 - 8 - 1 = -2 5
The set of corresponding elements y in Y is called the range of f.
(ii)
f(1 + x) = (1 + x)3 - 2(1 + x)2 + 4(1 + x) - 1
Unless stated to the contrary, we shall assume hereafter that the set X and Y consist of
(iii)
= 1 + 3x + 3x2 + x3 - 2 - 4x - 2x2 + 4 + 4x - 1
real numbers.

= x3 + x2 + 3x + 2

version: 1.1 version: 1.1

2 3
1. Quadratic Equations eLearn.Punjab 1. Quadratic Equations eLearn.Punjab
1. Functions and Limits eLearn.Punjab 1. Functions and Limits eLearn.Punjab

Explanation is given in the igure.


f(1/x) = (1/x) - 2(1/x) + 4 (1/x) - 1 = 3 - 2 + - 1, x ≠ 0
3 2
1 2 4
(iv)
x x x

Example 2: Let f(x) = x2. Find the domain and range of f.

Solution: f(x) is deined for every real number x.


Further for every real number x, f(x) = x2 is a non-negative real number. So
Domain f = Set of all real numbers.
Range f = Set of all non-negative real numbers.

x Method to draw the graph:


x -4
Example 3: Let f(x) = 2 . Find the domain and range of f. To draw the graph of y = f(x), we give arbitrary values of our choice to x and ind the

Solution: At x = 2 and x = -2, f(x) = 2


x corresponding values of y. In this way we get ordered pairs (x1 , y1) , (x2 , y2), (x3 , y3) etc. These
x -4
is not deined. So
Domain f = Set of all real numbers except -2 and 2 .
ordered pairs represent points of the graph in the Cartesian plane. We add these points and
join them together to get the graph of the function.
Range f = Set of all real numbers.

Let f(x) = x2 - 9 . Find the domain and range of f.


Example 5: Find the domain and range of the function f(x) = x2 + 1 and draw its graph.
Example 4:

Solution: We see that if x is in the interval -3 < x < 3, a square root of a negative number is
Solution: Here y = f(x) = x2 + 1
We see that f(x) = x2 +1 is deined for every real number. Further, for every real number
obtained. Hence no real number y = x2 - 9 exists. So
Domain f = { x d R : |x| 8 3 } = (-T, -3] j [3, + T)
x, y = f(x) = x2 + 1 is a non-negative real number.
Hence Domain f = set of all real numbers
Range f = set of all positive real numbers = (0, + T) and Range f = set of all non-negative real numbers except
the points 0 7 y < 1.
1.1.4 Graphs of Algebraic functions For graph of f(x) = x2 +1, we assign some values to x from its domain and ind the
corresponding values in the range f as shown in the table:
If f is a real-valued function of real numbers, then the graph of f in the xy-plane is
deined to be the graph of the equation y = f(x). x -3 -2 -1 0 1 2 3
y = f(x) 10 5 2 1 2 5 10
The graph of a function f is the set of points {(x, y)| y = f(x)} , x is in the domain of f in the
Cartesian plane for which (x, y) is an ordered pair of f. The graph provides a visual technique Plotting the points (x, y) and joining them with a smooth curve,
for determining whether the set of points represents a function or not. If a vertical line we get the graph of the function f(x) = x2 + 1, which is shown in the
intersects a graph in more than one point, it is not the graph of a function. igure.

version: 1.1 version: 1.1

4 5
1. Quadratic Equations eLearn.Punjab 1. Quadratic Equations eLearn.Punjab
1. Functions and Limits eLearn.Punjab 1. Functions and Limits eLearn.Punjab

(i) Polynomial Function


1.1.5 Graph of Functions Deined Piece-wise. A function P of the form P(x) = an xn + an-1 xn-1 + an-2 xn-2 + .... + a2 x2 + a1 x + a0
for all x, where the coeicient an, an-1, an-2, .... , a2, a1, a0 are real numbers and the exponents
When the function f is deined by two rules, we draw the graphs of two functions as
are non-negative integers, is called a polynomial function.
explained in the following example:
The domain and range of P(x) are, in general, subsets of real numbers.
If an ≠ 0 , then P(x) is called a polynomial function of degree n and an is the leading
Example 7: Find the domain and range of the function deined by:
coeicient of P(x) .
[ For example, P(x) = 2x4 - 3x3 + 2x - 1 is a polynomial function of degree 4 with leading
x when 0 7x 71
x - 1 when 1 < x 72
f(x) = also draw its graph.
coeicient 2.
Solution: Here domain f = [0, 1] j [1, 2] = [0, 2]. This function is composed of the following
two functions:
(ii) f(x) = x - 1 , when 1 < x 7 2
(ii) Linear Function
(i) f(x) = x when 0 7 x 7 1
If the degree of a polynomial function is 1, then it is called a linear function. A linear
function is of the form: f(x) = ax + b (a ≠ 0), a, b are real numbers.
To ind th table of values of x and y = f(x) in each case, we take suitable values to x in the
domain f. Thus
Table of y = f(x) = x - 1:
For example f(x) = 3x + 4 or y = 3x + 4 is a linear function. Its domain and range are the
Table for y = f(x) = x
set of real numbers.
x 0 0.5 0.8 1 x 1.1 1.5 1.8 2
y = f(x) 0 0.5 0.8 1 y = f(x) 0.1 0.5 0.8 1 (iii) Identity Function
For any set X, a function I : X " X of the form I(x) = x " x d X , is called an identity
function. Its domain and range is the set X itself. In particular, if X = R , then I(x) = x , for all x
d R , is the identity function.
Plotting the points (x, y) and joining them we get
two straight lines as shown in the igure. This is (iv) Constant function
the graph of the given function. Let X and Y be sets of real numbers. A function C : X " Y deined by C(x) = a , " x d X , a
d Y and ixed, is called a constant function.
For example, C : R " R deined by C(x) = 2, " x d R is a constant function.

1.2 TYPES OF FUNCTIONS (v) Rational Function


P(x)
A function R(x) of the form , where both P(x) and Q(x) are polynomial functions and
Q(x) ≠ 0, is called a rational function.
Q(x)
Some important types of functions are given below:
The domain of a rational function R(x) is the set of all real numbers x for which Q(x) ≠ 0.
1.2.1 Algebraic Functions
1.2.2 Trigonometric Functions
Algebraic functions are those functions which are deined by algebraic expressions.
We classify algebraic functions as follows: We denote and deine trigonometric functions as follows:
version: 1.1 version: 1.1

6 7
1. Quadratic Equations eLearn.Punjab 1. Quadratic Equations eLearn.Punjab
1. Functions and Limits eLearn.Punjab 1. Functions and Limits eLearn.Punjab

(i) y = sin x, Domain = R, Range -1 7 y 7 1. 1.2.6 Hyperbolic Functions


y = cos x , Domain = R, Range -1 7 y 7 1.
p
(ii)
sinh x = (ex - e-x) is called hyperbolic sine function. Its domain and range are
1
(iii) y = tan x, Domain = {x : xdR and x = (2n + 1) , n an integer}, Range = R (i)
2
y = cot x, Domain = {x : xdR and x ≠ np, n an integer}, Range= R
2 the set of all real numbers.

p
(iv)
(e + e-x) is called hyperbolic cosine function. Its domain is the set of
1 x
y = sec x, Domain = {x : xdR and x ≠ (2n + 1) , n an integer}, Range= R
(ii) cosh x =
(v) 2
y = csc x, Domain = {x : xdR and x ≠ np, n an integer}, Range = y 8 1, y 7 -1
2 all real numbers and the range is the set of all numbers in the interval [1, +T)
(vi) (iii) The remaining four hyperbolic functions are deined in terms of the hyperbolic
sine and the hyperbolic cosine function as follows:
e x - e- x
1.2.3 Inverse Trigonometric Functions
=
=
e x + e- x e x + e- x
sinh x 1 2
tanh x = sech x =
cosh x cosh x
e x + e- x
We denote and deine inverse trigonometric functions as follows:
=
=
p p e x - e- x e - e- x
cosh x 1 2

y = sin -1 x ⇔ x = sin y, where -


≤ y ≤ , -1 ≤ x ≤ 1
coth x = csch x =
sinh x sinh x x
(i)
2 2
(ii) y = cos x ⇔ x = cos y, where 0 ≤ y ≤ p, - 1 ≤ x ≤ 1
-1 The hyperbolic functions have same properties that resemble to those of
p p
(iii) y = tan -1 x ⇔ x = tan y, where - < y < , - ∞ < x < ∞
trigonometric functions.
2 2
1.2.7 Inverse Hyperbolic Functions

1.2.4 Exponential Function The inverse hyperbolic functions are expressed in terms of natural logarithms and we
shall study them in higher classes.

1  x +1 
A function, in which the variable appears as exponent (power), is
sinh -1 x = ln(x + x 2 + 1 ), for all x (iv) coth -1 x = ln  , x < 1
2  x -1 
called an exponential function. The functions, y = eax, y = ex, y = 2x = (i)

1 
ex ln 2, etc are exponential functions of x.
cosh -1 x = ln(x + x2 - 1 ) x ≥ 1 (v) sech -1 x = ln  + , 0 < x ≤1
1 - x2
x 
 
(ii)
1.2.5 Logarithmic Function x

1 1 + x  1 
ln  , x < 1 (vi) csch -1 x = ln  + , x ≠ 0
If x = ay , then y = loga x , where a > 0, a ≠ 1 is called Logarithmic Function of x. tanh -1 x =
1 + x2
2 1-x  x 
 
(iii)
x
(i) If a = 10, then we have log10 x (written as lg x) which is known as the common
logarithm of x.
(ii) If a = e, then we have loge x (written as In x) which is known as the natural 1.2.8 Explicit Function
logarithm of x.
If y is easily expressed in terms of the independent variable x, then y is called an explicit
function of x. For example
(i) y = x2 + 2x - 1 (ii) =
y x - 1 are explicit functions of x.
version: 1.1 version: 1.1

8 9
1. Quadratic Equations eLearn.Punjab 1. Quadratic Equations eLearn.Punjab
1. Functions and Limits eLearn.Punjab 1. Functions and Limits eLearn.Punjab

Symbolically it can be written as y = f(x). Note : In both the cases, for each x in the domain of f, -x must also be in the domain of f.

1.2.9 Implicit Function Example 1: Show that the parametric equations x = a cos t and y = a sin t represent
the equation of the circle x2 + y2 = a2
If x and y are so mixed up and y cannot be expressed in terms of the independent
variable x, then y is called an implicit function of x. For example, Solution: The parametric equations are
2
xy - y + 9 x = a cos t (i)
(i) x2 + xy + y2 = 2 (ii) = 1 are implicit functions of x and y.
xy y = a sin t (ii)
We eliminate the parameter “t” from equations (i) and (ii).
Symbolically it is written as f(x, y) = 0. By squaring we get, x2 = a2 cos2 t
y2 = a2 sin2 t
(ix) Parametric Functions By adding we get, x2 + y2 = a2 cos2 t + a2 sin2 t
= a2 (cos2 t + sin2 t)
Some times a curve is described by expressing both x and y as function of a third ∴ x2 + y2 = a2, which is equation of the circle.
variable “t” or “q” which is called a parameter. The equations of the type x = f(t) and y = g(t)
are called the parametric equations of the curve . Example 2: Prove the identities
The functions of the form: (i) cosh2 x - sinh2 x = 1 (ii) cosh2 x + sinh2 x = cosh 2x
x = at2 x = a cos t x = a cos q x = a sec q
y = b sin q y = a tan q
(i) (ii) (iii) (iv)
y = at y = a sin t e x - e- x
are called parametric functions. Here the variable t or q is called parameter.
Solution: We know that sinh x = (1)
2

1.2.10 Even Function and e x + e- x (2)


cosh x =
2
A function f is said to be even if f(-x) = f(x) , for every number x in the domain of f. Squaring (1) and (2) we have

e 2 x + e -2 x + 2
For example: f(x) = x2 and f(x) = cos x are even functions of x.
Here f(-x) = (-x)2 = x2 = f(x) and f(-x) = cos (-x) = cos x = f(x) e 2 x + e -2 x - 2
sinh 2 x = and cosh 2 x =
4 4
1.2.11 Odd Function e 2 x + e -2 x + 2 e 2 x + e -2 x - 2
cosh 2 x - sinh 2 x = -
Now (i) 4 4
A function f is said to be odd if f(-x) = -f(x) , for every number x in the domain of f. e + e + 2 - e - e -2 x + 2 4
2x -2 x 2x
= =


For example, f(x) = x3 and f(x) = sin x are odd functions of x. Here 4 4
f(-x) = (-x)3 = -x3 = -f(x) and f(-x) = sin(-x) = -sin x = -f(x)
2 2
cosh x - sinh x = 1

version: 1.1 version: 1.1

10 11
1. Quadratic Equations eLearn.Punjab 1. Quadratic Equations eLearn.Punjab
1. Functions and Limits eLearn.Punjab 1. Functions and Limits eLearn.Punjab

e2 x + e-2 x + 2 e2 x + e-2 x - 2
3. Express the following:
2 2
and (ii) cosh x + sinh x = 4
+
4 (a) The perimeter P of square as a function of its area A.
e2 x + e-2 x + 2 + e2 x + e-2 x - 2 (b) The area A of a circle as a function of its circumference C.
=
4 (c) The volume V of a cube as a function of the area A of its base.
2e 2 x + 2e -2 x e 2 x + e -2 x 4. Find the domain and the range of the function g deined below, and
g(x) = 2x - 5
= =

∴ cosh 2 x + sinh 2 x = cosh 2x


4 2 (i) (ii) g (x) = x2 - 4
(iii) g (x) = x+1 (iv) g (x) = x - 3

6 x + 7 , x ≤ - 2 x -1 , x < 3
Example 3: Determine whether the following functions are even or odd.
g (x) =  g (x) = 
 4 - 3 x , -2 < x 2 x + 1 , 3 ≤ x
(v) (vi)
3x
x + 1
(a) f(x) = 3x4 - 2x2 + 7 (b) f (x) = (c) f(x) = sin x + cos x
2

, x ≠ -1 , x ≠ 4
x 2 - 3x + 2 x 2 - 16
(vii) g (x) = (viii) g (x) =
Solution:
f(-x) = 3(-x)4 - 2(-x)2 + 7 = 3x4 - 2x2 + 7 = f(x)
x+1 x -4
(a) Given f(x) = x3 - ax2 + bx + 1
f(x) = 3x4 - 2x2 + 7 is even.
5.
Thus If f(2) = -3 and f(-1) = 0 . Find the values of a and b.
3( - x)
f ( - x) = - 2 = - f (x)
6. A stone falls from a height of 60m on the ground, the height h afterx seconds is
approximately given by h(x) = 40 - 10x2
3x
( - x) + 1 x + 1
(b) 2

(i) What is the height of the stone when:.


3x
x + 1
f (x) = is odd (a) x = 1 sec ? (b) x = 1.5 sec ? (c) x = 1.7 sec ?
Thus
f(-x) = sin(-x) + cos(-x) = -sin x + cos x ≠ ± f(x)
2
(ii) When does the stone strike the ground?
(c)
7. Show that the parametric equations:
Thus f(x) = sin x + cos x is neither even nor odd
(i) x = at2 , y = 2at represent the equation of parabola
y2 = 4ax
EXERCISE 1.1
(ii) x = acosq , y = bsinq represent the equation of ellipse
x2 y2
x+4
+ =1
1. Given that: (a) f(x) = x2 - x (b) f (x) = a2 b2
(iii) x = asecq , y = btanq represent the equation of hyperbola
Find (i) f(-2) (ii) f(0) (iii) f(x - 1) (iv) f(x2 + 4)

f(a + h) - f(a)
x2 y 2
- =1
2. Find and simplify where, a 2 b2
h

f(x) = 6x - 9
8. Prove the identities:
(i) (ii) f(x) = sin x sech2 x = 1 - tanh2 x
f(x) = x3 + 2x2 - 1
(i) sinh 2x = 2sinh x cosh x (ii)
(iii) (iv) f(x) = cos x (iii) csch2 x = coth2 x - 1

version: 1.1 version: 1.1

12 13
1. Quadratic Equations eLearn.Punjab 1. Quadratic Equations eLearn.Punjab
1. Functions and Limits eLearn.Punjab 1. Functions and Limits eLearn.Punjab

Solution:
fg (x) = f (g (x)) = f ( x2 - 1) = 2 (x2 - 1) +1 = 2x2 - 1
9. Determine whether the given function f is even or odd.
(i)
(ii) gf (x) = g (f(x)) = g (2x + 1) = (2x + 1)2 - 1 = 4x2 + 4x
(i) f(x) = x3 + x (ii) f(x) = (x + 2)2

x- 1
f (x) = x x 2 + 5 , x ≠ -1
x+ 1
(iii) f2(x) = f (f(x)) = f (2x + 1) = 2(2x +1) + 1 = 4x + 3
(iv) g2(x) = g(gx) = g (x2 - 1) = (x2 - 1)2 - 1 = x4 - 2x2
(iii) (iv) f (x) =

fg (x) ≠ gf(x)
x3 - x
We observe from (i) and (ii) that

x + 1
2 3
(v) f (x) = x +6 (vi) f (x) = 2
Note:
It is important to note that, in general, gf (x) ≠ fg (x) , because gf (x)means that f is
1.3 COMPOSITION OF FUNCTIONS AND 1.
applied irst then followed by g, whereas fg (x) means that g is applied irst then
INVERSE OF AFUNCTION followed by f.
2. We usually write f as f 2 and ff as f 3 and so on.
Let f be a function from set X to set Y and g be a function from set Y to set Z. The
composition of f and g is a function, denoted by gof, from X to Z and is deined by 1.3.2 Inverse of a Function
(gof)(x) = g(f(x)) = gf(x) , for all xdX.
Let f be a one-one function from X onto Y. The inverse function of f denoted by f -1, is
1.3.1 Composition of Functions Remember That: a function from Y onto X and is deined by:
Briely we write gof as gf. x = f -1(y), [ y d Y if and only if y = f(x) , [ x d X.
Explanation Illustration by arrow diagram
Consider two real valued functions f and g deined by The inverse function reverses the correspondence
f(x) = 2x + 3 and g(x) = x2 of the original function, so that
then gof(x) = g(f (x) ) = g(2x + 3) = (2x + 3)2 f -1(y) = x, when f(x) = y
and f(x) = y , when f -1(y) = x
The arrow diagram of two consecutive mappings, f
We can ind the composition of the functions f and
followed by g, denoted by gf is shown in the igure.
f -1 as follows:

Thus a single composite function gf(x) is equivalent (f -1 of)(x) = f -1(f (x)) = f -1(y) = x
to two successive functions f followed by g. and (fof -1)(y) = f (f -1(y)) = f(x) = y
We note that f -1 of and fof -1 are identity mappings on the domain and range of f and
Example 1: Let the real valued functions f and g be deined by f -1 respectively.
f(x) = 2x + 1 and g(x) = x2 - 1
Obtain the expressions for (i) fg (x) (ii) gf (x) (iii) f2 (x) (iv) g2 (x) 1.3.3 Algebraic Method to ind the Inverse Function

The inverse function can be found by using the algebraic method as explained in the
following example:
version: 1.1 version: 1.1

14 15
1. Quadratic Equations eLearn.Punjab 1. Quadratic Equations eLearn.Punjab
1. Functions and Limits eLearn.Punjab 1. Functions and Limits eLearn.Punjab

Example 2: Let f : R " R be the function deined by So the value of f(x) = 2 + x - 1 varies over the interval [2, +T).
f(x) = 2x + 1. Find f -1(x) Therefore range f = [2, +T)
By deinition of inverse function f -1, we have
Remember that: domain f -1 = range f = [2, +T)
The change of name of variable in the deinition of function does not change that function and range f -1 = domain f = [1, +T)
where the domain and range coincide.
EXERCISE 1.2
Solution: We ind the inverse of f as follows:
Write f(x) = 2x + 1 = y 1. The real valued functions f and g are deined below. Find
So that y is the image of x under f. (a) fog (x) (b) gof (x) (c) fof (x) (d) gog (x)
Now solve this equation for x as follows:
, x ≠1
y = 2x +1 3
⇒ 2x = y - 1
(i) f(x) = 2x + 1 ; g (x) =
x -1

g (x) = 2 , x ≠ 0
y-1
1

(ii) f (x) = x +1 ;
x
x=
, x ≠1
2 1
∴ (y) = ( y - 1 ) ∴ x = f -1 (y) 
-1 1 (iii) f (x) = ; g(x) = (x2 + 1)2
f x -1

f(x) = 3x4 - 2x2 , x≠ 0


2
2
-1 (iv) ; g (x) =
To ind f (x), replace y by x. x

∴ f -1 (x) =
1 2. For the real valued function, f deined below, ind
(x - 1)
2 (a) f -1(x) (b) f -1(-1) and verify f (f -1 (x)) = f -1 f(x)) = x
Veriication: (i) f(x) = -2x + 8 (ii) f(x) = 3x3 + 7

f ( f -1 (x) ) = f  (x - 1)  = 2  (x - 1)  + 1 = x
1  1 
2x + 1
(iii) f(x) = (-x + 9)3 (iv)
2  2 
f (x) = ,x>1
x -1

f -1 ( f (x) ) = f -1 ( 2x + 1) = ( 2x + 1 - 1) = x
1 3. Without inding the inverse, state the domain and range of f -1.
and

, x ≠ -3
2
1
(i) (iii)
Without inding the inverse, state the domain and range of f -1, where
f (x) = x+2 f (x) =
Example 3: x+3
f (x) = 2 + x -1 (ii) f (x) =
x -1
, x≠ 4 (iv) f(x) = (x - 5)2 , x 8 5
x -4
Solution: We see that f is not deined when x < 1.
∴ Domain f = [1, +T)
As a varies over the interval [1, +T), the value of x - 1 varies over the interval [0, +T).
version: 1.1 version: 1.1

16 17
1. Quadratic Equations eLearn.Punjab 1. Quadratic Equations eLearn.Punjab
1. Functions and Limits eLearn.Punjab 1. Functions and Limits eLearn.Punjab

1.4.4 Concept of Limit of a Function


1.4 LIMIT OF A FUNCTION AND THEOREMS
ON LIMITS (i) By inding the area of circumscribing regular polygon
Consider a circle of unit radius which circumscribes a square (4-sided regular polygon)
The concept of limit of a function is the basis on which the structure of calculus rests.
as shown in igure (1).
Before the deinition of the limit of a function, it is essential to have a clear understanding of
The side of square is 2 and its area is 2 square unit. It is clear that the area of inscribed
the meaning of the following phrases:
4-sided polygon is less than the area of the circum-circle.

1.4.1 Meaning of the Phrase “x approaches zero”

Suppose a variable x assumes in succession a series of values as

1, 1 , 1 , 1 , 1 ,... i.e., 1, 1 , 1 , 1 , 1 , ... , 1 ,...


2 4 8 16 2 22 23 24 2n
We notice that x is becoming smaller and smaller as n increases and can be made as small
as we please by taking n suiciently large. This unending decrease of x is symbolically written
as x " 0 and is read as “x approaches zero” or “x tends to zero”. Bisecting the arcs between the vertices of the square, we get an inscribed 8-sided
polygon as shown in igure 2. Its area is 2 2 square unit which is closer to the area of
Note: The symbol x " 0 is quite diferent from x = 0 circum-circle. A further similar bisection of the arcs gives an inscribed 16-sided polygon as
(i) x " 0 means that x is very close to zero but not actually zero. shown in igure (3) with area 3.061 square unit which is more closer to the area of circum-
(ii) x = 0 means that x is actually zero. circle.
It follows that as ‘n’ , the number of sides of the inscribed polygon
1.4.2 Meaning of the Phrase “x approaches ininity” increases, the area of polygon increases and becoming nearer to
3.142 which is the area of circle of unit radius i.e., pr 2
= p(1)2
Suppose a variable x assumes in succession a series of values as = p c 3.1 42.
1 ,10 ,100 ,1000 ,10000 .... i.e., 1,10,102,103.......,10n,... We express this situation by saying that the limiting value of the area o f the inscribed
It is clear that x is becoming larger and larger as n increases and can be made as large polygon is the area of the circle as n approaches ininity, i.e.,
as we please by taking n suiciently large. This unending increase of x is symbolically written Area of inscribed polygon " Area of circle
as “x "T” and is read as “x approaches ininity” or “x tends to ininity”. as n "T
Thus area of circle of unit radius = p = 3.142 (approx.)
1.4.3 Meaning of the Phrase “x approaches a” (ii) Numerical Approach
Consider the function f(x) = x3
Symbolically it is written as “x " a” which means that x is suiciently close to but diferent
from the number a, from both the left and right sides of a i.e; x - a becomes smaller and
The domain of f(x) is the set of all real numbers.
Let us ind the limit of f(x) = x3 as x approaches 2.
smaller as we please but x - a ≠ 0.
version: 1.1 version: 1.1

18 19
1. Quadratic Equations eLearn.Punjab 1. Quadratic Equations eLearn.Punjab
1. Functions and Limits eLearn.Punjab 1. Functions and Limits eLearn.Punjab

The table of values of f(x) for diferent values of x as x approaches 2 from left and Theorem 2: The limit of the diference of two functions is equal to the diference
of their limits.
Lim  f ( x ) - g ( x )  = Lim f ( x ) - Lim g ( x ) = L - M
right is as follows:
from left of 2 2 from right of 2

Lim ( x - 5 ) = Lim x - Lim 5 = 3 - 5 = - 2


x →a x →a x →a

x 1 1.5 1.8 1.9 1.99 1.999 1.9999 2.0001 2.001 2.01 2.1 2.2 2.5 3
For example, x →3 x →3 x →3
3
f(x)=x 1 3.375 5.832 6.859 7.8806 7.988 7.9988 8.0012 8.012 8.1206 9.261 10.648 15.625 27
Theorem 3: If k is any real number, then
Lim  kf ( x )  = k Lim f ( x ) = kL
The table shows that, as x gets closer and closer to 2 (suiciently close to 2), from

Lim ( 3 x ) = 3 Lim (x ) = 3 (2 ) = 6
both sides, f(x) gets closer and closer to 8. x →a x →a

We say that 8 is the limit of f(x) when x approaches 2 and is written as: For example:
f ( x ) → 8 as x → 2
x →2 x →2
3
or lim ( x ) = 8
x →2
Theorem 4: The limit of the product of the functions is equal to the product of
1.4.5 Limit of a Function their limits.

Lim  f ( x ) g ( x )  =  Lim f ( x )   Lim g ( x )  = LM


Let a function f(x) be deined in an open interval near the number “a” (need not be x →a  x →a   x →a 
For example: Lim ( 2 x )( x + 4 ) =  Lim ( 2 x )   Lim ( x + 4 )  = ( 2 )( 5 ) =10
 x→1   x→1 
at a).
x →1
If, as x approaches “a” from both left and right side of “a”, f(x) approaches a speciic
number “L” then “L”, is called the limit of f(x) as x approaches a. Theorem 5: The limit of the quotient of the functions is equal to the quotient of
Symbolically it is written as:
Lim f ( x ) = L read as “limit of f(x), as x " a , is L”.
their limits provided the limit of the denominator is non-zero.

 f ( x )  Lim f ( x)
g ( x ) ≠ 0, M ≠ 0
x →a

Lim  
( ) ( )
x →a L
 
It is neither desirable nor practicable to ind the limit of a function by numerical = = ,
x →a g x Lim g x M
x →a

 3 x + 4  Lim
approach. We must be able to evaluate a limit in some mechanical way. The theorems on
Lim  =
(3 x + 4) 6 + 4 10
x →2

 x+3 
limits will serve this purpose. Their proofs will be discussed in higher classes. For example: = = =2
x →2 Lim (x + 3) 2+3 5
x →2

Limit of  f ( x )  , where n is an integer


1.4.6 Theorems on Limits of Functions

( )
n
Theorem 6:

Let f and g be two functions, for which Lim f ( x ) = L and Lim g ( x ) = M , then Lim  f ( x )  = Lim f ( x ) = Ln
n n

x →a x →a x →a x →a

Theorem 1: The limit of the sum of two functions is equal to the sum of their limits.
Lim  f ( x ) + g ( x )  = Lim f ( x ) + Lim g ( x ) = L + M
( )
Lim ( 2 x - 3) = Lim ( 2 x - 3) = (5)3 = 125
3
3
For example:
x →4 x →4

x →a x →a x →a

Lim ( x + 5 ) = Lim x + Lim 5 = 1 + 5 = 6


We conclude from the theorems on limits that limits are evaluated by merely
For example,
x →1 x →1 x →1
substituting the number that x approaches into the function.

version: 1.1 version: 1.1

20 21
1. Quadratic Equations eLearn.Punjab 1. Quadratic Equations eLearn.Punjab
1. Functions and Limits eLearn.Punjab 1. Functions and Limits eLearn.Punjab

Example : If P(x) = anxn + an-1 x n-1


+ .... + a1 x + a0 is a polynomial function of degree n, Case II: Suppose n is a negative integer (say n = -m) , where m is a

Lim P ( x ) = P ( c )
positive integer.
then show that
x →c
xn - a n x-m - a -m
Solution: Using the theorems on limits, we have Now =

-1  x m - a m 
x -a x -a
=
Lim P=
(x) Lim+ (an x n an-1 x +
n -1
+ .... = m m  (a ≠ 0)
x a  x -a 
a1 x + a0
x →c x →c

= an Lim x n + an-1 Lim x n-1 + .... + a1 Lim x + Lim a0  -1   x - a 


∴ Lim = Lim  m m   
x →c x →c x →c x →c
xn - a n m m

= an c n + an-1 c n-1 + .... + a1c + a0


x →a x -a x →a
 x a  x - a 
∴ Lim P(x) = P(c) -1
x →c
= m m
.( ma m-1 ), (By case 1)
a a
= -ma - m-1
1.5 LIMITS OF IMPORTANT FUNCTIONS
∴ Lim = na n-1 (n = - m)
xn - a n
0 x →a
If, by substituting the number that x approaches into the function, we get   , then we
x -a
0
evaluate the limit as follows:

+a a

We simplify the given function by using algebraic technique of making factors if possible 1.5.2 Lim =
and cancel the common factors. The method is explained in the following important limits.
0
By substituting x = 0, we have   form, so rationalizing the numerator.
0
xn - an
= na n-1 where n is an integer and a > 0  x + a - a  
x - a ∴
1.5.1
= Lim   
Lim
x →a x+a - a x +a + a

  a
Lim
x →0 x x →0 x x +a +
Case 1: Suppose n is a positive integer. x +a -a
= Lim
x →0 x(
0
x+a + a)
By substituting x = a , we get   form. So we make factors as follows:
0
x
xn - an = (x - a) (xn-1 + axn-2 + a2 xn-2 + .... + an-1)
= Lim
x →0 x( x+a + a)

xn - an (x - a ) ( ax n-1 + ax n-2 a 2 x n-3 + . . . . + a n-1 )



1
x-a x-a
= Lim
x →0
Lim = Lim
x →a x →a

= Lim (x + ax + a x + .... + an-1) (polynomial function)


x+a + a
x →a
n-1 n-2 2 n-3 1 1
= =
= an-1 + a.an-2 + a2.an-3 + .... + an-1 a+ a 2 a
= an-1 + an-1 + an-1 + .... + an-1 (n terms)
= nan-1
version: 1.1 version: 1.1

22 23
1. Quadratic Equations eLearn.Punjab 1. Quadratic Equations eLearn.Punjab
1. Functions and Limits eLearn.Punjab 1. Functions and Limits eLearn.Punjab

Example 1: Evaluate (b) Limit as x " -T. This type of limits are handled in the same way as limits as x " +T.

i.e. = 0, where x ≠ 0
1
x2 - 1 x -3 Lim
(i) Lim 2 (ii) Lim x →-∞ x
x →1 x - x x →3 x- 3
The following theorem is useful for evaluating limit at ininity.
0
  form
2
x -1 Theorem: Let p be a positive rational number. If xp is deined, then
0
Solution: (i) Lim (By making factors)
x →1 x2 - x
a a

Lim = 0 and Lim p = 0 ,where a is any real number.
x2 - 1 ( x - 1)( x + 1) x+1 1+1 x →+∞ p x →-∞ x

-5 -5
Lim 2 = Lim = Lim = =2 x
x →1 x - x x →1 x( x - 1) x →1 x 1
x -3 0
6
(By making factors of x - 3)
For example, Lim 3 = 0 , Lim = Lim 1/ 2 = 0
x →±∞ x x →-∞ x →-∞ x
  form
x - 3 0
(ii) Lim x
x →3
1 1
∴ Lim
and Lim = Lim =0
x -3 ( x + 3 )( x - 3) x →+∞ x →+∞
5 1
= Lim x
x →3 x- 3 x →3 x- 3 x5
= Lim ( x + 3)
x →3 1.5.4 Method for Evaluating the Limits at Ininity
= ( 3 + 3)
=2 3 In this case we irst divide each term of both the numerator and the denominator by
the highest power of x that appears in the denominator and then use the above theorem.
1.5.3 Limit at Ininity
5 x 4 - 10x 2 + 1
x →+∞ -3 x 3 + 10x 2 + 50
Example 2: Evaluate Lim
, when x " c (a number)
f(x)
We have studied the limits of the functions f(x), f(x) g(x) and
g(x)
Let us see what happens to the limit of the function f(x) if c is +T or -T (limits at ininity)
Solution: Dividing up and down by x3 , we get

i.e. when x " +T and x " -T. ∞-0+0


=∞
5 x 4 - 10x 2 + 1 5 x - 10/x + 1/x3
x →+∞ -3 3 + 10x 2 + 50 x →+∞ -3 + 10/x + 50/ 3 -3 + 0 + 0
Lim = Li =

(a) Limit as x " +T


4 x 4 - 5x 3
f (x) = , when x ≠ 0
Example 3: Evaluate Lim
1 x →-∞ 3 x 5 + 2x 2 + 1
Let
Since x < 0, so dividing up and down by (-x)5 = -x5,
x
Solution:
This function has the property that the value of f(x) can be made as close as we please we get

-4 / x + 5/x 2 0+0
to zero when the number x is suiciently large.
4 x 4 - 5x 3
=0 x →-∞ -3 - 2/x - 1/x -3 - 0 - 0
Lim = Lim = =0
x →-∞ 3 x + 2x + 1
1 5 2 3 5
We express this phenomenon by writing Lim
x →∞ x

version: 1.1 version: 1.1

24 25
1. Quadratic Equations eLearn.Punjab 1. Quadratic Equations eLearn.Punjab
1. Functions and Limits eLearn.Punjab 1. Functions and Limits eLearn.Punjab

Example 4: Evaluate
x →0
Deduction Lim (1 + x )1 / x = e

 1
2 - 3x 2 - 3x
We know that Lim 1 +  = e
(i) L im (ii) L im n
x →-∞ x →+∞

 n
2 2
3 + 4x 3 + 4x (i)
x →∞

Solution: (i) Here x 2 = x = -x as x < 0 1 1


put n = , then = x in (i)

∴ Dividing up and down by -x, we get


x n

-2/x + 3
When x → 0, n → ∞
2 - 3x 0+3 3
Lim = L im = =
x →-∞ x →-∞

 1
3 + 4x 2 3 / x2 + 4 0+4 2
Lim 1 +  = e
n

 n
(ii) Here x == x == --xx as
a xx > 0 As

x →∞

∴ Lim (1 + x )
Dividing up and down by x, we get
1/ x

-3
=e
x →0
2 - 3x 2/x + 3 0-3
Lim = Lim = =
x →+∞ x →+∞

ax - 1
3 + 4x 2 3 / x2 + 4 0+4 2

1.5.6
x →0
Lim = loge a
 1
1 +
n x
x →+∞
 n 
1.5.5 Lim = e.
Put ax - 1 = y (i)
then ax = 1 + y
By the Binomial theorem, we have
So x = loga (1 + y)
 1 1 n( n - 1 )  1  n( n - 1 )( n - 2 )  1  when x " 0, y " 0
1 +  = 1 + n     +   +...
n 2 3 From (i)
 n n n n
+

1  1 1 1  2 
2! 3!

= 1 +1 + 1 -  + 1 - 1 -  + . . .
2!  n  3 !  n  n  ∴
x
a -1 y 1
Lim = Lim = Lim
x →0 y →0 log ( 1 + y) y →0

( )
x 1
when n 
→ ∞,
1 2 3
a log a ( 1 + y)
, , , . . . all tend to zero. y

 1
n n n 1 1
 Lim( 1 + y)1/ y = e
∴ L im 1 +  = 1 + 1 +
= Lim = = log a
y →0 log ( 1 + y) y →0
n
1 1 1 1 1 / y e

 n
+ + + +... a log a e
x →∞
2! 3! 4! 5!
= 1 + 1 + 0.5 + 0.166667 + 0.0416667 + ... = 2.718281 ...  ex - 1 
Deduction Lim   = loge e = 1.
x→0  x 
As approximate value of e is = 2.718281.

 1
∴ Lim 1 +  = e .
n

 n
x
a -1
x →+∞ We know that Lim = log e a (1)
x →0 x
version: 1.1 version: 1.1

26 27
1. Quadratic Equations eLearn.Punjab 1. Quadratic Equations eLearn.Punjab
1. Functions and Limits eLearn.Punjab 1. Functions and Limits eLearn.Punjab

Put a = e in (1), we have 1.5.7 The Sandwitch Theorem


ex - 1
Lim = log e e = 1. Let f, g and h be functions such that f(x) 7 g(x) 7 h(x) for all numbers x in some open
x →0 x
interval containing “c”, except possibly at c itself.
Important Results to Remember If Lim f (x) = L and Lim h(x) = L, then Lim g ( x) = L
x →c x →c x →c

 1 
Lim (e x ) = ∞ Lim (e x ) = Lim  - x  = 0,
e 
(i) (ii) Many limit problems arise that cannot be directly evaluated by algebraic techniques. They
x →∞ x →-∞ x →-∞

a
require geometric arguments, so we evaluate an important theorem.
Lim   = 0 , where a is any real number. sinq
x If q is measured in radian, then Lim
(iii)
q
x →±∞
1.5.8 =1
q →0

Proof: To evaluate this limit, we apply a new technique. Take q a positive acute central angle
Example 5: Express each limit in terms of the number ‘e’

 3
Lim 1 + 
of a circle with radius r = 1. As shown in the igure, OAB represents a sector of the circle.
2n 1

 n
(a) (b) Lim (1+2h) h
n→+∞ h →0
Given OA = OB = 1 (radii of unit circle)

∴ In rt ∆OCB, sinq =
Solution: (a) Observe the resemblance of the limit with BC
= BC (  OB = 1)
OB
 1
Lim 1 +  = e In rt ∆OAD, tanq =
n

 n
AD
n→∞ = AD (  OA = 1)
OA

    In terms of q, the areas are expressed as:


 3       
  Produce OB to D so that AD ⊥ OA. Draw BC ⊥ OA. Join AB
6 6

 1 +  =  1 +   =  1 +
n n
2n
3 3 1 3

 n  n  n / 3
   
Area of ∆OAB = OA BC = (1)(sinq ) = sinq
1 1 1
 put m = n/3 
(i)
 3  1   when n → ∞ ,
2 2 2
∴ Lim 1 +  = Lim 1 + 
6

 Area of sector OAB = r 2q = (1)(q ) = q


2n m

 n  
 m    m → ∞ 
= e6 1 1 1
n→+∞ m→+∞ (ii) (  r = 1)

2 2 2
Area of ∆OAD = OA AD = (1)(tanq ) = tanq
1 1 1
and (iii)
2 2 2
1
(b) Observe the resemblance of the limit with Lim (1 + x) = e , x
From the igure we see that
x →0

Area of ∆OAB < Area of sector OAB < Area of ∆OAD


 
∴ Lim (1 + 2h) = Lim (1 + 2h)  (put m = 2h, when h → 0, m → 0
2

q
1 1

  ⇒ sinq < < tanq


h 2h
h →0 h →0 1 1

  As sin q is positive, so on division by sin q, we get


2 2 2
= Lim (1 + m)  = e 2
2
1 1

 
m
m →0 2
version: 1.1 version: 1.1

28 29
1. Quadratic Equations eLearn.Punjab 1. Quadratic Equations eLearn.Punjab
1. Functions and Limits eLearn.Punjab 1. Functions and Limits eLearn.Punjab

EXERCISE 1.3
q  p
 q 
sinq cos q
1
 2
1< < 0 < <

sinq sinq
1. Evaluate each limit by using theorems of limits:
> cos q cos q < (ii) Lim (3x 2 - 2x + 4)
q q
i.e., 1> or <1 (i) Lim (2x + 4) (iii) Lim x2 + x + 4
x →3 x →1 x →3
when q " 0, cos q " 1
Sin q x2 - 4
2 x 3 + 5x
q
(iv) Lim (v) Lim ( x3 + 1 - x2 + 5 ) (vi) Lim
x →2 x →2 x →-2
Since is sandwitched between 1 and a quantity approaching 1 itself.
3x - 2
So, by the sandwitch theorem, it must also approach 1. 2. Evaluate each limit by using algebraic techniques.
sinq
x3 - x  3 x3 + 4x  x3 - 8
q (ii) Lim  2 
i.e., lim =1
q →0
x →2 x + x - 6
 x +x 
(i) Lim (iii) Lim 2
x →-1 x →0
Note: The same result holds for -p/2 < q < q
x+1
x 3 - 3 x 2 + 3x - 1  x3 + x 2  2x 2 - 32
sin 7q Lim  2 
x3 - x  x - 1 x - 4x 2
(iv) Lim (v) (vi) Lim 3
x →1 x →-1 x →4

q x- 2 x+h - x
Example 6: Evaluate: lim
q →0

sinq
xn - an
x - 2
(vii) Lim (viii) Lim (ix) Lim
x →2 h →0 x →a
q
Solution: Observe the resemblance of the limit with lim =1 h xm - am
q →0
Let x = 7q so that q = x/7
when q " 0 we have x " 0
3. Evaluate the following limits

1 - cos q
,

sin 7q
sin x 0
sinq
sin 7 x

(i) Lim (ii) Lim (iii) Lim
x →0 q →0
q
x →0
sin x sin x
Lim = Lim = 7 Lim = (7)(1) = 7 x x
q →0 x →0 x/7 x →0 x
x →p p - x
sin x sina x x
1 - cos q
(iv) Lim (v) Lim (vi) Lim
x →0 sinbx x →0 tan x

Example 7: Evaluate: Lim


q →0 q 1 - cos 2 x 1 - cos x sin 2 q
1 - cos q 1 - cos q 1 + cos q q
(vii) Lim (viii) Lim (ix) Lim
x →0 x2 x →0 sin 2 x q →0

q q 1 + cos q sec x - cos x 1 - cos p q tanq - sinq


Solution: = .
q →0 1 - cos qq sin3 q
1 - cos 2 q sin 2 q  sinq 
(x) Lim (xi) Lim (xii) Lim
q →0

x →0

= sinq 
x
q (1 + cos q ) q (1 + cos q )  
 q  1 + cos q 
1
= =

1 - cos q sinq
4. Express each limit in terms of e:
∴ = lim sinq lim
q q q →0 1 + cos q
1
 1  1 2  1
lim lim
q →0 q →0 q →0
Lim 1 +  Lim 1 +  (iii) Lim 1 - 
n
2n n

 n  n  n
(i) (ii)
1 n→+∞ n→+∞ n→+∞

1+1
= (0)(1)( )
 1   4
(iv) Lim 1 +  Lim 1 +  (vi) Lim (1 + 3 x ) x
n n
2

 3n   n
= (0) (v)
n→+∞ n→+∞ x →0

version: 1.1 version: 1.1

30 31
1. Quadratic Equations eLearn.Punjab 1. Quadratic Equations eLearn.Punjab
1. Functions and Limits eLearn.Punjab 1. Functions and Limits eLearn.Punjab

(vii) Lim (1 + 2 x )  x 
(viii) Lim (1 - 2h )
Example 1: Determine whether Lim f ( x) and Lim f ( x) exist, when
x →2 x →4
(ix) Lim  
1 x
1

x →∞ 1 + x
 
2 x2

0≤ x ≤2
x →0 h →0
2 x + 1 if
h

e1/ x - 1 e1/ x - 1 
, x<0 , x>0 f ( x) = 7 - x if 2≤ x ≤4
+1 +1 
(x) Lim 1/ x (xi) Lim 1/ x
x →0 e x →0 e

 x if 4≤ x ≤6
Solution:
1.6 Continuous and Discontinuous Functions
(i) Lim- f ( x) = Lim- (2 x + 1) = 4 + 1 = 5
x →2 x →2

1.6.1 One-Sided Limits Lim+ f ( x) =Lim+ (7 - x) =7 - 2 = 5


x →2 x →2

=
Since Lim =
f ( x) Lim f ( x) 5
-
x →2 +
x →2
In deining Lim f ( x) , we restricted x to an open interval containing c i.e., we studied
x →c ⇒ Lim f ( x) exists and is equal to 5.
x →2
the behavior of f on both sides of c. However, in some cases it is necessary to investigate (ii) Lim- f ( x) =Lim- (7 - x) =7 - 4 = 3
x →4 x →4

Lim+= =
one-sided limits i.e., the left hand limit and the right hand limit.
f ( x) Lim ( x) 4
x →4 +
x →4

(i) The Left Hand Limit Since Lim- f ( x) ≠ Lim+ f ( x)


x →4 x →4

Lim f ( x) = L is read as the limit of f(x) is equal to L as x approaches c from the left i.e., Therefore Lim f ( x) does not exist.
x →4
x →c
for all x suiciently close to c, but less than c, the value of f(x) can be made as close as we We have seen that sometimes Lim f ( x) = f (c) and sometimes it does not and also sometimes
x →c
please to L.
f (c) is not even deined whereas Lim f ( x) exists.
x →c

(ii) The Right Hand Limit


1.6.3 Continuity of a function at a number
Lim f ( x) = M is read as the limit of f(x) is equal to M as x approaches c from the right
x →c
i.e., for all x suiciently close to c, but greater than c, the value of f(x) can be made as close as (a) Continuous Function
we please to M. A function f is said to be continuous at a number “c” if and only if the following three
conditions are satisied:
Note: The rules for calculating the left-hand and the right-hand limits are the same as
(i) f (c) is deined. (ii) Lim f ( x) exists, (iii) Lim f ( x) = f (c)
we studied to calculate limits in the preceding section. x →c x →c

(b) Discontinuous Function


1.6.2 Criterion for Existence of Limit of a Function
If one or more of these three conditions fail to hold at “c”, then the function f is said to

= = =
be discontinuous at “c”.
Lim f ( x) L if and only if Lim f ( x) Lim+ f ( x) L
x →c x →c
-
x →c

version: 1.1 version: 1.1

32 33
1. Quadratic Equations eLearn.Punjab 1. Quadratic Equations eLearn.Punjab
1. Functions and Limits eLearn.Punjab 1. Functions and Limits eLearn.Punjab

It is noted that there is no break in the graph. (See igure (i))


x2 - 1
x - 1 if x ≠ 3
Example 2: Consider the function f ( x ) = x2 - 9
(b) g( x ) =
x -3
Solution: Here f (1) is not deined

As g (x) is not deined at x = 3
⇒ g (x) is discontinuous at x = 3
f (x) is discontinuous at 1.
(See igure (ii)).
x2 - 1 It is noted that there is a break in the graph at x = 3
x - 1
Further Lim f ( x ) = lim = lim (x + 1) = 2 (finite)
x →1 x →1 x →1

Therefore f (x) is continuous at any other number x ≠ 1 Example 5: Discuss continuity of f at 3,

For f (x) = 3x2 - 5x + 4, discuss continuity of f at x = 1 x - 1 , x < 3


when f ( x) = 
Example 3: if
2 x + 1 , if 3 ≤ x
Solution: Lim f ( x) =Lim (3 x 2 - 5x + 4)
- =3 =
5+4 2.
x →1 x →1 A sketch of the graph of f is shown in the igure (iii).
and f(1) = 3 - 5 + 4 = 2
Solution:
We see that there is a break in the graph at the point when x = 3
⇒ Now f (3) = 2(3) + 1 = 7
⇒ Condition (i) is satisied.
Lim f ( x) = f (1)
x →1

∴ f (x) is continuous at x = 1
Lim- f ( x) = Lim- f ( x - 1) = 3 - 1 = 2
x →3 x →3

Lim+ f ( x) = Lim+ f (2 x + 1) =6 + 1=7


Example 4: Discuss the continuity of the function f (x) and g (x) at x = 3.
x →3 x →3

Lim f ( x)≠≠ Lim


 x2 - 9

 if x ≠ 3
f ( x)

x →3-
-

x →3
+ +

f(x) =  x -3 g( x) = if x ≠ 3
2
x -9 i.e. condition (ii) is not satisied
 6 if x = 3
(a) (b)
 ∴
x -3
Lim f ( x) does not exist
x →3
Solution: (a) Given f (3) = 6
∴ the function f is deined at x = 3.
Hence f(x) is not continuous at x = 3

x2 - 9 EXERCISE 1.4
Now Lim f( x ) = Lim
x →3 x →3

( x + 3)( x - 3)
x -3
1. Determine the left hand limit and the right hand limit and then, ind the limit of the
following functions when x " c
= Lim
= Lim ( x + 3) = 6
x →3 x -3
x →3
x2 - 9
f(x) = 2x + x - 5, c = 1 , c = -3
x - 3
2
(i) (ii) f( x ) =
f( x ) = x - 5 ,
As Lim f( x ) = 6 = f( 3 )
x →3

∴ f (x) is continuous at x = 3 (iii) c=5

version: 1.1 version: 1.1

34 35
1. Quadratic Equations eLearn.Punjab 1. Quadratic Equations eLearn.Punjab
1. Functions and Limits eLearn.Punjab 1. Functions and Limits eLearn.Punjab

2. Discuss the continuity of f(x) at x = c: 1.7 Graphs

2 x + 5 if x ≤ 2

We now learn the method to draw the graphs of the Explicit Functions like y = f(x) ,
f ( x) = 
4 x + 1 if
(i) ,c =2 where f(x) = ax, ex, loga x , and loge x.
 x 2
3 x - 1 if x < 1 1.7.1 Graph of the Exponential Function f(x) = ax

f ( x) = 

(ii) 4 if x = 1, c = 1
 2 x if x > 1 Let us draw the graph of y = 2x, here a = 2.
We prepare the following table for diferent values of x and f(x) near the origin:
-4 -3 -2 -1
 if x ≤ - 2
x 0 1 2 3 4
 2
3x
If f ( x) =  x - 1 if - 2 < x < 2
x
y = f(x) = 2 0.0625 0.125 0.25 0.5 1 2 4 8 16


3.
 if x ≥ 2
Discuss continuity at x = 2 and x = -2
3 Plotting the points (x, y) and joining them with smooth
curve as shown in the igure, we get the graph of y = 2x.
From the graph of 2x the characteristics of the graph
x + 2 , x ≤ -1
of y = ax are observed as follows:

If f ( x) = 
If a > 1, (i) ax is always +ve for all real values of x.

4. , find "c" so that Lim f ( x) exists.
x →-1

c + 2 , x > -1
(ii) ax increases as x increases.
(iii) ax = 1 when x = 0
(iv) ax " 0 as x "-T
5. Find the values m and n, so that given function f is continuous
at x = 3.

1.7.2 Graph of the Exponential Function f(x) = ex


 mx x < 3
 mx if x < 3
if
(i) f ( x) =  n x = 3 (ii) f ( x) =  2
-2 x + 9 if  x if x ≥ 3
if
 x > 3 As the approximate value of ‘e’ is 2.718
The graph of ex has the same
characteristics and properties as that of ax when
 2x + 5 - x + 7
 , x≠ 2
a > 1 (discussed above).
If f ( x ) = 

6. We prepare the table of some values of x and f(x)
, x=2
x -2
 k near the origin as follows:
Find value of k so that f is continuous at x = 2.

version: 1.1 version: 1.1

36 37
1. Quadratic Equations eLearn.Punjab 1. Quadratic Equations eLearn.Punjab
1. Functions and Limits eLearn.Punjab 1. Functions and Limits eLearn.Punjab

x -3 -2 -1 0 1 2 3 1.7.5 Graphs of Implicit Functions


x
y = f(x) = e 0.05 0.135 0.36 1 2.718 7.38 20.07
Plotting the points (x, y) and joining them with smooth curve as shown, we get the (a) Graph of the circle of the form x2 + y2 = a2
graph of y = ex.
Example 1: Graph the circle x2 + y2 = 4 (1)
1.7.3 Graph of Common Logarithmic Function f(x) = lg x.
Solution: The graph of the equation x2 + y2 = 4 is a circle of radius 2, centered at the
If x = 10y, then y = lg x origin and hence there are vertical lines that cut the graph more than once. This can also be
Now for all real values of y, 10y > 0 ⇒ x > 0 seen algebraically by solving (1) for y in terms of x.
This means lg x exists only when x > 0 y = ± 4 - x2
⇒ Domain of the lg x is +ve real numbers. The equation does not deine y as a function of x.
For example, if x = 1, then y = ± 3 .
Hence ( (1, 3) ) and ( (1, - 3) ) are two points on the circle and vertical line passes through
Note: lg x is undeined at x = 0.

these two points.


For graph of f(x) = lg x, we ind the values of lg x from
We can regard the circle as the union of two semi-circles.
y = 4 - x 2 and y = - 4 - x 2
the common logarithmic table for various values of x > 0.

Table of some of the corresponding values of x and f(x) is as under: Each of which deines y as a function of x.
x "0 0.1 1 2 4 6 8 10 "+T
y = f(x) = lg x "-T -1 0 0.30 0.60 0.77 0.90 1 "+T
Plotting the points (x, y) and joining them with a smooth curve we get the graph as
shown in the igure.

1.7.4 Graphs of Natural Logarithmic


We observe that if we replace (x, y) in turn by (-x, y), (x, -y) and (-x, -y), there is no
Function f(x) = In x:
change in the given equation. Hence the graph is symmetric with respect to the y-axis, x-axis
The graph of f(x) = In x has similar properties as that
and the origin.
y2 = 4 ⇒ y = ±2
of the graph of f(x) = lg x.
x = 0 implies
y2 = 3 ⇒ y = ± 3
By using the table of natural logarithm for various values
x = 1 implies
y2 = 0 ⇒ y = 0
of x, we get the graph of y = In x as shown in the igure.
x = 2 implies
By assigning values of x, we ind the values of y. So we prepare a table for some values
of x and y satisfying equation (1).
version: 1.1 version: 1.1

38 39
1. Quadratic Equations eLearn.Punjab 1. Quadratic Equations eLearn.Punjab
1. Functions and Limits eLearn.Punjab 1. Functions and Limits eLearn.Punjab

Solution: For the choice of t in [-2, 2], we prepare a table for


x 0 1 2 -1 - -2 some values of x and y satisfying the given equation.
-2 -1
3 3

± 3 ± 3
t 0 1 2
y ±2 ±1 0 ±1 0
x 4 1 0 1 4
-2 -1
Plotting the points (x , y) and connecting them with a smooth curve as shown in the
y 0 1 2
igure, we get the graph of a circle.
We plot the points (x, y) , connecting these
x2 y2 points with a smooth curve shown in igure, we
(b) The graph of ellipse of the form 2 + 2 = 1 obtain the graph of a parabola with equation
a b
y2 = x.

1.7.6 Graphs of Discontinuous Functions


2 2
Example 2: Graph x 2 + y2 = 1 i.e., 9x2 + 4y2 = 36
2 3
x when 0 ≤ x ≤ 1
Graph the function deined by y = 
 x - 1 when 1 < x ≤ 2
Solution: We observe that if we replace (x, y) in turn by (-x, y),
(x,- y) and (-x, -y) , there is no change in the given equation. Hence the
Example 1:

graph is symmetric with respect to the y-axis, x-axis and the origin.
x2 = 4 ⇒ x = ±2
Solution: The domain of the function is 0 7 x 7 2
y = 0 implies
y2 = 9 ⇒ y = ±3
For 0 7 x 7 1, the graph of the function is that of y = x
x = 0 implies and for 1 < x 7 2 , the graph of the function is that of y = x - 1
Therefore x-intercepts are 2 and -2 and y-intercepts are 3 and -3 We prepare the table for some values of x and y in 0 7 x 7 2 satisfying the equations y
By assigning values of x, we ind the values of y. So we prepare = x and y = x - 1
a table for some values of x and y satisfying equation (1). x 0 0.5 0.8 1 1.5 1.8 2
y 0 0.5 0.8 1 0.5 0.8 1
x 0 1 2 -1 -2

± ±
y ±3 27 0 27 0
4 4
Ploting the points (x, y), connecting these points with a smooth curve as shown in the
igure, we get the graph of an ellipse.

1.7.5 Graph of parametric Equations

(a) Graph the curve that has the parametric equations


x = t2 , y = t -2 7 t 7 2 (3)

version: 1.1 version: 1.1

40 41
1. Quadratic Equations eLearn.Punjab 1. Quadratic Equations eLearn.Punjab
1. Functions and Limits eLearn.Punjab 1. Functions and Limits eLearn.Punjab

Plot the points (x, y). Connecting these points we get two straight lines, which is the Scale for graphs
graph of a discontinuous function. p
x2 - 9
Along x-axis, length of side o f small square = radian
, x≠3
6
x - 3
Example 2: Graph the function deined by y = Along y-axis, length of side of small square = 0.1 unit
Two points (0, 0) and ( (p/3,1) lie on the line y = x
Solution: The domain of the function consists of all real numbers except 3.
We prepare a table for some values of x and y in the interval -p 7 x 7 p it satisfying the
0
When x = 3, both the numerator and denominator are zero, and is undeined. equation y = cos x.

x 2 - 9 ( x - 3)( x + 3)
0

= x + 3 provided x ≠ 3 . -p -5p/6 -2p/3 -p/2 -p/3 -p/6 0 p/6 p/3 p/2 2p/3 5p/6 p
x - 3 x - 3
Simplifying we get y = = x
y = cos x -1 -.87 -.5 0 -.5 .87 1 .87 .5 0 -.5 -.87 -1
We prepare a table for diferent values of x and y satisfy the equation y = x + 3 and x ≠ 3.
X -3 -2 -1 0 1 2 2.9 3 3.1 4
Y 0 1 2 3 4 5 5.9 6 6.1 7

Plot the points (x, y) and joining these points we get


the graph of the function which is a straight line except the
point (3, 6).

The graph is shown in the igure. This is a broken


straight line with a break at the point (3, 6).

1.7.7 Graphical Solution of the Equations

(i) cos x = x (ii) sin x = x (iii) tan x = x


We solve the equation cos x = x and leave the other two equations as an exercise for The graph shows that the equations y = x and y = cos x intersect at only where

p radian = 0.73
the students.
43
x=
180
Solution: To ind the solution of the equation cos x = x,
 43 
Check: cos  p  = cos 43o = 0.73
we draw the graphs of the two functions
y = x and y = cos x : -p 7 x 7 p  180 
version: 1.1 version: 1.1

42 43
1. Quadratic Equations eLearn.Punjab
1. Functions and Limits eLearn.Punjab

Note: Since the scales along the two axes are diferent so the line y = x is not equally
inclined to both the axes.

EXERCISE 1.5

1. Draw the graphs of the following equations

2 2
x2 y2
(i) x +y =9 (ii) + =1
16 4

(iii) y = e2x (iv) y = 3x


2. Graph the curves that has the parametric equations given below
(i) x = t , y = t2 , -3 7 t 7 3 where “t” is a parameter
(ii) x = t -1 , y = 2t -1, -1 < t < 5 where “t” is a parameter
(iii) x = sec q , y = tan q where “q” is a parameter
3. Draw the graphs of the functions deined below and ind whether they are continuous.

 x - 1 if x < 3
y=
2x + 1 if x ≥ 3
(i)

x2 - 4
x ≠ 2
x-2
(ii) y=

x + 3 if x ≠ 3
y=
 2
(iii)
if x = 3
x 2 - 16
x ≠ 4
x-4
(iv) y=

4. Find the graphical solution of the following equations:


(i) x = sin 2 x

= cos x
x
(ii)
2

(iii) 2 x = tan x

version: 1.1

44
version: 1.1

CHAPTER

2 DIFFERENTIATION

Animation 2.1: Increasing and Decreasing Functions


Source and credit: eLearn.Punjab
1. Quadratic Equations eLearn.Punjab 1. Quadratic Equations eLearn.Punjab
2. Diferentiation eLearn.Punjab 2. Diferentiation eLearn.Punjab

Usually the small changes in the values of the variables are taken as increments of variables.
2.1 INTRODUCTION
Note: In this Chapter we shall discuss funcions of the form y = f(x) where xdDf and is
The ancient Greeks knew the concepts of area, volume and centroids etc. which are
called an independent variable while y is called the dependent variable.
related to integral calculus. Later on, in the seventeenth century, Sir Isaac Newton, an English
mathematician (1642-1727) and Gottfried Whilhelm Leibniz, a German mathematician,
2.1.1 AVERAGE RATE OF CHANGE
(1646-1716) considered the problem of instantaneous rates of change. They reached
independently to the invention of diferential calculus. After the development of calculus,

some ixed point) after times t and t1 are given by s ( t ) and s ( t1 ) , then the distance traveled in
Suppose a particle (or an object) is moving in a straight line and its positions (from
mathematics became a powerful tool for dealing with rates of change and describing the

the time interval t1 - t where t1 > t is s ( t1 ) - s ( t )


physical universe.

s ( t1 ) - s ( t )
Dependent and Independent Variables

t1 - t
In diferential calculus, we mainly deal with the rate of change of a dependent variable and the diference quotient (i)
with respect to one or more independent variables. Now, we irst explain the terms dependent
represents the average rate of change of distance over the time interval t1 - t .
We usually write y ∈f ( x ) where f ( x ) is the value of f at x D f (the domain of the function
and independent variables.
If t1 - t is not small, then the average rate of change does not represent an accurate rate

v f ( x=
f ). Let us consider the functional relation = ) x 2 + 1 .......
of change near t. We can elaborate this idea by a moving particle in a straight line whose
(A)
For diferent values of x ∈ D f , f ( x ) or the expression x 2 + 1 assumes diferent values.
s (t =
) t2 + t
position in metres after t seconds is given by

For example; if x = 1, 1.5, 2 etc., then


f (1=
) (1) + 1= 2 , f (1.5=
) (1.5) + 1= 2.25 + 1= 3.25 We construct a table for diferent values of t as under:
2 2

f ( 2) = ( 2) + 1 = 4 + 1 = 5
s ( 5 ) - s ( 3) ( 25 + 5 ) - ( 9 + 3) 30 - 12
Interval Average rate of change (i.e. average speed)
t = 3 secs to t = 5 secs
2

5-3
the value of y or f ( x ) is given by
We see that for the change 1.5 - 1 = 0.5 in the value of x , the corresponding change in = = =9
2 2

f (1.5 ) - f (1=
) 3.25 - 2= 1.25 t = 3 secs to t = 4 secs s ( 4 ) - s ( 3) (16 + 4 ) - 12 20 - 12
It is obvious that the change in the value of the expression x + 1 (or f ( x ) ) depends 4-3
2
= = =8
1 1
t = 3 secs to t = 3.5 secs  49 7 
+  - 12
s ( 3.5 ) - s ( 3)  4 2 
upon the change in the value of the variable x . As x behaves independently, so we call it the

independent variable. But the behaviour of y or f ( x ) depends on the variable x , so we call it
15

3.5 - 3
= = 4 =7.5
the dependent variable. 0.5 0.5
The change in the value of x (positive or negative) is called the increment of x and is
denoted by the symbol d x (read as delta x ). The corresponding change in the dependent We see that none of average rates of change approximates to the actual speed of the
variable y or f ( x ) for the change d x in the value of x is denoted by d y or d f = f ( x + d x ) - f ( x ) . particle after 3 seconds.

version: 1.1 version: 1.1

2 3
1. Quadratic Equations eLearn.Punjab 1. Quadratic Equations eLearn.Punjab
2. Diferentiation eLearn.Punjab 2. Diferentiation eLearn.Punjab

f ( x1 ) - f ( x )
Now we construct a table by taking small intervals.

((3.1) )
Interval Average rate of change
+ 3.1 - 12 x1 - x
lim
x1 → x
12.71 - 12 0.71
t = 3 secs to t = 3.1 secs
2

3.1 - 3
= = =7.1

((3.01) )
0.1 0.1 provided this limit exists, is called the instantaneous rate of change of f with respect to x
+ 3.01 - 12 12.0701 - 12 0.0701 at x and is written as f ' ( x ) .
t = 3 secs to t = 3.01 secs
2

3.01 - 3 If x1 = x + d x i.e., x1 - x = d x ,then the expression (i) can be expressed as


= = =7.01

((3.001) )
0.01 0.01

t = 3 secs to t = 3.001 secs


+ 3.001 - 12 12.007001 - 12 0.007001 f ( x + d x) - f ( x)
2

3.001 - 3
dx
= = =7.001 (ii)
0.001 0.001

and

f ( x + d x) - f ( x)
The above table shows that the average rate of change after 3 seconds approximates
to 7 metre/sec. as the length of the interval becomes very very small. In other words, we can
dx
lim (iii)
say that the speed of the particle is 7 metre/sec. after 3 seconds. d x →0

If t1 = t + d t
provided the limit exists, is deined to be the derivative of f (or diferential coeicient
of f ) with respect to x at x and is denoted by f ' ( x ) (read as “f-prime of x ”). The domain of
then the diference quoteint (i) becomes

s (t + d t ) - s (t ) f ‘consists of all x for which the limit exists. If x ∈ D f and f ' ( x ) exists, then f is said to be
dt diferentiable at x . The process of inding f ‘ is called diferentiation.
which represents the average rate of change of distance over the interval d t and

s (t + d t ) - s (t )
Several notations are used for derivatives. We have used the functional symbol f ' ( x ) ,
Notation for Derivative
dt
lim
d t →0 , provided this limit exists, is called the instantaneous rate of change
of distance ‘s’ at time t . for the derivative of f at x . For the function y = f ( x ) .
y + d y = f ( x + d x) - f ( )
2.1.2 Derivative of a Function
where d y is the increment of y (change in the value of y ) corresponding to d x ,the
Let f be a real valued function continuous in the interval ( x,x1 ) ⊆ D f (the domain of change in the value of x , then
d y = f ( x + d x) - f ( x)
Dividing both the sides of (iv) by d x , we get
f ), then (iv)

f ( x1 ) - f ( x )
diference quotient
x1 - x
(i) dy f ( x + d x) - f ( x)
dx dx
= (v)

represents the average rate of change in the value of f with respect to the change x1 - x in
Taking limit of both the sides of (v) as d x → 0 , we have
the value of independent variable x .
If x1 , approaches to x , then
version: 1.1 version: 1.1

4 5
1. Quadratic Equations eLearn.Punjab 1. Quadratic Equations eLearn.Punjab
2. Diferentiation eLearn.Punjab 2. Diferentiation eLearn.Punjab

dy f ( x + d x) - f ( x)
2.2 FINDING f’(x) FROM DEFINITION
d x →0 d x dx OF DERIVATIVE
lim = lim (vi)
d x →0

dy
= f ' ( x)
d x →0 d x Given a function f , f ' ( x ) if it exists, can be found by the following four steps
dy dy
lim is denoted by , so (vi) is written as

Find f ( x + d x )
dx dx

Simplify f ( x + d x ) - f ( x )
Step I
Step II

f ( x + d x) - f ( x)
dy

Divide f ( x + d x ) - f ( x ) by d x to get
Note: The symbol is used for the derivative of y with respect to x and here it is not a
dx
dx
Step III and simplify it

f ( x + d x) - f ( x)
dy
quotient of dy and dx. is also denoted by y ’.
dx
dx
Step IV Find lim
d x →0

Now we write, in a table the notations for the derivative of y = f ( x ) used by diferent The method of inding derivatives by this process is called diferentiation by deinition
mathematicians: or by ab-initio or from irst principle.
Name of Leibniz Newton Lagrange Cauchy
Mathematician Find the derivative of the following functions by deinition
f ( x) c (b) f ( x )
Example 1:
f ( x) f ' ( x) Df ( x ) =
=
dy df (a) x2
Notation used for derivative
dx or dx
x +dx
Solution: (a) For f ( x ) = c
by a, then the expression
f ( x + d x ) - f ( x ) becomes f ( x ) - f ( a ) . and the change d x in the independent variable, in this
If we replace by x and x

case, is x - a . f ( x + d x) =
f ( x + d x) - f ( x) = c - c = 0
(i) c

f ( x + d x) - f ( x) f ( x) - f (a)
(ii)

dx x-a f ( x + d x) - f ( x) 0
So the expression is written as (vii)
= = 0
dx dx
(iii)
Taking the limit of the expressiom(vii) when x → a , gives
f ( x + d x) - f ( x)
f ( x) - f (a)
= lim = ( 0) 0
dx
= f ' ( a ) . Here f ' ( a )
(iv) lim
d x →0 d x →0

x-a
lim
x →a

is called the derivative of f at x = a . f ' ( x ) = 0 , that is, (c) = 0


d
Thus
dx
(b) For f(x) = x2
f ( x + d x ) =( x + d x )
2
(i)
f ( x + d x ) - f ( x ) = ( x + d x ) - x 2 = x 2 + 2 xd x + (d x ) - x 2
2 2
(ii)
= 2 xd x + (d x )2 =( 2 x + d x )d x
version: 1.1 version: 1.1

6 7
1. Quadratic Equations eLearn.Punjab 1. Quadratic Equations eLearn.Punjab
2. Diferentiation eLearn.Punjab 2. Diferentiation eLearn.Punjab

f ( x + d x ) - f ( x ) ( 2 x + d x )d x
in f ( x ) f (a)
or
= + ≠2 x d x ,
= (d x 0) = x a=
=
dx dx
f ( x) - f (a) = x - a
(iii) Putting x , gives a

f ( x + d x) - f ( x)
= lim ( 2 x + d x= ) 2x
So

dx
(iv) lim Using alternative form for the deinition of a derivative, we have
d x →0 d x →0

f ( x) - f (a)
f ' ( x) = 2x =
x- a
x-a x-a
( )( )
i.e.,

x at x = a from irst principle. x- a x+ a

( )
=
Example 2: Find the derivative of
( x - a) x+ a
(rationalizing the numerator)

f ( x) = x-a
(x a)
( )
=
= ≠
f ( x + d x ) = x + d x and ( x - a) x + a
Solution: If x , then 1
x+ a
(II)

f ( x + d x) - f ( x) = x + d x -
(i)
Taking limit of both the sides of (II)as x → a, gives

( )( )
(ii) x

x +dx - x +dx +  rationalizing the  f ( x) - f (a)


lim = = lim
=  numerator 
1 1
x-a x+ a a+ a
x x
x +dx +  
x →a x →a

( x + d x) - x f '( a ) =
x

=
1
i.e.,
x +dx +
2 a

dx
x

f ( x + d x) - f ( x) =
x +dx +
i.e., (I)
If y = at x= - 1 by ab-initio method.
x
Dividing both sides of(1)by d x , we have
1 dy
Example 3: 2
, then find
(iii) x dx

f ( x + d x) - f ( x) dx
== ≠ (d x 0) Solution: Here y =
1
dx d x( x + d x + x +dx +
1 , so (i)
x) x x2
Taking limit of both the sides as d x → 0 , we have
y +d y =
( x + d x)
(iv) 1

f ( x + d x) - f ( x)
(ii)
 
2

= lim  
dx  x +dx +
1
x
lim
d x →0 d x →0 Subtracting (i) from (ii), we get

=f ' ( x) = ( x > 0) 1 x - ( x + d x)
1 1
x+ x 2 x dy
= =
- 2
i.e.,
( ) x x2 ( x + d x )
2 2
1
+ d
f '( a ) =
( x + ( x + d x ))( x - ( x + d x ))
2 2
1 x x
and
=
x2 ( x + d x )
2 a
2

version: 1.1 version: 1.1

8 9
1. Quadratic Equations eLearn.Punjab 1. Quadratic Equations eLearn.Punjab
2. Diferentiation eLearn.Punjab 2. Diferentiation eLearn.Punjab

=
=
( 2 x + d x )( -d x ) -d x ( 2 x + d x )
  3
( )   

x2 ( x + d x ) x2 ( x + d x ) + d -
( x + d x)
3
2 3
(iii) 2

   - x2
2 2
x x x
=
=
3
2

Dividing both sides of (iii) by d x, , we have


( x + d x) + ( x + d x ) .x + x ( x + d x) + ( x + d x ) .x + x
4 2 2 4 4 2 2 4

d y -d x ( 2 x + d x ) -( 2x + d x)
3 3 3 3 3 3 3 3

=
= 2 ≠ (d x 0)
d x ( 2x + d x)
d x x ( x + d x )2 d x x ( x + d x)
2
f ( x + d x) - f ( x) =
( x + d x ) + ( x + d x ) .x + x 3
2
i.e., (i)
Taking limit as d x → 0 , , gives
4 2 2 4

Dividing both the sides of (i) by d x , we get


3 3 3

dy -( 2x + d x)
= lim 2
d x →0 d x
x ( x + d x) f ( x + d x) - f ( x) 2x + d x
lim
d x →0
=
2

dx
( x + d x)
+ ( x + d x ) .x + x
(ii)

- ( 2x )
4 2 2 4
3 3

Taking limit of both the sides as d x → 0 , we have


x2 ( x2 )
3 3

= (Using quotient theorem of limits)

dy -2 -2 -2
= = |x =-1 = 3= f ' (=
x) = =
( -1) -1
dy 2x 2x 2
i.e., and 2
x + x .x + x
4 2 4 2 4 1
dx x3 dx 3 3 3 3
3x 3 3x 3

=
f '(8) =
2 1
at x = -1 is written as
dy dy and 1
Note: The value of | . 3
dx dx x=-1 3.(8) 3

Example 5: Find the derivative of x3 + 2 x + 3 .


2

Solution: Let y = x3 + 2 x + 3. Then


Example 4: Find the derivative of x and also calculate the value of derivative at x = 8.
3

y + d y =( x + d x ) + 2 ( x + d x ) + 3
Solution: Let f ( x ) = x .Then
3
(i)

d y = ( x + d x ) + 2 ( x + d x ) + 3 -  x3 + 2 x + 3
2
3

 
f ( x + d x ) =( x + d x )
3
(ii)

= ( x + d x ) - x3  + 2 ( x + d x ) - x  + ( 3 - 3)
2
3

 
3
and
= ( x + d x ) - x  ( x + d x ) + ( x + d x ) x + x 2  + 2d x
 
2

  
 ( x + d x ) - x  ( x + d x ) + ( x + d x ) . x + x 
d y d x ( x + d x ) + ( x + d x ) x + x  + 2d x
 2
2 2 4 2 2 4

f ( x + d x ) - f ( x) = ( x + d x ) - x3 =   
3 3 3 3 3 3

=
2
2 2

( x + d x ) 3 + ( x + d x ) 3 .x 3 + x 3 dx dx
3
4 2 2 4 (iii)

version: 1.1 version: 1.1

10 11
1. Quadratic Equations eLearn.Punjab 1. Quadratic Equations eLearn.Punjab
2. Diferentiation eLearn.Punjab 2. Diferentiation eLearn.Punjab

= ( x + d x ) + ( x + d x ) x + x2 + 2
( x ) = nx n-1 reduces to ( x ) 0=
2

= x 0-1 0 i.e.,
d n d 0
dy
lim = lim ( x + d x ) + ( x + d x) x + x 2 + 2 
Note: If n = 0, then the formula
dx dx
d x →0 d x d x →0  
2

(1) = 0 which is correct by example 1 part (a).


(iv) d

= (x) 2 + ( x ) x + x 2 + 2
dy dx

( x + 2 x + 3) = 3 x 2 + 2
dx

Let y = x n where n is a negative integer.


d 3
i.e., (b)
dx

Let n = -m (m is a positive integer). Then


2.2.1 Derivation of xn where ndZ.
=
y x=
-m 1
(i)
(a) We ind the derivative of x when n is positive integer.
n xm
y = x n . Then y +d y =
( x + d x)
(a) Let 1

y + d y =( x + d x )
and m
(ii)
n

dy= ( x + d x ) - xn Subtracting (i) from (ii). gives


n
and

1 x - ( x + d x)
Using the binomial theorem, we have
dy
= =
-
( x + d x ) xm xm ( x + d x )
m

x ( (d x 2+) ...
+ (d x) n )- x n
m
1
 n(n - 1) n-2 
d y =  x n + nx n-1.d x +
m m

   m ( m - 1) m-2 m
x m -  x m + mx m-1d x + x (d x ) + ... + (d x ) 
2

 
2

=
  x ( x + d x)
2
n(n - 1) n-2
d y d x  nx n-1 +
i.e., = x+ .+d x ... (d x) n-1 
m m

 
(expanding ( x + d x ) by binomial theorem)
(i)
2 m

Dividing both sides of (i) by d x , gives


 m ( m - 1) m-2 m -1 
dy n( n - 1 ) n-2 -d x  mx m-1 + x d x + ... + (d x ) 
= nx n-1 + x . d+ x +... ( d x )n-1  
dx =
x .( x + d x )
(ii) 2
2

Note that each term on the right hand side of (ii) involves d x except the irst term, so
m m

dy -1  m-1 m ( m - 1) m-2 
= +  mx + + x .d x ... (d x ) 
m -1

taking the limit as d x → 0 , we get


d x xm ( x + d x )  
and
= nx n-1
m
.
dy 2

Taking limit when d x → 0 , we get


( x ) n.x n-1
dx
=
=
= m m ( mx m-1 )
d n
-1
As y x , so n

(all terms containing d x ,vanish)


dx dy
dx x .x
version: 1.1 version: 1.1

12 13
1. Quadratic Equations eLearn.Punjab 1. Quadratic Equations eLearn.Punjab
2. Diferentiation eLearn.Punjab 2. Diferentiation eLearn.Punjab

=( -m ) x m-1.x -2 m =( -m ) x(
- m )-1
=nx n-1 [ ]
m- n= 2.2.2 DIFFERENTIATION OF EXPRESSIONS OF THE TYPES:

( x ) = nx n-1 ( ax + b ) n=
( ax + b )
d n
1
or n
and , 1,2,3...
dx

[ x ] = nx n-1 , if n ∈ Z
n

We ind the derivatives of ( ax + b ) and


d
from the irst principle when n ∈ N
n

( ax + b )
So far we have proved that n 1
dx n

The above rule holds if n ∈ Q - Z


Find from deinition the diferential coeicient of ( ax + b ) w.r.t. ‘ x ‘ when n
n
Example 1:
d  23  2 23 -1
=
x  =
2 is a positive integer.
For example
dx   3
x 1
3
3x Solution: Let y = (ax + b)n, (n is a positive integer)
d n
 x  = nx n-1 when n ∈ Q - Z is left as an exercise. Then y + d y=  a ( x + d x ) + b  = ( ax + b ) + ad x 
n n
The proof of
dx Using the binomial theorem we have

n n
 x  = nx n-1 is called power rule. ( ax +b )
y +d y =
+ ( ax b ) ( ad+x )   +( ax b ) ( ad +x ) + ...
1  + ( ad x )
d n n -1 n-2

  2
n 2 n
Note that
dx
n n
dy= ( y + d y) - y = 1  ( ax + b ) ( ad x ) +  2  ( ax + b ) .a (d x ) + ... + a (d x )
n -1 n-2

   
2 2 n n

Exercise 2.1
 n  n n -1 
= d x   ( ax + b ) .a +   ( ax + b ) .a 2d x + ... + a n (d x ) 
n -1 n-2

 1  2 
1. Find by deinition, the derivatives w.r.t ‘x’ of the following functions deined as:

2 x2 + 1 2- x
d y n n
So =   ( ax + b ) a +   ( ax + b ) .a 2d x + ... + a n (d x )
1 1 1
x-a
(i) (ii) (iii) (iv) (v)
n -1 n-2 n -1

d x 1 
x x3
2
x ( x - 3) (viii) ( x + 4 )
3 5

Taking limit when d x → 0 , we have


1
2
(ix) (x)
2 2
(vi) (vii) 3 x x
x4
xm , m ∈ N -100
dy  n  n n -1 
lim   ( ax + b ) .a +   ( ax + b ) .a 2d x + ... + a n (d x ) 
1
x ,m∈ N
40
(xi) (xii) (xiii) x (xiv) x
=
n -1 n-2

d x →0 d x
m

 1  2 
lim
d x →0

dy  n 
=   ( ax + b ) .a [All other terms tends to zero when d x → 0 ]
dy n -1
from irst principle if
dx 1 
2. Find Or
dx

( ax + b ) = n ( ax + b ) .a
d n n -1
Thus
x+2
1 dx
x+a
(i) (ii)
version: 1.1 version: 1.1

14 15
1. Quadratic Equations eLearn.Punjab 1. Quadratic Equations eLearn.Punjab
2. Diferentiation eLearn.Punjab 2. Diferentiation eLearn.Punjab

n
+   ( ax + b ) .a 2d x + ... + a n (d x ) ]
n-2 n -1

( ax + b )
1
Find from irst principle, the derivative of
2
Example 2: n
w.r.t. ‘ x ’,

dy n
= ( ax- b ) .a +
n -1
n  
( ax + b ) + ad x  ( ax + b )  
dx
1
Solution: Let y =
and [

( ax + b )
1 n
1
(when n is a positive integer). Then
n
+   ( ax + b ) .a 2d x + ... + a n (d x ) ]
n

n-2 n -1

y +d y = 2
 a ( x + d x ) + b 
1
and
Using the product and sum rules of limits when d x → 0 , we have
n

d y = y +d y - y = -
( ax + b ) + ad x  ( ax + b )
1 1
n n

dy=
( ax + b ) - ( ax + b + ad x )  d y dy
=

 d x→0 d x dx 
n n

( ax + b ) + ad x  ( ax + b )

 
lim and
n
n  (
- b ) .a
or
=  all other +terms containing 
n
n -1
n

( ax + b ) ( ax + b ) 1 
dy 1
-1
+ n x ( ax b ) ad x  ( ax b) ]  d x vanish 
. ax
dy + + -n
n

( ax + b ) + ad x  ( ax + b )
dx
 
n n
or (I)
 

d   -na
n ( ax b )
Using the binomial theorem, we simplify the expression
-( n +1)
=  n
=
- + n+1 =
( ax + b ) + ad x  - ( ax + b ) ,That is, dx  ( ax + b )  ( ax + b )
1
n n or .a

n
( ax + b ) + ad x  - ( ax + b ) = [( ax + b ) +   ( ax + b ) ( ad x )
n n -1

1 
n n
Exercise 2.2

n
+   ( ax + b ) .a 2 (d x ) + ... + ( ad x ) ]
n-2
1. Find from irst principles, the derivatives of the following expressions w.r.t. their
2
2 n
respective independent variables:

n n ( ax + b ) ( 2 x + 3)
=   ( ax + b ) .ad x +   ( ax + b ) .a 2 (d x ) + ... + a n (d x )
3 5
n -1 n-2
(i) (ii)

1  2
2 n

( 3t + 2 )
-2
 n  n n -1 
= d x   ( ax + b ) .a +   ( ax + b ) a 2d x + ... + a n (d x )  ( ax + b )
1
(iii) (iv)
n -1 n-2
5

 1  2 
( az - b )
1
(v) 7
Now (I) becomes

dx n
dy= ( - ) +
n -1
 
( ax + b ) + ad x  ( ax + b )  
n n
[ ax b .a
1

version: 1.1 version: 1.1

16 17
1. Quadratic Equations eLearn.Punjab 1. Quadratic Equations eLearn.Punjab
2. Diferentiation eLearn.Punjab 2. Diferentiation eLearn.Punjab

d  43 
2.3 THEOREMS ON DIFFERENTIATION
 3x 
dx  
Example 1: Calculate
We have, so far proved the following two formulas:
d  43  d  43 
 3x  = 3  x 
( c ) = 0 i.e.. the derivative of a constant function is zero. dx   dx  
dy Solution: (Using Formula 3)
1.

( x ) = nx n-1
dx
= 3x =
4 43 -1 1
d n x 4x3 (Using power rule)
2. power formula (or rule) when n is any rational 3
dx 4. Derivative of a sum or a Diference of Functions:
If f and g are diferentiable at x , then f + g, f - g are also diferentiable at x
number.

 f ( x ) + g ( x )  = f ' ( x ) + g ' ( x ) ,
Now we will prove other important formulas (or rules) which are used to determine
 f ( x ) + g ( x=
)  f ( x )  +  g ( x ) 
derivatives of diferent functions eiciently. Henceforth, in all subsequent discussion, f, g, h and
'
that is,
d d d
Also
etc. all denote functions diferentiable at x, unless stated otherwise.
Derivative of y = cf ( x )  f ( x ) - g ( x )  = f ' ( x ) - g ' ( x ) . that is, d  f ( x ) - g ( x=
dx dx dx
)  f ( x )  -  g ( x ) 
' d d

Let y = cf ( x ) . Then
3.

( x ) f ( x ) + g ( x ) . Then
dx dx dx

Let f=
y + d y= cf ( x + d x ) and
Proof:

f ( x + d x ) = f ( x + d x ) + g ( x + d x ) and
Proof:

y cf ( x + d x ) - cf ( x )
(i)
y +d y -=
(i)
f ( x + d x ) - f ( x ) = f ( x + d x ) + g ( x + d x ) -  f ( x ) + g ( x ) 
d y= c | f ( x + d x ) - f ( x ) |
(ii)
(ii)
=  f ( x + d x ) - f ( x )  +  g ( x + d x ) - g ( x )  (rearranging the terms)
or (factoring out c)

dy  f ( x + d x) - f ( x) 
= c  f ( x + d x) - f ( x) f ( x + d x) - f ( x) g ( x + d x) - g ( x)
dx  dx  = +
(iii)
dx dx dx
(iii)
Taking limit when d x → 0 Taking the limit when d x → 0
f ( x + d x) - f ( x)  f ( x + d x) - f ( x) g ( x + d x) - g ( x) 
dy  f ( x + d x) - f ( x)  f ( x + d x) - f ( x) (iv) = +lim  
d x →0 d x
lim c.  dx  dx dx 
dx dx
lim
d x →0 d x →0
 
f ( x + d x) - f ( x) g ( x + d x) - g ( x)
(iv) lim c. lim
d x →0 d x →0

= +lim
A constant factor can be taken out from a limit sign. dx dx
lim
d x →0 d x →0

= c f ' ( x ) ,that is, c f ( x )  = cf ' ( x )


(The limit of a sum is the sum of the limits)

f ' x f ' ( x ) + g ' ( x ) , that is  f ( x ) + g ( x )  = f ' ( x ) + g ' ( x )


dy '

=
Thus '
dx

= cf ' ( x ) c f ( x )  = cf ' ( x )  f ( x ) + g ( x=


)  f ( x )  +  g ( x ) 
dy ' d d d
or = or
dx dx dx dx

The proof for the second part is similar.

version: 1.1 version: 1.1

18 19
1. Quadratic Equations eLearn.Punjab 1. Quadratic Equations eLearn.Punjab
2. Diferentiation eLearn.Punjab 2. Diferentiation eLearn.Punjab

( )(
Find the derivative of y = 2 x + 2 x - x with respect to x. )
Note: Sum or diference formula can be extended to ind derivative of more than two
functions. Example 3:

(
Solution: y = 2 x + 2 x - x )( )
( ) ( ) ( )( )
Find the derivative of y = x + x + x + 2 x + 5 w.r.t. x .
3 4 2 3 1 2
x +1 . x x=
-1 2 x x +1 x -1
Example 1:
4 3 2 =2

 32 
Solution: y = x + x + x + 2x + 5 = 2 x ( x + 1=
) 2 x - x 2 
1
3 4 2 3 1 2
4 3 2  
Diferentiating with respect to x, we have Diferentiating with respect to x , we have

x  +  x  +  x  + ( 2 x ) + ( 5)
dy  3 4 2 3 1 2  d 3 4  d 2 3 d 1 2  d
 x + x + x + 2 x=
+ 5  dy d   32 
= 2  x - x 2 
d
dx  4  dx  4  dx  3  dx  2  dx
1

dx dx   
3 2 dx

 d  32  d  12    3 23 - 1 1 12 - 1 
(Using formula 4)

( x )+ ( x )+ ( x ) + 2 ( x) + 0
= 2   x  -  x  = 2  x - x 
       
3 d 4 2 d 3 1 d 2 d
= (Using formula 3 and 1) dx dx 2 2
1 3x - 1
= ( 4 x 4-1 ) + ( 3 x3-1 ) + ( 2 x 2-1 ) + 2 (1.x1-1 )
-1
4 dx 3 dx 2 dx dx
= 3x 2 - x 2 = 3 x - =
1
3 2 1
(By power formula) x x
4 3 2

= 3x3 + 2 x 2 + x + 2
5. Derivative of a product. (The product Rule)

( x + 5)( x + 7 ) with respect to x.


If f and g are diferentiable at x, then fg is also diferentiable at x and
Find the derivative of y =
 f (=
x ) g ( x )  f ' ( x ) g ( x ) + f ( x ) g ' ( x ) , that is,
2 3
Example 2: '

Solution: y =( x2 + 5)( x3 + 7 ) =x5 + 5 x3 + 7 x 2 + 35 =  f ( x ) g ( x )  +  f ( x )   g ( x )


d 
f ( x )   g ( x )  
d 
dx 
d
Diferentiating with respect to x, we get  dx   dx 

f ( x ) = f ( x ) g ( x ) . Then
=  x + 5 x3 + 7 x 2 + 35

f ( x + d x) = f ( x + d x) g ( x + d x)
dy d 5 Proof: Let

=  x5  + 5 ( x3 ) + 7 ( x 2 ) + [35] (Using formulas 3 and 4)


dx dx

f ( x + d x) - f ( x) = f ( x + d x) g ( x + d x) - f ( x) g ( x)
(i)
d d d d

Subtracting and adding f ( x ) g ( x + d x ) in step (ii), gives


(ii)
dx dx dx dx

f ( x + d x) - f ( x) = f ( x + d x) g ( x + d x) - f ( x) g ( x + d x) + f ( x) g ( x + d x) - f ( x) g ( x)
= 5x + 5 x 3x + 7 x 2x
5-1 3-1 2-1
+0

=  f ( x + d x ) - f ( x )  g ( x + d x ) + f ( x )  g ( x + d x ) - g ( x ) 
= 5x4 + 15x2 + 14x

version: 1.1 version: 1.1

20 21
1. Quadratic Equations eLearn.Punjab 1. Quadratic Equations eLearn.Punjab
2. Diferentiation eLearn.Punjab 2. Diferentiation eLearn.Punjab

( ) ( )
f ( x + d x) - f ( x)  f ( x + d x) - f ( x)   g ( x + d x) - g ( x)   1  1 
(iii) =  + +  g ( x d x) f ( x)   = 2 x- x + x + 1 x 1 - 
dx  dx   dx    2 x 

( )  2 2 x x- 1 
2 x
Taking limit when d x → 0 x- x  
= 2 + x +1
f ( x + d x) - f ( x)  2 x  
1 
dx x - x + 2x - x + 2 x - 1
x
(iv) lim
d x →0

 f ( x + d x) - f ( x) g ( x + d x) - g ( x) 
=

= lim  .g ( x + d x ) + f ( x ) . 
3x - 1
d x →0
 dx dx 
=
f ( x + d x) - f ( x) g ( x + d x) - g ( x)
. lim g ( x + d x ) + lim f ( x ) . lim
x

dx dx
= lim 6. Derivative of a Quotient (The Quotient Rule)
d x →0 d x →0 d x →0 d x →0

If f and g are diferentiable at x and g ( x ) ≠ 0 , for any x ∈ D ( g ) then


(Using limit theorems) f
is diferentiable
= f '( x ) f '( x ) g ( x ) + f ( x ) g '( x )  lim g ( x + d x ) =
g ( x )
g
 d x →0 
 f ( x)  f '( x ) g ( x ) - f ( x ) g '( x )
Thus

 =
'
at x and 
 f ( x ) .g ( x )  +  f ( x )  .g ( x ) f ( x )  g ( x )  g ( x) 
d 
=  g ( x ) 
d d
 dx 
2
or
d
( ) 
( ) ( ) d
 g ( x )  

dx dx
  -
d f ( x)
( )( )
      
 dx   dx 
=
f x g x f x

Example: Find derivative of y = 2 x + 2 x - x with respect to x dx  g ( x )   g ( x ) 
that is, 2

(
Solution: y = 2 x + 2 x - x )( ) Proof: Let f ( x ) =
f ( x)
g ( x)
2 ( x + 1)( x - x )
Then
=
f ( x + d x)
f ( x + d x) =
g ( x + d x)
Diferentiating with respect to x, we get (i)

( )( )
f ( x + d x) f ( x) f ( x + d x) g ( x) - f ( x) g ( x + d x)
= 2  x +1 x - x  f ( x + d x ) -=
f ( x) -=
dx   g ( x + d x) g ( x) g ( x) g ( x + d x)
dy d
(ii)

( )( ) ( ) dxd ( x - x )
dx
 d
= 2 

x +1  x - x + x +1 Subtracting and adding f ( x ) g ( x ) in the numerator of step (ii), gives
 dx 
f ( x + d x) g ( x) - f ( x) g ( x) - f ( x) g ( x + d x) + f ( x) g ( x)
( ) ( ) f ( x + d x) - f ( x) =
 1 12 - 1   1 12 - 1  
= 2  x + 0 x - x + x + 1 × 1 - x  g ( x) g ( x + d x)
 2   2 
( f ( x + d x ) - f ( x ) ) g ( x ) - f ( x ) ( g ( x + d x ) - g ( x ) ) 
g ( x) g ( x + d x) 
1
=

version: 1.1 version: 1.1

22 23
1. Quadratic Equations eLearn.Punjab 1. Quadratic Equations eLearn.Punjab
2. Diferentiation eLearn.Punjab 2. Diferentiation eLearn.Punjab

f ( x + d x) - f ( x)  f ( x + d x) - f ( x) g ( x + d x) - g ( x) 
.g ( x ) f ( x ). Using the product rule to f ( x ) .
g ( x)
-
g ( x) g ( x + d x) 
 
1
dx dx dx
(iii) 1

, we have

Taking limit when d x → 0 d  1  d d  1 


 f ( x ).  +  f ( x )   . f ( x ).
 1
=  
g ( x )   dx  g ( x) dx  g ( x ) 
f ( x + d x) - f ( x)
dx 

dx  f ( x )  -  g ( x ) 
(iv) lim
d x →0

+ f ( x ) dx
d d

g ( x)  g ( x ) 
= dx
  f ( x + d x) - f ( x) g ( x + d x ) - g ( x ) 
( ) ( )
2

lim  - 
x →0 g ( x ) g ( x + d x )

 f ( x )   g ( x ) - f ( x )   g ( x )  
d d d  d 
1

d  f ( x )   dx
    
. g x f x .
  dx 
x x
 =
dx  g ( x )   g ( x ) 
i.e.,

( lim g ( x )
2
Using limit theorems, we have

f '( x )  f ' ( x ) g (=
x ) f ( x ) g +' ( x )  d x) g ( x)
( )
-
g ( x ) .g ( x )   32 
1
x + 1  x - 1
d x →0

if y =  ,
( x ≠ 1)
dy
Example 2: Find
x2 -1
 dx  f ( x )   g ( x ) - f ( x )  dx  g ( x )  
d  d 
1
dx
 f ( x)  f '( x ) g ( x ) - f ( x ) g '( x ) d  f ( x) 
'
   
 g ( x)   g ( x )  dx  g ( x )   g ( x ) 
Thus 2
or 2 Solution: Given that

( )
 23 
x + 1  x - 1 ( x +1  )( ) - (1) 
First Alternative Proof:
 
f ( x)  
3 3

=
=
f ( x) = can be written as f ( x ) = f ( x ) g ( x )
x

x -1
g ( x)
y
x2 -1
( )( )( )
1

( )( )
x +1 x -1 x +1+ x
Using the procedure used to prove product rule, quotient rule can be proved. = x +1 x +1+ x
x -1
( )( )( ) ( x + 1) + ( )
=
Second Alternative Proof: We irst prove the reciprocal rule and then use product rule to
x +1 x -1 x +1+ x= x +1 x
2
prove the quotient rule. =

= x + 1 + 2 x + x x + x= x + 2x + 2x + 1
The reciprocal rule. If g is diferentiable at x and g ( x ) ≠ 0 , then
3 1
2 2
1
is diferentiable at x and
dy d  32  d  32  d d  12  d
 x  + ( 2 x ) +  2 x  + (1)
g
=  x + 2 x + 2 x += 1
1

dx dx     dx dx   dx
2

 g ( x ) 
dx
-
d  1  dx  = x + 2 (1) + 2. += x +2+
d
=
3 12

dx  g ( x ) 
1 3 1

 g ( x ) 
0
2
(Proof of reciprocal rule is left as an exercise) 2 2 x 2 x

version: 1.1 version: 1.1

24 25
1. Quadratic Equations eLearn.Punjab 1. Quadratic Equations eLearn.Punjab
2. Diferentiation eLearn.Punjab 2. Diferentiation eLearn.Punjab

( )  32   2 x +1 x + x +1
x + 1  x - 1 x -
  with respect to x.  2 x  2x + x - x - x - 1 x -1
=
=
=
2 x
Example 3: Diferentiate
x2 - x2
3 1 3
x x.2 x 2
2x

( )
 32  2 x3 - 3x 2 + 5
x + 1  x - 1 x2 + 1
Example 4: Diferentiate with respect to x .

Solution: Let y =  
2 x3 - 3x 2 + 5
Solution: Let f ( x ) =
x2 - x2 x2 + 1
( )
3 1

f ( x ) = 2 x3 - 3 x 2 + 5 and g ( x=) x2 + 1
. Then we take
 32 
x + 1  x - 1
 2 x3 - 3 x 2 + 5= 2 ( 3 x 2 ) - 3 ( 2 x ) + 0= 6 x 2 - 6 x
 
x ( x - 1) f ' ( x )=
= d

( )( )( ) ( x - 1) ( x + )
Now
dx
x +1 x -1 x + x +1 x +1 g' ( x ) =  x + 1 = 2 x + 0 = 2x

( ) ( )
dx 
and d 2

x -1 x -1
f '( x ) g ( x ) - f ( x ) g '( x )
Using the quotient formula: f ' ( x ) =
x x
x + x +1  g ( x ) 
,we obtain

d  2 x 3 - 3 x 2 + 5  ( 6 x - 6 x )( x + 1) - ( 2 x + 3 x + 5 ) ( 2 x )
2
=
x

( x 2 + 1)
=
dx  x 2 + 1 
2 2 3 2
Diferentiating with respect to x , we have

6 x 4 - 6 x3 + 6 x 2 - 6 x - ( 4 x 4 - 6 x3 + 10 x )
2

dy d  x + x + 1 
=  
(x + 1)
dx dx   =

( ) ( ) ( x)
x 2
2

x + x +1 - x + x +1
6 x 4 - 6 x3 + 6 x 2 - 6 x - 4 x 4 + 6 x3 - 10 x
d d

( ) (x + 1)
=
x
= dx dx
2 2
2

( )
x

 1 - 12   1 - 12  2 x 4 + 6 x 2 - 16 x
(x + 1)
x 1 + x + 0  - x + x + 1 . x  =
=  2  2 
2 2

( )
x
 1 
x 1 +  - x + x +1
1
 2 x
EXERCISE 2.3
=
2 x
x Diferentiate w.r.t. x

a+x
x 4 + 2 x3 + x 2 x -3 + 2 x -3/2 + 3
a-x
1. 2. 3.

version: 1.1 version: 1.1

26 27
1. Quadratic Equations eLearn.Punjab 1. Quadratic Equations eLearn.Punjab
2. Diferentiation eLearn.Punjab 2. Diferentiation eLearn.Punjab

Diferentiating (ii) and (iii) w.r.t x and u respectively, we have.


2x - 3  1 
( x - 5)( 3 - x )  -   g ( x )  g ' ( x )
2

2x + 1 = =
 x dx dx 
4. 5. 6. x du d

( 
) 
1+ x  x - x2 
(x + 1)
= =  f ( u )  f ' u
du du 
3
dy d
  x2 + 1
2 2 and

x2 - 1 x2 - 3
7. 8. 9.
x Thus (i) can be written in the following forms
1+ x 2x - 1 a-x
1- x x +1 a+x ( f (u )) = f '(u )
10. 11. 12. d du
2 (a)

x2 + 1 1+ x - 1- x x a+x
dx dx

=
1+ x + 1- x a-x
dy dy du
x2 - 1
13. 14. 15. (b) .
dx du dx
If = x- , show that 2 x + y =
1 dy The proof of the Chain rule is beyond the scope of this book.
16. y 2 x
x dx
If y =x 4 + 2 x 2 + 2 , prove that= 4 x y - 1 = 1. Let y  g ( x )  and u
= g ( x)
dy
17. Note:
n
dx

=Then y u= nu n-1
n dy
and (power rule)
2.4 THE CHAIN RULE du

= = nu n-1
dy dy du du
But .

( )
The composition fog of functions f and g is the function whose values f [g(x)], are found dx du dx dx

for each x in the domain of g for which g(x) is in the domain of f . f  g ( x )  is read as f of g d
 g ( x )  = n  g ( x )  .g' ( x )
n -1
 = g' ( x ) 
 du 
 dx 
n
or
of x). dx

2. Reciprocal rule can be written as


Theorem. If g is diferentiable at the point x and f is diferentiable at the point g( x ) then
the composition function fog is diferentiable at the point x and ( fog ) ' ( x ) = f '  g ( x )  .g ' ( x ) . d  1  d
 =  g ( x )  = - 1.  g ( x )  .g' ( x )
-1 -1-1

dx  g ( x )  dx
(=
fog )( x ) f  g ( x )  , then
The proof of the chain rule is beyond the scope of this book.
= = ( -1)  g ( x ) .g' ( x )
-2
If y

( fog ) ' ( x )  f  g ( x )  '


=
=
dy

Find the derivative of ( x3 + 1) with respect to


dx
⇒ f '  g ( x )  .g ' ( x )
=
9
dy Example 1:
(i)

u = g ( x)
dx

Then y = f ( u )
( x3 1)+ =
Let y+= and u x=
Let (ii) 93
Solution: 1 Then y u 9
(iii)
version: 1.1 version: 1.1

28 29
1. Quadratic Equations eLearn.Punjab 1. Quadratic Equations eLearn.Punjab
2. Diferentiation eLearn.Punjab 2. Diferentiation eLearn.Punjab

a+x + a-x
=
= if y = ( x ≠ 0)
du dy dy
a+x - a-x
Now 3 x 2 and 9u 8 (Power formula) Example 3: Find
dx du dx

= 9u 8
a+x + a-x
dy du
y=
Using the formula , we have

( x + 1) =9 ( x3 + 1) ( 3 x 2 )  u =x3 +1 and
a+x - a-x
dx dx
 
Solution:
=3 x 2 
d 3 du
 
9 8

= 27 x ( x + 1)
or
a + x - a - x , gives
dx dx
Multiplying the numerator and the denominator by

( )( )
2 3 8

a+x + a-x a+x - a-x

( a - x )( a - x)
a-x
,( x ≠ -a ) with respect to x y=
a+x a+x - a+x -
Example 2: Diferentiate

a-x
y= =
=
a-x
( a+x ) (
- a-x )
(a + x) - (a - x)
( )
1

a+x a+x
2 2

= =
Solution: Let and u . Then y u 2

( a + x ) + ( a - x ) - 2 a 2 - x 2 2a - 2 a 2 - x 2
2x
2 a - a2 - x2
=
= =
dy 1 12 - 1 1 - 12
Now u u
du 2 2
d
du d  a - x   dx
( - ) 
 ( + ) - ( - ) d
 dx ( + ) 

that is, y =
x
a - a2 - x2
= =
a x a x a x a x

dx dx  a + x  (a + x) Let f ( x ) =x and g ( x ) =- a 2 - x 2 , then


and 2

(=
0 - 1)( a + x ) - ( a - x )( 0 + 1)
0- - (=
a x- ) ( a -x ) ( x2 )
a
-a - x - a + x -2a
= = 2 f ( x)' =
1 and -g' ( x ) = 2 2 -1 d

(a + x) (a + x) (a + x)
1 1
d 2 2 2 1 2
2 2 a2
dx 2 dx

-= -x ( x)
2=
1 x
= . , we have 2 a2 - x2 a2 - x2
dy dy du

dy f ' ( x ) g ( x ) - f ( x ) g ' ( x )
Using the formula
dx du dx
=
d  a - x  1 - 12  -2a  1  a - x  -2a  a-x  g ( x ) 
Using the formula , we have
-

= u  2
=  × u =
1 2

( )
  2  
dx
dx  a + x  2  ( )  2  a + x  (a + x)  a+x
2

+
1. a - a 2 - x 2 - x.
a x

( )
x

( a - x ) × -a = -a = a2 - x2
- dy
=
1

(a + x) ( ) (a - x) (a + x)2
a - a2 - x2
2

+
2
dx
-
1 2 1 3

a a2 - x2 - ( a2 - x2 ) - x2
2 a x 2

( ) ( )
a a2 - x2 - a2
=
a -x a- a -x a2 - x2 a - a2 - x2
2
= 2
2 2 2 2

version: 1.1 version: 1.1

30 31
1. Quadratic Equations eLearn.Punjab 1. Quadratic Equations eLearn.Punjab
2. Diferentiation eLearn.Punjab 2. Diferentiation eLearn.Punjab

(
-a a - a 2 - x 2 ) y =( ax + b ) ( ax + b ) =

(a - ) (a - )
-m
-a
( ax + b )
1
=
n
= m
(i)

a -x a -x a -x a -x
2
= 2
2 2 2 2 2 2 2 2

( ax + b ) . Let+u =
d m

(1 + 2 x ) .x
We first find ax b. Then

( ) ( u ) dx
dx
if y= ( ax + b =
) u=
3
dy 3
d m d m d m du
Example 4: Find 2
(using chain rule)
dx

( ) ( )
dx dx dx

= +mu m-1 x a=m ( ax b )  ( ax + b ) =


  12    d 
a
m -1
Solution: + y =
1 +2 =
x .x  1 2 x  x  
3

 dx 
3 3
.a
  
2

(1 + 2 x ).x
Now diferentiating (i) w.r.t.’ x ’, we have

u=  (i )
1


(1) .( ax + b ) - 1. ( ax + b )
2
Let
Then y = u 3 ( ii )  dy d  
d m d m

=
=  
Differentiating (ii) with respect to u , we have  dx dx  ( ax + b )m  ( ax + b )m 
1 dx dx

 
2


(

) ( ) 0.( ax + b ) - 1.m ( ax + b ) .a
m -1
+== 3u 2 3+ =1 2 x x 2  3 1 2 x .x
2

=
1

( ax + b )
dy 2 m

 

( b ) .a ) x+ ( ax- b=
dx 2m

- =+ ( ax ) + m ( ax b )
Diferentiating (i) with respect to x , gives m -1 -2 m m -1- 2 m

( )
m .a
du  1  12 =
(-m ) ( ax + b )- m-1 . a+=
n( ax b )n-1 .a =
=  0 + 2.  x + 1+ 2 x
1 ( -m n)
dx  2 x 2 x
1+ 2 x 2 x +1+ 2 x 1+ 4 x
+= 1 = =
,q≠0
dy p
2 x 2 x 2 x Example 6: if y = x n where n =
Find
dx q

Solution: Given that y = ≠n = ,q =


dy dy du
Using the formula = . ,we have p p
x n where 0. putting n ,we have

( ) ( )
dx du dx

d   1+ 4 x 
q q

 + + = x 
3

y =x
3 2 p

dx    2 x 
2
1 2 x . x 3 1 2 x .x

( ) ( )
q
(i)
Taking qth power of both sides of (i), we get
+= 1 2 x+ x 1 4 x
3 2

(1 2 x+) ( x )
2 yq = x p (ii)

+=
Diferentiating both sides of (ii) w.r.t. ‘ x ‘ , gives

( x ) (Using chain rule)


4x
d q d p d q dy d p
dy (y ) = (x ) or (y ) . =
Example 5: If y = (ax + b) where n is a negative integer, ind
n
using quotient theorem dx dx dy dx dx
⇒ q y q -1
dx
= px p -1
Solution: Let n = -m where m is a positive integer. Then
dy
(iii)
dx
version: 1.1 version: 1.1

32 33
1. Quadratic Equations eLearn.Punjab 1. Quadratic Equations eLearn.Punjab
2. Diferentiation eLearn.Punjab 2. Diferentiation eLearn.Punjab

Multiplying both sides of (iii) by y, we have Now we explain the method of inding derivatives of functions given in the form of
parametric equations by the following examples.
= py x p -1 or = p . x x p -1
dy dy
q . yq q. x p (using (i) and (ii))
dx dx

⇒ × xq
dy
. p . x q x p -1 =
p p
dy p 1 p +p-1-p Example 1: Find if x = at 2 and y = 2at.
= dx
dx q x q
 p 
x = nx n-1  q = 
p
p q -1 dy
 
=  n Solution: We use the chain rule to find
q dx

(x ) n x n-1 .
d n dy d
Thus Here = (2at ) = 2a.1=2a
dx dt dt
dx d
and = (at 2 ) = a (2t ) = 2at
dt dt
2.5 DERIVATIVES OF INVERSE FUNCTIONS

( 2a = y )
dy
If for each x d Df , f(x) = y and for each y d Dg, g(x) = x, then f and g are inverse of each dy dy dt 2a 2a
so = . = dt = =
other, that is, dx dt dx dx 2at y
( g o f= )( x ) g( = =
f ( x )) g( y) x (i) dt
and ( f o= f (g= f=  y 
Eliminating t, we get x = a   = = ⇒ y2 =
2
g )(y ) (y )) (x) y (ii) y2 y2
 2a 
a. 4ax (i)
Using chain rule, we can prove that 4a 2 4a
f '( x ). g'( y ) = 1 Diferentiating both sides of (i) w.r.t. ‘ x ’ we have

⇒ (y ) =
1 d 2 d
f ' (x) = (4ax)
g' (y ) dx dx

 dy  ⇒ 2y
d 2 dy d dy
 ⇒
dx 
(y ) . = 4a (x) = 4a (1)
 dx dx dx dx
 
f (x ) = y f ' (x ) =


dy 1
 and g (y ) = x ⇒ g'(y ) = dx 
= dy 2a
 dy 
dx dx =

dx y
dy

2.6 DERIVATIVE OF A FUNCTION GIVEN IN Example 2: Find


dy
if x 1 - t 2 and y = 3t 2 - 2t 3 .
THE FORM OF PARAMETRIC EQUATIONS dx

Solution: Given that x = 1 - t2 ...... (i) and y = 3t2 - 2t2


The equations x = at and y = 2at express x and y as function of t . Here the variable t
(ii)
2

is called a parameter and the equations of x and y in terms of t are called the parametric Diferentiating (i) w.r.t. ‘ t ’ ,we get
equations.
version: 1.1 version: 1.1

34 35
1. Quadratic Equations eLearn.Punjab 1. Quadratic Equations eLearn.Punjab
2. Diferentiation eLearn.Punjab 2. Diferentiation eLearn.Punjab

= (1- t 2 ) = (1) - ( t 2 ) =-
 ( 2t )  (1 + t ) - 2t x (1 + t )
0 2t =- 2t d 
dy d d d
d  2t 
d
 dt 
2 2

(1 + t 2 )
=
= 
dt dt dt dt

dy
dt  1 + t 2 
dt
Diferentiating (ii) w.r.t. ‘ t ’ ,we have
2
dt

2 (1 + t 2 ) - 2t ( 2t ) 2 + 2t 2 - 4t 2 2 (1 - t 2 )
= ( 3t 2 - 2t 2 )= ( 3t 2 ) - ( 2t 3 ) 2 - 2t 2
(1 + t ) (1 + t 2 ) (1 + t ) (1 + t )
= = = =
dy d d d

=3 ( 2t ) - 2 ( 3t 2 ) =6t - 6t 2 =6t (1 - t )
dt dt dt dt 2 2 2 2 2 2 2

2 (1 - t 2 )
( ) 2 (1 - t 2 ) t 2 - 1
Applying the formula
+
dy
= .= = = =
2 2
dy dy dt 1 t
-4t
dt

(1 + t )
dx dt dx dy - 4t 2t
dy
= =
2
dy dy dt dt dx 2

.
dx dt dx dx

6t (1- t )
dt

-3 (1- t ) =3 ( t -1)
2.7 Differentiation of Implicit Relations
= =
-2t
Sometimes the functional relation is not explicitly expressed in the form y = f ( x )
1- t 2
Find= if x =
dy 2t
1+ t 2 1+ t
Example 3: , y but an equation involving x and y is given. To ind dy from such an equation, we diferentiate
dx dx

(1+ t )
each term of the equation and use the chain rule where it is required.The process of inding

=
= (i ) ( ii )
2
2t
1+ t 1+ t 2
Solution: Given that x 2
and y dy
in this way, is called implicit diferentiation. We explain the implicit diferentiation in the
dx
Diferentiating (i) w.r.t. ‘ t ’ ,we get following examples.

d

d  1- t   dt
(1- t 2 )  (1+ t 2 ) - (1- t 2 ) . (1+ t 2 )


d
= =  if x 2 + y 2 =
2 
dx 2 dy
dt  1+ t  (1 + t )
dt Example 1: Find 4
2 2 dx

( -2t ) (1 + t 2 ) - (1- t 2 ) ( 2t ) 2t ( -1 - t 2 - 1 + t 2 )
dt

-4t Solution: Here x 2 + y 2 =

(1 + t ) (1 + t ) (1 + t )
=
=
4 (i)
2 2 2 2 2 2

Diferentiating both sides of (i) w.r.t. ‘ x ‘ , we get


Diferentiating (i) w.r.t. ‘ t ’ ,we have

version: 1.1 version: 1.1

36 37
1. Quadratic Equations eLearn.Punjab 1. Quadratic Equations eLearn.Punjab
2. Diferentiation eLearn.Punjab 2. Diferentiation eLearn.Punjab

2x + 2 y = dy 2 ( 2 - x ) 2 - x
⇒ =4 - 2x ⇒ = =
dy dy
0 2y (ii)
dx dx dx 2y y

or x + y =
0 ⇒- =
dy dy x
dx dx y Note: Solving (i) for y , we have

y2 =5 + 4x - x ⇒ y =± 5 + 4 x - x 2
Solving (i) for y in terms of x, we have
Thus y = 5 + 4 x - x 2
y±=-4 x 2
(iii)

y=
- 5 + 4 x - x2
⇒ y= 4- x 2
or (iv)
(ii)
Each of these equations (iii) and (iv) deines a function.
or y-=-4 x
y = f1 ( x ) = 5 + 4 x - x2
2
(iii)
Let (v)

y=f1 ( x ) =- 5 + 4 x - x2 .
dy
found above represents the derivative of each of functions deined as in dx and (vi)
dx
(ii) and (iii) Diferentiation (v) w.r.t. ‘ x ‘ , we get

f1 ' ( x )= ( 5 + 4 x - x ) × ( 4 - 2 x )=
2- x
x ( --2 x ) =
2 - 2
=
1
1
5 + 4x - x2
dy 1 x
dx 2 4 - x 2 4 - x2

( )
From (ii) 2
2- x
= -  4 - x2 = = (v)=
, 5 + 4 x - x 2 y, f1 ' ( x )
x
y From so

( )
y y
-x
=- x ( -2 x ) = =-  - 4 - x =y
Also f 2 ' ( x ) = ( ) × ( 4 - 2x) =
2- x
dy 1 x
2 4 - x2 - 4 - x2 - + -
From (iii)
2 - 2
1
dx y 1
- 5 + 4 x - x2
5 4 x x
2
2- x
From (vi) -=
5 +=
4 x - x 2 y, so f2 ' ( x )
,if y 2 + x 2 - 4 x =
dy

Thus (ii) represents the derivative of f1 ( x ) as well as that of f 2 ( x ) .


Example 2: Find 5. y
dx

Solution: Given that y + x - 4 x =


52 2
(i)

Diferentiating both sides of (i) w.r.t. ‘ x ’ ,we get if y 2 - xy - x 2 + 4 =


dy
Example 3: Find 0.

 y 2 + x 2 - 4 x  = ( 5 )
dx

Solution: Given that y2 - xy - x2 + 4 = 0


d d
(i)

( y ) = ( y )
dx dx
 d 2 dy 
+ 2x - 4 = =
 2y 
dy d 2 dy
dx 
or 2y 0
dx dx dx dx Diferentiating both sides of (i) w.r.t. ‘ x ‘ , gives
version: 1.1 version: 1.1

38 39
1. Quadratic Equations eLearn.Punjab 1. Quadratic Equations eLearn.Punjab
2. Diferentiation eLearn.Punjab 2. Diferentiation eLearn.Punjab

 y - xy - =
x 2 + 4  ( 0=
) 1  2 ( x - 1) 2 ( x 2 - 1)( x 2 + 1)
= 2 x + ( -2 ) . 3 = 2  x - 3  =

=
d 2 d 4
0 dy 1
 x 
dx dx
dx x x3 x3
dy  dy  1 x2 + 1
- 1.y + x  - 2 x + 0 = = 1 - ( -1) . 2 = 1 + 2 =
dx  dx 
or 2y 0 du 1

dy dx 2 ( x - 1)( x + 1) 2 ( x 2 - 1)
and
dx x x x2
dy 2 x + y
⇒ (2 y - x) =
+ ⇒ = Thus = .= = . 2 =

- 2 x
1

2 2
dy
dx 2 y - x
dy x2
x +1  x
2x y
dx du dx du x 3
x

if y 3 - 2 xy 2 - x 2 y + 3 x =
dy EXERCISE 2.4
Example 4: Find 0.
dx
dy
Solution: Diferentiating both sides of the given equation w.r.t. ‘x’ we have 1. Find by making suitable substitutions in the following functions deined as:
dx
1- x a+x
 y - 2 xy 2 + x 2 y + 3=
x  ( 0 )= 0 y=
1+ x
= x+ x y=x
a-x
d 3 d (i) (ii) y (iii)

( y ) - ( 2 xy 2 ) + ( x 2 y ) + ( 3 x ) = ( 3x 2 - 2 x + 7 )
dx dx
a2 + x2
y=
d 3 d d d
a2 - x
6
or 0 (iv) (v)

( y ) - 2 1.y 2 + x ( y 2 )  +  2 xy + x 2  + 3 =
dx dx dx dx
d 3  d   dy 
   dx 
0

( y ) and ( y ) , we have
dx dx dy
2. Find if:
d 3 d 2 dx
3x + 4 y + 7 = xy + y 2 =
Using the chain rule on
dx dx (i) 0 (ii) 2
  dy   x 2 - 4 xy - 5 y = 4 x 2 + 2hxy + by 2 + 2 gx + 2 fy + c =
- 2  y 2 + x  2 y   + 2 xy + x 2 +3=
y ( x 2 - 1=
) x x2 + 4
dy dy (iii) 0 (iv) 0
  dx  
2
3y 0
x 1+ y + y 1+ x =
(3 y - 4 xy + x 2 )
dx dx
(v) 0 (vi)
= 2 y 2 - 2 xy - 3
2 dy
or
dx dy
2 y 2 - 2 xy - 3
3. Find of the following parametric functions
⇒ =
dy dx
3 y 2 - 4 xy + x 2
a (1 - t 2 )
dx

x= q + and y = q +1 =
=
q
1 2bt
Diferentiate x 2 + w.r.t. x - 1+ t2 1+ t2
1 1 (i) (ii) x , y
Example 5:
x2 x

1- t2
+ y=
x-2 = Prove that y = +x 0 = =
dy 2t
1+ t 1+ t
1 1 4. if x , y
Solution: Let and u x . Then dx 2
x2 x

version: 1.1 version: 1.1

40 41
1. Quadratic Equations eLearn.Punjab 1. Quadratic Equations eLearn.Punjab
2. Diferentiation eLearn.Punjab 2. Diferentiation eLearn.Punjab

5. Diferentiate

x2 - (1 + x ) dx
dx 
sin    dx 
 2 → 0 
1

2 n

 2 
(i) w.r.t x 4 (ii) w.r .t x2
= +lim cos  x  dlim
2  →0 d x
2

 
x
x2 + 1 x -1 ax + b ax 2 + b 
 when d x → 0 
dx
→0
x

x -1 x +1 cx + d ax 2 + d
(iii) w.r .t (iv) w.r .t 2 2
2
dx
2

x2 + 1  
  dx 
x -1 =
Thus = cos=
x+.1. lim cos  x 1
sin
 cos x and
(v) w.r .t x3
dx
dy
 
2 2
 
lim
d x / 2 →0 d x / 2 →0

 
dx 2

Let y= cos x, then y + d y= cos ( x + d x )


2
2.8 DERIVATIVES OF TRIGONOMETRIC
FUNCTIONS and d y = cos ( x + d x ) - cos x
= cos x cos d x - sin x sin d x - cos x
While inding derivatives of trigonometric functions, we assume that x is measured in
 1 - cos d x 
1 - cos x - - x sin d x cos x 
= 
==  dx 
sin x sin
0 are used to ind the derivative
dy sin d x  1- cos d x 
radians. The limit theorems lim 1 and lim

( sin x ) .
x →0 x →0

= - - 
x x
cos x 
dx dx  dx 
formulas for sin x and cos x.
We prove from irst principle that
dy  sin d x  1 - cos d x  
-lim ( sin x ) -
=
( sin x ) = cos x and ( cox x )= - sin x
cos x  
d x →0 d x dx  dx 

lim
d x →0
d d

y = sin x Then y + d y = sin ( x + d x ) = lim ( - sin x )


 sin d x    1 - cos d x  
dx dx

 - lim  - cos x  
d x  d x →0   dx
and d y = sin ( x + d x ) - sin x
d x →0  
Let

 x +dx + x   x +dx - x   dx  dx 


sin d y
= 2 cos  =
   +   sin    
     2   2   d x→0 d x = 1and 
sin 2 cos x

( sin- x ) .1 ( cos x )( 0 )

 
lim
Thus - =
2 2
- d
dy
   
 dx  dx  dx   dlim   = 0
2 cos  x + dx
dx 1 cox x
 sin   sin      
dy  2  +cos  x d x   2 
x →0

=  =
2 
 
dx dx  2  dx ( cos x ) = - sin x
d
or
dx

( sin x ) = cos x and ( cos x )= - sin x, we prove that


2
 d x 
d d
 sin   
dy dx  2 
Now using

= lim cos  x +  dx 
dx dx
d x →0 d x
    =
= ( sec x ) sec x tan x and ( cot x ) cosec 2 x
lim
d x →0
d d
 
2
2 dx dx

version: 1.1 version: 1.1

42 43
1. Quadratic Equations eLearn.Punjab 1. Quadratic Equations eLearn.Punjab
2. Diferentiation eLearn.Punjab 2. Diferentiation eLearn.Punjab

Now we write the derivatives of six trigonometric functions


( sec x ) = sec x tan x.
(1) ( sin x ) = cos x ( 2) ( cos x ) = sin x
d
Proof of d d
dx

Let = =
dx dx

( 3) ( tan x ) = sec 2 x ( 4) ( cot x ) = - cosec 2 x


1
y sec x (i) d d
cos x
dx dx

( 5) ( cosec x ) = - cosec x cot x ( 6) ( sec x ) = sec x tan x


Diferentiating (i) w.r.t. ‘ x ’ , we have d d
dx dx

d  1   dx 
d
 (1)  cos x - 1. ( cos x )
  Using 
( y) =
d
=  
 
( cos x )  quotient 
d dx
dx  cos x   formula 
Example 1: Find the derivative of tan x from irst principle.
 
0.cos x - 1.( - sin x )
2
dx

= Solution: Let y= tan x, then +y =d x tan +( x d x ) and


d y = y + d x - y = tan ( x + d x ) - tan x
cos 2 x

=
=
1 sin x

sin ( x + d x ) sin x sin ( x + d x ) cos x - cos ( x + d x ) sin x


. sec x tan x
cos x cos x
= -=
( sec x ) = sec x tan x cos ( x + d x ) cos x cos ( x + d x ) cos x
d
Thus

sin ( x + d x - x )
dx
sin d x
( cot x ) = cos ec 2 x =
=
cos ( x + d x ) .cos x cos ( x + d x ) cos x
d
Proof of
dx

= = dy sin d x
=
d x cos ( x + d x ) .cos x d x
cos x 1
Let y cot x (i) .
sin x

dy    sin d x 
= lim 
Diferentiating (i) w.r.t. ‘ x ’ , we get
d x →0 d x d x →0 cos ( x + d x ) .cos x
 . lim 
d x →0  d x

1
  
or lim

d
( ) 
- ( sin x )  Using 
d  cos x   dx   lim cos ( x + d x ) = cos x 
( y) =
d
=  quotient 
cos x sin x cos x

dx  sin x  ( sin x )    d x →0 
d dx
 formula  =
=
dx ( cos x )( cos x )  sin d x 
dy 1
  =1
2

 and dlim 
dx Thus .1 sec 2 x
 dx 
( - sin x ) sin x - cos x ( cos x )
x →0

=
( tan x ) = sec 2 x
- ( sin x + cos x )
= sec 2 x
sin 2 x dy d
Thus or
= -2 - = =2
2 2
1 dx dx
cos ec 2 x
sin x sin x

( cot x ) = cos ec 2 x
d
Thus
dx
version: 1.1 version: 1.1

44 45
1. Quadratic Equations eLearn.Punjab 1. Quadratic Equations eLearn.Punjab
2. Diferentiation eLearn.Punjab 2. Diferentiation eLearn.Punjab

Example 2: Diferientiate ab-initio w.r.t. ‘ x ‘

 x +dx + x   x +dx - x 
cos   sin  
(i) cos 2x (ii) sin x (iii) cot 2 x
 2 .  2 
x +dx + x x +dx - x
y cos 2 ( x + d x )
y cos 2 x, then y + d=
Solution: (i) Let=
d y= cos ( 2 x + 2d x ) - cos 2 x
2

    x +dx - x 
x +dx + x  sin  
and
 
2 x + 2d x + 2 x 2 x + 2d x - 2 x   
2+sin ( 2 x d x ) sin d x
lim
- = - = = →
lim  . x + d x - x
cos
0
x +dx - x 
2
x +dx + x
dy
dx d x→0  
Thus 2
 
2 sin sin
   
 
2 2
dy sin d x
2+sin ( 2 x d x ) .
 
2
- =
2
dx dx  x+ x 
Now
   x +dx - x 
sin d x   → 
lim  2 sin ( 2+x d x ) .
 =
=  .1
cos
=-

 
dy cos x
d x 
0 when
dx  x+ x 
2
 
dy
dx d x→0  dx→0
2
   
Thus 2 x
 
sin d x
- 2+lim ( sin 2 x d x ) . lim
=
d x →0 d x →0 d x ( iii ) Let y = cot 2 x, then
sin d x  y cot 2 ( x + d x )
y + d=
( 2 sin-2 x=
-= ) .1 2 sin 2 x  dlim sin ( 2 x + d x ) = 1 =
y cot 2 ( x + d x ) - cot 2 =
x cot ( x + d x ) + cot x  x cot
+ ( x - d x ) cot x 
 x →0 d x →0 d x  d=
sin 2 x and lim

Let y +sin x , then y + d y = sin x d x  cos ( x + d x ) cos x 


= cot ( x + d x ) + cot x  . - 
( )
(ii)

d=
y sin x + d x - sin x  + d 
sin x cos ( x + d x ) - cos x sin ( x + d x )
and sin x x sin x

= cot ( x + d x ) + cot x  ×
 x +dx + x   x +d x - x 
= 2 cos   sin   sin ( x + d x ) sin x
   
 sin x cos ( x + d x ) - cos x sin ( x + d x ) 
d y  cot ( x + d x ) + cot x  - sin d x
( )( )
2 2

x + d x - x =( x + d x ) - x =d x, ( )
=  . 
As x +dx + x d x  sin ( x + d x ) sin x  d x
=
 - ( + d ) = ( -d ) =
- d 

 cot ( x + d x ) + cot x
sin x x x sin x sin x
 x +dx - x  dy sin d x 
= -lim  .( 1) 
 sin ( x + d x ) sin x
sin  
 x +dx + x  d x →0 d x dx 
dy  
lim
d x →0
= 2 cos  .
2
dx dx
   lim cot ( x + d x ) =
So
cot x + cot x cot x 
.( 1) . 1
2
 x +dx + x   x +d x - x  = -  
 and lim sin ( x + d x ) =
dy d x →0
2 cos   sin   sin x 
Thus
 
( )( )
   
dx sin x sin x
= -2 cot x
d x →0

= .1 = -2 cot x co sec x
2 2
x +dx + x x +dx - x
2

sin 2 x
version: 1.1 version: 1.1

46 47
1. Quadratic Equations eLearn.Punjab 1. Quadratic Equations eLearn.Punjab
2. Diferentiation eLearn.Punjab 2. Diferentiation eLearn.Punjab

 p p
Differentiate sin3 x w.r.t. cos 2 x
= = x Sin ∈ - x
Example 3:
sin y for y 
==  2 2 
3 2
Then y or , (ii)
Solution: Let y sin x and u cos x

-2 cos x ( sin x )
Diferentiating both sides of (ii) w.r.t. ‘ x ’ , we get
Now = 3 sin 2 x cos=
( sin y ) (=
sin y )
dy du
= 1 =
x and d d dy dy
dx dx cos y
 
dx dx dx dx

( 3 sin2 x cos x ) .=
 dx 1 
Thus= =  dx   p p
=
⇒ ∈ - for
dy dy dx 1
-2 cos x sin x  du  y  , 
. dy 1
 du   2 2
du dx du
dx cos y
  p p 
= - sin x. =  ∈  - , 
3 1
1 - sin 2 y   2 2 
 cos y is positive for y

( sin -1 x )
2

Thus = - < < for


d 1
1 - x2
1 x 1
dx
2.9 DERIVATIVES OF INVERSE
Proof of (2). Let y = Cos -1 x
[0, p ]
TRIGONOMETRIC FUNCTIONS (i)
=
=
Then ∈ y or x
x Cos cos y for y (ii)
Here we want to prove that
Diferentiating both sides of (ii) w.r.t. ‘ x ’ , gives
 sin -1 x  = x ∈ ( -1 , 1) or - 1 < x < 1
( cos y ) = ( cos y ) = - sin y
d 1
1- x 1=
1. , d d dy dy
dx 2

Cos -1 x  = - x ∈ ( -1 , 1) or - 1 < x < 1


dx dx dx dx

y ∈ ( 0 ,p )
d 1
1 - x2 ⇒ - =
2. , dy 1
dx for
Tan -1 x  = - x ∈R
dx sin y
d 1
 sin y is positive for y ∈ ( 0 ,p ) 
1 + x2
= -
3. ,
dx 1
1 - cos 2 y
Cosec -1 x  = - x ∈ [ -1 , 1]' , [ -1 , 1]' = ( -∞ ,-1) ∪ (1,∞ )
( Cos -1 x ) =-
d 1
| x | x2 - 1 -1 <x <1
4. ,
dx d 1
1 - x2
Thus for
 Sec -1 x  = - x ∈ [ -1 , 1]' , [ -1 , 1]' = ( -∞ ,-1) ∪ (1,∞ )
dx
d 1
| x | x2 - 1
5. ,
Proof of (3). Let y = Tan -1 x
dx
(i).
Cot -1 x  = - x ∈R
 p p
d 1
1 + x2
= ∈ y- for
6. ,
y  , 
dx
 2 2
Then x=Tan y or x tan (ii)
Proof of (1). Let y = Sin -1 x (i).
Diferentiating both sides of (ii) w.r.t. ‘ x ’ , we have

version: 1.1 version: 1.1

48 49
1. Quadratic Equations eLearn.Punjab 1. Quadratic Equations eLearn.Punjab
2. Diferentiation eLearn.Punjab 2. Diferentiation eLearn.Punjab

= = ( tan y ) (=
tan y )  p 
When y ∈  - ,0  ,cosec y and cot y are negative
d d dy dy
 2 
1 sec 2 y
dx dx dx dx
 p p
=⇒ ∈ 2- y  ,  As cosec y = x, so x is negative in this case
dy 1
 2 2
for
dx sec y
and cot y = - cosec 2 y - 1 = - x 2 - 1 when x < -1
= = for x ∈ R
1 1
1 + tan y 1 + x 2
2

( )
-1
Thus = Tan -1 x  ∈ 2 for x R Thus= Co sec -1 x  < - (x 1)
d 1
1+ x
d
dx dx x - x2 - 1

Let y = Co sec -1 x -1
= <- (x 1)
( - x ) x2 - 1
Proof of (4). (i)

 p p
= =x Co sec∈y -or x cos
- ec y for y  {0}
 2 2  cosec -1 x  = - x ∈ [ -1, 1]'
Then , (ii) d 1
| x | x2 - 1
for
dx

 p p  p   p
 - 2 , 2  - {0} is also written as  - 2 0  ∪ 0 , 2 
Proof of (5). is left as an exercise
Proof of (6). is similar to that of (4)
Diferentiating both sides of (ii) w.r.t. ‘ x ’ , we get

x
=
= ( cosec y ) ( cosec y ) if =y x Sin -1  + a 2 + x 2
dy
a
d d dy Example 1: Find
1
dx dx dx dx

= ( - cosec y cot y )
dy
x
Solution: Given that=y x Sin -1   + a 2 + x 2
dx
 p p
⇒ =- y ∈ - ,  -{0} a
dy 1
 2 2
for
dx cosec y cot y
Diferentiating w.r.t. x , we have

 p
When y ∈  0 ,  , cos ec y and cot y are positive.
 2 =
dy d 
dx dx 
x Sin -1 x

a
+ a=2
+ x 2 d 
 dx  x Sin -1 x 
+
a  dx
(
d 2
a + x )
2 1/ 2

.(+a ) (+a x2 )
As cosec y = x , so x is positive in this case d x
= 1 . Sin + x.
-1
. +  2 2 -1
1
x 1 1 2 d 2
and cot y= co sec y - 1= x -1 for all x > 1  x  dx  a 
x
1-  
2 2 a 2 2 dx
a
=
d
( Co sec -1 x ) >
-1
x x2 - 1
Thus for x 1
dx

version: 1.1 version: 1.1

50 51
1. Quadratic Equations eLearn.Punjab 1. Quadratic Equations eLearn.Punjab
2. Diferentiation eLearn.Punjab 2. Diferentiation eLearn.Punjab

dy

.( -2 x )
3. Find if
Sin -1 +x . +
x 11 1 dx
x2 a 2 a2 - x2 y = x cos y x = y sin y
1- 2
a (i) (ii)
a

Sin -1 + x . - = Sin -1
x a 1 1 x
a2 - x2 a a2 - x2
4. Find the derivative w.r.t. x
a a
1+ x 1 + 2x
1 + 2x 1+ x
4 (1 + y 2 )
(i) cos (ii) sin

 x
=
= If y tan  2 Tan -1  ,show that
dy
4+ x
Diferentiate
 2
Example 2: 5.
2
dx
(i) sin x w.r.t. cot x (ii) sin 2 x w.r.t. cos 4 x

u = 2 Tan -1 , then If tan y (1 + tanx-) =


1 tan x,show that - =
x dy
Solution: Let 6. 1
2 dx
y =tan u ⇒ =sec 2 u =1+ tan 2 u =
1 + y2 If= tan x + tan x + tan x + ...∞ ,prove that -( 2 y=1)
dy dy
7. y sec 2 x.
du dx
d  -1 x  d x If =x a cos 3 q =
, y b sin3 q , show that
+ a= btanq
== =
 2 Tan  2. = .  
du 1 2 1 4 dy
dx  2  x  dx  2  1 + x 2 4 + x2
and . 8. 0
1+  
2 2 dx

a ( cos t + sin t ) , y =a ( sin t - t cos t )


dx
2 if x =
4 (1 + y )
4 dy

(1 y2 ) .
9. Find
dx
Thus + == . =
2
dy dy du 4
dx du dx 4 + x2 4 + x2 10. Diferentiate w.r.t. x

Cos -1 Cot -1 Sin -1


x x 1 a
(i) (ii) (iii)
EXERCISE 2.5 a a a x
 x2 + 1   2x 
Sin -1 1 - x 2 Sec -1  2  Cot -1  2 
 x -1 1- x 
1. Diferentiate the following trigonometric functions from the irst principle, (iv) (v) (vi)
sin 2 x + cos 2 x
 1 - x2 
(i) sin x (ii) tan 3 x (iii) (iv) cos x 2
Cos  -1
2 
1+ x 
(v) tan x2
(vi) tan x (vii) cos x (vii)

Diferentiate the following w.r.t. the variable involved


= =
2.
tan q sec q Tan -1
dy y y x
(i) 2
x sec 4 x (ii) 3 2 11. if

( sin 2q - cos 3q )
dx x x y

(+p Tan-1-x ) , show


+ that (1 x 2 ) y1 p (1 y 2 ) 0
cos x + sin x
= =
2
(iii) (iv)
12. If y tan
version: 1.1 version: 1.1

52 53
1. Quadratic Equations eLearn.Punjab 1. Quadratic Equations eLearn.Punjab
2. Diferentiation eLearn.Punjab 2. Diferentiation eLearn.Punjab

 ah - 1 
2.10 DERIVATIVE OF EXPONENTIAL FUNCTIONS:
= a .( ln a )  Using lim= = log a e ln a 
 
x
h →0

( a ) = a x .( ln a )
h

f ( x) = ax
A function f deined by
d x
or
a > 0 , a ≠ 1 and x is any real number.
dx

is called an exponential function


if : (i) y = e x +1 (ii) y = a
If a = e , then y = a becomes y = e . e is called the natural exponential function.
dy 2
x
x x x Example 1: Find
dx
Now we ind derivatives of e x and a x from the irst principle:
1. Let y = e x then Solution: ( i ) Let =
u x 2 + 1 , then
y + d y =e x +d x and d y =y + d y - y =e x +d x - e x =e x . ed x - e x
y = eu ....( A ) and =
du d 2
( x +=
1) 2 x

e x ( ed x 1)
dy  ed x - 1 
That is , d y-= = ex
dx dx
. 
dx  dx  Diferentiating both sides of ( A ) w.r.t. 'x' , we have
and

dy xe -1 x  ed x - 1 
( e ) ( e ) . dx
dx
=
=
d x →0 d x
lim e 
 dx 
 e . dlim 
 dx 
 = ( y) =
Thus lim d d u d u du
d x →0 x →0 (Using the chain rule)

 Using ( e ) e 
 x 
dx dx du
 x
 dx→0 e =e  =
= eu .
lim
 
 du d x
 
x

dx dx
 eh - 1 
e x +1 .( 2 x )
 
=
= e x .1  Using lim 1 = +u x2 1 =
= 2x 
dy dy du
   
2
Thus and
h →0

( e ) = ex
dx h dx dx

or
d x ( ii ) Let u x
=
= Then y au ( A)

( x ) =
dx
y = a x , then and= =
du d 1/ 2 1 -1/ 2 1

y + d y= a x +d x and d y = a x +d x - a x= a x . ad x - a x= a x ( ad x - 1)
2. Let x

Diferentiating both sides of ( A ) w.r.t. 'x' , gives


dx dx 2 2 x

Dividing both sides by d x , we have

dy  a -1dx = =
dy d u
( a ) =( a ) dx
d u du  dy

 dx
dy du 
. 
du dx 
= ax  
dx  dx 
= ( a u ln a ) . ( a ) a x ln a 
dx dx du
 
= 
 ad x - 1  x  ad x - 1 
du d x
  
Using
=
= =
( ) (a )
lim a x     ax 
lim
dx d x→0  d x   d x  d x → 0
dy dx dx


 1 
Thus a . lim ax
d x →0
Thus = ln a = =
 u 
d 1 du
 2 x
a x x
. x and
dx 2 x dx

version: 1.1 version: 1.1

54 55
1. Quadratic Equations eLearn.Punjab 1. Quadratic Equations eLearn.Punjab
2. Diferentiation eLearn.Punjab 2. Diferentiation eLearn.Punjab

dy 1  dx
= Now= 1 + 
dx dx
ln a 1
 x 
x
.a . ln
2 x
 d x  1  d x d x
= . ln 1 + = ln 1 +
x


x dx 
1 x
Example 2: Differentiate y = a x w.r.t. x. x  x  x 
   
dy  1  dx  1   dx x
lim = lim ln 1 +  = lim ln 1 +
d d
x x

d x →0 d x d x →0  x

x   x d x →0   x  
x

Solution: Here y = a x 
Thus
   
= e x ln a  
 d x d x 
= + . ln  lim
x

1 
dy 1
 d x →0  x  
Diferentiating w.r.t. ‘ x ‘ , we have
x 
dx x

= e x ln a , ( x ln a )  dx 
dy d

( e x In a ax )
 → 0 when d x → 0 
= a .( In a )
=  x 
dx dx

( e ax )
x

= a x .( In a )
= = ln e
 lim
( + ) z =

 z →0 e
1
x In a 1
 
 1 z

( log 1)
x
= = =
1 1 e
.1 e
x x
2.11 DERIVATIVE OF THE LOGARITHMIC FUNCTION Now we ind derivative of the general logarithmic function.
y = log a x then
y log a ( x + d x ) and
Let
y + d=
Logarithmic Function:
If a > 0 a ≠ 1 and x =
 x +dx   dx
d y = log a ( x + d x ) - log a=
( x 0)
a , then the function defind by
log  =
 log a 1 + 
= y >log a  x   x 
x
x

dy 1  dx 1 x  dx
= log a 1 + = . log a 1 +
is called the logarithm of x to the base a.
 
dx dx  x  x dx  x 
x x
The logarithmic functions log e and log are called natural and common logarithms
respectively, y =log e x is written as y = ln x .
10

 d x d x
= log a 1 +
x


1
 x 

( In x ) .
x
   
 d    d  x
d

Thus = lim log a 1 + = 
lim log a 1 +
d d
x x


We first find dy 1 x 1 x
dx d x→0  x x   x d x →0  x  
x

 
dx
Let y = ln x Then    
y + d y= In ( x + d x ) lim x 
 dx x
= + log a  d x 1
and
d

 
x  
1
 x +dx   dx 
d y = ln ( x + d x ) - ln x =   = ln 1 +   x →0 
x
 x   x 
version: 1.1 version: 1.1

56 57
1. Quadratic Equations eLearn.Punjab 1. Quadratic Equations eLearn.Punjab
2. Diferentiation eLearn.Punjab 2. Diferentiation eLearn.Punjab

 z →0 ( )z =
 
2.12 LOGARITHMIC DIFFERENTIATION
= log a x + e
1
1
 
 lim 1 z
x
Algebraic expressions consisting of product, quotient and powers can be often
 1 
log a=  = =
simpliied before diferentiation by taking logarithm.
 log 
d 1 1 1
 ln a 
x e
or . a
dx x ln a log e a
Differentiate y = e
f ( x)

if y log10 ( ax 2 + bx + c )
Example 1: w.r.t.' x '.
=
dy
Example 1: Find
y=e
dx f ( x)
Solution: Here (i)
Solution: Let u = ax 2 + bx + c Then

In y = f ( x ) . In e
Taking logarithm of both sides of (i), we have

y= log10u ⇒ =
= f ( x) ( In e = 1)
dy 1 1

( ax + bx + c ) = a ( 2 x ) + b (1) = 2ax + b
du u In 10

= Diferentiating w.r.t x , we get


du d
and

. = f ' ( x)
dx dx

dy dy du  1 1  du
1 dy
= =  . 
dx du dx  u ln 10  dx
Thus . y dx
×= ( x ) e×f ( x ) f ' ( x )
y f '=
dy

( 2ax b )
( ax + bx + c ) ln 10 ( )
So
= + 2
dx
e = e ( ) × f ' ( x)
1
d f ( x) f x
or

( )
dx
d 
+ +  = 2 2ax + b
dx   (ax + bx + c) ln 10 x x2 + 3
2
or log10 ax bx c

x2 + 1
Differentiate ln ( x 2 + 2 x ) w.r.t. ' x '.
Example 2: Find derivative of

x x2 + 3
......( i )
Example 2:

( x + 1)
Solution: Let y = 2
Solution: Let =y ln ( x 2 + 2 x ) , then
Taking logarithm of both sides, we have

ln ( x 2 + 2 x =
) ( x + 2x)
( )
( Using chain rule )
( )
dy d 
- ln ( x 2 + 1)
=   x x2 + 3 
1 d 2
+ = y ln  2 =  ln x x 2 + 3
.
 x +1 
2 ( x + 1)
2
dx dx dx
 
x 2 x ln

= .(=
2x + 2)
ln ( x 2 + 3) - ln ( x 2 + 1)
1
x + 2x x2 + 2 x
or ln y= ln x + ......( ii )
2 ( x + 1)
2

( )
1
d  =
+
2
dx   x2 + 2 x
2
Thus ln x 2 x
Diferentiating both sides of (ii) w.r.t ‘ x ‘,
version: 1.1 version: 1.1

58 59
1. Quadratic Equations eLearn.Punjab 1. Quadratic Equations eLearn.Punjab
2. Diferentiation eLearn.Punjab 2. Diferentiation eLearn.Punjab

[ In=
y] In ( x 2 + 3) - In ( x 2 + 1) 
d  
2.13 DERIVATIVE OF HYPERBOLIC FUNCTIONS
 +
d 1
dx  
In x
dx 2
The functions deined by:
= + . 2 × 2x - 2 × 2x
1 dy 1 1 1 1
x 2 x +3 x +1 ex - e
-x
e x + e- x
sinh x = , x ∈ R ; cosh x = ; x∈R
y dx

+= 2-
1 x 2x
x x +3 x +1
2 2
sinh x e x - e - x
(x + 3)( x 2 + 1) + x . x ( x 2 + 1) - 2 x . x ( x 2 + 3)
= = ;x∈ R
2

cosh x e x + e - x
tanh x

x ( x 2 + 3)( x 2 + 1)
=
2

are called hyperbolic functions.


x4 + 4 x2 + 3 + x4 + x2 - 2 x4 - 6 x2 3 - x2
x ( x 2 + 3)( x 2 + 1) x ( x 2 + 3)( x 2 + 1)
=
The reciprocals of these three functions are deined as:

= x - x , x ∈ R - {0} ;
y (3 - x )
cos ech=
1 2
x x2 + 3 3 - x2 sinh x e - e
dx x ( x 2 + 1)( x 2 + 1) x 2 + 1 x ( x 2 + 3)( x 2 + 1)
x
=
=
2
dy
Thus .
= = x , x∈R
1 2
cosh x e + e - x
sec hx

3 - x2 e x + e- x
, x ∈ R - {0}
x 2 + 3 . ( x 2 + 1)
= = = x
1
tanh x e - e - x
2
coth

Derivatives of sin h x, cos h x and tan h x are found as explained below:


Differentiate ( ln x ) w.r.t. ' x '.
( sinh x ) =
d 1 x
( )
-x 
  ( e + e - x ) = cosh x
x

- = - - =
Example 3:
   
d 1 x -x 1 x
dx  2  2
e e e e ( 1 )
dx 2
Solution: Let y = ( ln x )
( ) ( e - e - x ) = sinh x
x

( cosh x ) =
d 1 x -x 
(i)

 +  = 
 + -x
- 
 =
d 1 x 1 x
dx  2  2
Taking logarithm of both sides of (i) , we have e e e e .( 1 )

In y In ( In x )  x In ( In x )
(e + e- x ) ( e x + e- x ) - ( e x - e- x ) ( e x - e- x )
=
=
dx 2
 
x

d  e x - e- x 
[tanh x ]
(e + e )
=
=
dx  e x + e - x 
x
d
Diferentiating w.r.t x , -x 2

+ 2 - (e + e - 2)
dx x

e 2 x + e -2 x
= 1 . In ( In x ) + x . . ( In x )
-2 x

(e + e ) (e + e- x )
=
=
2x
1 dy 1 d 4
y dx In x dx x -x 2 x 2

In ( In x ) + x . In ( In x ) +
 2 
= . = = = -x 
2
1 1 1
e +e 
x
sec h 2 x.
In x x In x

 1   1 
= y  In ( In x ) + = ( In x )  In ( In x ) + In x 
In x 
dy The following results can easily be proved.
  
x

dx
version: 1.1 version: 1.1

60 61
1. Quadratic Equations eLearn.Punjab 1. Quadratic Equations eLearn.Punjab
2. Diferentiation eLearn.Punjab 2. Diferentiation eLearn.Punjab

( cos eh x ) = ( sec h x-)


2.14 DERIVATIVES OF THE INVERSE
= -
d d
coth x cos ech x ; tanh x sec h x
dx dx HYPERBOLIC FUNCTIONS:
( coth x ) = - cos ech2 x.
d
dx The inverse hyperbolic functions are deined by:
=
1. =
y sinh' -1 x∈ if and if x
[0, ]
sinh y ; x, y R
if y = sinh 2 x = = -1
∈ y ∞ ; x∈ 1∞, ) , y
dy
Example 1: Find
- if x ∈tanh y ; x ( 1,1) , y
2. y cosh x if and only if x
cosh
= = x if and∈only
dx
-1
3. y tanh R

[ 1,1] , y R
Solution: Let u = 2 x, then
= y =
coth -1 x if and∈only
- if x ∈coth- y {0}
'

; x (0 ,1` ] , y [0 ,
4. ; x
= y sec∈h -1 x if and
∈ only
∞ if x= sec h y
∈ech- y ; x ∈R -{0} , y {0}
5. )
= =
⇒ = y cos ech=
-1
dy
y sinh u cosh u 6. x if and only if x cos R

( ) ( )
du

( 2 x ) 2.
The following two equations can easily be derived:
= =
du d

(i ) sinh -1 x = In x + x 2 + 1 ( ii ) cosh -1 x= In x+ x 2- 1
and
dx dx
= = cosh= cosh (=
2 x )  .2 2 cosh 2 x
dx 
dy dy du du
Thus . u.

[ sinh 2 x ] = 2 cosh 2 x .
dx du dx
Proof of (i).
=Let y ∈sinh -1 x for x, y R,then
d
or
dx
e y - e- y
y = tanh ( x 2 )
dy =x sinh y ⇒=x
Example 2 : Find if 2
⇒ 2 xe y =e 2 y - 1
dx

e 2 y - 2 xe y - 1 =
Solution: Let =
u x ,then =
y tanh u ⇒ = sec h 2 u
dy or 0
2

du Solving the above equation for e y , we have

2 x ± 4 x2 + 4
= =
and
du d
( x ) 2x ey =
2
2 x ± 2 x2 + 1
dx dx
= =
x ± x2 + 1
sec h×2 ( x 2 ) 
du 
= = sec h 2= 
dy dy du 2
Thus . u. 2x
As e y is positivefor y ∈ R, so we discard
dx du dx dx
tanh x 2  = 2 x sec h 2 x 2
 x - x2 + 1
d

( )
or
dx

Thus e y =
x + x2 + 1 ⇒ y =
In x + x 2 + 1

version: 1.1 version: 1.1

62 63
1. Quadratic Equations eLearn.Punjab 1. Quadratic Equations eLearn.Punjab
2. Diferentiation eLearn.Punjab 2. Diferentiation eLearn.Punjab

( )
Derivative of cosh -1 x:
⇒ sinh x = In x + x + 1
∈ -1∞x ; ∈ x ∞[1 ) , y [0,
-1

=
Proof of ( ii )
2

Let y cosh )

Let y cosh -1 x for x ∈ [1 , ∞ ), y ∈ [ 0 ,∈ ), then


= Then x = cosh y

e y + e- y
=
x cosh y ⇒
= ⇒ e 2 y - 2e y =
+1 0 ......( I )  
 
x
 dy 1 
2
2 x ± 4 x2 - 4 2 x ± 2 x2 - 1 = sinh y ⇒
= =
Solivng ( I ) gives, e =  dx dx 
dx dy 1
= =± x2 - 1 .

( )
and
 
y

 dy 
x dy dx sinh y

2 2
e y =- x 2 - 1 can be written as y =In x -x 2 1-

( ) ( sinh y > 0, as y > 0 )


x
= =
If x = 1, then y = ln 1 - 1 - 1= ln (1=
) 0 but
dy 1 1
cosh y - 1

( )
or

( > -1 x )
dx sinh y 2

ln x - x 2 - 1 is negative for all x > 1, that is =


= (x 1)
dy d 1
x2 - 1

( )
for each x ∈ (1,∞ ) , y ∉ ( 0 ,∞ ) ,so we discard this value of e y
Thus cosh
dx dx

( )
As cosh -1 x = In x + x 2 - 1 , so
Thus e y =+ x 2 + 1 which give y =
In x + x 2 - 1 , that is

( )
x
 2x  x2 - 1 + x
cosh -1 x = In x + x 2 - 1 . cosh x =
dx   1 + =  =
-1

x + x 2 - 1  2 x 2 - 1  x + x 2 - 1
d 1 1 1
x2 - 1 x2 - 1
.

Derivative of sinh -1 x :
Derivative of tanh -1 x :
=Let y ∈sinh -1 x ; x , y
Let y = tanh -1 x ; x ∈ ( -1, 1) , y ∈ R
R

Then x = sinh y  
 
Then x = tanh y and = sec h 2 ⇒ = = dy 1 
   dx dx 
dx dy 1

   
sec h 2 y
 dy 
dy dx

= cosh y =
⇒ =  dy 1  
 dx dx 
dx dy 1




dy 
= = (sec h= y - 1 tanh 2 y )
dy dx cosh y
dy 1 1
 dx 1 - tanh 2 y 1 - x2
2

or = = > ( cosh y 0) = ( tanh -1 x ) <


dy 1 1 d 1
1 + sinh y 1 - x2
Thus ; -1<x <1or x 1
dx cosh y 2
dx

(
∈sinh -1 x ) (x R)
The following diferentiation formulae can be easily proved.
=
=
( coth -1 x ) =
dy d 1
1 + x2
or - ; x >1
dx dx d 1 1
dx 1 - x2 x -1
2

version: 1.1 version: 1.1

64 65
1. Quadratic Equations eLearn.Punjab 1. Quadratic Equations eLearn.Punjab
2. Diferentiation eLearn.Punjab 2. Diferentiation eLearn.Punjab

( sec h -1 x ) =-
EXERCISE 2.6
0 <x <1
d 1
x 1 - x2
Find f ' ( x ) if
;

( x) =
dx
1.
- -1
>
f ( x) x e ( x ≠ 0) f ( x ) = e x ( I + ln x )
d 1
x 1 + x2 f ( x) = e (ii) =
cosech ;x 0 1
x -1

( cos ech -1 x ) =-
dx

x ∈R -{0}
3
(i) x
(iii)

ln ( e + e )
d 1
x 1 + x2 e ax - e - ax
f ( x) = -x
or ;
-x
fx = ax
dx ex
e +1 e + e - ax

( )
x
(iv) (v) (vi)

Find= if y sinh -1 ( ax + b ) (vii) = ln ( e 2 x + e -2 x ) e 2 x + e -2 x


dy
Example 1: f (x) (viii) f (x) = ln
dx

Solution: Let =
u ax + b , then
dy
2. Find if
= =⇒
dx
sinh -1 u
dy 1
1 + u2 y = x ln x y=
y
y = x 2 ln x
dx x
(i) (ii) (iii)

( )
= =
dy dy du 1 du ln x
1 + u dx
x2 - 1
. .

y = ln 2 y = ln x + x 2 + 1
dx du dx 2

y = x ln
 sinh -1 ( ax +=
b) +(=
ax b )
x +1
1
 du 
(iv) 2
(v) (vi)
=
y= e - x ( x3 + 2 x 2 + 1)
 a
y ln ( 9 - x 2 )

1 + ( ax + b )
d 1 d x
 dx  (viii) y = e -2 x sin 2 x
Thus .a
(vii) =
dx 2 dx
(ix)
(x) y = x e sin x (xi) y = 5e3 x -4 (xii) = ( x + 1)
x

( sec x )
y
= ≤ -≤ x p /2
x 2 - 1 ( x + 1)
dy
Example 2: Find if y cosh 1
0

(xiii) y = ( ln x )
(x +1)
y=
dx
(xiv)
ln x

Solution: Let u = sec x, then


3 3/ 2

=y cosh -1 u ⇒
=
dy 1 dy
3. Find if
u2 -1
dx dx

=
du d
= ( sec x ) sec x tan x (i) y = cosh 2 x (ii) y = sinh 3 x

y = sinh -1 ( x3 )
and
p p
- -1 <( sin< x )
dx dx
= =
dy dy du 1 du =
(iii) y tanh (iv)
u - 1 dx
Thus . . x
2 2
x
dx du dx

y = ln ( tanh x )
2

=
=
= ( sec x tan x ) ( sec x tan x ) sec x y = sinh -1  
2
1 1
(v) (vi)
sec x tan x

cosh -1 ( sec x )  = sec x


d
or
dx

version: 1.1 version: 1.1

66 67
1. Quadratic Equations eLearn.Punjab 1. Quadratic Equations eLearn.Punjab
2. Diferentiation eLearn.Punjab 2. Diferentiation eLearn.Punjab

( )
2.15 SUCCESSIVE DIFFERENTIATION
if y = ln x + x 2 + a 2
d3y
Example 2: Find
(OR HIGHER DERIVATIVES): 3

( )
dx

Sometimes it is useful to ind the diferential coeicient of a derived function. If we Solution: Give that y = ln x + x 2 + a 2 (i)
denote f ’ as the irst derivative of f, then (f ’)’ is the derivative of f ’ and is called the second
Diferentiating both sides of (i) w.r.t. ‘ x ‘ , we have

( )
derivative of f .For convenience we write it as f”.
Similarly (f ”)’. the derivative of f ”, is called the third derivative of f and is written as f ’”.
= + +
In general, for n ≥ 4 , the nth derivative of f is written as f . (n)
dy 1 d
x + x 2 + a 2 dx
x x2 a2
dx
 1× 2 x 
Here we state diferent notations used for derivatives of higher orders..
= + . 1 
1
x + x 2 + a 2  2 x 2 + a 2 
1st derivative 2nd derivative 3rd derivative nth derivative
y’ y ’’ y ’’’ y (n)
 x2 + a2 + x 
= ×  
 
1
x+ x +a  2 x +a 
dy d2y d3y dny 2 2 2 2
dx dx 2 dx3 dx n

That is, dy =
y1 y2 y3 yn 1
x +a
(ii)
dx 2 2

Dy D 2y D 3y D ny
Diferentiating (ii) w.r.t. ‘ x ’, we have

d2y d  2
= ( + -)
2 -1/ 2 
=
+ ( × ) 2x
df d2 f d3 f dn f
2 -3 / 2

dx 2 dx  
1 2
dx dx 2 dx 3 dx n x a x a
2

( x2 + a2 )
= -
d 2y x
Example 1: Find higher derivatives of the polynomial or 2 3/ 2
(iii)

f ( x )=
dx
x - x + x + 2x + 7
1 4 1 3 1 2
12 6 4 Diferentiating (iii) w.r.t. ‘ x ’ , we get

1 . ( x2 + a2 ) - x. ( x + a 2 ) .2 x
( 4 x3 ) - ( 3 x 2 ) + ( 2 x ) + 2 + 0=
3 2

( ) 
3/ 2 1/ 2

f ' ( x )= x - x + x+2 = -
3
d y
+
1 1 1 1 3 1 2 1 2
Solution: dx 3 2 2 3/ 2

f '' ( x ) = ( 3 x 2 ) - ( 2 x ) + (1) + 0 = x 2 - x + ( x + a ) ( x + a ) - 3x -


12 6 4 3 2 2 x a

a2 - 2 x2
1 1 1 1

(x + a ) ( x2 + a2 )
= = -
2 2 1/ 2 2 2 2

f "' ( x= ) 2x - 1
3 2 2 2
2 2 3 5/ 2

f iv ( x ) = 2 2 x2 - a2
(x + a2 )
=
d3y
5/ 2
dx 3 2
All other higher derivatives are zero.

version: 1.1 version: 1.1

68 69
1. Quadratic Equations eLearn.Punjab 1. Quadratic Equations eLearn.Punjab
2. Diferentiation eLearn.Punjab 2. Diferentiation eLearn.Punjab

Example 1: -a (q sinq=
If x = )+, y a (1 cosq ) . Then
if y3 + 3ax 2 + x 3 =
d2y
Example 3: Find 0
+a=
dx 2 d2y
Solution: Given that y3 + 3ax 2 + x3 =
show that y 2 0
0 (i) dx 2

Solution: Given that= a (q + sinq )


y a (1 + cos q )
Diferentiating both sides of (i) w.r.t. ‘ x ‘ , gives x (i)
=
( 0 )= 0 Differentiating ( i ) and ( ii ) w.r.t 'q ' , we get
and (ii)
 y 3 + 3ax 2 + x 3= 
d d

+ 3a ( 2 x ) + 3 x 2 = + ( 2ax x 2 )
dx dx
⇒ -y 2 = = a (1 + cos q )
dy dy
dq
3y2 dx
0 (iii)
dx dx
2ax + x 2
⇒ = - and = a ( -sinq )
dy
dq
(ii) dy
(iv)
dx y2

Diferentiating both sides of (ii) w.r.t. ‘ x ‘ , gives


dy dq dq
dy
Using= =
( 2a + 2 x ) y 2 - ( 2ax + x 2 )  2 y  dq dx
dy
. we have
d  2ax + x 2  dq
( 1) 
dy dx
 dx -
dx

(y )
= - =
dx  y 2 
d2y
- a sin q -sin q
2 2

=
=
dx 2
a (1 + cos q )
2 ( a + x ) y 2 - ( 2ax + x 2 ) . 2 y × -
 2ax + x  1 + cos q

2

= -   sinq
= -
y2
1 + cosq
dy
(v)

( 2ax + x 2 )( 2ax + x 2 ) 
4 That is,
y dx

2 ( a + x ) y +  Diferentiating (v) w.r.t. ‘ x ’
 
2

= -
y
d2y d  sin q   sin q  dq
-2 =
-  = ×
2 ( a + x ) y 3 + ( 2ax + x 2 ) 
 
y4
dx  1+cos q  dq  1+cos q  dx
d

 
= - 
2
dx

cos q (1 + cos q ) - sin q ( -sin q ) dq


( )
2 ( a + x ) -3ax 2 - x 3 + x 2 ( 2a + x ) 
4
y .y

(-y )
= -
- =   =- (1 + cos q )
2
3 2 3 2
.
3ax x dx

2 x 2  - ( a + x )( 3a + x ) + ( 4a 2 + x 2 + 4ax ) 
cos q +cos 2 q +sin 2q dq
5
y
= -
(1 + cos q )
d2y
= -
2
.
dx 2 dx

2 x 2  - ( 3a 2 + 4ax + x 2 ) + 4a 2 + x 2 + 4ax 
1 + cosq
 = a (1 + cosq ) 
 dx 
y5
- =
×
(1 + cos q ) a (1 + cos q )  dq
1
= - 
2

y5
2 x 2  a 2  -2a 2 x 2
- =
=
y5 y5
version: 1.1 version: 1.1

70 71
1. Quadratic Equations eLearn.Punjab 1. Quadratic Equations eLearn.Punjab
2. Diferentiation eLearn.Punjab 2. Diferentiation eLearn.Punjab

= [ -ay ] ⇒ - 2 = ( a )( ay- ) =


dy  dy  d 
  =
a- - ay 
d2y dy dy
dx  dx  dx  dx 

 y
Now
= - = -
a (1 + cos q )2 1 + cos q = 
1 1 1 1 dx dx
a  y 2
 a
. .
 
a
= a2 y
d2y
=- × 2 =- 2
1 a2 a
or 2
(i)
dx
a y y
Diferentiating (i) w.r.t. ‘ x ‘ we get
= -a ⇒y +a =
2d2y d2y2
or y
d d2y d 2
0

a 2 ( -ay ) =
dx 2
=  a y  ⇒ 3 =
dx 2
 2 
a 2 -=
d3y dy
dx  dx  dx
Find the irst four derivatives of cos ( ax + b ) .
a3 y
dx dx
Example 5:

= y cos ( ax + b ) , then + a3 y =
d3y
Solution: Let Thus 3
0
dx
y1 = cos ( ax +
- b )  =+sin ( ax b ) . + ( ax b )
that y2 -x ( a 2 )
d d
-3
= -1
=
- sin ( ax + b ) × ( a + 0 ) =
- a sin ( ax + b )
dx dx x
=
2
Example 7: If y Sin , then show x 2
a

y2 =
-a sin ( ax + b )  =
( -a ) cos ( ax + b ) × ( a + 0 ) 
dx 
d
Solution: y = sin -1
x
- a+2 cos ( ax b )
=
, so
a

y3 =
- a2
d
(
cos ( ax + b )  = - a 2 )  - sin ( ax + b ) × ( a + 0 ) 
d  x d  x
= = = Sin -1  ×  
dy 1
dx  a dx  a 
dx
= a 3 sin ( ax + b ) x
y1
1-  
dx 2

a
sin ( ax+ b ) = a 3× cos ( ax+ b ) × a= a 4 cos ( ax+ b )
(a x2 )
y4 = a 3
d
= = . = .-
1 1 a 1 -1/2

a2 - x2 a a2 - x2 a
dx 2

y2 = ( a 2 - x 2 )  =- ( a 2 - x 2 ) × ( -2 x ) =x ( a 2 - x 2 )
= 6: =
If y e - ax +, then show that
d3y a2
Example a3 y -1/2 -3/2 -3/2
0

dx  
3
dx d 1

( e )= e - ax . ( - ax )= e - ax . ( - a )
2
Solution: As y= e - ax , so =
dy d - ax d
dx dx dx

That is
dy
dx
= -ay ( e - ax
= y)

version: 1.1 version: 1.1

72 73
1. Quadratic Equations eLearn.Punjab 1. Quadratic Equations eLearn.Punjab
2. Diferentiation eLearn.Punjab 2. Diferentiation eLearn.Punjab

EXERCISE 2.7 f ( x ) =a0 + a1 x + a2 x 2 + a3 x3 + a4 x 4 + a5 x5 + ...... + an x n + ..... f ( 0 ) =a0


f ' ( x ) = a1 + 2a2 x + 3a3 x 2 + 4a4 x3 + 5a5 x 4 + ...... + na x n 1 + ..... f ' ( 0 ) = a1
f '' ( x ) = 2a2 + 6a3 x + 12a4 x 2 + 20a5 x3 + ... + n ( n - 1) an x n-2 + ... f '' ( 0 ) = 2a2
1. Find y2 if

f ''' ( x ) =6a3 + 24a4 x + 60a5 x 2 + .... f ''' ( 0 ) = 6a3


y= 2 x - 3 x + 4 x + x - 2 (ii) = ( 2 x + 5) = x+
( x) ( 0 ) = 24a4
3/2 1
f(=
)
24a4 + 120a5 x ........ f(
4)
5 4 3
(i) y (iii) y 4
x

f '' ( 0 ) f ''' ( 0 ) f( ( 0)
4)
2. Find y2 if
= ( 0 ) , a1 f=
a0 f= ( 0 ) , a2
= =
 2x + 3 
So we have
y = x 2 . e- x y = ln 
'
, a3 , a4

 3x + 2  f n ( 0)
2! 3! 4!
(i) (ii)
Following the above pattern, we can write an =
n!
3. Find y2 if
(i) x2 + y 2 =
a2 (ii) x3 - y 3 =
a =
(iii) cosq , y a sin q
x a= Thus substituting these values in the power series, we have

(iv)=x at= x 2 + y 2 + 2 gx + 2 fy + c = f '' ( 0 ) 2 f ''' ( 0 ) 3 f ( ) ( 0 ) 4 f n ( 0) n


f ( x=
) f (0) + f (0) x + x + x + x + .... + x + ....
2
, y bt 4 (v) 0 '
4

y ln ( x 2 - 9 )
4. Find y4 if 2! 3! 4! n!

(i) y = sin 3 x (ii) y = cos3 x (iii) = This expansion of f ( x ) is called the Maclaurin series expansion.

- that -(1 x+2 ) =


The above expansion is also named as Maclaurin’s Theorem and can be stated as:
5. =
If x Sin q=
, y Sin mq , Show y2 xy1 m2 y 0 If f ( x ) is expanded in ascending powers of x as an ininite series, then

f '' ( 0 ) 2 f ''' ( 0 ) 3 f ( ) ( 0 ) 4 f n ( 0) n
= If y = - x, show
+ that f ( x=
) f (0) + f ( 0) x + x + x + x + .... + x + ....
4
d2y dy
6. e sin x
2 2y 0 '

( a 2 b2 ) y
2
dx dx 2! 3! 4! n!
= -= + that
+
d2y dy
Note that a function f can be expanded in the Maclaurin series if the function is deined

( ) ( )
7. If y e ax sin bx, show 2a 0
in the interval containing 0 and its derivatives exist at x = 0 .
2
dx dx
= If y= - -1 x -,prove- that 1 x 2 y2 xy1 2 0
2
8. Cos
The expansion is only valid if it is convergent.
If y = a cos (ln x) + b sin (ln x), prove that x 2 d y2 + x dy + y =
2
9. 0.

Expand f ( x ) = 1
dx dx

1+ x
Example 1: in the Maclaurin series.

at x 0 that is, f ( 0 ) 1 . Now we ind successive derivatives of f and


2.16 SERIES EXPANSIONS OF FUNCTIONS
= =
Solution: f is deined
A series of the form a0 + a1 x + a2 x 2 + a3 x3 + a4 x 4 + ...... + an x n + ..... is called a power series their values at x = 0 .
expansion of a function f ( x ) where a0 ,a1 ,a2 , ... an , ... are constants and x is a variable. f ' ( x) =
( -1)(1 + x ) f ' ( 0) =
and -
-2

f '' ( x ) =
( -1)( -2 ) (1 + x ) and f '' ( 0 ) (=- 1) 2
1,
-3
We determine the coeicient a0 , a1 , a2 , ..., an , ... to specify power series by inding 2

successive derivatives of the power series and evaluating them at x = 0 . That is, f ''' ( x ) =
( -1)( -2 ) ( -3)(1 + x ) and f ''' ( 0 ) (=- 1)
-4 3
3
version: 1.1 version: 1.1

74 75
1. Quadratic Equations eLearn.Punjab 1. Quadratic Equations eLearn.Punjab
2. Diferentiation eLearn.Punjab 2. Diferentiation eLearn.Punjab

f(
4)
( x) =
( -1)( -2 ) ( -3)( -4 )(1 + x ) and f (
4)
( 0 ) (=-
1)
f '' ( 0 ) 2 f ''' ( 0 ) 3 f ( ) ( 0 ) 4 f ( ) ( 0 )
-5

f ( x) =f ( 0) + f ( 0) x +
4
4
x + x + x + + ...,we have
4 5
'

Following the pattern, we can write f ( n ) ( 0 ) = ( -1)n n


2 3 4 5
-1 3 -1 7
sin x+= + .x +x +x +x +x + x+ +x
( 0) = ( 0) =
1, f '' ( 0 ) ( 1) 2 .
0 2 0 4 1 5 0 6
Now substituting f - -1, f ' =
2
0 1 ...
2 3 4 5 6 7
- f ''' (-0 ) =
( 1-)= ( 0 ) ( 1) 4,.... f (
3, f ( ) = ( 0 ) ( 1)
n)
=x - + - + ......
3 4 4 n
n in the formula. x3 x5 x7

f '' ( 0 ) 2 f ''' ( 0 ) 3 f ( ) ( 0 ) 4 f ( ) ( 0) x
3 5 7

f ( x) =
f ( 0) + f ( 0) x + x + x + + x = ... +
4 n
'
x ,...
2 3 4 n
Example 3: Expand ax in the Maclaurin series.

+
f(
n)
( 0 ) x x + ,... we have Solution: Let f ( x ) = a x , then
=
=
=f ' ( x ) a x ln a, f '' ( x ) a x ( ln a ) , f ''' ( x ) a x ( ln a )
n

= 1 + ( -1) x + ( -1) x + ( -1) x3 + ( -1)


( -1) n x n + ...
2 3

x + ... + f ( ) ( x ) a x( ln a ) , ..., f ( ( x ) a x ( ln a )( ) .
n

=
=
n)
1 2 2 2 3 3 4 4 4

1+ x
4 4 n

f ( x ) , f ' ( x ) , f '' ( x ) , f ''' ( x ) , f ( ) ( x ) , ... f ( ) ( x ) , we get


2 3 4 n
Putting x = 0 in
4 n

Thus, the Maclaurin series for 1 is the geometric series with the irst term 1 and
1+ x
common ratio -x. f ( 0=
) a=0 1, f ' ( 0=) a 0 ln =
a ln a, f '' ( 0=
) ( ln a ) , f ''' ( 0 ) ( ln a )
2 3

=
=f ( ) ( 0 ) ( ln a ) , ... , f ( ( 0 ) ( ln a )
4 4 n) n
.
= 1 , we have
a
1- r
Note: Applying the formula S Substituting these values in the formula

f '' ( 0 ) 2 f ''' ( 0 ) 3 f ( ) ( 0) n
1 - x + x2 - x=
3 + ... = f ( x=
) f ( 0) + f ( 0) x + x + x + ... + x + ..., we have
1 - (-x) 1 + x
n
1 1 '

( ln a ) ( ln a ) ( ln a )
2 3 n

=1 + ( ln a ) .x + + + ... + x n + ...
2 3 n
x 2 3
a x x
2 3 n
Example 2: Find the Maclaurin series for sin x
Note: If we put a = e in the above expansion, we get

( x ) sin x. Then
Solution: Let f = = f (=
0 ) sin 0 0.
e x =1 + x + + + ... + + . .. ( In e = 1)
f ' ( x) =
cos x and f ' ( 0 ) - 1 ; f '' (-x )= x and f '' ( 0 )
x 2 x3 xn
= cos 0 = =
f ''' ( x ) =- cos x and f ''' ( 0 ) =-cos 0 =-1 ; f ( ( x ) =-( -sin x )
sin sin 0 0; 2 3 n
=-sin x
4)

( 0 ) sin ( 0 ) 0.
Replacing x by 1, we have
and f = = ( 4)

e =1+1 + + + ... +
f ( ) = ( x ) = cos x and f ( ( 0 )= cos 0= 1, f ( ( x )= - sin x
5) 6)
1 1 1
5

f ( ) ( 0) = ( 0-)
2 3 n
and= 0; f( ) cos x and f ( =-
6 7 7)
1
Putting these values in the formula
version: 1.1 version: 1.1

76 77
1. Quadratic Equations eLearn.Punjab 1. Quadratic Equations eLearn.Punjab
2. Diferentiation eLearn.Punjab 2. Diferentiation eLearn.Punjab

f ' ( x) = 2 ( x +a )
+ a1 2a- 3a-3 ( x +a ) 4(x +
4a- a ) + ... na-
n (x a+)
n -1
Example 4: Expand (1 + x)n in the Maclaurin series. 2 3

f '' ( x )= 2a2 + 6a3 ( x - a ) + 12a4 ( x - a ) + ... + n ( n - 1) an ( x - a )


...

Solution: Let f ( x =
) (1 + x ) , then + ...
2 n-2

f ''' ( x ) = 6a3 + 24a4 ( x - a ) + ......


n

f ' (=
x ) n (1 + x ) f '' ( x ) = n ( n - 1) (1 + x )
n -1 n-2

f ''' ( x ) =n ( n - 1)( n - 2 )(1 + x ) ( x ) =n ( n - 1)( n - 2 )( n - 3)(1 + x ) f '' ( a )


,
,f(
4)

Putting x = a , we get f ' ( a =


) a1 ; f '' ( a =) 2a2 ⇒ a2= =; f ''' ( a ) 6a3
n -3 n-4

Putting x = 0 , we get
f ( 0 ) =(1 + 0 ) = ( 0 ) =n (1 + 0 ) =n , f ''' ( a )
2
n -1

⇒ a3 =
n '

f '' ( 0 ) = n ( n - 1)(1 + 0 ) = n ( n - 1)
1, f
n-2
3
f ''' ( 0 ) = n ( n - 1) ( n - 2 )(1 + 0 ) = n ( n - 1)( n - 2 ) ,
n -3

f(
4)
( 0 ) = n ( n - 1)( n - 2 )( n - 3)(1 + 0 )
n-4
= n ( n - 1)( n - 3) Following the above pattern , we have
f( )
(a)

Substituting these values in the formula


Substituting the values of a0 ,a1 ,a2 ,a3 ,..., , w e g e t
( 0 )+x 2 ( 0 )+x3
f ( x) =
f (+0 ) ( 0 )+. x f '' ( a ) f ''' ( a )
f ( x=
) f ( a ) + f ' ( a )( x - a ) + ( x - a) + ( x - a ) + ...
'' '''
' f f
f ... , we have 2 3

n ( n - 1) 2 n ( n - 1)( n - 2 ) 3
2 3

(1 + x )
2 3
(a)
+
= 1 n+. x +x x + ...
f(
n)

( x - a)
n

+ + ...
2 3 n

2.17 TAILOR SERIES EXPANSIONS This expansion is the Taylor series for f at x = a . The expansionisonly valid if it is
OF FUNCTIONS: convergent .
If a = 0, then the above expansion becomes

f '' ( 0 ) 2 f '' ( 0 ) 3 f ( ) ( 0) n
f ( x=
) f ( 0) + f ' ( 0) x +
If f is deined in the interval containing ' a' and its derivatives of all orders exist at
x = a , then we can expand f ( x ) as x + x + ... + x + ...
n

2 3 n

f ( x )= f ( a ) + f ' ( a )( x - a ) +
(a) (a)
( x - a) + ( x - a) which is the Maclaurin series for f at x = a .
'' '''
f f
Replacing x by x + h and a by x , the expansion in (A) can be written as
2 3

(a) (a)
2 3
f(
4)
f(
n)

+ ( x - a) + ... + ( x - a) + ...
f '' ( x ) 2 f ''' ( x ) 3 f ( ) ( x) n
f ( x + h=
) f ( x) + f ' ( x) h +
4 n

h + h + ... + h + ... (B)


n
4 n

f ( x ) = a0 + a1 ( x - a ) + a2 ( x - a ) + a3 ( x - a ) + a4 ( x - a ) + ...
2 3 n
2 3 4
Let
+ an ( x - a ) + ...
The expansions in (B) is termed as Taylor’s Theorem and can be stated as: If x and h
are two independent quantities and f ( x + h ) can be expanded in ascending power of h as
n

Obviously f ( a ) = a.0 (  putting x = a , all other terms vanish )


an ininite series, then
version: 1.1 version: 1.1

78 79
1. Quadratic Equations eLearn.Punjab 1. Quadratic Equations eLearn.Punjab
2. Diferentiation eLearn.Punjab 2. Diferentiation eLearn.Punjab

f '' ( x ) 2 f ''' ( x ) 3 f ( ) ( x) n Now taking the successive derivative of sin x and evaluating them at p , we have
f ( x + h=
) f ( x) + f ' ( x) h + h + h + ... + h + ...
n

6
p  p
( x)
2 3 n
= cos x f '= =
  cos
3
6
'
f and
6 2

f '' ( x ) = - sin x p  p -1
Example 1: Find the Taylor series expansion of In (1 + x) at x = 2.
and f '' -  -
= =
6
sin
Solution: Let f ( x ) = ln (1 + x ) , then f ( 2 )= ln (1 + 2 )= ln 3
6 2

p  p
Finding he successive derivatives of ln (1 + x ) and evaluating them at x = 2 ( x) = -cos x and f ''' -  -==
3
6
'''
f cos
6 2

f ' ( x) = ( 2) 4 p  p 1
and f '= = f(
4)
( x ) =- ( - sin x ) =sin x and f ( ) = =
1 1 1
1+ x 1+ 2 3   sin
6
f '' ( x-) =
( 1+) (1 x ) and f '' ( 2 ) =- (1 + 2 ) =
6 2
-
-2

Thus the Taylor series expansion at a = p is


1
9
f ''' ( x ) =-
( 1)( -2 )(1 + x ) and f ''' ( 2 ) = 2 . (1 + 2 ) =
-3 -3 2 6

p  -2  p - 2  p
27

( x) =( -1) ( -2 )( -3)(1 + x ) ( -1) 3(1 + x ) and f ( 2)


3
1 3
( 4)
= ( 4)
= - sin x = + x- + x-  +  x -  + ...
-4 -4
2 3
3 1
2  6 2  6 3  6
3
f
81 2

The Taylor series expansions of f at x = a is 3 p 1  p 3 p


= + x- - x-  -  x -  + ....
2 3
1
2  6  2 2 6 2 3 6
f '' ( a ) f ''' ( a ) = ( 31
- 0 300 ) =≈10 .017455
p
2

f ( x=
) f (a) + f (a) .( x - a) + ( x - a) + ( x - a ) + ...... For x =-
310 , x
' 2 3
6

( .017455) - ( .017455) - ( .017455)


2 3
sin 310 ≈ +
1 3 1 2 3 3
Now substituting the relative values, we have 2 2 4 12
≈ .5 + .015116 - 0.000076 ≈ .5150
- -
ln (1 + x ) = ln 3 + ( x - 2 ) + 9 ( x - 2 ) + 27 ( x - 2 ) + 81 ( x - 2 ) + ....
1 2 3
1 2 3 4

x - 2 ( x - 2) ( x - 2 ) - ( x - 2 ) + .... x  
3 2 3 4
Prove that e x += e 1 + h + +
h2 h3
+ ....
- +
2 3 4

 
Example 3: h
= ln 3 + 2 2 3
1.3 2.3 3.33 4.34

Example 2: Use the Taylor series expansion to ind the value of sin 310. =
Solution: Let f ( x=
+ h ) e x + h , then f ( x ) e x ...(i)

Solution: We take a = 30° = p


f ' ( x ) e= , f '' ( x ) e= , f ''' ( x ) e x
By successive derivatives of (i) w.r.t ‘x’ we have
6 = x x
etc .
p  p
( x ) sin x, then
Let f= =f  =  sin
1
6
By Taylor’s Theorem we have
6 2
version: 1.1 version: 1.1

80 81
1. Quadratic Equations eLearn.Punjab 1. Quadratic Equations eLearn.Punjab
2. Diferentiation eLearn.Punjab 2. Diferentiation eLearn.Punjab

2.18 GEOMETRICAL INTERPRETATION


f ( x + h )= f ( x ) + h f ' ( x ) + f '' ( x ) + + f ''' ( x ) + ...
h2 h3
2 3
OF A DERIVATIVE

Let AB be the arc of the graph of f deined by the equation y = f ( x ) .


Putting the relative values, we get

Let P ( x, f ( x ) ) and Q ( x + d x. f ( x + d x ) )
x+h
=
e +h e + e + e + ...
x h 2 x h3 x
x
e
2 3 be two
 
= e x 1 + h + + + ...
h 2 h3 neighbouring points on the arc AB where x ,
 2 3  x + d x ∈ Df .
The line PQ is secant of the curve and it makes
∠XSQ with the positive direction of the x -axis. (See
EXERCISE 2.8 the igure 2.21.1)
Drawing the ordinates PM , QN and
1. Apply the Maclaurin series expansion to prove that: perpendicular PR to NQ , we have

ln (1 + x ) = x - + - + ......
RQ = NQ - NR = NQ - MP = f ( x + d x) - f ( x)
x 2 x3 x 4
(i)

and PR = MN = ON - OM = x + d x - x = d x
2 2 2

=1 - + - + ......
x2 x4 x6
Thus tan m ∠XSQ = tan m ∠RPQ
(ii) cos x

f ( x + d x) - f ( x)
2 4 6

1+ x = 1+ - + + ...... = =
x x 2 x3
dx
(iii) RQ
2 8 16 PR
=1 + x + + + ......
x 2 x3
PQ1 , PQ2 , PQ3 ,... are shown in the igure 2.21.2. Points Qi ( i = 1,2,3,...) are getting closer and
Revolving the secant line PQ about P towards P, some of its successive positions
(iv) ex
2 3

=+
1 2x + + + ...... closer to the point P and PRi i.e; d xi (i = 1, 2, 3, ...) are approaching to zero.
2 3
4 x 8x
(v) e2 x
2 3 In other words we can say that the
revolving secant line approaches the tangent
line PT as its limiting position at P while d x
2. Show that:

cos ( x + h )= cos x - h sin x - cos x + sin x + ......


h2 h3 approaches zero, that is,
2 3 tan m ∠ XSQ → tan m ∠XTP when d x → 0
f ( x + d x) - f ( x)
and evaluate cos 61°.
→ tan m∠XTP as d x → 0
dx
( ln 2 ) ( ln 2 )
or
2 {1 + ( ln 2 )
= h+ + + ...} f ( x + d x) - f ( x)
2 3
x+h h2 h3
= tan m∠XTP
x
3. Show that 2
dx
2 3 so lim
d x →0

version: 1.1 version: 1.1

82 83
1. Quadratic Equations eLearn.Punjab 1. Quadratic Equations eLearn.Punjab
2. Diferentiation eLearn.Punjab 2. Diferentiation eLearn.Punjab

Find the equations of the tangents to the curve x 2 - y 2 - 6 y =


( x ) tan m ∠XTP
f '=
Example 2: 0 at the point

Thus the slope of the tangent line to the graph of f at ( x, f ( x ) ) is f ' ( x ) .


or
whose abscissa is 4.

Solution. Given that x 2 - y 2 - 6 y =


Discuss the tangent line to the graph of the function | x | at x = 0 .
0 (i)
Example 1:
We irst ind the y-coordinates of the points at which the equations of the tangents are to
be found. Putting x = 4 is (i) gives
f ( x) = | x |
16 - y - 6 y =
2
0 ⇒ y + 6 y - 16 =
2
0

f (=
0 ) |=
Solution: Let
-6 ± 36 + 64 -6 ± 100 -6 ± 10
= =
f (0 + d x) =
0 | 0 and, or y = , that is ,
| 0 + d x |=
| d x |, -6 + 10 4
2 2 2
-6 - 10 -16
y= = = 2
f ( 0 + d x ) - f ( 0) =
or y = = = -8
so |d x | - 0 2 2 2 2

f ( 0 + d x ) - f ( 0) | d x |
Thus the points are (4, 2) and (4, - 8).
=
dx dx
and Diferentiating (i) w.r.t. ‘ x ’ we have

dx
Thus f ' ( 0 ) = lim 2x - 2 y -6 = ⇒ 2 ( y + 3) = ⇒ =
dx
dy dy dy dy x
d x →0
dx y + 3
0 2x
dx dx dx
Because d x = d x when d x > 0 = =
4 4
d x = - d x when d x < 0 2+3 5
The slope of the tangent to (i) at (4, 2) = .
and
Therefore, the equation of the tangent to (i) at (4, 2) is

y-2 = ( x - 4) ⇒ 5 y - 10 = 4 x - 16
so we consider one-sided limits 4

dx dx 5=
y 4x - 6
5
= =
d x d x →0 d x
Lim Lim 1 or
d x →0 + +

dx -d x
= Lim- = - 1
d x d x →0 d x = -
4 4
-8 + 3
and Lim-
d x →0 The slope of the tangent to (i) at (4, - 8) =
5
Therefore the equation of the tangent to (i) at (4, - 8) is
dx
dx y - ( -8 ) =-
( x - 4)
The right hand and left hand limits are not equal, therefore, the Lim does not
d x →0 4
exist.
This means that f ' ( 0 ) ,the derivative of f at x = 0 does not exist and there is no tangent 5 y + 40 =
-4 x + 16 ⇒ 4 x + 5 y + 24 =
5
0
line to the graph of f and x = 0
(see the igure 2.21.3).

version: 1.1 version: 1.1

84 85
1. Quadratic Equations eLearn.Punjab 1. Quadratic Equations eLearn.Punjab
2. Diferentiation eLearn.Punjab 2. Diferentiation eLearn.Punjab

2.19 INCREASING AND DECREASING f ( x2 ) - f ( x1 ) < 0 ( x2 - x1 > 0 and x 2 + x1 < 0 )


FUNCTIONS ⇒ f ( x2 ) < f ( x1 )
⇒ f is decreasing on the interval ( -∞ ,0 )
Let f be deined on an interval (a, b) and let x1 , x2 ∈ ( a, b ) . Then
If x1 ,x2 ∈ ( 0 ,∞ ) and x2 > x1 , then
f ( x2 ) - f ( x1 ) > 0 ( x2 - x1 > 0 and x 2 + x1 > 0 )
(i) f is increasing on the interval (a, b) if f(x2) > f(x1) whenever x2 > x1

⇒ f ( x2 ) > f ( x1 )
(ii) f is decreasing on the interval (a, b) if f(x2) < f(x1) whenever x2 > x1

⇒ f is increasing on the interval ( 0, ∞ )

Here f ' ( x ) <2 x and f ' ( ∈


= x ) -∞ ( ,0 ) , therefore,
f is decreasing on the interval ( -∞ ,0 )
0 for all x

Also f ' ( x) > 0 x ∈ ( 0 ,∞ ) ,


( 0, ∞ ) .
for all so f is increasing on the interval

f ' ( x1 ) < 0 ⇒ f is decreasing at x1


From the above theorem we can conclude that
We see that a diferentiable function f is increasing on (a,b) if tangent lines to its graph

f ' ( x1=
) 0 ⇒ f is neither increasing nor decreasing at x1
1.
at all points (x, f(x)) where xd(a, b) have positive slopes, that is,
2.
f ' ( x1 ) > 0 ⇒ f is increasing at x1
and f is decreasing on (a, b) if tangent lines to its graph at all points ( x, f ( x ) ) where
f ’ (x) > 0 for all x such that a < x < b
3.

x ∈ ( a, b ) , have negative slopes, that is, f ' ( x ) < 0 for all x such that a < x < b
Now we illustrate the ideas discussed so far considering the function f deined as
f ( x=
) 4 x - x2
Now we state the above observation in the following theorem.
(I)
To draw the graph of f, we form a table of some ordered pairs which belongs to f
-1
Theorem:

y = f ( x)
x 0 1 2 3 4 5
-5 -5
Let f be a diferentiable function on the open interval (a,b). Then
f is increasing on (a,b) if f ' ( x ) > 0 for each x ∈ ( a,b )
0 3 4 3 0

(ii) f is decreasing on (a,b) if f ' ( x ) < 0 for each x ∈ ( a,b )


(i)

Let f ( x ) = x 2 , then
f ( x2 ) - f ( x1 ) = x2 2 - x12 = ( x2 - x1 )( x2 + x1 )
If x1 ,x2 ∈ ( -∞ , 0 ) and x2 > x1 ,, then
version: 1.1 version: 1.1

86 87
1. Quadratic Equations eLearn.Punjab 1. Quadratic Equations eLearn.Punjab
2. Diferentiation eLearn.Punjab 2. Diferentiation eLearn.Punjab

The graph of f is shown in the igure 2.22.1. i.e. 4 - 2 x > 0 ⇒ -2 x > - 4 ⇒x<2
Thus it is increasing in the interval ( -∞ , 2 ) . Similarly we can show that it is decreasing,
in the interval ( 2, ∞ ) .

Now we give an analytical approach to the above discussion.

Let f be an increasing function in some interval in which it is diferentiable. Let x and


x + d x be two, points in that interval such that x + d x > x .
As the function f is increasing in the interval, it conveys the fact that f(x + dx) > f(x).

Consequently we have, f ( x + d x ) - f ( x ) > 0 and ( x + d x ) - x > 0 , that is,


f(x + dx) - f(x) > 0 and dx > 0

f ( x + d x) - f ( x)
> 0
dx
or
The above diference quotient becomes one-sided limit

f ( x + d x) - f ( x)
dx
lim+
From the graph of f, it is obvious that y rises from 0 to 4 as x increases from 0 to 2 and d x →0
y falls from 4 to 0 as x increases from 2 to 4. As f is diferentiable, so f ‘ (x) exists and one sided limit must equal to f ‘ (x).
In other words, we can say that the function f deined as in (I) is increasing in the
interval 0 < x < 2 and is decreasing in the interval 2 < x < 4.
Thus f ‘ (x) > 0

The slope of the tangent to the graph of f at any point in the interval 0 < x < 2 , in which
Example 1: Determine the values of x for which f deined as f ( x ) = x 2 + 2 x - 3 is
the function f is increasing is positive because it makes an acute angle with the positive
(i) increasing (ii) decreasing.
direction of x-axis. (See the tangent line to the graph of f at (1, 3)).
(iii) ind the point where the function is neither increasing nor decreasing.
But the slope of the tangent line to the graph of f at any pointin the interval
2 < x < 4 in which the function f is decreasing is negative as it makes an obtuse angle with the
Solution: The table of some ordered pairs satisfying f ( x ) = x 2 + 2 x - 3 is given below:

As we know that the slope of the tangent line to the graph of f at ( x, f ( x ) ) is f ' ( x ) , so
positive direction of x-axis. (See the tangent line to the graph of f at (3, 3)).

-4 -3 -2 -1
the derivative of the function f i.e., f ' ( x ) , is positive in the interval in which f is increasing and
x 0 1 2
-3 -4 -3
f ' ( x ) , is negative in the interval in which f is decreasing.
y = f(x) 5 0 0 5

The function f under consideration is actually increasing at each x for which f ' ( x ) > 0 .

version: 1.1 version: 1.1

88 89
1. Quadratic Equations eLearn.Punjab 1. Quadratic Equations eLearn.Punjab
2. Diferentiation eLearn.Punjab 2. Diferentiation eLearn.Punjab

( x - 1) ( x - 3) > 0 ‘ in the intervals ( -∞ ,1) and ( 3,∞ )


The graph of f is shown in the igure2.22.2.
f ' ( x=
) 2x + 2
f ' ( x) < 0 ⇒ ( x - 1)( x - 3) < 0
The condition f ' ( x ) > 0 ⇒ 2x + 2 > 0
( x - 1)( x - 3) < 0 if x > 1 and x < 3 that is 1 < x < 3
(i)
⇒ 2x > -2
which gives x > -1 , so the function f deined as
f ( x ) = x 2 + 2 x - 3 is increasing in the interval ( - 1,∞ ) . 2.20 RELATIVE EXTREMA

(ii) And the condition f ' ( x ) < 0 ⇒ 2 x + 2 < 0 Let ( c - d x, c + d x ) ⊆ D f , , (domain of a function f), where
d x is small positive number.
⇒ 2 x < -2
If f ( c ) ≥ f ( x ) for all x ∈ ( c - d x, c + d x ) then the function
which gives x < - 1 , so the function f under
f is said to have a relative maxima at x = c .
Similarly if f ( c ) ≤ f ( x ) for all x ∈ ( c - d x, c + d x ) , then
consideration in the example I is decreasing in the
interval ( -∞ ,-1) .
the function f has relative minima at x = c .
(iii) The function is neither increasing nor decreasing where f ' ( x ) = 0 , that is, Both relative maximum and relative minimum are
2x + 2 = ⇒x= -1.
- 1 then f ( -1) =-
( 1) + 2 ( -1) - 3 =-4 . Thus f is neither increasing nor deceasing at
0 called in general relative extrema.
If x = The graph of a function is shown in the adjoining igure.
It has relative maxima at x = b and x = d . But at x = a and
2

x = c , it has relative minima.


the point (-1, -4).
Any point where f is neither increasing nor decreasing is called a stationary
Note that the relative maxima at x = d is not the highest point of the graph.
Note:
point, provided that f ‘ (x) = 0 at that point.

2.21 CRITICAL VALUES AND


f ( x ) =x3 - 6 x 2 + 9 x
Example 2: Determine the intervals in which f is increasing or it is decreasing if
CRITICAL POINTS

f ' ( x ) = 3 x 2 -12 x + 9
If c ∈ Df and f ' ( c ) =
0 or f ' ( c ) does not exist, then the number c is called a critical value

= 3 ( x 2 - 4 x + 3)
Solution.
for f while the point (c. f(c)) on the graph of f is named as a critical point.

=3 ( x - 1)( x - 3) Note: There are functions which have extrema (maxima or minima) at the points
where their derivatives do not exist. For example, the derivatives of the function f and f
f ' ( x) > 0
deined as.
⇒ x2 - 4 x + 3 > 0
⇒ ( x - 1)( x - 3) > 0

version: 1.1 version: 1.1

90 91
1. Quadratic Equations eLearn.Punjab 1. Quadratic Equations eLearn.Punjab
2. Diferentiation eLearn.Punjab 2. Diferentiation eLearn.Punjab

f ( x) = x We note that f ' ( x ) > 0 before x = 0 , f =


' ( x ) 0=
at x < 0 and f ' ( x ) 0 after x = 0 .
The graph of f shows that it has relative maxima at x = 0.
2 - x x > 0
and f ( x ) =  = at x c if f ' ( x ) > 0 , before
2 + x x ≤ 0
f ' ( c ) 0 and f ' ( x ) < 0 after x = c.
Thus we conclude that a function has relative maxima
=x c=
do not exist at (0, 0) and (0, 2) respectively.
But f has minima at (0, 0) and f has maxima at
Considering an interval (2 - dx, 2 + dx) in the neighbourhood of x = 2 we ind the values
of f ‘ (2-e) and f ‘ (2 + e) when 2 - ed(2 - dx, 2) and 2 + ed(2, 2 + dx)
(0, 2). See the adjoining igures.

f ' ( x ) = 0 are called stationary points of f.


Those critical points on the graph of f at which

f ' ( 2 - e )= 3 ( 2 - e )( 2 - e - 2 ) ' ( x ) 3 x ( x - 2 ) 
 f =
= 3 ( 2 - e )( -e )
Now we discuss relative maxima and relative

-3e ( 2 - e ) < 0 ( e > 0, 2 - e > 0 )


minima of the diferentiable function f deined as:
y =f ( x ) =x3 - 3 x 2 + 4 ....(1) =
f ' ( 2 + e )= 3 ( 2 + e )( 2 + e - 2 )
= 3e ( 2 + e ) > 0 ( e > 0, 2 + e > 0 )
and
Graph of f is drawn with the help of some ordered pairs tabulated as below:
-3/2 -1 -1/2
We see that f ' ( x ) < 0 before x = 2, f ' ( x ) = 0 at x = 2 and f ' ( x ) > 0 after x =
X 0 1/2 1 3/2 2 5/2 3
Y -49/8 0 25/8 4 27/8 2 5/8 0 7/8 4 2.

Now diferentiating (i) w.r.t. ' x' we get It is obvious from the graph that it has relative minima at x = 2 .
f ' ( x ) = 3x 2 - 6 x = 3x ( x - 2 ) = at x c if f ' ( x ) < 0 before
f ' ( x) = 0 ⇒ 3x ( x - 2 ) = c, f ' ( x) = c and>f ' ( x ) 0 =
Thus we conclude that a function has relative minima

= x 0 or= x= =
Now we consider an interval ( -d x , d x ) in the neighbourhood of x = 0 . Let 0 - e is a
0 x 2 0 at x after x c.

point in the interval ( -d x,0 ) We see that


Let f be diferentiable in neighbourhood of c where f ' ( c ) = 0.
First Derivative Rule:

f ' ( 0 - e ) = 3 ( -e ) ( -e - 2 ) ( f =
' ( x) 3x ( x - 2 ) )
= 3e ( e + 2 ) > 0 ( e > 0, e + 2 > 0 ) If f ' ( x ) changes sign from positive to negative as x increases through c, then
f ( c ) the relative maxima of f.
1.

That is f ' ( x ) is positive for all x ∈ ( -d x, 0 ) .


Let 0 + e1 is a point in the interval ( 0, d x ) , then we have If f ' ( x ) changes sign from negative to positive as x increases through c, then
f ' ( 0 +=
e1 ) 3 ( e1 )( e1 - 2 )
f ( c ) is the relative minima of f.
2.

-=3e1-( 2 e<1 ) 0 ( 2 - e1 > 0, e1 > 0 ) , that is,

f ' ( x ) is negative for all x ∈ ( 0 ,d x )

version: 1.1 version: 1.1

92 93
1. Quadratic Equations eLearn.Punjab 1. Quadratic Equations eLearn.Punjab
2. Diferentiation eLearn.Punjab 2. Diferentiation eLearn.Punjab

Examine the function deined as


f ( x ) =x 3 - 6 x 2 + 9 x for extreme values.
Note: 1. A stationary point is called a turning point if it is either a maximum point or Example 1:

f ' ( x ) = 3 x 2 - 12 x + 9
a minimum point.

= 3 ( x 2 - 4 x + 3) = 3 ( x - 1)( x - 3)
2. If f ‘ (x) > 0 before the point x = a, f ‘ (x) = 0 at x = 0 and f ‘ (x) > 0 after x = 0, Solution:
then f does not has a relative maxima.

f ' ( x ) = 3 x 2 , that is,


See the graph of f (x) = x3. In this case, we have

f ' ( 0 - e ) =3 ( -e ) =3e 2 > 0


First Method
If x = 1 - e where e is very very small positive number, then
2

and f ' ( 0 + e ) = 3 ( e ) = 3e 2 > 0 ( x - 1)( x - 3) =(1 - e - 1)(1 - e - 3) =( -e )( -e - 2 ) =e ( 2 + e ) > 0 that is ,


f ' ( x ) > 0 before x =1. For x =
2

1 e , we have +
( x - 1)( x - 3) =(1 + e - 1)(1 + e - 3) =(e )( -2 + e ) =-e ( 2 - e ) < 0
The function f is increasing before x = 0 and also
it is increasing after x = 0.
Such a point of the function is called the That is, f ' ( x ) < 0 after x =
As f ' ( x ) > 0 before x = 1, f ' ( x ) =at x 1 and f ' ( x ) <0 after
1
0= =x 1
point of inlexion.

Thus f has relative maxima at x =1 and f (1) =-1 - 6 + 9 =4.


Let x= 3 - e , then
( x - 1)( x - 3) =( 3 - e - 1)( 3 - e - 3) =( 2 - e )( -e ) =-e ( 2 - e ) < 0
We have noticed that the irst derivative f ' ( x ) of a function changes its sign from
Second Derivative Test:

For x = 3 + e
That is f ‘(x) < 0 before x = 3.
positive to negative at the point where f has relative maxima, that is, f ‘ is a decreasing

(x - 1) (x - 3) = (3 + e - 1)(3 + e - 3)= (2 + e)(e) > 0


function in the neighbouring interval containing the point where f has relative maxima.

That is, f ' ( x ) > 0 after x =


Thus f '' ( x ) is negative at the point where f has a relative maxima.
As f ' ( x ) < 0 before x = 3 , f ' ( x ) at x = 3 and f ' ( x ) > 0 after x =3 ,
3.

But f ' ( x ) of a function f changes its sign from negative to positive at the point where f
x 3. and f (=
Thus f has relative minima at= 3) 3 ( 3) - 12 ( 3) +=
2

Second Method: f '' ( x ) = 3 ( 2 x - 4 ) = 6 ( x - 2 )


has relative minima, that is, f ’ is an increasing function in the neighbouring interval containing 9 0

f '' (1) =6 (1 - 2 ) =- 6 < 0 , therefore,


the point where f has relative minima.

Thus f '' ( x ) is positive at the point where f has relative minima.

f has relative maxima at x =1 and f (1) =(1) - 6 (1) + 9 (1)


Let f be diferential function in a neighbourhood of c where f ' ( c ) = 0 . Then
Second Derivative Rule 3 2

=1 - 6 + 9 = 4
f has relative maxima at c if f '' ( c ) < 0 .
f '' ( 3) =6 ( 3 - 2 ) =6 > 0 , therefore f has relative minima at x = 3 and f ( 3) = 27 - 54 + 27 = 0
f has relative minima at c if f '' ( c ) > 0 .
1.
2.

version: 1.1 version: 1.1

94 95
1. Quadratic Equations eLearn.Punjab 1. Quadratic Equations eLearn.Punjab
2. Diferentiation eLearn.Punjab 2. Diferentiation eLearn.Punjab

Examine the function deined as f ( x ) = 1 + x3 for extreme values


f '' ( -x ) =- x ( cos×-2 x =
) 2- sin x
Example 2: 1
sin 2 cos 2 x

Solution: Given that f ( x ) = 1 + x3


2
p  p
f ''   =- sin - 2 cos p =-1 - 2 × ( -1) = 2 - 1 > 0
Diferentiating w.r.t. ' x' we get f ' ( x ) = 3 x 2 2
As

f ' ( x) = 0
2
 3p  3p
⇒ 3x 2 = ⇒x= and f ''   =- sin - 2 cos 3p =- ( -1) - 2 ( -1) =1 + 2 > 0
f '' ( x ) = 6 x ( 0 ) 6=
( 0) 0
0 0
and f ''=  2  2

p 3p
Thus f ( x ) has minimum values
= =
The second derivative does not help in determining the extreme values.
f ' ( 0 - e ) = 3 ( 0 - e ) = 3e >0
2 2
for x and x
2 2
f ' ( 0 + e ) = 3 ( 0 + e ) = 3e 2 > 0 p 
As f ''  - = sin-
p
- =-
p 1
-= 2<. 0
1
2

As the irst derivative does not change sign at x = 0 , therefore (0, 0) is a point 4
2 cos 0
4 2 2 2
 3p  3p 3p
of inlexion. and f ''  - = sin - - =- -= 2<. 0
1 1
 4 
2 cos 0

( x ) sin x +
4 2 2 2
Example 3: Discuss the function deined as f =
1
cos 2 x for extreme values in
the interval ( 0 , 2p ) . p 3p
Thus f ( x ) has minimum values
= =
2 2
for x and x

( x ) sin x +
4 4
Solution: Given that f =
1
cos 2 x
2 2
EXERCISE 2.9

f ' ( x ) =cos x + ( -2 sin 2 x ) =cos x -


1 1
sin 2 x 1. Determine the intervals in which f is increasing or decreasing for the domain
2 2 2
= ( 2 sin x -cos x ) cos x = -
f ( x ) = sin x x ∈ ( -p ,p )
1 mentioned in each case.

( )
cos x 2 sin x cos x
2 (i) ;
= cos x 1 - 2 sin x
 -p p 
f ( x ) = cos x x ∈ , 
 2 2
( )
;
f ( x )= 4 - x 2 x ∈ ( -2 ,2 )
(ii)

Now f ' ( x ) = 0 ⇒ cos x 1 - 2 sin x =


;
f ( x ) = x 2 + 3x + 2 x ∈ ( -4 , 1)
(iii)
0

p 3p
(iv) ;
⇒ cos x =
0 ⇒ x=,

p 3p
2 2
Find the extreme values for the following functions deined as:
f ( x ) = 1 - x3 f ( x) =
2.
1 - 2 sin x = ⇒ sin x = ⇒ x=, x2 - x - 2
1
or 0

f ( x ) = 5x2 - 6 x + 2 f ( x ) = 3x 2
4 4 (i) (ii)
2

f ( x ) = 3x 2 - 4 x + 5 f ( x ) = 2 x 3 - 2 x 2 - 36 x + 3
(iii) (iv)
Diferentiating (i) w.r.t. ‘ x ’ , we have
(v) (vi)
version: 1.1 version: 1.1

96 97
1. Quadratic Equations eLearn.Punjab 1. Quadratic Equations eLearn.Punjab
2. Diferentiation eLearn.Punjab 2. Diferentiation eLearn.Punjab

f ( x=
) x4 - 4 x2 f ( x) =( x - 2 ) ( x - 1)
2
= 3[ 2 x - 12] = 6 ( x - 6 )

f ( x ) = 5 + 3x - x3
(vii) (viii)

f '' ( 3) =6 (3 - 6) =6 ( -3) =
(ix)
As -18 which is negative.
Find the maximum and minimum values of the function deined by the following Thus f ( x ) gives the maximum value if x = 3 , so the other positive integer is 6 because
equation occurring in the interval [ 0 ,2p ]
3.
9 - 3 = 6.

( x ) sin x + cos x.
f=
Example 2: What are the dimensions of a box of a square base having largest
Show that y = has maximum value at x = e .
ln x volume if the sum of one side of the base and its height is 12 cm.
4.
x

Show that y = x x has a minimum value at x = . Solution: Let the length of one side of the base (in cm) be x and the height of the box (in
1
5.
e cm) be h, then V=x 2 h
It is given that x + h = ⇒ h = 12 - x
Thus V=x 2 (12 - x ) and
Application of Maxima and Minima 12
Now we apply the concept of maxima and minima to the practical problems. We irst

= 2 x (12 - x ) + x 2 ( -1)= 24 x - 3 x 2= 3 x ( 8 - x )
form the functional relation of the form y = f(x) from the given information and ind the
dV
maximum or minimum value of f as required. Here we solve some examples
dx

= 0 ⇒ 3 x ( 8 - x ) = 0 . In this case x cannot be zero,


relating to maxima and minima problems.
dV
dx
Example 1: Find two positive integers whose sum is 9 and the product of one with
the square of the other will be maximum. so 8 - x = 0 ⇒ x = 8.

= 24 - 6 x which is negative for x = 8


Solution: Let x and 9 - x be the two required positive integers such that
d 2V

Thus V is maximum if x = 8(cm) and h = 12 - 8 = 4(cm)


x ( 9 - x ) will be maximum.
dx 2
2

f (=
x) x ( 9 - x ) . Then
2
Let
f ' (=
x ) 1 . ( 9 - x ) + x . 2 ( 9 - x ) × ( -1)
Example 3: The perimeter of a triangle is 20 centimetres. If one side is of length 8
2 centimetres, what are lengths of the other two sides for maximum area of the triangle?

= ( 9 - x ) [9 - x - 2 x ] = ( 9 - x )( 9 - 3 x ) = 3 ( 9 - x )( 3 - x )
f ' ( x ) = 0 ⇒ 3 ( 9 - x )( 3 - x ) = 0 ⇒ x = 9 or x = 3
Solution: Let the length of one unknown side (in cm) be x , then the length of the other
unknown side (in cm) will be 20 - x - 8 = 12 - x .
In this case x = 9 is not possible because Let y denote the square of the area of the triangle, then we have
9 - x = 9 - 9 = 0 which is not positive integer.
y 10 (10 - 8 )(10 - x )(10 - 12 + x )
= 
=
s = 10 and area of the triangle s ( s - a )( s - b )( s - c ) )
f '' ( x ) = 3 ( -1)( 3 - x ) + ( 9 - x ) × ( -1)  = 3[ -3 + x - 9 + x ]
= 10.2 (10 - x )( x - 2 ) = 20 ( - x 2 + 12 x - 20 )
20
 2

version: 1.1 version: 1.1

98 99
1. Quadratic Equations eLearn.Punjab 1. Quadratic Equations eLearn.Punjab
2. Diferentiation eLearn.Punjab 2. Diferentiation eLearn.Punjab

= 20 ( -2 x + 12 ) =-40 ( x - 6 ) is negative for x = 5 because 12 ( 2 × 5 - 23)= 12 ( -13)


dy d 2V
dx dx 2
=0 ⇒x=
dy
6 Thus V will be maximum if the length of a side of the corner square to be cut of is 5 cm.
dx

is -ve,so x = 6 gives the maximum area of the triangle. Find the point on the graph of the curve y = 4 - x2 which is closest to
d2y
As Example 5:
The length of other unknown side = 12 - 6 = 6 ( cm )
dx 2
the point (3, 4).

Solution: Let l be distance between a point ( x, y ) on the curve y= 4 - x 2 and the point (3 ,
Thus the lengths of the other two sides are 6 cm and 6 cm.

Example 4: An open box of rectangular base is to be made from 24 cm by 45cm


4). Then l = ( x - 3) + ( y - 4 )
2 2

( x - 3) + ( 4 - x2 - 4) ( ( x, y ) is on the curve y= 4 - x2 )
cardboard by cutting out square sheets of equal size from each corner and bending the
=
2 2
sides. Find the dimensions of corner squares to obtain a box having largest possible

= ( x - 3) + x4
volume. 2

Solution: Let x (in cm) be the length of a side of each square sheet to be cut of from each
comer of the cardboard. Then the length and breadth of the resulting box (in cm) will be
45 - 2 x and 24 - 2 x respectively. Obviously the height of the box (in cm) will be x . Thus the
volume V of the box (in cubic cm) will be given by
V = x ( 24 - 2 x )( 45 - 2 x ) = 2 x (12 - x )( 45 - 2 x )
= 2 x ( 540 - 69 x + 2 x 2 )

= 2 1.( 2 x 2 - 69 x + 540 ) + x ( 4 x - 69 ) 
dV

( )
and
= 2 6 x 2 - 138 x + 540
dx

= 12  x 2 - 23 x + 90  = 12 ( x - 5 )( x - 18 )

=0 ⇒ 12 ( x - 5 )( x - 18 ) = =
⇒ x 5 or
=
dV Now we ind x for which l is minimum.
0 x 18

. ( 2 ( x 3) 4 x 3 ) 
dx

⇒x= 5 [ if x =18, then 12-x =12 - 18 =-6 , that is , = - +


dx 2. ( x - 3) + x 4
dl 1
2

.2 ( 2 x3 + x - 3)
V is negative which is not possible]
=
12 ( 2 x - 23)
1
=
2
d y 2l
dx 2

version: 1.1 version: 1.1

100 101
1. Quadratic Equations eLearn.Punjab 1. Quadratic Equations eLearn.Punjab
2. Diferentiation eLearn.Punjab 2. Diferentiation eLearn.Punjab

= ( 2 x 3 + x - 3)
1 6. Find the lengths of the sides of a variable rectangle having area 36 cm 2 when its
perimeter is minimum.

= ( x - 1) ( 2 x 2 + x - 3)
l
1
A box with a square base and open top is to have a volume of 4 cubic dm. Find the

= 0 ⇒ ( x - 1) ( 2 x 2 + 2 x + 3) =
l 7.

0 =
⇒ x - 1 0 or 2x 2 +
= 2x + 3 0
dl 1 dimensions of the box which will require the least material.

1 ( 2 x 2 + 2 x + 3 = 0)
dx l

⇒x=
8. Find the dimensions of a rectangular garden having perimeter 80 metres if its area

l is positive for 1 - e and 1+e where e is very very small positive real number.
is to be maximum.
9. An open tank of square base of side x and vertical sides is to be constructed to

 1 5  1 5
contain a given quantity of water. Find the depth in terms of x if the expense of lining
Also 2 x + 2 x + 3 = 2  x 2 + x +  + = 2  x +  + is positive,for x =1 -e
2

 4 2  2 2
2
the inside of the tank with lead will be least.

and x = 1 + e
10. Find the dimensions of the rectangle of maximum area which its inside the

depends on the factor x - 1 .


dl semi-circle of radius 8 cm as shown in the igure.
The sign of
x - 1 is negative for x = 1 - e because x - 1 = 1 - e - 1 = - e ..... (i)
dx

x - 1 is positive for x = 1 + e because x - 1 = 1 + e - 1 = e ..... (ii)

changes sign from -ve to +ve at x = 1.


dl
From (i) and (ii), we conclude that
dx
Thus l has a minimum value at x = 1.
Putting x= 1 in y= 4 - x 2 , we get the y-coordinate of the required point which
is 4 - (1) =
2
3

Find the point on the curve y = x2 - 1that is closest to the point (3, -1).
Hence the required point on the curve is (1, 3).
11.

EXERCISE 2.10
12. Find the point on the curve y = x2 + 1 that is closest to the point (18, 1).

1. Find two positive integers whose sum is 30 and their product will be maximum.
2. Divide 20 into two parts so that the sum of their squares will be minimum.
3. Find two positive integers whose sum is 12 and the product of one with the square
of the other will be maximum.
4. The perimeter of a triangle is 16 centimetres. If one side is of length 6 cm, what are
length of the other sides for maximum area of the triangle?
5. Find the dimensions of a rectangle of largest area having perimeter 120 centimetres.
version: 1.1 version: 1.1

102 103
version: 1.1

CHAPTER

3 Integration

Animation 3.1: Integration


Source and credit: eLearn.Punjab
1. Quadratic Equations eLearn.Punjab 1. Quadratic Equations eLearn.Punjab
3. Integration eLearn.Punjab 3. Integration eLearn.Punjab

3.1 INTRODUCTION 3.1.2 Distinguishing Between dy and dy.

When the derived function (or diferential coeicient) of a function is known, then The tangent line is drawn to
the aim to ind the function itself can be achieved. The technique or method to ind such the graph of y = f(x) at P(x, f(x) and
a function whose derivative is given involves the inverse process of diferentiation, called MP is the ordinate of P, that is,
anti-derivation or integration. We use diferentials of variables while applying method MP = f(x). (see Fig. 3.1)
of substitution in integrating process. Before the further study of anti-derivation, we irst Let dx be small number, then the
discuss the diferentials of variables. point N is located at x + dx’on the x-axis.
Let the vertical line through N cut the
tangent line at T and the graph of f at Q.
Then the point Q is (x + dx, f(x + dx)), so
3.1.1 Differentials of Variables

Let f be a diferentiable function in the interval a < x < b, deined as y = f(x), then dx = dx = PR
= d y f (x + d x) - f ( x) and dy = RQ = RT + TQ

dy f (x + d x) - f (x)  RT 
=
= f ′(x), that is = tan jdx + TQ  tanj = 
d x →0 d x dx  PR 
and lim lim
d x →0

= f ′( x ) where j is the angle which the tangent PT makes with the positive direction of the x-axis.
dy

dy dy = f ‘ (x)dx + TQ (∴ tan jdx = f ‘ (x))


dx
or
⇒ dy = dy + TQ
number e.
dx
We know that before the limit is reached, difers from f ‘ (x) by a very small real
We see that dy is the rise of f for a change dx in x at x where as dy is the rise of the
tangent line at P corresponding to same change dx in x.
where e is very small
dy
= f ′( x ) + e The importance of the diferential is obvious from the igure 3.1. As dx approaches 0,
dx
Let

d y = f ′ ( x )d x + e d x
the value of dy gets closer and closer to that of dy, so for small values of dx,

The term f ' ( x ) d x being more important than the term e dx, is called the diferential of
dy = dy
or (i)

or dy = f ‘ (x)dx [a dy = f ‘ (x)dx] (iv)


We know that dy = f(x + dx) - f(x)
the dependent variable y and is denoted by dy (or df)
Thus dy = f ' ( x ) d x (ii) f(x + dx) = f(x) + dy
=
As dx (=x)' d x (1)d x, so But dy c dy, so
the diferential of x is denoted by dx and is deined by the relation dx = dx. f(x + dx) c f(x) + dy (v)
The equation (ii) becomes f(x + dx) c f(x) + f ‘ (x)dx (vi)
dy = f ’ (x) dx (iii)

Note. Instead of dy, we can write df, that is, df = f ‘ (x) dx where f ‘ (x) being coeicient of
diferential is called diferential coeicient.
version: 1.1 version: 1.1

2 3
1. Quadratic Equations eLearn.Punjab 1. Quadratic Equations eLearn.Punjab
3. Integration eLearn.Punjab 3. Integration eLearn.Punjab

Example: Find dy and dy of the function deined as


1 y  x+ y 1 x+ y
dy = + =
  =
   dx
f(x) = x2 , when x = 2 and dx = 0.01 1
x x   x  x x 
or 2
dx 2
dx
x
x+ y
⇒ dy =   dx
Solution: As f(x) = x2, so f ‘ (x) = 2x
dy = f(x + dx) - f(x) = (x + dx)2 - x2  x 
x+ y
= 2x dx + (dx)2 = 2x dx + (dx)2 (a dx = dx) =  dy = f ' ( x ) dx 
dy
Thus f(2 + 0.01) - f(2) = 2(2) (0.01) + (0.01)
Thus
2 dx x
= 0.04 + 0.0001 = 0.0401, that is
dy = 0.0401 when x = 2 and dx = dx = 0.01
3.1.4 Simple application of differentials

Also dy = f ‘ (x) dx Use of diferentials for approximation is explained in the following examples.
= 2(2) x (0.01) = 0.04 (a f ‘ (x) = 2x, x = 2 and dx = 0.01)
Thus dy - dy = 0.0401 - 0.04 = 0.0001.
Example 1: Use diferentials to approximate the value of 17 .

dy Solution: Let f(x) = x


Then f (x + dx) = x + d x
3.1.3 Finding by using differentials
dx
We explain the process in the following example. As the nearest perfect square root to 17 is 16, so we take x = 16
and dx = dx = 1
when - In x = Inc
dy y
Example: Using diferentials ind Then y = f(16) = 16 = 4
Using f (x + dx) c f (x) + dy
dx x

Solution: Finding diferentials of both sides of the given equation, we get c f(x) + f ‘ (x) dx. we have

 1 
d  - lnx = d [ lnc ]= 0 f (16 + 1) ≈ f (16 ) + ( )
y  × (1) =
 
1
x   2 x
 f ' x
2 16

≈ 4 + = 4 + = 4.125
1 1
2× 4
using d(f ± g) = df ± dg, we have
8

d   - d ( ln x⇒
)=
 y d  1 17 ≈ 4.125
y.- =
dx  x 
1
x
0 . dx 0 Hence
x
Example 2: Use diferentials to approximate the value of 3
8.6
Using d(fg) = fdg + gdf, we get

1 1
yd   + dy - dx =
1 Solution: Let f (x) = 3
x then
x x
0

y + d y = f ( x + d x) = + dx (d x dx ) and f ' ( x )


x
 1  1 x + d x = 3=
x=
y ×  - 2 dx  + dy - dx = 0 ⇒ dy = dx + 2 dx
3 1
1 1 1 y
 x  x
2
x x x x 3x 3

version: 1.1 version: 1.1

4 5
1. Quadratic Equations eLearn.Punjab 1. Quadratic Equations eLearn.Punjab
3. Integration eLearn.Punjab 3. Integration eLearn.Punjab

As the nearest perfect cube root to 8.6 is 8, so we take x = 8 Solution: Let x be the side and V be the volume of the cube, then
and dx = 0.6, then V = x3 and dV = (3x2) dx
Taking x = 20 (cm) and dx = 0.12 (cm), we get

f (=
8) = 2 = (8)
f '= =2
dV = [3(20)2] (0.12) = 1200 x (0.12) = 144 (cubic cm)
3(8) 3
1 1 1
3× 4
3
8 and ,
12 The error 144 cubic cm in volume calculation of a cube is either positive or negative.

dy =f ' ( x ) dx = × ( 0.6 ) =0.05


1
so EXERCISE 3.1
f ( x + d x ) = f ( x ) + dy, we have
12

Find dy and dy in the following cases:


Using

f ( 8 + 0.6 ) = f ( 8 ) + 0.05
1.
(i) y = x2 - 1 when x changes from 3 to 3.02
+=
2 0.=
05 2.05 (ii) y = x2 + 2x when x changes from 2 to 1.8
But using calculator, we ind that 3
8.6 is approximately equal to 2.0488. (iii) y= x when x changes from 4 to 4.41

dy dx
Example 3: Using diferentials, ind the approximate value of sin 460 2. Using diferentials ind and in the following equations
dx dy

Solution: Let y = sinx, then (i) xy + x = 4 (ii) x2 + 2y2 = 16


y + dy = sin (x + dx) = sin (x + dx) (dx = dx) (iii) x4 + y2 = xy2 (iv) xy - lnx = c

p
3. Use diferentials to approximate the values of
We take x = 450 = and dx = 10 =0.01745 (i) 4
17 (ii) (31)1/5
4

 ( sin x ) = cos x 
 d 
(iii) cos 290 (iv) sin 610

 dx 
Hence dy = cos x dx 4. Find the approximate increase in the volume of a cube if the length of its each edge
changes from 5 to 5.02.

c ( cos 45° )( 0.01745 ) = ( 0.01745)


1 5. Find the approximate increase in the area of a circular disc if its diameter is ?
2

c 0.7071 (0.01745)
3.2 INTEGRATION AS ANTI - DERIVATIVE
c 0.01234
(INVERSE OF DERIVATIVE)
Using f (x + dx) c f(x) + dy we have
In chapter 2, we have been inding the derived function (diferential
sin (460) c sin 45° + dy c 0.7071 + 0.01234 = 0.71944
coeicient) of a given function. Now we consider the reverse (or inverse) process
c 0.7194
i.e. we ind a function when its derivative is known. In other words we can say that if
f’(x) = f(x), then f(x) is called an anti-derivative or an integral of f(x). For example, an
Using calculator, we ind sin 460 is approximately equal to 0.71934.

anti-derivative of f(x) = 3x2 is f(x) = x3 because f’(x) =


d 3
(x ) = 3x2 = f(x).
Example 4: The side of a cube is measured to be 20 cm with a maximum error of 12 cm dx
in its measurement. Find the maximum error in the calculated volume of the cube.
version: 1.1 version: 1.1

6 7
1. Quadratic Equations eLearn.Punjab 1. Quadratic Equations eLearn.Punjab
3. Integration eLearn.Punjab 3. Integration eLearn.Punjab

The inverse process of diferentiation i.e. the process of inding such a function whose
derivative is given is called anti-diferentiation or integration. ∫ cosec 2 ( ax + b ) dx = - cot ( ax + b ) + c ∫ cosec 2 xdx
- =+ x c
1
5.
While inding the derivatives of the expressions such as x2 + x, x2 + x + 5, x2 + x - 3
cot
a
∫ sec ( ax + b ) tan ( ax + b=
) dx sec ( ax + b ) + c ∫ sec xtan xdx =
+ sec x c
etc., we see that the derivative of each of them is 2x + 1, that is, 1
6.
a
(x + x - 3) = 2x + 1 7. ∫ cosec ( ax + b ) cot ( ax + b ) dx =
- cosec ( ax + b ) + c ∫ cosecx cot -x dx =
+
d 2 d 2 d 2 1
(x + x) = (x + x + 5) = cosec x c

∫ eλ x + µ dx = × eλ x + µ + c ( λ ≠ 0 ) ∫ e dx=
a
dx dx dx
ex + c
λ
1
8. x
Now if f(x) = 2x + 1 (i)
Then f(x) = x + x
∫ a=
λ x+ µ
.a λ x + µ + c.( a〉 0 ,a ≠ 1,λ ≠ 0 ) .a + c.( a〉 0 ,a ≠ 1)
2
∫ a x dx
=
λ ln a
1
9.
1 x
dx
is not only anti-derivative of (i). But all anti-derivatives of f(x) = 2x + 1 are included in ln a

∫ ax + b dx = ∫ (ax + b) ∫ dx = ln x + c,x ≠ 0
x + x + c where c is the arbitrary constant which can be found if further information is given.
2
-1
As c is not deinite, so f(x) + c is called the indeinite integral of f(x) , that is,
1 1
10. dx

∫ f (x) dx = Φ (x) + c
x
= ln ax + b + c,( ax + b ≠ 0 )
(ii) 1
a
∫ tan ( ax =
+ b ) dx ln sec ( ax + b ) + c ∫ tan xdx= ln sec ( x ) + c
In (ii), f(x) is called integrand and c is named as the constant of integration.

∫ .... dx indicates that integrand is to be integrated w.r.t. x.


1
11.
The symbol a
-= ln cos ( + ax b+) c
∫ .... dx are inverse operations of each other. - =
+ cos x
1
d ln c
Note that and a
∫ cot ( ax =
+ b ) dx ln sin ( ax + b ) + c ∫ cotx dx = ln sinx + c
dx
1
12.
a
∫ sec ( ax=
+ b ) dx ln sec ( ax + b ) + tan ( ax + b ) + c ∫ sec xdx= ln sec x + tan x + c
3.2.1 Some Standard Formulae for Anti-Derivatives 13. 1
a
∫ cos ec (=
ax + b ) dx ln co sec ( ax + b ) - cot ( ax + b ) + c ∫ cosec xdx
= ln cosec x - cot x + c
We give below a list of standard formulae for anti-derivatives which can be obtained 14. 1
from the corresponding formulae for derivatives: a

General Form Simple Form


In formulae 1-7 and 10-14, a ≠ 0
These formulae can be veriied by showing that the derivative of the right hand side of
each with respect to x is equal to the corresponding integrand.
( ax + b )
∫ ( ax=
+ b ) dx + c,( n ≠ -1) + c ( n ≠ -1)
n +1
x n+1
∫ x=
a ( n + 1) n +1
1.
n n
dx
Examples:

∫ sin ( ax + b ) dx = - cos ( ax + b ) + c
1
∫ sin xdx
- =+ x c ∫ x dx =
x5+1
5 +1
+c=
x6
+c
 d  1 6  1
 dx  =x 
 6  6
= ( x6 ) =
1 6-1

2. cos 1. 5
dx .6 x x5
a 6 6

∫ cos ( ax + b= ) dx sin ( ax + b ) + c ∫ cos xdx =


sin x + c
1
3.

∫ ∫=
 d  -2  1

 = -2 dx ( x ) 2 
- +1
3

=
a -
 dx 
∫ sec 2 ( ax + b= ) dx tan ( ax + b ) + c
3
1 x 2 d
∫ sec 2 xdx = +x c  x
2.
- +1  
2
1 dx x dx
4. tan x3 3
a 2
version: 1.1 version: 1.1

8 9
1. Quadratic Equations eLearn.Punjab 1. Quadratic Equations eLearn.Punjab
3. Integration eLearn.Punjab 3. Integration eLearn.Punjab

x2 + a2 + x 1 
× = 
 1  - 2 -1 x 2 + a 2 
-
1
-3
x + x2 + a2 x2 + a2
1

= + c =- +c =
-2. -  x =
= 1
2 1
x 2
-  2
x2
1 x x3
 d  
(+2 x 3) dx
2 3.2.2 Theorems on Anti-Derivatives
∫ dx∫ =   - 
-4

( 2 x + 3)  dx  6 ( 2 x + 3)3  
1 1
3.
  

( )
4

( 2 x + 3)= ( 2 x + 3)
I. The integral of the product of a constant and a function is equal to the product of the

( 2 x 3)
-4+1 -3

= +c -
+c =
+
2 ( -4 + 1)
1 d -3 constant and the integral of the function.
-6 In symbols,

∫ af ( x ) dx=
6 dx
a ∫ f ( x ) dx
- =
+ - ( -3)( 2 x + 3) ( 2 ) =
=
-3-1

6 ( 2 x + 3) ( 2 x + 3)
1 1 1 where a is a constant.
3
c 4
6
 d 1 
 dx  2 sin 2 x  = 2 dx ( sin 2 x ) 
 1 d
∫ cos 2 xdx
= =
+c sin 2 x + c
sin 2 x 1 II. The integral of the sum (or diference) of two functions is equal to the sum (or diference)
   
4. of their integrals.
2 2
= ( cos 2= x ) × 2 cos 2 x
In symbols,

∫  f ( x ) ± f ( x ) dx ∫ f ( x ) dx ± ∫ f ( x ) dx
1
2 =
- cos 3 x  d  1 

( cos 3x ) 
1 2 1 2

∫ sin 3 xdx = + c =- cos 3 x + c  dx  - 3 cos


- 3x  =
1 1 d
5.
   
Anti-Derivatives of [f(x)]n f ’(x) and [f(x)]-1 f ’(x)
3 3 3 dx
 
∫ cos ec xdx
- =+ x c  (- cot x) =-(- cosec 2 x) = cosec 2 x 
d 3.2.3
 dx 
6. 2
cot

 f ( x ) 
 d  sec 5 x   1
∫ sec 5 xtan 5 xdx =
+ =  = ×( sec 5 xtan 5 x ) 5 sec 5 xtan 5 x
∫  f
n +1

( x )= f ' ( x ) dx + 
 dx 
7. sec 5 x
  5  5 (n ≠ -1)
c

n +1
n
5 Prove that: (i)

( a ) e ax +b 
c,
 d  e ax +b  1 ax +b 
∫  f ( x ) f '= ( x ) dx ln f ( x ) + c,
e ax +b
∫e dx = +c   = e ×=
-1

ax +b
8. (ii) (f(x) > 0)
a  dx  a  a 

( 3λ x ( ln 3) λ ) 3λ x 
3λ x  d  3λ x  
∫ 3 dx =
λx
+c = =
  
λ ln 3  dx  λ ln 3  λ ln 3
1 Proof:
9.

d
(i) Since ([f(x)n+1) = (n + 1) [f(x)]n f ‘ (x)

(+ax b ) dx
 d 1
( )
1  ∴ by deinition, ∫ ( n + 1)  f ( x )  f ' =
( x ) dx  f ( x )  + c1
dx
n +1
∫ dx∫ =  = + = 
-1
 ax + b  
n
1 1 1
ax + b  dx  a a ax + b
10. 
( n + 1) ∫  f ( x ) f ' ( x ) dx =
+ f ( x ) 
ln ax b . .a
n +1

= ln ( ax + b ) + c,( ax + b > 0 )
n
1 c1 (by theorem I)

or ∫ [ f ( x)] f ' =
( )
[ f ( x)]n +1
(( ))
+c = ≠ -1 (n
a
c
 d   n +1 n +1
n

∫ dx = ln x + x 2 + a 2 + c
( x)dx where c 1)
 ln x + =
x2 + a2 1 + × 2x 
11. 1
x2 + a2
1 1
 dx x+ x +a  2 x +a
2 2 2 2

version: 1.1 version: 1.1

10 11
1. Quadratic Equations eLearn.Punjab 1. Quadratic Equations eLearn.Punjab
3. Integration eLearn.Punjab 3. Integration eLearn.Punjab

d 1

(ii) Since [In f(x)] = . f ‘ (x)
x 0+1
= x - x 2 - 3x + c ∫1 dx
∫ =x 0= + c2 
dx f(x) 1 3

dx

∫x x - 1 dx = ∫ ( x - 1) x dx
3 1
By deinition, we have 1

(>f ( x ) 0)
(ii)


2 2

. f ' ( x ) dx =
ln+f ( x ) c
2

f ( x)
∫   2 f ' ( x ) dx
( ) i (If×( f )( x=
1
=   ) x 2 - 1. =
1

or ∫ [ f ( x)]-1 f '=
f x
( x)dx I n f ( x) + c.
= ∫  f ( x )  2 f ' ( x ) then f ' ( x )= 2 x⇒ x= f ' ( x) )
1
1 1
Thus we can prove that 2 2

∫=
x n+1
+ c, (n ≠ -1)
1  f ( x )  2
( x + 1) 2 + c.
n +1
(i) n
x dx 3

= +=
3

( ax + b )
1 2

∫ ( ax + b ) =
c
n +1

+ c, (a ≠ 0, n ≠ -1)
3

a ( n + 1)
2 3
(ii)
n
dx 2
x+2-2
( x > -2 )
∫ x dx ln x + c
∫ dx = ∫
=
x
x+2 x+2
1 (iii) dx,
(iii)

∫  x + 2  dx = ∫ dx - 2 ∫ ( x + 2 ) .1 dx = x - 2 ln ( x + 2 ) + c
 2 
∫ ax + b= ln ax + b + c, (a ≠ 0) =  -
1 1 -1
(iv) dx 1
a

(x 0)
( )
∫ dx = ∫ >
1 1 1
x +1 x +1
Examples: Evaluate (iv) . dx

∫ ( x + 1)( x - 3) dx ∫x x - 1 dx
x x

∫  f ( x )

.2 f ' ( x ) dx =
f ' ( x) =if f ( +x )
(i) (ii)
=
2
-1 1

x 1

∫ x + 2 dx, ( x > -2 ) ∫ ( x > 0)


( )
2 x

= 2 ∫  f ( x )  f ' ( x ) dx

2 f ' ( x )
x 1
x x +1 =
(iii) (iv) dx, -1 1

( )

or

∫ ∫ cos2 x sin x dx
sinx + cos3 x
( x > 0) = 2 ln f ( x ) + c = 2 ln
x
x +1 + c
dx
x +1 - x
(v) , (vi)

∫ 1 + cos 2 x dx,
3 - cos 2 x
(vii) ( cos 2 x ≠ -1) ∫
dx
( x > 0)
x +1 - x
(v) ,

∫ ( x + 1)( x - 3) dx= ∫ ( x - 2 x - 3) dx
Solution: Rationalizing the denominator, we have
(i) 2

=∫ x dx - 2 ∫ x dx - 3∫1 dx ∫ =∫
x +1 + x
( )( )
dx
x +1 - x x +1 - x x +1 + x
2
(By theorems I and II) dx

 x n+1
= - 2. - 3.x + c  ∫ x dx = + c1 and
x3 x2
 n +1
n

3 2
version: 1.1 version: 1.1

12 13
1. Quadratic Equations eLearn.Punjab 1. Quadratic Equations eLearn.Punjab
3. Integration eLearn.Punjab 3. Integration eLearn.Punjab

EXERCISE 3.2
 
∫ ∫ 
x +1 + x
= dx= ( + )+  dx
1 1

x +1- x 
2
x 1 2 x
1. Evaluate the following indeinite integrals

∫+( x 1) dx ∫x
∫ ( 3x - 2 x + 1) dx
= +
∫ 
 1 
( x > 0)
1 1

x +  dx,
2 2
dx
x
(i) (ii)

∫ x( )
( x + 1)
2

+ c= ( x + 1) + x + c ( x > 0) ∫ ( 2x + 3) dx
3

= +
3

x + 1 dx,
2 x 2
2 3
2 23 1
2 (iii) (iv) 2

∫( )
3 3 3 3

∫  x -
 1 
( x > 0) ( x > 0)
2 2
x + 1  dx,
∫ cos 2 x sin x dx
sin x + cos 3 x
2
2

x
(v) dx, (vi)

y ( y + 1)
(vi)

∫ x dx, ∫
3x + 2
(vii) ( x > 0) (viii) dy, ( y > 0 )

( ) (1 - )
 sin x cos 3 x 
Solution: ∫ dx +∫ 
sin x + cos 3 x
y
=  dx q -1
 cos x sin x cos 2 x sin x 
∫ dq , (q > 0 ) ∫ ( x > 0)
2 2
x
q
cos 2 x sin x 2
(ix) (x)
= ∫
dx,
 1 cos x 
+  dx
x

∫ e x dx
e + ex
 
= ∫ sec x dx + ∫ cot x dx
2x
2
cos x sin x (xi)
2

=
tan+ x ln sin+x c 2. Evaluate

∫ 1 + cos 2 x dx, ( cos 2 x ≠ -1)


3 - cos 2 x  x + a > 0
∫ 1 + x 2 dx
1 - x2
∫  
(vii) dx
x + a + x + b x +b > 0
(i) (ii)

Solution: ∫ = =
4 - (1 + cos 2 x ) ∫ , ( x > 0 ,a > 0 ) ∫(a - 2 x ) dx
∫ 1 + cos- 2 x dx ∫  1 + cos 2 x 1 dx
3 - cos 2 x
3

 
dx
x +a +
(iii) (iv) 2

(1 + e )
4
1 + cos 2 x
x

= ∫ dx - ∫1= ∫ 2 sec x dx - ∫1 dx ∫ ∫ sin ( a + b ) x dx


x 3

4 2 (v) dx (vi)
dx ex
= 2 tan x - x + c
∫ 1 - cos 2 x dx, (1 - cos 2 x > 0 ) ∫ ( ln x ) × dx , ( x > 0 )
2 cos 2 x
1
(vii) (viii)
x
 p p
∫ sin x dx ∫ 1 + cos x dx, - < x < 
1
 2 2
(ix) 2
(x)

∫ ax ∫ cos3x sin2 x dx
ax + b
+ 2bx + c
(xi) 2
dx (xii)

(xiii) ∫ ∫ tan
cos 2 x - 1
dx, (1 + cos 2 x ≠ 0 ) (xiv)
1 + cos 2 x
2
x dx

version: 1.1 version: 1.1

14 15
1. Quadratic Equations eLearn.Punjab 1. Quadratic Equations eLearn.Punjab
3. Integration eLearn.Punjab 3. Integration eLearn.Punjab

3.3 INTEGRATION BY METHOD Example 3: Evaluate ∫x x - a dx , ( x > a)


OF SUBSTITUTION
Solution: Let x - a = t ⇒ x = a + t
Sometimes it is possible to convert an integral into a standard form or to an easy ⇒ dx = dt
integral by a suitable change of a variable. Now we evaluate ∫ f ( x ) dx by the method of Thus ∫x x - a dx = ∫ ( a + t ) t dt
substitution. Let x be a function of a variable t, that is,
 12 
= ∫  at + t  dt = a ∫ t 2 dt + ∫t
3 1 3
if
x = f(t), dx = f’(t) dt  
2 2
dt
then
Putting x = f(t) and dx = f’(t) dt, we have 3 5

∫ f ( x)dx = ∫ f (f (t )f '(t ) dt.


= a + + c= t + t + c
t 2
t 2
2a 32 2 52
3 5 3 5
Now we explain the procedure with the help of some examples. 2 2
= 2t 2  + t  + c = 2 ( x - a)2  + ( x - a)  + c
a 1  a 
∫ 2 at + b (at + b > 0 )
3 3
1
a dt 3 5  3 
( ) + = c 2 -x
Example 1: Evaluate 5
3  5a + 3 x - a 
= 2( x - a)2   ( a ) 2 ( 5+ 3-x 3a+)
3

 
a c
Solution: Let at + b = u. Then 15 15
= ( x - a ) 2 ( 2a + 3 x ) + c
3
a dt = du 2

Thus ∫ = ∫2 u 2∫
15
1 -21
=
adt du

∫ x dx ,
2 at + b
( x > 0) .
u du
cot x
Example 4: Evaluate

 - 1 +1   1 
1 u 2  1  u2  x = z,
=  1  + c =   + c = + c = at + b + c
1
Solution: Put

( x)
2 - +1 2 1 
2
u
 2   2  =
⇒dz
1
=


then d dx dz
x 2 x
4 + x2 dx = 2dz
Example 2: Evaluate dx. 1
or
x

∫ x dx ∫ cot x . x dx ∫ cot z.( 2dz )


Solution: Put 4 + x2 = t
==
cot x 1
thus
⇒= or =
=2 ∫ cot z dz 2∫ 2 ∫ ( sin z ) cos z dz
1
2 x dx dt x dx dt, therefore
=
=
2 cos z -1

∫ ∫ t  2 = 2∫
( z > 0 as x > 0 )
1 1
dz
1 -21
= = + . = 2 ln sin z + c,
x 1 t 1/ 2 sin z
4 + x2
dx dt t dt c

= +2 ln sin x
2 1/ 2
= t + c = 4 + x2 + c
c
version: 1.1 version: 1.1

16 17
1. Quadratic Equations eLearn.Punjab 1. Quadratic Equations eLearn.Punjab
3. Integration eLearn.Punjab 3. Integration eLearn.Punjab

∫ cosec x dx ∫ sec x dx  p p
∫ 1 + sin x dx ,  - < x < 
Example 5: Evaluate (i) (ii)

cos ec x ( cos ec x - cot x )  2 2


Solution: ∫ cos ec x dx = ∫
Example 7: Evaluate

cos ex x - cot x
t, then ( cosec x cot cosec 2 x )=
dx

Put cosec x cot x =- + x


∫ ∫+ 1 ∫ 1 - sin x dx
1 - sin x 1 - sin 2 x
1 + sin x dx = dx =
or cosec x ( cosec x - cot x ) dx =
dx dt
1 - sin x
Solution: sin x .


dt
=
cosec x ( cosec x - cot x )
cos x
1 - sin x
so ∫ ∫ t=
dx
= ln t + c
( cosec x - cot x )
1
dx dt
Put sin x = t, then cos x dx = dt, therefore

Thus cosec x dx = ln cosec x - cot x + c [ t = cosec x - cot x ]


∫ ∫ 1 - sin x .cos x dx-=
1 + sin x dx = ∫ 1- t =
∫ (1 t ) 2 dt
-
1
1 dt

sec x ( sec x + tan x )


∫ sec x dx = ∫ ( sec x + tan x ) (1 - t ) 2
( )
(ii) - +1

= +c = -2 1 -t +c
1
dx

sec x =
+ tan x t, then sec xtan x + sec
=
+ 1 ( - 1)
 1 
-
sec x ( sec x + tan x ) dx =
2
Put x dx dt
or dt  2 
-=2- 1 + x
sec x ( sec x tan x )
∫ ( sec x tan x ) = ∫ t=
sin c
ln t + c
∫ x ( ln 2 x ) ( x > 0)
so
Thus ∫ sec x dx = ln sec x + tan x + c
dx dt

( t = sec x + tan x )
dx
Example 8: Find 3
,

Evaluate ∫ cos x sin x dx , ( sin x > 0 ) .


Solution: Put In 2x = t, then
Example 6: 3

dx = dt
1 1
 1 
.2 dx = dt or
Put= =  2 sin x .cos x  dx
2x x
 
∫ ( ln 2 x ) ∫ t3 ∫t
Solution: sin x t, then dt
t -2
=
=  sin x t  . = = =
-3
+ c

1 1 1
-2
or 2t dt cos x dx Thus dx .dt dt
= =
3
x
Putting t and cos x dx 2t dt in the integral, we have,

∫ (1 - t ) . t × 2t dt, ( cos- ) =- + c =- + c
sin x

∫ cos 2 ( ln 2 x )
1 1
x sin x cos x dx = x =
1=
- sin x

= 2 ∫ ( t 2 - t 6 ) dt = 2 ∫ t 2 dt - 2 ∫ t 6 dt
2 4 2 2 4 2
1 t 2t 2

Find ∫ a x x dx , ( a > 0, a ≠ 1)
2
Example 9:
= - 2 + c
3 7
t t
x 2 = t, then x dx =
2. 1
3 7 Solution: Put dt

( sin x ) 2 ( sin x ) 2 = Thus ∫ a x x = ∫ a × 2 dt


= - + c sin x - sin 2 x + c
3 7 3 7 2
2 2 2 2
2 2
t 1
3 7 3 7 dx
version: 1.1 version: 1.1

18 19
1. Quadratic Equations eLearn.Punjab 1. Quadratic Equations eLearn.Punjab
3. Integration eLearn.Punjab 3. Integration eLearn.Punjab

2∫
3.4 SOME USEFUL SUBSTITUTIONS
= = += + c
2
1 t 1 at ax
a dt c
2 ln a 2 ln a
We list below suitable substitutions for certain expressions to be integrated.
Example 10: Evaluate Expression Involving Suitable Substitution
a2 - x2 x = a sin q
∫ (- a < x < a) ∫x dx, ( x > a or x < - a )
(i)
x2 - a2 x = a sec q
1 1
a - x x - a
(i) dx, (ii) (ii)
x = a tan q
2 2 2 2

(iii) a2 + x2
where a is positive.
(iv) x + a (or x - a) x + a = t (or x-a =
t)
Solution: (i) Let x = a Sin q, that is, (v) 2ax - x 2 a sin q
x - a =
p p 2ax + x 2 a sec q
x + a =
x = a Sinq for - <q < then dx =a cosq dq
(vi)
,

∫ ( a > 0)
2 2
a cos q dq
∫ ∫
1
= a + x
dx Example 1. Evaluate dx
a2 - x2 a 2 - a 2 sin 2 q
Thus 2 2

a cos q dq a cos q dq
=∫
= ∫ a cosq = a tan q for -
p
<q <
p
a 1 - sin q
Solution: Let x . Then

∫1 dq=
2

q + c
2 2
= dx = a sec2 q dq

x  x 
Thus
=
Sin  +  = Sin q 
-1

a  a  a sec 2 q dq
∫ ∫ ∫a
c
= × a sec q =
dq
1 1
a2 + x2 a 2 + a 2 tan 2 q 1 + tan 2 q
2
dx
p p
Put x = a Sec q i.e., x = a sec q for 0 < q < <q <p. a sec 2 q dq
=∫
= ∫ secq dq
(ii) or
Then dx = a sec q tan q dq a secq
2 2

secq ( secq + tanq )


a secq tanq d q = ∫ =dq ln ( sec +q tan q+)
∫x = ∫ a secq secq + tanq

(
dx c1
x2 - a2 a 2sec 2q - a 2
Thus

a ( sec q 1)
a secq tanq d q  a2 + x2 x
-∫
 a2 + x2
=ln  + +c1  sec q 1=+
tan q 1=+ 2 = 2
a secq .a tanq
x2
 a 

)
i.e.,

2 2 2 2



a a a
= 1 dq= .q + c = a 2 tan 2 = a tanq  a 2 + x 2 + x+ +  + a2 + x2
1 1
==ln  = + = +  c1secq c1secq q as secq is
 a a 
a a
  

) -) ln- aln +a c+ pos


 x

( (
= Sec -1+ q
 Sec= 
a a
p p p p 
1 x
 a
< << q< q  
c.
==ln x + +a 2 + x+2 - -for
a a
2 
c positive
1 1
2
version: 1.1 version: 1.1

20 21
1. Quadratic Equations eLearn.Punjab 1. Quadratic Equations eLearn.Punjab
3. Integration eLearn.Punjab 3. Integration eLearn.Punjab

( ( ) + c+wh ) + c=where- c =c
Evaluate the following integrals:
=ln x + =a 2 + x+2 - ln a

∫ ∫ (1 + x ) Tan ∫
1

(1 + x )
dx 1 1+x
x + a 2 + x 2 is always positive for real values of a.
-1
1 - x
9. 3
10. dx 11. dx
Note:
2
2 2 x

∫ (x> 0) sin q
∫ 1 + cos2 q dq ∫ ∫
dx
Example 2. Evaluate , ax dx
a2 - x4 7 - 6 x - x2
2x+ x 2 12. 13. 14.

∫ ∫ ∫ sin x ln ∫ cos x  sin x  dx


 ln sin x 
( x + 1)
dx dx cos x
2x + x -1
Solution:
2
=
2
15. dx 16.
sin x

 p ∫ 4 + 2 x + x2 ∫ x 4 + 2 x 2 + 5 dx
x dx x
x + 1 = sec q . Then < q <
17. 18.
Let  0
2 
∫ cos ∫
  x   1  x + 2
dx = sec q tan q dq  x -  ×  - 1 dx
 2   x  x + 3
19. 20. dx

sec q tan dq sec q tan dq


∫ ∫ ∫ = ∫ secq dq
( x + 1) ∫ sin x ∫1
tan q
dx
-1 sec 2 q - 1

( ) +c
Thus = =
2 dx
+ cos x
2
21. dx 22.
+ ln ( secq + tan q ) + c = ln x + 1 + 2 x
sin x +
3
2 cos x
x 1 2 2

EXERCISE 3.3
3.5 INTEGRATION BY PARTS
Evaluate the following integrals:
We know that for any two functions f and g.

∫ ∫ x + 4x +13 ∫ 4 + x 2 dx
- 2x
( x ) g ( x ) +f ′ ( x ) g ( x ) f ( x ) g ′ ( x )
2
dx x
4 - x2  f=
1. dx 2. 3.
dx 
2 d

∫ x ln x dx ∫ e x + 3 dx f ( x ) g′( x )  f ( x ) g ( x )  f ′( x ) g ( x )
1 ex
= -
4.
dx 
5. d
or

∫ ∫
x + b sec 2 x
(x + 2bx + c )
6. 1
dx 7. dx Integrating both the sides with respect to x, we get,
tan x

( f ( x ) g ( x ))
( )+c ∫ ∫  dx
( ) ( ) d
f ′ ( x ) g ( x )  dx

2

= ′ -
∫ x - a2 = ln x + x2 - a2 
dx f x g x dx
8.
= -∫  ∫ f ′ ( x ) g ( x ) dx
(a) Show that
 f ( x ) g ( x )   dx
d 
2

∫ a - x= Sin 1 + a - x + c  dx 
2 2 a x x 2 2
(b) show that dx
a a
version: 1.1 version: 1.1

22 23
1. Quadratic Equations eLearn.Punjab 1. Quadratic Equations eLearn.Punjab
3. Integration eLearn.Punjab 3. Integration eLearn.Punjab

= f ( x) g ( x) + c - ∫ f ′ ( x ) g ( x ) dx (By Deinition) ∫ x tan


2
Example 3. Evaluate

∫ f ( x ) g′( x ) ∫ g ( x ) f ′ ( x+) dx
x dx
f ( x ) g-( x )
=

∫ ∫ g ( x ) f ′ ( x ) dx ∫x = ∫ x ( sec x - 1) dx ( 1 + tan x = sec x )


i.e., c (i)
f ′ ( x=
) g ′ ( x ) dx - f ( x) g ( x)
2 2 2 2
Solution: tan x dx

= -∫ x sec ∫ x dx
or (i)

A constant of integration is written, when ∫ g ( x ) f ′ ( x ) dx is evaluated. The equation (i)


2
x dx
(I)
Integrating the ist integral by parts on the right side of (I), we get
or (i)’ is known as the formula for integration by parts.
 x2 
If we put u = f(x) and dv = g ’ (x) dx
∫x tan x dx= [x tan x - ∫ tan x . 1 dx] - 
 2
+ c1 

2

then du = f ‘ (x) dx and v = g(x).


 x2 
= x tan x-dx + ∫ cos x=
-. ( sin+x ) dx + - c  - x tan x + ln cos x
The equation (i) and (i)’ can be written as

∫ u dv =-uv ∫ v du
1 x2
 2 
+
c2 c1
2

=x tan x + ln cos x - + c, where c =c2 - c1


∫ u dv= uv - ∫ v du
c (ii)
x2
(ii)' 2

∫ x cos x dx.
Example 4. Evaluate ∫x
5

∫x ∫ ( ln x ) x
ln x dx
ln x dx =
Example 1. Find
5 5
Solution: dx

=( ln x ) ∫ 6 . x dx =6 ln x - 6 ∫ x dx
f(x) = x and g ‘ (x) = cos x,
-
Solution: If
x6 x6 1 x6 1
f ’(x) = 1 and g(x) = sin x
5

∫ x cos= ∫ ( sin x ) (1) dx


then
x sin x -
6
1  x6 
Thus
= x sin x - ( - cos x ) + c
= ln x -  + c1 
x dx
x6
6 6 
= +x sin x + cos x
6

= - -
c x 6
x 6
c1

∫x e
ln x + c where c =

( )
6 36 6

Evaluate ∫ ln x +
x
Example 2. Find dx
Example 5. x 2 + 1 dx

( )
Solution: Let
u=x and dv = e dx, x

Let f ( x ) = ln x + x 2 + 1 and g ′ ( x ) =
then du = 1 .dx and v = ex
Solution: 1. Then
Applying the formula for integration by parts, we have

∫x e x e x ∫ e x x 1 dx -= x e x
dx = -
f ′( x )
= × +

+
1 (x 1) 2 . 2 x 
-1 
x
ex + c 1 1 2 1

x + x2 + 1  2 
 
= + . 1 
1 x
x + x2 + 1  x2 + 1 

version: 1.1 version: 1.1

24 25
1. Quadratic Equations eLearn.Punjab 1. Quadratic Equations eLearn.Punjab
3. Integration eLearn.Punjab 3. Integration eLearn.Punjab

Find ∫ e ax cos bx dx .
 x2 + 1 + x 
and g ( x ) = x
Example 7.
= ×  =
 x 2 + 1 
1 1
x + x2 + 1  x2 + 1 Solution: Let f(x) = eax and g ’ (x) = cos bx

Using the formula ∫ f ( x)=


g ′ ( x ) dx f ( x ) g ( x ) - ∫ g ( x ) f ′ ( x ) dx, we get == f ′( x ) and g ( x )

(x + )
sin bx
then a . e ax

∫ ln ∫x .
b

Thus ∫ e ax cos bx dx× = ∫


x 2 + 1 . 1 dx = [ln( x + x 2 + 1)] .x -  sin bx   sin bx 
 - ×
1
x + 1    (ae ) dx

(x + )
dx
 b   b 
e ax ax

x + 1 x - ∫ ( x + 1) 2 ( 2 x ) dx
2

∫ ln
b∫
-
= - e ax sin bx
1
2 1 2 1 a ax

Integrating ∫ e ax sin bx dx , by parts, we get


e sin bx dx (I)

1  ( x + 1) 2
2
 2 
b


1

= x ln (x + x + 1) -  + c1 
∫ e sin bx dx =  - ∫ -
 cos bx   cos bx 
2

2  e ax ×  -  × (ae ) dx + c1
1
   b   b 

( )
ax ax
2
=x ln x + x2 + 1 - x 2 + 1 + c1 , where c =- c1 = - ∫ e ax cos+bx dx c1 (II) +
1 1 ax a
e cos bx
2 b b

Putting the value of ∫ e ax sin bx dx


∫x .a e
Example 6. Evaluate 2 ax
dx in (I), we get

∫e ∫e
a 1 
= e sin bx -  - e ax cos bx + + bx dx c1 
1 ax a
b b 
ax ax
Solution: If we put f(x) = x2 and g ’ (x) = a eax, then cos bx dx
b b
cos

= e sin bx + 2 e cos bx - 2 ∫ e ax cos bx dx -


f ’ (x) = 2x and g(x) = eax

∫ f ( x)=
g ′ ( x ) dx f ( x ) g ( x ) - ∫ g ( x ) f ′ ( x ) dx, we get
1 ax a ax a2 a
.c1
Using the formula b b b b
 a 2  ax
or 1 2  ∫ e cos bx dx = e ax sin bx + 2 e ax cos bx -
∫ x . ax = x 2 e ax - ∫ e ax .( 2 x ) dx
1 a a
 b 
2
dx
ax .c1

x 2e ax - 2 ∫ x e ax dx
b b b
= i.e. ∫ e ax=
b 2  1 ax
+ e sin bx

-e cos bx ×
a 2 + b 2  b 2
a ax b2 a
 a 2 + b2
cos bx dx 2
. c1
b b
 e ax   e ax 
∫ x e dx = x  a  - ∫  a  × 1 . dx [b sin bx + a cos bx-] + c,
b ( a 2 + b2 )
But = where c =
ax
e ax ab
a 2 + b2
c1

1  e ax 
= xe ax - ∫ e dx =a x e - a . a  + c1 a = r cos q b = r sin q;
1 1 1
ax ax
If we put and
then a 2 + b 2 = r 2 ⇒ r = a 2 + b 2
a a

∫x a e
1 
dx =x 2e ax - 2  .x e ax - 2 e ax + c1 
1
a  r sin q
Thus 2 ax

= = tan q ⇒ q= tan -1
a
r cosq
b b
=x 2e ax - .xe ax + 2 e ax + c1
2 2 a a
where c = 2c1
a a
version: 1.1 version: 1.1

26 27
1. Quadratic Equations eLearn.Punjab 1. Quadratic Equations eLearn.Punjab
3. Integration eLearn.Punjab 3. Integration eLearn.Punjab

and a cos bx + b sin bx = r cos q cos bx + r sin q sin bx


= r [cos bx cos q + sin bx sin q] = r cos (bx - q) = -∫ sin 2 x dx ∫ sin
2
x cos 2 x dx

= -∫ ∫
1 - cos 2 x
 b  b
=
+ a -  bx q
, = tan -1 
-1 dx sin 2 x cos 2 x dx (I)

Integrating ∫ sin 2 x cos 2 x dx by parts, we have


 a  a
2 2
b cos tan 2

∫ sin x cos 2 x dx = ∫ cos x sin 2 x cos x dx


The answer can be written as:

∫ e cos
 b
2

= e ax cos  bx - tan -1  + c
1
a 2 - b2  a
× ( - sin x ) dx
ax
bx dx
 sin x 
∫ 3
sin 3 x
= - cos x  
3
[  If f (x) = cos x and


 3 

( )
Example 8. Evaluate a 2 + x 2 dx g' (x) = sin 2 x cos x.

∫ a 2 + x 2 . 1 dx = a2 + x2 x - ∫x . ( + ) . 2 x dx = ∫ ..... (II) + then f ' (x) = -


1
1 2 2 2 1 1
Solution: a x cos x sin 3 x sin 4 x dx sin x
2 3 3
sin 3 x 
= x a +x - ∫ 3 
x2
a2 + x2
2
2 2
dx and g (x) = sin

Putting the value of ∫ sin 2 x cos 2 x dx in (I), we obtain,



a2 + x2 - a2
= x a +x -
a2 + x2
2 2
dx

∫ sin -∫  ∫
1 cos 2 x  1 
x dx = - dx +
∫ ∫  3 
1
= x a +x - a + x dx + 2 2 
4 3 4
a2 cos x sin x sin x dx
a + x
2 2 2 2
dx 3

∫ 2∫ ∫
2 2

= -1 dx cos 2-x - 3x
2 ∫ a 2 + x 2 dx= ∫
1 1 1 1
a2 + x2 + a2.
dx cos x sin sin 4 x dx

( )
1
a2 + x2
x dx 2 3 3

or 1 +  ∫ sin 4 x dx=
 1 1  sin 2 x 
= x a 2 + x 2 + a 2 ln x + a2 + x2 + c1  × -  + c-1
1 1
   3 2 2 
cos x sin 3 x
2 3

∫ sin x=
3 1 
 × - - cos x +sin 3 x c

( )
(See Example 1 Article 3.4) 1 1
4 2 
4
dx sin 2 x

∫ a + x dx = 2 a + x + 2 ln x + a + x + c, where c = 2
4 3
=
- x - 2x cos x + =
2 2 x 2 2 a2 2 2 a 2c1 3 3 1 3
sin 3 x

Similarly integrals ∫ a 2 - x 2 dx and ∫ x 2 - a 2 can be evaluated.


sin c where c c1

e x (1 + sin x )
8 16 4 4

Example 10. Evaluate ∫ dx .

Evaluate ∫ sin 4 x dx .
1 + cos x

 x
Example 9.
e x 1 + 2 sin cos 
e x (1 + sin x )
∫ 1 + cos x dx -∫
x
 2  

-∫ sin 2 x (1 cos 2 x ) dx
=

∫= ∫ sin x . sin
dx  1 + cosx = 1 + 2cos 2 1
Solution: 2 x
 
sin x dx =
2 x 2
Solution: 4 2 2
x dx 2cos
2

version: 1.1 version: 1.1

28 29
1. Quadratic Equations eLearn.Punjab 1. Quadratic Equations eLearn.Punjab
3. Integration eLearn.Punjab 3. Integration eLearn.Punjab

e (1 + sin x )
∫ 1 + cos x ∫ 2
EXERCISE 3.4
x1 x
= +   dx
x
2 x

2
i.e. dx e sec tan
1. Evaluate the following integrals by parts add a word representing all the
= + ∫ e x sec 2 dx ∫ e tan 2 dx
2
1 x x x
(I) functions are deined.
2 2

∫  tan ∫e ∫ x sin x dx ∫ ln x dx ∫ x ln x dx
 x x  x x  2 x 1
But  . e dx =
 tan -
 .e  sec  .+ dx c, (Integrating by parts)
2  2  2 2
(i) (ii) (iii)
x

∫ e tan
x
dx =
e x-tan
x 1
∫ e sec+
x (iv) ∫ x ln x dx (v) ∫ x ln x dx (vi) ∫ x ln x dx
2 3 4

∫ Tan x dx ∫ x sin x dx ∫ x Tan x dx


i.e. x x 2
dx c (II)
-1 -1

Putting the value of ∫ e x tan


2 2 2 2
(vii) (viii) 2
(ix) 2

∫ x Tan x dx ∫ x Tan x dx (xii) ∫ x cos x dx


x
dx in (I), we get -1 -1
(x) (xi) 3 3

e x (1 + sin x )
∫ Sin x dx ∫ x Sin x dx
2

∫ 1 + cos x = ∫ ∫
  -1 -1
e sec + dx e x tan
- e sec += +
dx c  e x tan
1 x 2x x 1 x 2x x (xiii) (xiv)

∫ e sin x cos x dx ∫ x sin x cos x dx


 
dx c
2 2 2 2 2 2
(xv) x
(xvi)

∫ x cos x dx ∫ x sin x dx
Show that ∫ e ax a f ( x ) + f ' ( x )  dx = e ax f ( c ) + c .
(xvii) 2
(xviii) 2

∫ (ln x) dx ∫ (ln ( tan x ) sec x dx


Example 11. 2
(xix) (xx) 2

∫e ∫ e . a f ( x ) dx + ∫ e . f ′ ( x ) dx ...(i)
 a f ( x ) + f ′ ( x )  =
In the second integral, let j ( x ) = e ax and g ′ ( x ) = f ′ ( x ) ,

Solution: ax
dx ax ax
x Sin-1 x
1 -x
(xxi) dx

( eax ) × a and g ( x ) = f ( x )
2

′( x )
j=

∫ e . f ′ ( x ) dx =e × f ( x ) - ∫ f ( x ) × ( ae ) dx + c
2. Evaluate the following integral.

∫ tan x dx ∫ sec x dx ∫e
then

so ax ax ax
(i) 4
(ii) 4
(iii) x

∫ tan x sec x dx ∫ x e dx
sin 2 x cos x dx

=-e ax f ( x ) ∫ a e f ( x+) dx c ∫e
-x
(iv) 3
(v) 3 5x
(vi)

∫ e cos3x dx ∫ cosec x dx
ax
sin 2 x dx

thus ∫ e ax  a f ( x ) + f ′ ( x )  dx
= ∫ ae ax f ( x ) dx + ∫ e ax f ′ ( x ) dx + c (vii) (viii)
2x 3

= ∫ a e f ( x ) dx + e ax f ( x ) - ∫ a e ax f ( x ) dx + c 
Show that ∫ e ax=
 b
- e ax sin  bx + Tan -1 
ax

e f ( x ) + c.
1
a 2 + b2  a
3.
=
sin bx dx c.
ax

4. Evaluate the following indeinite integrals.

(i) ∫ a 2 - x 2 dx (ii) ∫ x -a
2 2
dx

(iii) ∫ 4 - 5x 2 dx (iv) ∫ 3 - 4x
2
dx

(v) ∫ x 2 + 4 dx (vi) ∫ x e dx
2 ax

version: 1.1 version: 1.1

30 31
1. Quadratic Equations eLearn.Punjab 1. Quadratic Equations eLearn.Punjab
3. Integration eLearn.Punjab 3. Integration eLearn.Punjab

5. Evaluate the following integrals. EXAMPLES OF CASE I

∫e ∫ e ( cos x
1 
+ sin x ) dx
∫ 2x
-x + 6
 + ln x  dx
x 
( x > 2)
- 7x + 6
(i) x
(ii) x
Example 1: Evaluate 2
dx ,

∫e
   3 sin x - cos x 
∫e  a Sec x +  dx   dx
The denomicator 2x2 - 7x + 6 = (x - 2) (2x - 3),
-1 1
(iii) (iv)
 - 
ax 3x
Solution:
2 sin 2 x
-x + 6
x x 1

∫ e [ - sin x + 2cos x ] dx ∫ (1 + x ) =
( x - 2 )( 2 x - 3)
+
A B
x - 2 2x - 3
x ex Let
(v) 2x
(vi) 2
dx

-x + 6 = A(2x - 3) + B(x - 2) which is true for all x


∫ e ( cos x - sin x ) dx (viii) ∫
( )
-1 or
-x e m Tan x
+
(vii) dx Putting x = 2, we get
-2 + 6 = A(4 - 3) + B x 0 ⇒ A = 4
2

e x (1 + x )
1 x

∫ 1 - sin x dx ∫ (2 + x) A( 0) + B  - 2 
3 
2x
(ix) (x)
= , we get - + 6=
dx
2 3 3
2 
and Putting

∫  1 -
 1 - sin x  x
x

 e dx
2 2
cos x   1
(xi)
=B  -  ⇒ B =- 9
9
 2
or
2

∫ (x dx = ∫ 
-x + 6  4 -9 
+
- 2 )( 2 x - 3)
 dx
x - 2 2x - 3 
3.5 INTEGRATION INVOLVING Thus

=4 ∫ ( x - 2 ) 1 . dx -
2 ∫
( 2 x - 3) . 2dx
PARTIAL FRACTIONS -1
-1 9

If P(x), Q(x) are polynomial functions and the denominator Q(x)( ≠ 0), in the rational
= 4 ln ( x - 2 ) - ln ( 2 x - 3) + c, ( x > 2 )
9
P(x) 2
function ,can be factorized into linear and quadratic (irreducible) factors, then the rational
Q(x)

Evaluate ∫
2 x3 - 9 x 2 + 12 x
( x > 2)
function is written as a sum of simpler rational functions, each of which can be integrated by
2 x2 - 7 x + 6
methods already known to us. Example 2: dx,
Here we will give examples of the following three cases when the denominator Q(x)
contains Solution: After performing the division by the denominator, we get

∫ 2 x2 - 7 x + 6 ∫
2 x 3 - 9 x 2 + 12 x  -x + 6 
 - +  dx
2x - 7x + 6 
Case I. Non-repeated linear factors. dx = x 1 2

∫ x dx - ∫1 dx + ∫ (x ∫ 2x
-9
Case II. Repeated and non-repeated linear factors.
= dx +
- 2)
4
- 3
Case III. Linear and non-repeated irreducible quadratic factors or non repeated dx, (See the Example 1)
irreducible quadratic factors.

= - x + 4 ln (x - 2) - ( 2 x - 3 ) + c, ( x > 2)
x2 9
2 2
version: 1.1 version: 1.1

32 33
1. Quadratic Equations eLearn.Punjab 1. Quadratic Equations eLearn.Punjab
3. Integration eLearn.Punjab 3. Integration eLearn.Punjab

∫x ( x > a)  ( x - 1)-1 
2 ln ( x - 1) - ln ( x + 1)  + 3 
2a
- a2 = + 
Example 3: Evaluate (i) dx ,

 - 
2

∫ a 2 - x 2 dx, ( x < a)
c
1
 x - 1
2a
=  - + c
(ii)
2 ln 
3
 +  -
Solution: (i) The denominator x2 - a2 = (x - a)(x + a),
x 1 x 1

e x ( x 2 + 1)
( x - a )( x + a ) x - a
+
∫ (x + 1)
2a A B
x + a
Let = Example 5: Evaluate dx
2

= - e x ( x 2 + 1)  
∫ (x dx = ∫ e x 1 -
1 1
x - a x + a + 2 
(Applying the method of partial fractions)

( x + 1)
,
+ 1)  ( x + 1) 
Solution: 2 2

e x ( x 2 + 1)
dx, (By Partial Fractions)

∫ (x ∫  x ∫( x a ) -.1 dx + (x a)
2

 1 
= -  dx -=
- a )( x + a )
-1 -1

∫ (x dx = ∫ e dx - 2 ∫ ∫ (x
2a 1
- a x + a
Thus
⇒ dx + 2
dx . 1 dx

+ 1) + 1)
ex ex
x + 1
x

x - a
( x > a)
dx (I)
= ln x - a - ln x + a + = + c,
2 2

x + a
c ln
We integrate by parts the last integral on the right side of (I).
(ii) It is left as an exercise.
(x + 1)  ( x + 1)-1  x
∫ e ( x + 1) - ∫
-1

 . e dx
-2

-1  - 
x x

 
EXAMPLES OF CASE II dx = e .
1

∫(x (x 1) ∫ ( x + 1)2 - =+ ∫ x + 1 dx
ex ex ex
- 1) ( x + 1) x + 1
7 1
Example 4: Evaluate dx or dx (II)

Solution: We write Using (II), (I) becomes

e x ( x 2 + 1)
∫ ( x - 1)2 ( x + 1) dx = x - 1 +
7x - 1  
(x - 1)
+ ∫ (x + 1)
dx = ∫ e dx - 2 ∫ dx + 2  - + ∫ x + 1 dx 
A B C ex ex ex
x + 1 x + 1  +
x
2 2
x 1

= ( e + c ) - 2∫ + 2∫
 Applying the method 
= + -  of Partial Fractions  dx -
(x - 1)
ex 2e x ex
x + 1 x + 1 x + 1
2 3 2
x - 1 x + 1  
x
dx

e x ( x - 1)
2

 2  xe x + e x - 2e x
∫ ( x - 1)2 ( x + 1) ∫  x - 1 +
7x - 1 = e - + c = + c = + c.
2e x
-  dx x + 1 x + 1 x + 1
(x - 1)
Thus
x
3 2
 x + 1 
dx = 2

= 2 ∫ ( x - 1) .1 dx + 3∫ ( x - 1) .1 dx - 2 ∫ ( x + 1) .1 dx
∫ x 3 - 1 dx
-1 -2 -1

(x - 1)
1
Example 6: Evaluate
2 ln ( x - 1) + - 2 ln ( x + 1) + c ( x > 1)
-2+1

=
-2 + 1 x3 - 1 = (x - 1 )(x2 + x + 1),
3
Solution: The denominator
version: 1.1 version: 1.1

34 35
1. Quadratic Equations eLearn.Punjab 1. Quadratic Equations eLearn.Punjab
3. Integration eLearn.Punjab 3. Integration eLearn.Punjab

ln x 2 - 1 - ln ( x 4 + x 2 + 1) -
Bx + C  2 x2 + 1 
( x - 1) ( x 2 + x + 1) x - 1 x 2 + x + 1
+ = Tan -1  + 
1 A 1 1 1
 
Let = c
3 6 3 3

 1
- x -
∫ x( x - 1)
dx , x ≠ 0, x ≠ - 1
2
 3 , ( Applying the method of partial fractions )
1
= +
3
Example 8: Evaluate
x - 1 x + x + 1
3 3 3
2

= - .
Cx + D
1 1 1 x+2
3 x - 1 3 x + x + 1
( )
=
+ +
. 2 ,
3 A B
Solution: Let x x3 - 1 x - 1 x + x + 1

1 2x + 4 
( x - 1) ( x 2 + x + 1)
x
 -  dx -3 2x + 1
1 1 1
3 x - 1 6 x + x + 1
Thus
=+ +
dx = . . 1
x - 1 x + x + 1
2
(By the method of partial fractions)

∫  3 .
1 2x + 1 
2
x

∫ x ( x - 1) x 2 + x + 1 ∫  x x - 1 x 2 + x + 1  dx
= 1.dx - -  dx  -3 2x + 1 
( )
+ +
1 1 1 3
x - 1 6 x + x + 1 6 x + x + 1
. 2 . 2 3 1
Let dx =

= ∫ ( x - 1) dx - ∫(x + x + 1) ( 2 x + 1) dx - ∫
∫(x + x + 1)
-1 -1

=- 3∫ ( x ) 1 . dx + ∫(x - 1) 1 . dx + ( 2x + 1) dx
1 1 1 1
 1  3
2
dx
-1 -1 -1
x+  +  
2
3 6 2 2

(x + 1)
 2
2

 2 
= - 3ln x + ln x - 1 + ln + x + c
 1
= - 3ln x + ln x - 1 ( x 2 + x + 1) + c
2

+
ln x - 1 - ln ( x 2 + x + 1) -
 
= Tan -1  +2 
1 1 1 1 x

 3 
. c
  = - 3ln x + ln x3 - 1 + c
3 6 2 3
 2 

ln ( x 2 + x + 1) -
2
 2x + 1 
Evaluate ∫ 2
= ln x - 1 - +  2 x2 + 6 x
( x + 1) ( x + 2 x + 3 )
Tan -1 
1 1 1
 3 
c Example 9: dx
3 6 3

Solution: We write
Note: x2+ x + 1 is positive for real values of x.

2 x2 + 6 x Ax + B Cx + D
Evaluate ∫ 6
( x2 + 1)( x2 + 2 x + 3) x2 + 1 x2 + 2 x + 3
+
2x
x - 1
Example 7: dx Let =

2x + 1 2x + 3
Put x2 = t, then 2x dx = dt and - 2
x + 1 x + 2x + 3
Solution: = (Applying the method of partial fractions)
2

∫x dx = ∫ 3 ∫ (t - 1) ( t + t + 1) Thus ∫ ( x 2 + 1)( x 2 + 2 x + 3) ∫  x
2 x2 + 6 x  2x + 1 2x + 3 
- 2  dx
2x 1 1
- 1 t - 1
+ 1 x + 2x + 3 
6
dt = dx =

ln t - 1 - ln ( t 2 + t + 1) -
2
2

 2t + 1 
= +
∫x ∫x ∫x ∫x
Tan -1  2x + 2
1 1 1
 3  = dx + - dx -
c 2x 1 1
+ 1 + 1 + 2x + 3 + 2x + 3
3 6 3 dx
(See the example 6)
2 2 2 2

version: 1.1 version: 1.1

36 37
1. Quadratic Equations eLearn.Punjab 1. Quadratic Equations eLearn.Punjab
3. Integration eLearn.Punjab 3. Integration eLearn.Punjab

∫ ( x - 3) ( x 2 + 4 ) dx ∫
1 + 4x
∫ ( x + 1) ( 2 x ) dx + ∫ x2 + 1 dx - ∫ ( x + 2 x + 3) ( 2 x + 2 ) dx - ∫
( )
=
-1 -1
12

(x + 1) +
2 1 1 21. 22. dx
2
dx +8
2

ln ( x + 1) + Tan x - ln (x + 2 x + 3) - ∫ x3 - 8 dx
9x + 6
∫ (x
2
x + 1 2 x 2 + 5x + 3
- 1) ( x 2 + 4 )
= -1
Tan -1 + 24.
2 2 1 23. dx
c 2
2 2

∫ ( x + 2) ( x ∫ ( 4 x 2 + 1)( x 2 - x + 1) dx
2 x2 - x - 7 3x + 1
+ x + 1)
EXERCISE 3.5 25. 2 2
dx 26.

∫ ( x 2 + 4 )( x 2 + 4x + 5) dx ∫ ( x 2 + a 2 )( x 2 + 4a 2 ) dx
Evaluate the following integrals. 4x + 1 6a 2
27. 28.

∫x ∫ ( x + 3 ) ( 2x
3x + 1 5x + 8
∫ ( x 4 + x 2 + 1) dx
2 x2 - 2
∫ (x
3x - 8
- 1)
- x + 2 )( x 2 + x + 2 )
- x - 6
1. dx 2. dx 29. 30. dx

(a- b) x
2

∫ x 2 + 2x - 15 dx
2

∫ ( x - a )( x - b ) dx,
x 2 + 3 x - 34
(a > b)
∫ ( x 2 + x + 1)( x 2 + 2x + 3) dx
3. 4. 3x3 + 4 x 2 + 9x + 5
31.

∫ 1 - x - 6 x 2 dx ∫ x 2 - a 2 dx
3 - x 2x
5. 6.

∫ 6 x 2 + 5 x - 4 dx ∫ 2 x 2 - 3x - 2 dx
1 2 x3 - 3x 2 - x - 7
7. 8. 3.6 THE DEFINITE INTEGRALS

∫ ( x - 1) ( x - 2 ) ( x - 3) dx ∫ x ( x - 1)( x - 3) dx
3 x 2 - 12 x + 11 2x - 1 We have already discussed in section 3.2 about the indeinite integral that is, if f‘ (x) =
9. 10.
f(x), then

∫ ( x 2 - 1) ( 2 x + 3) dx
5 x 2 + 9x + 6
∫ (1 + x ) ( 2
4 + 7x
∫ f ( x=
) dx f ( x ) + c, where c is an arbitrary constant
+ 3x )
11. 12. 2
dx

If ∫ f ( x=
) dx f ( x ) + c, then the integral of f from a to b is denoted by ∫ f ( x ) dx
∫ (x ∫ (x
b

- 1) ( x + 1) - 1)( x + 1)
2 x2 1 (read as
13. 2
dx 14. 2
dx
intergral from a to b of f of x, dx) and is evaluated as: a

∫ x3 - 3x 2 + 4 dx ∫ ( x + 1) ( x
x3 - 6 x 2 + 25
- 2) ∫ f ( x ) dx = ∫ f ′ ( x ) dx ( if f ( x ) = f′( x ))
x + 4
15. 16. 2 2
dx b b


x3 + 22 x 2 + 14 x - 17
∫ ( x + 1) ( x 2 + 1) dx
x - 2
f ( x ) + c a = f ( b ) + c  - f ( a ) + c  = f ( b ) - f ( a )
(x - 3)( x + 2 )
=
a a

17. 3
dx 18. b

∫ (x ∫ ( x + 3) ( x 2 + 1) dx
9x - 7
- 1) ( x 2 + 1) ∫ f ( x ) dx has a deinite value f(b) - f(a), so it is called the deinite integral of f from a to b.
x b
19. dx 20.

f(b) - f(a) is denoted as f ( x )  a or f ( x )  a (read f(x) from a to b)


a
b b

version: 1.1 version: 1.1

38 39
1. Quadratic Equations eLearn.Punjab 1. Quadratic Equations eLearn.Punjab
3. Integration eLearn.Punjab 3. Integration eLearn.Punjab

The interval [a, b] is called the range of integration while a and b are known as the To explain the idea mentioned above, we irst

draw the graph of f deined as: f ( x ) =


lower and upper limits respectively.
1 2
x

As f(b) - f(a) is a deinite value, so the variable of integration x in ∫ f ( x ) dx


b 2
can be
The graph of f is shown in the igure. We divide
replaced by any other letter. a

the interval [1, 3] into four sub-intervals of equal length

∫ f ( x=
) dx ∫ f ( =
t ) dt f (b) - f ( a ) =
3 -1
=
b b
1
i.e. .
a a 4 2

Note: If the lower limit is a constant and the upper limit is a variable, then the integral is As the subintervals are
[x0, x1], [x1, x2], [x2, x3], [x3, x4], so

∫ f ( t ) dt = f (=
t) a f ( x) - f (a)
x
x0 = 1, x1 = 1.5, x2 = 2, x3 = 2.5, x4 = 3
In the igure MA = f(x0), NB = f(x1) and MN = dx, so it
x
a function of the upper limit, that is,
a
is obvious that the area of the rectangle AMNC < the area of the shaded region AMNB < area


of the rectangle DMNB, that is,
3t 2 dt = t 3 =
 x3 - a3
x
x
For Example, f(x0).dx < area of the shaded region AMNB < f(x1).dx
a
a

The relation f’ (x) = f(x) shows that f(x) gives the rate of change of f(x), so the total
* * * *
Let x 1 , x 2 , x 3 , x 4 be the mid point of four sub-
change in f(x) from a to b as f(b) - f(a) shows the connection between anti-derivatives and intervals mentioned above.
* *

∫ f ( x ) dx .
b
Then the value of f at x 1 , is f ( x 1 ) , so the area of the

rectangle FMNE = f ( x 1 ) dx
deinite integral *
a
(See the rectangle FMNE shown in the igure)

3.6.1 The Area Under The Curve


We observe that the area of the rectangle FMNE is
approximately equal to the area of the region AMNB under
About 300 B.C. and around this, mathematicians succeeded to ind area of plane
the graph of f from x0 to x1.
region like triangle, rectangle, trapezium and regular polygons etc. but the area of the
complicated region bounded by the curves and the x-axis from x = a to x = b was a challenge
for mathematicians before the invention of integral calculus.
Now we calculate the sum of areas of the rectangles shown in the igure, that is,
Now we give attention to the use of integration for evaluating areas. Suppose that a
f ( x1 ) d x + f ( x2 ) d x + f ( x3 ) d x + f ( x4 ) d x
∗ ∗ ∗ ∗
function f is continuous on interval a 7 x 7 b and f(x) > 0. To determine the area under the
graph of f and above the x-axis from x = a to x = b, we follow the idea of Archimedes
 ∗ 
 f ( x1 ) + f ( x2 ) + f ( x3 ) + f ( x4 )  d x
∗ ∗ ∗
=
(287-212 B.C.) for approximating the function by horizontal functions and the area under f
by the sum of small rectangles.  
version: 1.1 version: 1.1

40 41
1. Quadratic Equations eLearn.Punjab 1. Quadratic Equations eLearn.Punjab
3. Integration eLearn.Punjab 3. Integration eLearn.Punjab

Consider a function f which is continuous on the interval a 7 x 7 b and f(x) > 0.


 1  x0 + x1  2 1  x1 + x2  1  x2 + x3  1  x3 + x4   1
The graph of f is shown in the igure.
=    +   +   +   
2 2 2

 2  2  2 2  2  2   2
We deine the function A(x) as the area above the
x-axis and under the curve y = f(x) from a to x. Let dx
2 2

1  1 + 1.5   1.5 + 2   2 + 2.5   2.5 + 3   be a small positive number and x + dx be any number
=   +   +   +   
2 2 2 2

4  2   2   2   2   in the interval [a, b] such that a < x < x + dx.


Let P(xi f(x)) and Q(x + dx, f(x + bx)) be two points
(1.25 ) + (1.75 ) + ( 2.25 ) + ( 2.75 ) 
1
=
4 
2 2 2 2

on the graph of f. The ordinates PM and QN are drawn

= (1.5625 + 3.0625 + 5.0625 + 7.5625 )


1 and two rectangles PMNR, SMNQ are completed.
According to above deinition, the area above
the x-axis and under the curve y = f(x) from a to x + dx
4
= (17.25 ) = 4.3125
1
4 is A(x + dx), so the change in area is
A(x + dx) - A(x) which is shaded in the igure
 1 x3 
∫ x dx =  .  = ( 27 - 1) = Note that the function f is increasing in the interval [x, x + dx].
3 3
1 2 1 26
But
 2 3 1
= 4.3
From the igure, it is obvious that area of the rectangle PMNR < A(x + dx) - A(x) < area of
1
2 6 6

If we go on increasing the number of intervals, then the sum of areas of small rectangles the rectangle SMNQ, i.e.,
approaches closer to the number 4.3. f(x) dx < A(x + dx) - A(x) < f(x + dx) dx
*
Dividing the inequality by dx, we have
If we divide the interval [1, 3] into n intervals and take x i the coordinate of any point
of the ith interval and dx i = x i - x i - 1, i = 1, 2, 3, ..., n, then the sum of areas of n rectangles is A ( x + d x ) - A( x)
f ( x) < < f ( x + d x)


(I)
* 
 x i  d x which tends to the number 4.3 when n gT and each dxig0. lim f ( x ) = f ( x ) lim f ( x + d x ) =
f ( x)
n dx

 
f and
i =1 d x →0 d x →0

∑ f (x ) d x
∗ Since the limits of the extremes in (I) are equal, so

A( x + d x ) - A( x )
n
Thus lim = 4.3 and we conclude that
→ f ( x ) when d x → 0.
n→∞

i =1
i i
d xi →0

dx
A( x + d x ) - A( x )
= f ( x ).
∑ f (x ) d x ∫2 x

= dx
n 3
1 Thus lim
d x →0
2
lim dx.
n→∞
i =1
i i
d i x →0 1

Thus the area above the x-axis and under the curve y = f(x) from a to b is the deinite or A ‘ (x) = f(x)

∫ f ( x=
) dx A( x) + c

b
that is, A(x) is an antiderivative of f, so
integral

∫ f (=
x ) dx  A (=
x )  a A( x ) - A( a )
f ( x) dx.
x
x
and
a

version: 1.1 version: 1.1

42 43
1. Quadratic Equations eLearn.Punjab 1. Quadratic Equations eLearn.Punjab
3. Integration eLearn.Punjab 3. Integration eLearn.Punjab

Since A(x) is deined as the area under the curve y = f(x) from a to x, so A(a) = 0

∫ sin x dx = [ - cos x ]-p = - cos p- cos-( p )  =- -1 ( ) =


p
 - -1 0
A( x ) = ∫ f ( x ) dx
x p
Note:
Thus (I) -p
a

Putting x = b in the equation (I), gives


3.6.2 Fundamental Theorem and Properties

A(b ) = ∫ f ( x ) dx
b of Deinite Integrals

∫ f ( x ) dx
a b

which shows that the area A of the region, above the x-axis and under the curve y = f (x) from The Deinite integral
a to b is given by
a

gives the area under the curve y = f(x) from x = a to x = b and the x-axis (proof is given

∫ f ( x ) dx, that is, A = ∫ f ( x ) dx


in the article 3.6.1)
b b

a a

(b) Fundamental Theorem of Calculus


If f is continuous on [a, b] and f‘ (x) = f(x), that is,
*
If the graph of f is entirely below the x-axis, then the value of each f ( x i ) is negative and

each product f ( xi ) d xi , is also negative, so in such a case, the deinite integral is negative. f(x) is any anti-derivative of f on [a, b], then

∫ f ( x=
) dx f (b) - f ( a )
Thus the area, bounded in this case by the curve y = f(x), the x-axis and the lines b

= a= b is - ∫ f ( x ) dx.
b
a

Note that the diference f(b) - f(a) is independent of the choice of anti-derivative of the
x ,x
a

function f.
For example, sin x is negative for - p < x < 0
and is positive for 0 < x < p.
∫ f ( x ) dx = - ∫ f ( x ) dx
b a
(c)
Therefore the area bounded by the x-axis
(d) ∫ f ( x ) dx ∫ f<( x ) <dx + ∫ f ( x ) dx,
and graph of sin function from -p to p is given by
a b

=
b c b
a c b
a a c

 
- ∫ sin x dx + ∫ sin x dx = ∫ sin x dx + ∫ sin x dx  ∫a ( ) ∫b ( )
p -p p
=  -
0 b a

 
f x dx f x dx Proof of (c) and (d):
-p
(c) If f‘ (x) = f(x), that is, if f is an anti-derivative of f, then using the Fundamental Theorem
= [ - cos x ]0 + [ - cos x ]0 = - cos ( -p ) - cos0  +  - ( cos p - cos0 ) 
0 0 0

-p p
of Calculus, we get
= - ( -1) - 1 - ( -1) -1 = 2 + 2 = 4
∫ f ( x ) dx = f ( b ) - f ( a ) = - f ( a ) - f ( b )  = - ∫ f ( x ) dx
b a

a b

version: 1.1 version: 1.1

44 45
1. Quadratic Equations eLearn.Punjab 1. Quadratic Equations eLearn.Punjab
3. Integration eLearn.Punjab 3. Integration eLearn.Punjab

(d) If f’ (x) = f(x), that is, if f(x) is an anti-derivative of f(x), then applying the Fundamental
 x2 
 2  - [ x ]1 + 2 ln ( x + 1) 1
Theorem of Calculus, we have
=
2
2

 1
2

∫ f ( x ) dx f (c)
= f ( a ) and ∫ f-( x ) dx f (b) f (c) = -  ( 2 )2 (1) 
c b

=  -  - [ 2 - 1] + 2 ln ( 2 + 1) - ln (1 + 1) 
2

 
∫ f ( x ) dx + ∫ f ( x ) dx = f ( c ) - f ( a ) + f (b) - f (c )
a c
c b 2 2

 - 1 + 2 [ ln 3 - ln 2]
 1
= -
Thus

 2
2

= f (b) - f (c ) = ∫ f ( x ) dx
a c
b

= + 2ln
1 3
a
2 2
Other properties of deinite integrals can easily be proved by applying the Fundamental p

∫ ∫ sec x ( sec x + tan x ) dx


x + 9x + 1
3 3
Theorem of Calculus.
4

x2 + 9
Example 2: Evaluate (i) dx (ii)
Now we evaluate some deinite integrals in the following examples. 0 0

∫(x + 3 x 2 ) dx ∫
x2 + 1
2
3
Solution:

x + 1
3

 x3 + 9 x 1 
∫ ∫0  x 2 + 9 + x 2 + 9  dx
Example 1: Evaluate (i) (ii)
x3 + 9 x + 1
dx
-1 1 3 3

x2 + 9
(i) dx =

 x ( x2 + 9)
Solution: 0

1 
∫(x ) dx = ∫ 2 ∫0 
 1 
+ 3x = ∫x dx + ∫ 3x + 2  dx =  +  dx
3 3

 + +  x2 + 9 
3 3 3

0  
(i) 3 2 3 2
dx x 9 x 9
x
-1 -1 -1

 ( 3)4 ( -+1) ∫ x dx ∫x
 x4  
( ) (
=  4  +  x  -1 = - 4  - 3 - 1)  = +
3 3 3
4 1
 + 9
3 3 dx
  -1  4 
3 3
2

 
0 0

 x2 
3  0  ∫ x 2 + ( 3)
81 - 1 1 -1 x 
+  27 - ( -1)= + ( 27 + 1)
 81 1 =
   + c +
3

 3
-1 x
3

= -  
1 1
 4 4   2 0


Tan dx = Tan

( )
2
3 3
4
 3 2 2 
= 20 + 28 = 48
=  -
( 0)  1
+  Tan -1 - Tan -1 
0
 2 2 
3
 3 3
∫ ∫1 x + 1 dx
x2 + 1 x2 - 1 + 2  
3
dx =
2 2

x +1 1 
(ii)
3 
=  - 0  +  Tan -1 - Tan -1 0 
1
 x2 - 1 2  2  3 
∫ ∫  x
1

 2 
=  +  dx= - 1 +  dx
2 2
3
x + 1 x + 1 x + 1 3 1p  3 p p 
= +  - 0 = +  Tan -1 = and Tan -1 0= 0 
1
2 3 6  2 18  
∫ x dx ∫ 1 dx ∫
1 1

= - + 2
2 2 2 3 6
1
x + 1
dx
1 1 1

version: 1.1 version: 1.1

46 47
1. Quadratic Equations eLearn.Punjab 1. Quadratic Equations eLearn.Punjab
3. Integration eLearn.Punjab 3. Integration eLearn.Punjab

∫ ∫ ∫
∫ ( sec x + sec x tan x ) dx
p p

∫ sec x ( sec x + tan x ) dx =


0 2 2
4 4 = 0 dx + 2 x dx = 0 + 2 x dx
(ii) 2
-1 0 0

 x2  4 0
= 2   = 2 -  = 4
0 0
p p
2

= +∫ sec x dx ∫ sec x  2 0 2 2
4 4
2
tan x dx

p  p
[ tan x ]0 -+ [sec x ]0+ =  tan -  
0 0


p p

= tan 0   sec sec 0 


7

   
4 4
3x

( )
x +9
Example 5: Evaluate dx

= (1 - 0 ) +
4 4 2

2 - 1 =
0

2 Solution: Let f(x) = x2 + 9. Then f ‘ (x) = 2x, so

∫ ∫ ∫ (x
3
p
= dx = +
(2 x)

Evaluate ∫
-
1
3x 3
x +9 x +9
4 2 dx 2
9)
2
(2 x) dx
1
-
Example 3: dx 2 2 2

= ∫ [ f ( x)] f ( x) dx
0
1 sin x -
1
3
p p

Solution: ∫ ∫0 (1 - sin x )(1 + sin x ) dx


2

4
1 4
1 + sin x 2
-
dx = - + 1
1

= = 3 [ f ( x)] + c = 3( x + 9) + c
2 1 1
0
1 sin x 3 [ f ( x)] 2

2 - 1 +1
2 2
p p

∫ ∫
1 + sin x 1 + sin x
=
4 4

1 - sin 2 x
dx = dx 2
 2 

 2 
cos 2 x
dx = 3( x + 9)  = 3 (7 + 9) - (0 + 9) 
7
0 0 7 1 1 1
3x

∫0  cos2 x cos2 x  dx =∫0 ( sec sec x tan x ) dx


p p
x2 + 9  0  
2

 1 sin x 
2
Thus
=  + + 2x
4 4

 2 
0

= 3 (16) - (9)  = 3(4 - 3) =


1 1

=  
2
3
2 (See the solution of example 2(ii))


Sin -1 x
3

x ≠ - 1, 1
∫ (x
2
Example 6: Evaluate
+ x ) dx 1 - x
2 dx ,
Example 4: Evaluate
2
1

-1

p p
2

t = Sin -1 x. Then for - ≤ t ≤

∫ ( x + x ) dx = ∫ ( x + x ) dx + ∫ ( x + x ) dx
Solution: Let x = sin t
2 0 2 2 2
 p p
Solution:
1 - sin 2 t dt - ≤ ≤
(by property (d))
 2 
-1 -1 and 
 -x = x if x < 0
dx = cos t dt = cos t is +ve for t

= ∫ [ x + (- x)] dx + ∫ ( x + ( x)] dx
0
2
 
0 2

-1 0  = x if x > 0  = 1 - x 2 dt

version: 1.1 version: 1.1

48 49
1. Quadratic Equations eLearn.Punjab 1. Quadratic Equations eLearn.Punjab
3. Integration eLearn.Punjab 3. Integration eLearn.Punjab

p p p
(x ≠ -1, 1) =  sin + cos  - (0 sin 0 + cos 0)
1
1 - x2 6 6
or dx = dt
6
1 p p p
x = , then = ⇒t = Sin -1 = - (0+1) = - 1
1 1 1 3 3
if S in t = . + +
2 2 2 6 6 2 2 12 2

⇒ t = Sin -1 =
∫ x In x dx
3ð 3 3
and if x = , then = Sin t e
2 2 2 3 Example 8: Evaluate

∫ ∫ (Sin
Sin -1 x
3 3 1
-1
2 2
1
1 - x2 1 - x2
Thus dx = x) . dx Applying the formula

∫ f ( x) f ' ( x) dx ∫ f ( x)
Solution:
= f ( x) f ( x) -
1 1
2 2
f ' ( x) dx, we have
p


 x2  1
= t=
dt ⇒ (  x = Sin t Sin -1 x ∫ (In x) x dx = (In x) . - ∫  .
3
x2
 
t) dx
p 2 2 x
1  x2 
= x In x - ∫ x In x-  +
p
6

t2  3 1  p  p   1 p2 p2 
1 2 1 1 2
=   =   - - 2 2 
  
x dx = c
 
2 2

 2 p 2  3   6   2  9 36 
= 2 2 2
1 2 x2 
∫1
= -  2 x In x 4 1
e e

1  4p 2 - p 2  3p 2 p2
6


Thus x In x dx
=  =
2  1 2 e 1 2 (1) 2 
=
=  e In e -  -  (1) In 1 - 
36 72 24
2 4 2 4 
p
 e2 e2  1 1
∫ x cos x dx
=  .1 -  -  . 0 -  (  In e = 1 and In 1 = 0)
2 4
6

2 4
Example 7: Evaluate
0

+
e2 1
Applying the formula =

∫ f ( x) f ' (x) dx ∫ f ( x) f
Solution:
= f (x) f ( x) -
4 4

∫ x cos x dx ∫ (sin x) (1) dx ∫ ∫ ∫ g ( x ) dx


' ( x) dx, we have
= x sin x -
1 3 1
Example 9: If f ( x ) dx = 5, f ( x ) = 3 and = 4, then
-2 -2

= x sin x - [(- cos x) + c1]


1

= x sin x + cos x + c where c = - c1,


evaluate th e following deinite integrals:

∫ f ( x ) dx ∫ [2 f ( x ) + 3 g (x )] dx
3 1
p

∫ x cos x dx = [x sin x + cos x ]


p
(i) (ii)
-3 -2
6

∫ 3 f ( x ) dx ∫ 2 g ( x ) dx
6
Thus
-
0 1 1
0
(iii)
-2 -2

version: 1.1 version: 1.1

50 51
1. Quadratic Equations eLearn.Punjab 1. Quadratic Equations eLearn.Punjab
3. Integration eLearn.Punjab 3. Integration eLearn.Punjab

∫ f ( x) dx ∫ f ( x) dx ∫
p p p

sec q
∫ cos q ∫ (1 + cos ∫ sin q + cos q
+
3 1 3

sin q dq q ) tan q dq dq
Solution: (i) = f (x) dx = 5 + 3 = 8 3
2
4
2 2
4

-2 -2
19. 20. 21.

∫ [2 f ( x) + 3g ( x)] dx = ∫ 2 f ( x) dx ∫ 3 g (x) dx
1

+
0 0 0
1 1 1

 13 
(ii)
 + 
2

-2 -2 -2

∫ x-3 ∫1/8 23 dx ∫1 x + 1 dx
  x2 - 2
∫ f ( x) dx ∫
x 2
= 2
5 1 3
1 1
+3 g (x) dx 22. dx 23. 24.
-1
-2 -2

= 2(5) + 3(4) = 10 + 12 = 22
x
p p

∫0 cos2 x ∫0 1 + sin x dx
sin x - 1
∫2 ( x - 1)( x 2 + 1) dx
3x - 2 x + 1
3 2 4 4
1
26.

∫ 3 f ( x) dx ∫ 2 g ( x) dx = 3 ∫ f ( x) dx ∫ g (x) dx
25. 27.
- - 2
1 1 1 1
(iii)
-2 -2 -2 -2 p p

∫ ∫ ∫0 (1 + cos x)(2 + cos x) dx


= 3 × 5 - 2 × 4 = 15 - 8 = 7
1 2 2
3x cos x sin x
4 - 3x
28. dx 29. dx 30.
0 p sin x (2 + sin x )

EXERCISE 3.6 6

Evaluate the following deinite integrals.


3.7 APPLICATION OF DEFINITE INTEGRALS.

∫ (x + 1) dx ∫ (x + 1) dx ∫ (2 x
2 1 0
1
- 1) 2
Here we shall give some examples involving area bounded by the curve and the x-axis.
2 1/3
1. 2. 3. dx
-1 -2

∫ ∫ ∫
1

3 - x dx x x 2 - 1 dx Example 1. Find the area bounded by the curve y = 4 - x2


2 5
4. 5. (2t 1) dt 6.
-6 and the x-axis.

∫1 x 2 + 2 dx ∫2  x - x  dx ∫  x + 2 
2

 1  1
x 2 + x + 1 dx
2 3 2 1
x
7. 8. 9.
-1
Solution: We irst ind the points where
the curve cuts the x-axis. Putting y = 0,
p

∫ ∫p cos t dt ∫1  x  1 - x 2  dx
 1 2  1  we have
+
1

4 - x2 = 0 ⇒ x = ± 2.
3 3 2
dx
x + 9
10. 2
11. 12. x
0
p So the curve cuts the x-axis at (-2, 0) and (2, 0)
cos q + sin q The area above the x-axis and under the curve y = 4 - x2 is
∫ In x dx ∫ ∫
 
6

e - e  dx dq
-
2 2 x x 4

2cos 2 q
14.
 
2 2
13. 15.
1 0 0
shown in the igure as shaded region..

 

p p p

∫ cos q dq ∫p cos q cot ∫ cos


Thus the required area = - =
4 x - 
q dq  
6 4 4
(4 x ) dx
16. 3
17. 2 2
18. 4
t dt
0 0
6

version: 1.1 version: 1.1

52 53
1. Quadratic Equations eLearn.Punjab 1. Quadratic Equations eLearn.Punjab
3. Integration eLearn.Punjab 3. Integration eLearn.Punjab

f(x) 8 0 for - 2 7 x 7 0, that is, the area in the interval


 (2)3   (-2)3 
=  4(2) -  -  4(-2) -
[-2, 0] is above the x-axis and is equal to

 3   3 
 8  8 ∫ x(x - 4) dx
0

=  8 -  -  -8 + 
2

 3  3 -2

 -16   x2   x4 
= -  =  ∫
- = - 4  - = 
0

2x 
0 0
16 32 x4
 3   2  -2 4  -2
3 2
(x 4 x ) dx =
-2
3 3 4
 (-2) 4   16 
Example 2. Find the area bounded by the curve y = x3 + 3x2 and the x-axis. = 0 -  - 2(-2) 2  = 0 -  - 8  = - (4 - 8) = 4
 4   4 
Solution: Putting y = 0 , we have
x3 + 3x2 = 0
⇒ x2 (x + 3) = 0 ⇒ x = 0, x = -3 f(x) 7 0 for 0 7 x 7 2, that is, the area in the interval [0, 2 ] is below the x-axis and is

 x4 
equal to - ∫ ( x - 4) dx -= - 2x 
2 2

The curve cuts the x-axis at (-3, 0) and (0, 0)


4 0
3 2

(see the igure).


 16  
0

-= - 4 -  0
∫ (x   
2(4)
= + 3 x 2 ) dx
0

= - [ -4 - 8] = - (-4) = 4
Thus the required area 3

-3

 x4 3
= +
Thus the area of the shaded region = 4 + 4 = 8
4 
0

  -3
x

Find the area bounded by the curve f(x) = x3 - 2x2 + 1 and the x-axis in
0   (-3) 4 
Example 4:
=  + 0 -  + (-3)3 
4   4 
the 1st quadrant.

 81   81 - 108   27 
= 0 -  - 27  = -   = --  = Solution: As f(1) = 1 - 2 + 1 = 0, so x - 1 is factor of x3 - 2x2 + 1. By long division, we ind that
27
4   4   4 
x2 - x - 1 is also a factor of x3 - 2x2 + 1.
4

Example 3. Find the area bounded by y = x(x2 - 4) and the x-axis. Solving x2 - x - 1 = 0, we get

1 ± 1 + 4 1 ± 5
Solution: Putting y = 0, we have =
x(x2 - 4 ) ⇒ x = 0, x = ±2
x =
2 2
The curve cuts the x-axis at (-2, 0), (0, 0) and (2, 0). The graph of f is shown in the igure and 1+ 5 1- 5
Thus the curve cuts the x-axis at x = 1, and
we have to calculate the area of the shaded region. 2 2
f(x) = x(x2 - 4),

version: 1.1 version: 1.1

54 55
1. Quadratic Equations eLearn.Punjab 1. Quadratic Equations eLearn.Punjab
3. Integration eLearn.Punjab 3. Integration eLearn.Punjab

The graph of the curve is shown in the EXERCISE 3.7


adjoining igure and the required area is
shaded. 1. Find the area between the x-axis and the curve y = x2 + 1 from x = 1 to x = 2.
The required area A will be 2. Find the area, above the x-axis and under the curve y = 5 - x2 from x = -1 to x = 2.
Find the area below the curve y = 3 x and above the x-axis between x = 1 and x = 4.
A = ∫ (x - 2 x + 1) dx
1 3.

p p
3 2

Find the area bounded by cos function from x = -


0
4. to x =
= - 2 + x Find the area between the x-axis and the curve y = 4x - x2.
1 2 2
x4 x3 5.
4 3 6. Determine the area bounded by the parabola y = x2 + 2x - 3 and the x-axis.
1  3 - 8 + 12
0

=  - + 1 - 0 = =
2 7 7. Find the area bounded by the curve y = x3 + 1, the x-axis and line x = 2.
4 3  12 12 8. Find the area bounded by the curve y = x3 - 4x and the x-axis.
9. Find the area between the curve y = x(x - 1)(x + 1) and the x-axis.
10. Find the area above the x-axis, bounded by the curve y2 = 3 - x from x = -1 to x = 2

Find the area between the x-axis and the curve y2 = 4 - x in the irst
= the curve y - cos p to p
Example 5: 1
11. Find the area between the x-axis and x from x =
quadrant from x = 0 to x = 3.
p
2
12. Find the area between the x-axis and the curve y = sin 2x from x = 0 to x =
= 2ax - x 2 when a > 0.
Solution: The branch of the curve above the x-axis is 3
= 4 - x
13. Find the area between the x-axis and the curve y
y
The area to be determined is shaded in the adjoining igure.
3.8 DIFFERENTIAL EQUATIONS
= ∫ 4 - x dx
3
Thus the required area
0 An equation containing at least one derivative of a dependent, variable with respect to
Let 4 - x = t (i), then -dx = dt ⇒ dx = -dt an independent variable such as
Putting x = 0 and x = 3 (i). we get t = 4 and t = 1
+ 2x =
dy
y 0 (i)

Now the required area = ∫ t × (-dt ) = - ∫ t dt


1 1 1 1 dx

+ - 2x =
2 2
x d2y dy
4 4 or 0 (ii)

∫ t dt
dx 2 dx
= =
4 1 4
2
t 3/2 is called a diferential equation.
3/ 21 Derivatives may be of irst or higher orders. A diferential equation containing only
2  32 
[-8 =1]
1

= = (4)- (1)= 
2 3/2 4 3
2 14 derivative of irst order can be written in terms of diferentials. So we can write the equation
3 
2
t (square units)
3 1 3 3 (i) as y dy + 2x dx = 0 but the equation (ii) cannot be written in terms of diferentials.

version: 1.1 version: 1.1

56 57
1. Quadratic Equations eLearn.Punjab 1. Quadratic Equations eLearn.Punjab
3. Integration eLearn.Punjab 3. Integration eLearn.Punjab

Order: The order of a diferential equation is the order of the highest derivative in All solutions obtained from (iii) by putting diferent values of A, are called particular
the equation. As the order of the equation (i) is one so it is called a irst order diferential solutions of (v) while the solution (iii) itself is called the general solution of (v).
equation. But equation (ii) contains the second order derivative and is called a second order A solution of diferential equation is a relation between the variables (not involving
diferential equation. derivatives) which satisies the diferential equation.
Here we shall solve diferential equations of irst order with variables separable in the
3.8.1 Solution of a Differential Equation of irst order: forms

dy f ( x) dy g ( y)
Consider the equation = or =
dx g ( y) dx f ( x)
y = Ax2 + 4 (iii)
where A is a real constant
Diferentiating (iii) with respect to x gives Example 1: Solve the diferential equation (x - 1) dx + y dy = 0

dy
= 2Ax (iv) Solution: Variables in the given equation are in separable form, so integrating either terms,

y - 4
dx
we have

∫ (x - 1) dx + ∫ y dy = c ,
From (iii) A = , so putting the value of A in (iv), we get
x2 where c1 is a constant
 y - 4
1

= 2 x  x2 
dy
 x  or  - x + =
2 y2
 
dx c1 , which gives
⇒ = 2y - 8 which is free of constant A
2 2
Thus the required general solution is x2 + y2 - 2x = c,
dy
x
dx where c = 2c1

⇒ 2y - x
dy
=8 Example 2: Solve diferential equation
dx
Substituting the value of y and its derivative in x 2 (2 y + 1) - 1=0
dy
(v), we see that it is satisied, that is. dx
2(Ax2 + 4) - x(2Ax) = 2Ax2 + 8 - 2Ax2 = 8 Solution: The given diferential equation can be written as
which shows that (iii) is asolution of (v)
Giving a particular value to A. say A = -1. we get x 2 (2 y + 1)
dy
y = -x2 + 4
=1 (i)
dx
Dividing by x2, we have (2 y + 1) (x ≠ 0)
dy 1
We see that (v) is satisied if we put y = -x2 + 4 and = -2x, so y = -x2 + 4 is also a solution
dy = 2, (ii)
dx x
dx
of (v). Multiplying both sides of (i) by dx, we get
For diferent values of A, (iii) represents diferent parabolas with vertex at (0, 4) and the
 dy 
axis along the y-axis. We have drawn two members of the family of parabolas. (2y + 1)  dx  = 2 dx
1
y = Ax2 + 4 for A = -1, 1  dx  x
version: 1.1 version: 1.1

58 59
1. Quadratic Equations eLearn.Punjab 1. Quadratic Equations eLearn.Punjab
3. Integration eLearn.Punjab 3. Integration eLearn.Punjab

 dy 
(2 y + 1) dy =  dx =
dy 
 p 
1
 dx 
or dx
 
x2
 
0< y<
Integrating either side gives
Solve 2e tan y dx + (1 - e ) sec y dy = 0  3p
2

 or p < y < 
∫ (2 y + 1) dy = ∫ x 2 dx
x x 2
Example 5:
1  2 
Solution: Given that: 2ex tan y dx + (1 - ex) sec2 y dy = 0 (i)
 
∫x Dividing either term of (i) by tan y (1 - ex), we get
x -1
y +y= - + c 
-2
dx = + c
1
 -1 
2
or
x

Thus y + y = c -
2e x sec 2 y
1 - ex
1
2
is the general solution of the given diferential equation. dx +
tan y
dy = 0

-2e x
x
dx +
sec 2 y
ex - 1
or dy = 0
Example 3: Solve the diferential equation tan y
Integrating, we have
- 2y = x ≠ 0, y > 0
1 dy
0
 ex   sec 2 y 
∫ -2  e x - 1  dx + ∫  tan y  dy = c1 (e x - 1 > 0)
x dx
x
Solution: Multiplying the both sides of the given equation by dx, gives
y
1  dy   dy  -2 In (ex - 1) + In (tan y) = c1
 dx =- 2 x dx= = 2 x dx  dy 
or
⇒ In (ex - 1)-2 + In (tan y) = In c,
1
y  dx   dx 
0 or dy dx
where c1 = In c
In [(ex - 1)-2 tan y] = In c
y
Now integrating either side gives In y = x + c1
2
where c1 is a constant or
=
or y =
e x +c1
2 2
e x . ec1 ⇒ (ex - 1)-2 tan y = c ⇒ tan y = c{ex - 1)2.

Thus y = ce x where
2
ec1 = c Example 6: Solve (sin y + y cos y) dy = [x (2 Inx + 1)] dx
is the required general solution of the given diferential equation.
Solution: (sin y + y cos y) dy = (2x In x + x) dx (i)
2
dy y + 1
Example 4: Solve
e- x
= 1
dx or (1. sin y + y cos y ) dy = (2 x In x + x 2 . ) dx
x
 d  d 
⇒  ( y sin y )  dy =  ( x 2 In x)  dx (  ( y sin y ) = +
Solution: Separating the variables, we have d
   
1. sin y y cos y and
dy dx dy
1 1
y + 1
dy = - x dx = e x dx d 1
2
e ( x 2 In x ) 2x In x + x 2 . )
dx x
Now integrating either side gives Integrating, we have
Tan-1 y = ex + c, where c is a constant,

∫ = ( y sin y )  dy +∫ 
 d  d 
( x 2 In x)  dx
or y = Tan (ex + c)
which is the general solution of the given diferential equation. dy   dx 
version: 1.1 version: 1.1

60 61
1. Quadratic Equations eLearn.Punjab 1. Quadratic Equations eLearn.Punjab
3. Integration eLearn.Punjab 3. Integration eLearn.Punjab

⇒ y sin y = x2 In x + c Note: The general solution represents a system of parabolas which are vertically above
(or below) each other.
3.8.2 Initial Conditions
x + x - 3, if y = 0 when x = 2
dy 3 2
Example 2: Solve =
Diferential equations occur in numerous practical problems concerning to physical, dx 4
biological and social sciences etc.
Solution: Given that
The arbitrary constants involving in the solution of diferent equations can be determined

= x + x - 3
by the given conditions. Such conditions are called initial value conditions. dy 3 3
(i)
The general solution of diferential equation in variable separable form contains only dx 4
one variable. Here we shall consider those diferential equations which have only one initial Separating variables, we have
value condition.
3 2 
Note that the general solution of diferential equation of order n contains n arbitrary =  x + x - 3  dx
4 
dy (ii)
constants which can be determined by n initial value conditions.
Integrating either side of (ii) gives

∫= ∫  4 x
3 
Example 1: The slope of the tangent at any point of the curve is given by
+ x - 3  dx

2
dy
= 2 x - 2, ind the equation of the curve if y = 0 when x = -1.
dy
3  x3 
=   + - 3x + c
dx x2
4 3 
or y
2
Solution: Given that = 2x - 2 ⇒ y= x + x 2 - 3x + c
dy 1 3 1
(i) (iii)
dx 4 2
Now applying the initial value condition, we have
Equation (i) can be written as
dy = (2x - 2) dx
0 = (8) + (4) - 3(2) + c
(ii) 1 1
Integrating either side of (ii) gives
⇒ c=6-2-2=2
4 2

∫ dy = ∫ (2 x - 2) dx
y = x2 - 2x + c
Thus (iii) becomes
or (iii)
Applying the given condition, we have y = x + x 2 - 3x + 2
1 3 1
0 = (-1)2 - 2(-1) + c ⇒ c = -3
⇒ 4y = x + 2x - 12x + 8
4 2
3
Thus (iii) becomes
2

y = x2 - 2x - 3
which represents a parabola as shown in the Example 3: A particle is moving in a straight line and its acceleration is given by
adjoining igure. a = 2t - 7,
For c = 0, (iii) becomes y = x2 - 2x. (i) ind v (velocity) in terms of t if v = 10 m/sec, when t = 0
The graph of y = x2 - 2x is also shown in the igure. (ii) ind s (distance) in terms of t if s = 0, when t = 0.
version: 1.1 version: 1.1

62 63
1. Quadratic Equations eLearn.Punjab 1. Quadratic Equations eLearn.Punjab
3. Integration eLearn.Punjab 3. Integration eLearn.Punjab

Solution: Given that a = 2t - 7, that is

 dv  dp = ⇒ ln p =kt + c1
1
- = 7 =  a 
dv or k dt
 dt 
2t p
⇒ p= e kt +c1 =
⇒ dv = (2t - 7) dt
dt
e kt . ec1

Integrating, we have =
=or p ce kt (i) (where ec1 c)

∫ dv
= ∫ (2t - 7) dt
Applying the given condition, that is p = 100 when t = 0, we have
100 = ce(0)k = c (a e0 = 1)
⇒ v = t2 - 7t + c1 (1) Putting c = 100, (i) becomes p = 100 ekt (ii)
Applying the irst initial value condition, we get p will be 200 when t = 2(hours), so (ii) gives
10 = 0 - 0 + c1 ⇒ c1 = 10 200 = 100 e2k ⇒ e2k = 2
The equation (1) becomes
v = t2 - 7t + 10 which is the solution of (i) = =
⇒k
1
or 2k ln 2 ln 2
2
 ds 
= t 2 - 7t + 10  = 
ds
 dt 
Now  v Subsituting = ln 2 in (ii), we get
dt
1 

=
= 100 =
 ln 2 t
⇒ ds = (t2 - 7t + 10) dt
2 
1 1
ln 2
ln(2 2 )
(2)
2
p e 100e 100e

p = 100 (2 )
Integrating both sides of (2), we get
1

∫ ds ∫ (t
2

= - 7t + 10) dt
If t = 4 (hours), then =p 100 (2 =) 100× =4
2
4
2
400.
⇒ s = - 7 + 10t + c2
3 2
t t
(3)
3 2 Example 5: A ball is thrown vertically upward with a velocity of 1470 cm/sec
Applying the second initial value condition, gives Neglecting air resistance, ind
0 = 0 - 0 + 0 + c2 ⇒ c2 = 0 (i) velocity of ball at any time t
(ii) distance traveled in any time t
s = t - t + 10t
1 3 7 2
Thus is the solution of (ii) (iii) maximum height attained by the ball.
3 2

Example 4: In a culture, bacteria increases at the rate proportional to the number Solution.
of bacteria present. If bacteria are 100 initially and are doubled in 2 hours, ind the (i) Let v be the velocity of the ball at any time t, then by Newton’s law of motion, we have
number of bacteria present four hours later.
-=g⇒ -=
dv
dv g dt (i)

∫ dv= ∫ - g dt
dt
Solution: Let p be the number of bacteria present at time t, then
or (integrating either side of (i))
= >kp,
dp
(k 0) v = -gt + c1 (ii)
dt Given that v = 1470 (cm/sec) when t = 0, so
version: 1.1 version: 1.1

64 65
1. Quadratic Equations eLearn.Punjab 1. Quadratic Equations eLearn.Punjab
3. Integration eLearn.Punjab 3. Integration eLearn.Punjab

1470 = -g(0) + c1 ⇒ c1 = 1470 Solve the following diferential equations:


Thus (ii) becomes v = -gt + 1470 = 1470 - 980t (taking g = 980) 1 - x
= - y y dx + x dy = =
dy dy
(ii) Let h be the height of the ball at any time t, then 2. 3. 0 4.

x dy + y ( x - 1) dx =
dx dx y

 dh  5.= (y > 0) = 1
dy y dy
= 1470 - 980 t  v =
6.

dh , sin y cosec x 7. 0

 dt 
x2

(1 + y2 )
dx dx
x2 + 1
= > , ( x, y 0) = 2 x2 y = x2 - 1
dt
dh = (1470 - 980 t) dt
x dy 1 dy 1 dy
y +1
8. . 9. 10.

(x - yx 2 )
or y dx x dx 2 dx

+ = + y 2 + xy 2 =
dy 2 xy dy
h = 1470 t - 980 + c2 = 1470 t - 490 t 2 + c2 2y + 1
2
t 2 11. x 12. 0
dx dx
(iii)
 dy   2 dy 
sec 2 x tan y dx + sec 2 y tan x dy =  y - x  =2 y + 
2
14.
 dx   dx 
13. 0
h = 0 when t = 0, so we have
0 = 1470 x 0 - 490(0)2 + c2 ⇒  dy 
1 + cos x tan y = y - x = 3 1 + x 
c2 = 0 dy dy
16.
 dx 
15. 0

( e x + e- x ) dy
Putting c2 = 0 in (iii), we have dx dx
h = 1470 t - 4 9 0 t2 17. sec x + tan y
dy
=0 18. -= ex e- x
dx dx
(iii) The maximum height will be attained when v = 0, that is
- x =
dy
19. Find the general solution of the equation xy 2 Also ind the particular solution

1470 - 980 t = 0 ⇒ t=
dx
1470 3
= (sec) if y = 1 when x = 0.
980 2
= 2 x given that x = 4 when t = 0.
3 3
dx
20. Solve the diferential equation
Thus the maximum height attained in (cms) = 1470 ×   - 490 ×  
2
dt
2 2 21. Solve the diferential equation
ds
+ 2 st = 0 . Also ind the particular solution if s = 4e,
2205 - 1102.5 = 1102.5
dt
= when t = 0.
22. In, a culture, bacteria increases at the rate proportional to the number of bacteria
EXERCISE 3.8 present. If bacteria are 200 initially and are doubled in 2 hours, ind the number of
bacteria present four hours later.
1. Check that each of the following equations written against the diferential 23. A ball is thrown vertically upward with a velocity of 2450 cm/sec. Neglecting air
equation is its solution. resistance, ind
(i) velocity of ball at any time t
= 1 + y = cx - 1
dy
(i) x , y
dx (ii) distance traveled in any time t
x 2 ( 2 y + 1) - 1 =0 y2 + y = c - (iii) maximum height attained by the ball.
dy 1
(ii) ,
dx x
- e2 x = y 2 = e2 x + 2 x + c
dy
(iii) y 1 ,
dx
- 2y = y = ce x
1 dy
(iv) ,
2
0

tan ( e x + c )
x dx
y2 + 1
= =
dy
(v) ,
e- x
y
dx
version: 1.1 version: 1.1

66 67
version: 1.1

CHAPTER

4 Introduction to
Analytic Geometry

Animation 4.1: Coordinate System


Source and credit: eLearn.Punjab
1. Quadratic Equations eLearn.Punjab 1. Quadratic Equations eLearn.Punjab
4. Introduction to Analytic Geometry eLearn.Punjab 4. Introduction to Analytic Geometry eLearn.Punjab

that P has coordinates (x, y). It may be noted that x and y are the directed distances of P from
4.1 INTRODUCTION the y-axis and the x-axis respectively. The reverse of this technique also provides method for
associating exactly one point in the plane with any ordered pair (x, y) of real numbers. This
Geometry is one of the most ancient branches of mathematics. The Greeks
method of pairing of in a one-to-one fashion the points in a plane with ordered pairs of real
systematically studied it about four centuries B.C. Most of the geometry taught in schools is
numbers is called the two dimensional rectangular (or Cartesian) coordinate system.
due to Euclid who expounded thirteen books on the subject (300 B.C.). A French philosopher
and m athematician Rene Descartes (1596-1650 A.D.) introduced algebraic methods in
If (x, y) are the coordinates of a point P, then the irst member (component) of the
ordered pair is called the x - coordinate or abscissa of P and the second member of the
geometry which gave birth to analytical geometry (or coordinate geometry). Our aim is to

ordered pair is called the y - coordinate or ordinate of P. Note that abscissa is always irst
present fundamentals of the subject in this book.

element and the ordinate is second element in an ordered pair.


Coordinate System
Draw in a plane two mutually perpendicular The coordinate axes divide the plane into four equal parts called quadrants. They are
number lines x' x and y' y , one horizontal and the other deined as follows:
vertical. Let their point of intersection be O , to which we
call the origin and the real number 0 of both the lines is Quadrant I: All points (x, y) with x > 0, y > 0
represented by O. The two lines are called the coordinate
Quadrant II: All points (x, y) with x < 0, y > 0
axes. The horizontal line x'Ox is called the x-axis and the
Quadrant III: All points (x, y) with x < 0, y < 0
vertical line y' Oy is called the y-axis.
Quadrant IV: All points (x, y) with x > 0, y < 0
The point P in the plane that corresponds to an ordered pair
As in the case of number line, we follow the (x, y) is called the graph of (x, y).
convention that all points on the y-axis above x'Ox
Thus given a set of ordered pairs of real numbers, the graph of the set is the aggregate
are associated with positive real numbers, those
of all points in the plane that correspond to ordered pairs of the set.
below x'Ox with negative real numbers. Similarly,
all points on the x-axis and lying on the right of O Challenge!
will be positive and those on the left of O and lying
on the x-axis will be negative.
Suppose P is any point in the plane. Then P i- Write down the coordinates of the points
can be located by using an ordered pair of real if not mentioned.
numbers. Through P draw lines parallel to the

Locate (0, -1), (2, 2), (-4, 7) and (-3, -3).


coordinates axes meeting x-axis at R and y-axis at S.
Let the directed distance OR = x and the directed distance OS = y .
ii-

The ordered pair (x, y) gives us enough information to locate the point P. Thus, with
every point P in the plane, we can associate an ordered pair of real numbers (x, y) and we say
version: 1.1 version: 1.1

2 3
1. Quadratic Equations eLearn.Punjab 1. Quadratic Equations eLearn.Punjab
4. Introduction to Analytic Geometry eLearn.Punjab 4. Introduction to Analytic Geometry eLearn.Punjab

a= BC= (2 - 7) + ( -6 - 5 ) =
4.1.1 The Distance Formula 2 2
146

Let A (x1 , y1) and B (x2 , y2) be two points in the plane. We can ind the b= CA= 2 - ( - 1) 2 + ( -6 - 2 ) =
2
73
Note that :
distance d = AB from the right triangle AQB by using the Pythagorean Clearly: a=2 b 2 + c 2 .

AB stands for

theorem. We have Therefore, ABC is a right triangle with right angle at A.


m AB or AB

= = AQ + QB 2
d AB (1) Example 2: The point C (-5, 3) is the centre of a circle and
= RS
AQ = RO + OS P (7, -2) lies on the circle. What is the radius of the circle?
-=OR
+ OS
= x2 - x1 Solution: The radius of the circle is the distance from C to P.
QB = SB - SQ = OM - ON
By the distance formula, we have

= y2 - y1 Radius= CP= ( 7 - ( -5) ) + ( -2 - 3)


2 2

Therefore, (1) takes the form = 144 + 25= 13


d 2 = ( x2 - x1 ) + ( y2 - y1 )
2 2

( x2 - x1 ) + ( y2 - y1 )
4.1.2 Point Dividing the Join of Two Points in a given Ratio
d = AB =
2 2
or (2)

which is the formula for the distance d. The distance is always taken to be positive and Theorem: Let A (x1 , y1) and B (x2 , y2) be the two given points in a plane. The coordinates of
it is not a directed distance from A to B when A and B do not lie on the same horizontal or the point dividing the line segment AB in the ratio k1 : k2 are
 k1 x2 + k2 x1 k1 y2 + k2 y1 
 
vertical line.
 1 2 + k1 + k2 
,
If A and B lie on a line parallel to one of the coordinate axes, then by the formula (2), k k


Let P ( x, y ) be the point that divides AB in the ratio k1: k2


the distance AB is absolute value of the directed distance AB .
The formula (2) shows that any of the two points can be taken as irst point. Proof:
From A, B and P draw perpendiculars to the x-axis as shown in the igure. Also draw
Example 1: Show that the points A (-1, 2), B (7, 5) and BC ⊥ AQ . Since LP is parallel to CA, in the triangle ACB, we have
C (2, -6) are vertices of a right triangle.
AP CL QM x - x1
= = = =
k1
k2 PB LB MR x2 - x
x - x1
Solution: Let a, b and c denote the lengths of the sides BC,
=
k1
x2 - x
CA and AB respectively. So,
k2
By the distance formula, we have

( 7 - ( -1) ) + ( 5 - 2 )
k1 x2 - k1 x = k2 x - k2 x1
c= AB= =
( k1 + k2 ) x =k1 x2 + k2 x1
2 2 or
73
or
version: 1.1 version: 1.1

4 5
1. Quadratic Equations eLearn.Punjab 1. Quadratic Equations eLearn.Punjab
4. Introduction to Analytic Geometry eLearn.Punjab 4. Introduction to Analytic Geometry eLearn.Punjab

k1 x2 + k2 x1  -8 
x= Coordinates of the required point are  ,1
k1 + k2  5 
or
(ii) In this case

2 × 5 - 3 × ( -6 ) 2(-2) - 3(3)
Similarly, by drawing perpendiculars from A , B and P to the y-axis and
= x= - 13 =
k y +k y 2-3 2-3
y= 1 2 2 1
28 and y=
k1 + k2
proceeding as before, we can show that
Thus the required point has coordinates (-28, 13)
Theorem:
The centroid of a ∆ABC is a point that divides each median in the ratio 2 : 1. Using this
Note:
(i) If the directed distances AP and PB have the same sign, then their ratio is positive and P
is said to divide AB internally. show that medians of a triangle are concurrent.
(ii) If the directed distances AP and PB have opposite signs i.e, P is beyond AB. then their Proof: Let the vertices of a ∆ABC have coordinates as shown in the igure.
ratio is negative and P is said to divide AB externally.
= = - 1  x + x y + y3 
Midpoint of BC is D  2 3 , 2 .
AP k1 AP k
 
or
BP k2 PB k2 2 2

( )
Proceeding as before, we can show in this case that
k x -k x k y -k y Let G x, y be the centroid of the ∆ .
= =
k1 - k2 k1 - k2
x 1 2 2 1 y 1 2 2 1
Then G divides AD in the ratio 2 : 1. Therefore

x2 + x3
+ 1.x1 x + x + x
Thus P is said to divide the line segment AB in ratio k1 : k2 , internally or externally according

=
=
as P lies between AB or beyond AB. 2.
(iii) If k= k= 2 +1
x 2 1 2 3
1:1, then P becomes midpoint of AB and coordinates of P are :
y + y2 + y3
1 2 3
x1 + x2 y1 - y2 Similarly, y = 1
=
=
.
x , y 3
2 2
In the same way. we can show that coordinate of the point that divides BE and CF each
(iv) The above theorem is valid in whichever quadrant A and B lie.
 x + x + x y + y2 + y3 
in the ratio 2 : 1 are  1 2 3 , 1 .
Example 1: Find the coordinates of the point that divides the join of A (-6, 3) and B (5,  
-2) in the ratio 2 : 3.
3 3

(i) internally (ii) externally Thus ( x, y ) lies on each median and so the medians of the ∆ABC are concurrent.

Solution: (i) Here k1 =


-
2, k2 =
=
3, x1 =6, x2 5. Theorem: Bisectors of angles of a triangle are concurrent.
By the formula, we have

2 × 5 + 3 × ( -6 ) 2 ( -2 ) + 3 ( 3)
Proof: Let the coordinates of the vertices of a triangle be as shown in the igure.
-8 Suppose = BC a=, CA b and=
=
=
==
2+3 2+3
AB c
Let the bisector of ∠A meet BC at D. Then D divides BC in the ratio c : b. Therefore
x and y 1
5

version: 1.1 version: 1.1

6 7
1. Quadratic Equations eLearn.Punjab 1. Quadratic Equations eLearn.Punjab
4. Introduction to Analytic Geometry eLearn.Punjab 4. Introduction to Analytic Geometry eLearn.Punjab

2. Find in each of the following:


 cx + bx2 cy3 + by2 
coordinates of D are  3 
(i) the distance between the two given points
 b+c b+c 
,

The bisector of ∠B meets AC at I and I


(ii) midpoint of the line segment joining the two points
(a) A (3 ,1); B (-2 ,-4 )
divides AD in the ratio c : BD (b) A (-8 ,3); B (2, -1)

=
=
Now
BD c
DC b
or
DC
BD
b
c
(c)
 1
(
A  - 5, -  ; B -3 5,5
 3
)
DC + BD b + c
=
3. Which of the following points are at a distance of 15 units from the origin?

( )
or
 15 15 
BD c
b+c (10, -10)  , 
=
=  2 2
a ac (a) 176,7 (b) (c) (1, 15 ) (d)
b+c
or or BD
BD c

( )
4. Show that

ac 3,1 and C (0, -2) are vertices of a right triangle.


b+c
Thus I divides AD in the ratio c : or in the ratio b + c : a (i) the points A (0, 2), B
Coordinates of I are (ii) the points A (3, 1), B (-2, -3) and C (2, 2) are vertices of an isosceles triangle.
(iii) the points A (5, 2), B (-2, 3), C (-3, -4) and D (4, -5) are vertices of a parallelogram.
bx2 + cx3 by2 + cy3

 ( + ) + ( + ) +


Is the parallelogram a square?
b+c b+c The midpoints of the sides of a triangle are (1, -1), (-4, -3) and (-1, 1). Find coordinates
 
b c ax b c ay
5.
a+b+c a+b+c
1 1

 
,

( )
 
of the vertices of the triangle.

 ax1 + bx2 + cx3 ay1 + by2 + cy3  3, - 1 , B (0, 2) and C (h, -2) are vertices of a right
 
6. Find h such that the points A
 + + a+b+c 
i.e., ,
a b c
triangle with right angle at the vertex A.
The symmetry of these coordinates shows that the bisector
of ∠C will also pass through this point.
7. Find h such that A (-1, h ), B (3, 2) and C (7, 3) are collinear.
8. The points A (-5, -2) and B (5, -4) are ends of a diameter of a circle. Find the centre
Thus the angle bisectors of a triangle are concurrent.
and radius of the circle.
9. Find h such that the points A ( h , 1), B (2, 7) and C (-6, -7) are vertices of a right triangle
EXERCISE 4.1
with right angle at the vertex A.

Describe the location in the plane o f the point P ( x, y ) for which


A quadrilateral has the points A (9, 3), B (-7, 7), C (-3, -7) and D(5, -5) as its vertices.
1.
x>0 x > 0 and y > 0 x=0
10.
(i) (ii) (iii)
(iv) y = 0 x < 0 and y ≥ 0 x= y
Find the midpoints of its sides. Show that the igure formed by joining the midpoints
(v) (vi)
x= y (viii) x ≥ 3 x > 2 and y =
consecutively is a parallelogram.
(vii) (ix) 2
(x) x and y have opposite signs.
version: 1.1 version: 1.1

8 9
1. Quadratic Equations eLearn.Punjab 1. Quadratic Equations eLearn.Punjab
4. Introduction to Analytic Geometry eLearn.Punjab 4. Introduction to Analytic Geometry eLearn.Punjab

11. Find h such that the quadrilateral with vertices A (-3, 0), B (1, -2), C (5, 0) and D (1, h ) Draw PM and O’ N perpendiculars to Ox .
is parallelogram. Is it a square? From the igure, we have
12. If two vertices of an equilateral triangle are A (-3, 0) and B (3, 0), ind the third OM= x, MP= y, ON= h, NO =' k= MM '
vertex. How many of these triangles are possible? Now X =O'M' = NM = OM - OM - ON =- x h
13. Find the points trisecting the join of A (-1, 4) and B (6, 2). Similarly, Y = M 'P = MP - MM ' = y k
14. Find the point three-ifth of the way along the line segment from A (-5, 8) to B (5, 3). Thus the coordinates of P referred to XY-system are ( x - h, y - k )
15. Find the point P on the join of A (1, 4) and B (5, 6) that is twice as far from A as B is i.e. X= x - h
from A and lies Y= y - k
(i) on the same side of A as B does. Moreover, x =+ X h, + y =Y k.

(ii) on the opposite side of A as B does.


Example 1: The coordinates of a point P are (-6, 9). The axes are translated through the
16. Find the point which is equidistant from the points A (5, 3),
B (-2, 2) and C (4, 2). What is the radius of the circumcircle of the ∆ABC ?
point O’ (-3, 2). Find the coordinates of P referred to the new axes.

The points (4, -2), (-2, 4) and (5, 5) are the vertices of a triangle. Find in-centre of
Solution. Here h- = 3,=
17.
k 2

Find the points that divide the line segment joining A ( x1 , y1 ) and B ( x2 , y2 ) into
the triangle.
Coordinates of P referred to the new axes are (X, Y) given by
18. X = -6 - (-3) = -3 and Y = 9 - 2 = 7
four equal parts. Thus P (X, Y) = P (-3 ,7).

4.2 TRANSLATION AND ROTATION OF AXES Example 2: The xy -coordinate axes are translated through the point O’ (4, 6). The
coordinates of the point P are (2, -3) referred to the new axes. Find the coordinates of P
Translation of Axes referred to the original axes.
Let xy-coordinate system be given and
O ' (h, k ) be any point in the plane. Through Solution: Here X = -2, Y =
= 3, h=4, k 6 .
O’ draw two mutually perpendicular lines We have x =X + h =4 + 2 =6
O’X , O’Y such that O’X is parallel to Ox . The y =Y + k =-3 + 6 =3
new axes O’X and O’Y are called translation
of the Ox - and Oy - axes through the point
Thus required coordinates are P (6, 3).

O’. In translation of axes, origin is shifted Rotation of Axes


to another point in the plane but the axes Let xy-coordinate system be given. We rotate
remain parallel to the old axes. Ox and Oy about the origin through an angle
Let P be a point with coordinates ( x, y ) referred to xy -coordinate system and the axes q (0 < q < 90 ) so that the new axes are OX and
be translated through the point O '(h, k ) and O’X, O’Y be the new axes. If P has coordinates OY as shown in the igure. Let a point P have
(X, Y) referred to the new axes, then we need to ind X, Y in terms of x, y. coordinates (x, y ) referred to the xy-system of

version: 1.1 version: 1.1

10 11
1. Quadratic Equations eLearn.Punjab 1. Quadratic Equations eLearn.Punjab
4. Introduction to Analytic Geometry eLearn.Punjab 4. Introduction to Analytic Geometry eLearn.Punjab

coordinates. Suppose P has coordinates (X, Y) referred to the XY-coordinate system. We have
to ind X, Y in terms of the given coordinates x, y. Let a be measure of the angle that OP  5 3 + 7 -5 + 7 3 
 
 
makes with O. i.e., (X, Y)
2 2

From P, draw PM perpendicular to Ox and PM’ perpendicular to OX. Let OP = r , From the are the required coordinates.
right triangle OPM ', we have

OM=' X= r cos (a - q )  4

M ' P= Y= r sin (a - q ) 
Example 4: The xy-axes are rotated about the origin through an angle of arctan
lying
(1) 3
in the irst quadrant. The coordinates of a point P referred to the new axes OX and OY
Also from the ∆ OPM , we have are P (-1, -7). Find the coordinates of P referred to the xy-coordinate system.
x = r cos a , y = r sin a
System of equations (1) may be re-written as: Solution. Let P(x, y) be the coordinates of P referred to the xy-coordinate system.

=X +r cos a cosq r sin a sin q  Angle of rotation is given by arctan q = . Therefore, sin q = , cosq = .
4 4 3
=Y -r sin a cosq r cos a sin q 
(2) 3 5 5
From equations (3) above, we have
Substituting from (2) into the above equations, we have X= x cos q + y sin q- and +Y =
x sin q y cosq

=X x cos q + y sin q 
-1 = x + y and - 7 =- x + y
Y y cosq - x sin q 
3 4 4 3
=
(3) or
3 x + 4 y +=
5 0 and - 4 x + 3 y +=
5 5 5 5

i.e., ( X , Y ) =
( x+cosq ysin q , x+sin q y cosq )
or 35 0
Solving these equations, we have
are the coordinates of P referred to the new axes OX and OY.
= = ⇒ x= 5, y = -5
x y 1
125 -125 25
Thus coordinates of P referred to the xy-system are (5, -5).
Example 3: The xy-coordinate axes are rotated about the origin through an angle of
300. If the xy-coordinates of a point are (5, 7), ind its XY-coordinates, where OX and OY are
the axes obtained after rotation.
EXERCISE 4.2
Solution. Let (X, Y) be the coordinates of P referred to the XY-axes. Here q = 300.
1. The two points P and O’ are given in xy-coordinate system. Find the XY-coordinates
From equations (3) above, we have
X=
5cos 30 + 7sin-30 and +Y =
of P refered to the translated axes O’X and O’Y.
P ( 3, 2 ) ; O ' (1, 3) P ( -2, 6 ) ; O ' ( -3, 2 )
5sin 30 7cos30
(i) (ii)
-5
+X = + =
P ( -6, - 8 ) ; O' ( -4, - 6 )
3 5  1 7
5 3 7 7 3
P  , ; O ' - , 
or and Y
2 2  2 2
2 2 2 2 (iii) (iv)

version: 1.1 version: 1.1

12 13
1. Quadratic Equations eLearn.Punjab 1. Quadratic Equations eLearn.Punjab
4. Introduction to Analytic Geometry eLearn.Punjab 4. Introduction to Analytic Geometry eLearn.Punjab

2. The xy-coordinate axes are translated through the point whose coordinates are Note: (i) If l is parallel to x-axis , then a = 0°
given in xy-coordinate system. The coordinates of P are given in the XY-coordinate (ii) If l is parallel to y-axis , then a = 90°
system. Find the coordinates of P in xy-coordinate system.
(i) P (8, 10); O’ (3, 4) (ii) P (-5, -3) ; O’ (-2 ,-6) Slope or gradient of a line: When we walk on

 3 7 1 1
P  - , - ; O '  , - 
an inclined plane, we cover horizontal distance

 4 6 4 6
(iii) (iv) P (4, -3); 0‘ (-2, 3) (run) as well as vertical distance (rise) at the same
time.
3. The xy-coordinate axes are rotated about the origin through the indicated angle. It is harder to climb a steeper inclined plane. The
The new axes are OX and OY. Find the XY-coordinates of the point P with the given measure of steepness (ratio of rise to the run) is
xy-coordinates.
P (5, 3 ); q = 450 (ii) P (3, -7); q = 300
termed as slope or gradient of the inclined path
(i) and is denoted by m .

P (11, -15); q = 600 P (15, 10): q = arctan m= = = tan a


1 rise y
(iii) (iv)
3
In analytical geometry, slope or gradient m of a non-vertical straight line with a as its
run x
4. The xy-coordinate axes are rotated about the origin through the indicated angle and
the new axes are OX and OY. inclination is deined by: m : tana

( )
Find the xy-coordinates of P with the given XY-coordinates. If l is horizontal its slope is zero and if l is vertical then its slope is undeined.
(i) P(-5, 3); q = 300 (ii) P -7 2, 5 2 ; q =
45ο If 0 < a < 900, m is positive and if 900 < a < 1800, then m is negative

4.3.1 Slope or Gradient of a Straight Line Joining Two Points


4.3 EQUATIONS OF STRAIGHT LINES
If a non-vertical line l with inclination a
Inclination of a Line: The angle a ( 0 < a < 180 ) passes through two points P ( x1 , y1 ) and Q ( x2 , y2 )
ο ο
measured counterclockwise from
positive x-axis to a non-horizontal straight line l is called the inclination of l . , then the slope or gradient m of l
y2 - y1
is given= = tana
x2 - x1
by m

Proof: Let m be the slope of the line l .


Draw perpendiculars PM and QM‘ on x-axis and a perpendicular PR on QM‘
Then a , mPR =
∠RPQ = x2 - x1 and mQR =y2 - y1

y2 - y1
Observe that the angle a in the diferent positions of the line l is a, 0 and 90 The slope or gradient of l is deined as: m = tan a =
x2 - x1
0 0 .
respectively.
version: 1.1 version: 1.1

14 15
1. Quadratic Equations eLearn.Punjab 1. Quadratic Equations eLearn.Punjab
4. Introduction to Analytic Geometry eLearn.Punjab 4. Introduction to Analytic Geometry eLearn.Punjab

p
When 0 < a <
Theorem: The two lines l1 and l2 with respective Remember that:
Case (i). slopes m1 and m2 are
2 The symbol
In the right triangle PRQ , we have (i)  stands for “parallel”.
(i) parallel if m1 = m2
y2 - y1
(ii)  stands for “not parallel”.
= = a (iii) ⊥ stands for “perpendicular”.
x2 - x1 -1
perpendicular if m1 =
m tan
(ii)
m2

m1m2 + 1 =
p
<a <p
or 0
Case (ii) When
2
In the right triangle PRQ Example 1: Show that the points A(-3, 6), B(3, 2) and C(6, 0) are collinear.

y2 - y1
tan (p - a ) =
Solution: We know that the points A, B and C are collinear if Notice that:
x1 - x2 the line AB and BC have the same slopes. Here Slope of Slope of AB = slope of AC
y2 - y1
- tan a = 2-6 -4 -4 -2 0 - 2 -2
x1 - x2 = = = = and slope of=
BC =
3 - ( -3) 3 + 3 6
or
6-3 3
AB
y2 - y1 y2 - y1
3
tan a = m=
x2 - x1 x2 - x1
or or
a Slope of AB = Slope of BC

Thus if P ( x1 , y1 ) and Q ( x2 , y2 ) are two points on a line, then slope of PQ is given by:
Thus A, B and C are collinear.

Show that the triangle with vertices A (1, 1), B (4, 5) and C (12, -1) is a right
y2 - y1 y1 - y2
Example 2:
x2 - x1 x1 - x2
m or m triangle.

5 -1 4
= m= =
4 -1 3
Solution: Slope of AB 1

y2 - y1 y - y2 -1 - 5 -6 -3
m≠ and m ≠ 1 = m= = =
x1 - x2 x2 - x1 12 - 4 8
Note: (i) and Slope of BC 2

l is horizontal, if m = 0 (a a = 00)
4
 4  3 
Since m1m2 =
  --  =1, therefore, AB ⊥ BC
(ii)
(iii) l is vertical, if m is not deined (a a = 900)  3  4 
So ∆ABC is a right triangle.
(iv) If slope of AB = slope of BC, then the points A, B and C are collinear.

version: 1.1 version: 1.1

16 17
1. Quadratic Equations eLearn.Punjab 1. Quadratic Equations eLearn.Punjab
4. Introduction to Analytic Geometry eLearn.Punjab 4. Introduction to Analytic Geometry eLearn.Punjab

4.3.2 Equation of a Straight Line Parallel to the x-axis Proof: Let P (x, y) be an arbitrary point of the straight line l with slope m and y-intercept
(or perpendicular to the y-axis) c. As C (0, c) and P (x, y) lie on the line, so the slope of the line is:

y-c
m= or y - c = mx and y = mx + c
x-0
is an equation of l .
The equation of the line for which
c = 0 is
y = mc
In this case the line passes through the origin.

Example 1: Find an equation of the straight line if


(a) its slope is 2 and y-intercept is 5
All the points on the line l parallel to x-axis remain at a constant distance (say a) from

it is perpendicular to a line with slope -6 and its y-intercept is


x-axis. Therefore, each point on the line has its distance from x-axis equal to a, which is its
y-coordinate (ordinate). So, all the points on this line satisfy the equation: y = a
4
(b)
3
Solution: (a) The slope and y-intercept of the line are respectively:
Note: (i) If a > 0, then the line l is above the x-axis. m=2 and c=5
(ii) If a < 0, then the line l is below the x-axis. Thus y = 2x + 5 (Slope-intercept form: y = mx + c)
(iii) If a = 0, then the line l becomes the x-axis. is the required equation.
Thus the equation of x-axis is y = 0 (b) The slope of the given line is
m1 = -6
4.3.4 Derivation of Standard Forms of Equations of
∴ m2-==
Straight Lines 1 1
The slope of the required line is:
m1 6
Intercepts: The slope and y-intercept of the required line are respectively:
• If a line intersects x-axis at (a, 0), then a is called
m= (slope of ⊥ line is -6) and c =
1 4
x-intercept of the line. 6 3

( x+) =
• If a line intersects y-axis at (0, b), then b is called
+y =
1 4
y-intercept of the line. Thus or 6y x 8
6 3
1. Slope-Intercept form of Equation of a Straight Line: is the required equation.

Theorem: Equation of a non-vertical straight line with slope m and y-intercept c is 2. Point-slope Form of Equation of a Straight Line:
given by:
=
y mx + c
Theorem: Equation of a non-vertical straight line l with slope m and passing through a
point Q (x1 , y1) is
version: 1.1 version: 1.1

18 19
1. Quadratic Equations eLearn.Punjab 1. Quadratic Equations eLearn.Punjab
4. Introduction to Analytic Geometry eLearn.Punjab 4. Introduction to Analytic Geometry eLearn.Punjab

y - y1= m ( x - x1 )
4. Two-point Form of Equation of a Straight Line:

Proof: Let P(x, y) be an arbitrary point of the straight line


with slope m and passing through Q(x1 , y1). Theorem: Equation of a non-vertical straight line
As Q(x1 , y1) and P(x, y) both lie on the line, so the slope of passing through two points Q(x1 , y1) and R(x2 , y2) is

y2 - y1 y2 - y1
( x - x1 ) or y= ( x - x2 )
the line is
y - y1 =
y - y1 - y2
m= or y - y1= m ( x - x1 ) x2 - x1 x2 - x1
x - x1

which is an equation of the straight line passing through x1 , y1 with slope m.

3. Symmetric Form of Equation of a Straight Line: Proof: Let P (x, y) be an arbitrary point of the line passing through Q (x1 , y1) and

y - y1
R (x2 , y2). So
= tan a , where a is the inclination of the line.
x - x1 y - y1 y - y2 y2 - y1
We have
= =
x - x1 x - x2 x2 - x1
(P, Q and R are collinear points)
x - x1 y - y1
or = = r ( say )
cos a sin a We take

y - y1 y2 - y1
=
This is called symmetric form of equation of the line.
x - x1 x2 - x1
y2 - y1
=
y - y1 ( x - x1 ) the required equation of the line PQ .
Example 2: Write down an equation of the straight line passing through (5, 1) and
parallel to a line passing through the points (0,-1), (7, -15).
x2 - x1
or

or ( y2 - y1 ) x - ( x2 - x1 ) y + ( x1 y2 - x2 y1 ) =
0
Solution: Let m be the slope of the required straight line, then

-15 - ( -1)
m=
x y 1
7-0 y1 1 = 0
(a Slopes of parallel lines are equal)
We may write this equation in determinant form as: x1
= -2 x2 y2 1
As the point (5, 1) lies on the required line having slope -2 so, by point-slope form of Note: (i) If x1 - x2, then the slope becomes undeined. So, the line is vertical.
equation of the straight line, we have
y2 - y1
y - (1) = -2(x - 5) (ii) y=
- y2 ( x - x2 ) can be derived similarly.
or y = -2x + 11 x2 - x1

or 2x + y - 11 = 0
is an equation of the required line.

version: 1.1 version: 1.1

20 21
1. Quadratic Equations eLearn.Punjab 1. Quadratic Equations eLearn.Punjab
4. Introduction to Analytic Geometry eLearn.Punjab 4. Introduction to Analytic Geometry eLearn.Punjab

Example 3: Find an equation of line through the points (-2, 1) and (6, -4).
= + =1 - or
+
x y
2 -4
2x y 4 0
Solution: Using two-points form of the equation of straight line, the required equation is
which is the required equation.
-4 - 1
=
y -1  x - ( -2 ) 
6 - ( -2 )  Example 5: Find an equation of the line through the point

-5
( x + 2 ) or 5 x + 8 y +=
P(2, 3) which forms an isosceles triangle with the coordinate
or y=
-1 2 0 axes in the irst quadrant.
8

5. Intercept Form of Equation of a Straight Line: Solution: Let OAB be an isosceles triangle so
that the line AB passes through A = (a, 0) and
Theorem: Equation of a line whose non-zero x and B(0, a), where a is some positive real number.

a-0
y-intercepts are a and b respectively is
Slope of AB = = - 1 . But AB passes through P (2, 3).
+ =
x y 0-a
Equation of the line through P(2, 3) with slope -1 is
1
a b a

Proof: Let P(x , y) be an arbitrary point of the line y - 3 =-1( x - 2 ) or x + y - 5 =0


whose non-zero x and y-intercepts are a and b respectively.
Obviously, the points A(a, 0) and B(0, b) lie on the required 6. Normal Form of Equation of a Straight Line:
line. So, by the two-point form of the equation of line,
Theorem: An equation of a non-vertical straight line l , such that length of the perpendicular
from the origin to l is p and a is the inclination of this perpendicular, is
we have

b-0
y=
-0 ( x - a) x cos a + y sin a =
0-a
(P, A and B are collinear) p

or -ay = b ( x - a )
or bx + ay = ab Proof: Let the line l meet the x-axis and y-axis at the

+ =
x y points A and B respectively. Let P (x, y) be an arbitrary
or 1 (dividing by ab) point of AB and let OR be perpendicular to the line l .
Then OR = p .
a b
Hence the result.

From the right triangles ORA and ORB, we have,


Example 4: Write down an equation of the line which cuts the x-axis at (2, 0) and y-axis
at (0, -4). cos a =
cos a
p p
or OA =
OA
Solution: As 2 and -4 are respectively x and y-intercepts of the required line, so by cos(90 - a ) =
sin a
p p
and or OB =
OB
two-intercepts form of equation of a straight line, we have
version: 1.1 version: 1.1

22 23
1. Quadratic Equations eLearn.Punjab 1. Quadratic Equations eLearn.Punjab
4. Introduction to Analytic Geometry eLearn.Punjab 4. Introduction to Analytic Geometry eLearn.Punjab

[∴cos(90 - a ) =
sin a )]
Proof: Here a and b cannot be both zero. So the following cases arise:
Case I: a≠0 , b= 0
As OA and OB are the x and y-intercepts of the line AB, so equation of AB is
In this case equation (1) takes the form: Remember that:

+ = ax + c =0 or x =-
The equation (I) represents
p / cos a p / sin a
x y c
1 (Two-intercept form)

That is x cos a + y sin a =


a a straight line and is called
p is the required equation.
which is an equation of the straight line parallel to the general equation of a

the y-axis at a directed distance - from the y-axis.


c straight line.
Example 6: The length of perpendicular from the origin to a line is 5 units and the

Case II:= a 0 , b≠0


inclination of this perpendicular is 1200. Find the slope and y-intercept of the line. a

Solution. Here p = 5, a = 1200. In this case equation (1) takes the form:

bx + c = y=-
Equation of the line in normal form is c
x cos120 + y sin120 =
0 or
b
-c
5

⇒ - x+ y=
1 3 which is an equation of the straight line parallel to x-axis at a directed distance
5 b
from the x-axis.

2 2
x - 3 y + 10 =
0 (1) Case III: a≠0 , b≠0
In this case equation (1) takes the form:
To ind the slope of the line, we re-write (1) as:= +
x 10
-a
y
by =
- ax - c or y = x - =mx + c
3 3 c
which is slope-intercept form of the equation.
b b

Here=m =
1
and c
10 -a -c
3 3 which is the slope-intercept form of the straight line with slope and y-intercept
b b
.
4.3.5 A Linear Equation in two Variables Represents Thus the equation ax + by + c =0 , always represents a straight line.
a Straight Line

The linear equation ax + by + c =


4.3.6 To Transform the General Linear Equation
Theorem: 0 in two variables x and y represents a to Standard Forms
straight line. A linear equation in two variables x and y is
ax + by + c =0 (1) Theorem: To transform the equation ax + by + c = 0 in the standard form
where a, b and c are constants and a and b are not simultaneously zero.
1. Slope-Intercept Form.
We have
version: 1.1 version: 1.1

24 25
1. Quadratic Equations eLearn.Punjab 1. Quadratic Equations eLearn.Punjab
4. Introduction to Analytic Geometry eLearn.Punjab 4. Introduction to Analytic Geometry eLearn.Punjab

-a -a -c
5. Intercept Form.
by =
- ax - c or y = x - =mx + c, where m = , c =
c
ax + by =
-c + = + 1 =
ax by x y
-c -c -c / a -c / b
b b b b or 1 i.e
2. Point - Slope Form
which is an equation in two intercepts form.
-a
We note from (1) above that slope o f the line ax + by + c =0 is . A point on the
b
 -c 
6. Normal Form.
line is  ,0 
 a  The equation: ax + by + c =0 (1)
-a  c
=  + 
can be written in the normal form as:
b  a
Equation of the line becomes y x

ax + by -c
=
which is in the point-slope form.
± a +b ± a +b
(2)
2 2 2 2
3. Symmetric Form

-a
The sign of the radical to be such that the right hand side of (2) is positive.
= = a == a , cos a
a b
± a 2 + b2 ± a 2 + b2
m tan . sin Proof. We know that an equation of a line in normal form is
x cos a + y sin a =
b
 -c 
A point on ax + by + c = 0 is  ,0 
p (3)
 a  If (1) and (3) are identical, we must have
Equation in the symmetric form becomes
-c
 c = =
cos a sin a
a b
x -- 
 a y-0
p
= = p cos a sin a cos 2 a + sin 2 a
b / ± a 2 + b2 a / ± a 2 + b2 = = = =
r 1
-c ± a 2 + b2 ± a 2 + b2
i.e.,
is the required transformed equation. Sign of the radical to be properly chosen. a b

=
= cos a sin a
a b
± a +b ± a 2 + b2
4. Two -Point Form Hence, and

ax + by + c =. Substituting for cos a , sin a and p into (3), we have


2 2

We choose two arbitrary points on 0 Two such points are

 -c   -c  ax + by -c
 ,0  and  0,  . Equation of the line through these points is =
 a   b  ± a 2 + b2 ± a 2 + b2
Thus (1) can be reduced to the form (2) by dividing it by ± a 2 + b 2 . The sign of the
x+
y-0 -a  c
c
= =- y 0 + x 
radical to be chosen so that the right hand side of (2) is positive.
b  a
a
0+ - -0
i.e.,
c c
b a

version: 1.1 version: 1.1

26 27
1. Quadratic Equations eLearn.Punjab 1. Quadratic Equations eLearn.Punjab
4. Introduction to Analytic Geometry eLearn.Punjab 4. Introduction to Analytic Geometry eLearn.Punjab

Example 1: Transform the equation 5x - 12y + 39 = 0 into Example 2: Sketch the line
(i) Slope intercept form. (ii) Two-intercept form. 3 x + 2 y + 6 =.
0 (1)
(iii) Normal form. (iv) Point-slope form.
(v) Two-point form. (vi) Symmetric form. Solution: To sketch the graph of (1), we ind two points on it.
If y = 0, x = -2 and if x = 0 , y = -3 .
Solution: Thus x intercept = -2
y intercept = -3
We have 12 y = +5 x 39 or = +x = , y-intercept c = The points A(-2, 0), B(0, -3) are on (1). Plot these points in the
5 39 5 39
(i) y ,m
12 12 12 12
5 x - 12-y =39 or + = 1 or + = 1 is the required equation.
5 x 12 y x y plane and draw the straight line through A and B. It is the graph
-39 39 -39 / 5 39 / 12
(ii) of (1).

(iii) 5 x - 12-y =39 . Divide both sides by ± 52 + 12±2 =13 . Since R.H.S is to be Example 3: Find the distance between the parallel lines
positive, we have to take negative sign. 2x + y + 2 =0 (1)
6x + 3y - 8 =
+ =
and 0 (2)
5 x 12 y
-13 13
Hence = 3 is the normal form of the equation. Sketch the lines. Also ind an equation of the line parallel to the given lines and lying midway

 -39 
between them.
A point on the line is  ,0  and its slope is
5
 
(iv) .
5 12
5 39 
Solution: We irst convert both the lines into normal form. (1) can be written as
Equation can be written as: y -= x+  2x + y =-2
12  5 
0
Dividing both sides by - 4 + 1 , we have
 39   -39   39 
Another point on the line is  0,  . Line through  ,0  and  0,  is
 12   5   12  -2 -y
(v)
x+ =
2
(3)
x+
y-0
39 5 5 5
=
39 -39
5 which is normal form of (1). Normal form of (2) is
0- -0
+ =
12 5 6x 3y 8

 -39 
We have tan a
= = m, sin a
= ,cos a
=
45 45 45
. A point of the line is  ,0  .
5 5 12
 5  + =
(vi) 2x y 8
12 13 13 i.e., (4)
5 5 3 5
Equation of the line in symmetric form is

x + 39 / 5 y - 0
= = r (say)
Length of the perpendicular from (0, 0) to the line (1) is [ From (3)]
12 / 13 5 / 13
8
Similarly, length of the perpendicular from (0, 0) to the line (2) is [From (4)]
3 5

version: 1.1 version: 1.1

28 29
1. Quadratic Equations eLearn.Punjab 1. Quadratic Equations eLearn.Punjab
4. Introduction to Analytic Geometry eLearn.Punjab 4. Introduction to Analytic Geometry eLearn.Punjab

The point P ( x1 , y1 ) is above the line if y1 > y′ that is


From the graphs of the lines it is clear that the lines are on

y1 - y′ > 0
opposite sides of the origin, so the distance between them
equals the sum of the two perpendicular lengths.
 a c
y1 -  - x1 -  > 0
+ =  b b
2 8 14 i.e.
i.e., Required distance =
⇒⇒ ax1 + by1 + c > 0
5 3 5 3 5

Similarly P ( x1 , y1 ) is below the line if


The line parallel to the given lines lying midway between
them is such that length of the perpendicular
 a c
 2  y1 - y′ < 0 i.e. y1 -  - x1 - 
-  or - =  b b
8 7 7 1
3 5 3 5  3 5 5 3 5 ax1 + by1 + c < 0
from O to the line =

The point P ( x1 , y1 ) is on the line if


or
Required line is = = + or 6=x + 3y 1
2x y 1

ax1 + by1 + c =
5 5 3 5

Corollary 1. The point P is above or below l respectively if ax1 + by1 + c and b have the
4.3.7 Position of a point with respect to a line
same sign or have opposite signs.
Consider a non-vertical line l
l : ax + by + c = ax1 + by1 + c
Proof. If P is above l , then y1 - y′ > 0 >0
0
i.e.,
in the xy-plane. Obviously, each point of the plane is either above b
Thus ax1 + by1 + c and b have the same sign.
the line or below the line or on the line.

Let P ( x1 , y1 ) be a point in the plane not lying on


Similarly, P is below l if

ax1 + by1 + c
Theorem:
l y1 - y′ < 0 <0
l : ax + by + c =
i.e.,
b
Thus ax1 + by1 + c and b have opposite signs.
0 (1)

Corollary 2. The point P ( x1 , y1 ) and the origin are


then P lies
a) above the line (1) if ax1 + by1 + c > 0
b) below the line (1) if ax1 + by1 + c < 0 (i) on the same side of l according as ax1 + by1 + c and c have the same sign.
on the opposite sides of l according as ax1 + by1 + c and c have opposite signs.
Proof. (i) The point P ( x1 , y1 ) and O (0,0) are on the same side of l if ax1 + by1 + c and
(ii)
Proof: We can suppose that b > 0 (irst multiply the
equation by -1 if needed). Draw a perpendicular from P on a.0 + b.0 + c have the same sign.
x-axis meeting the line at Q( x1 , y′) . (ii) Proof is left as an exercise
Thus ax1 + by′ + c =0 so that

y′ =
- - x1
a c
b b
version: 1.1 version: 1.1

30 31
1. Quadratic Equations eLearn.Punjab 1. Quadratic Equations eLearn.Punjab
4. Introduction to Analytic Geometry eLearn.Punjab 4. Introduction to Analytic Geometry eLearn.Punjab

Check whether the point ( -2 , 4 ) lies above or below the line


l1  l2 ⇔ slope of l1 (m1 ) = slope of l2 (m2 ) .
Example 1:
4x + 5 y - 3 =
(i)
0 (1)

⇔ - =- 2
a1 a
Solution: Here b = 5 is positive. Also b1 b2
4 (-2) + 5(4) - 3 = -8 + 20 - 3 = 9 > 0
⇔ - =- 2 ⇔ a1b2 - b1a2 =0
(2) a1 a
The coeicient of y in (1) and the expression (2) have the same sign and so the point b1 b2

l1 ⊥ l2 ⇔ m1m2 =-1
(-2, 4) lies above (1).
(ii)

Check whether the origin and the point P (5, -8) lie on the same side or on  a  a 
⇔  - 1  - 2  =-1 ⇔ a1a2 + b1b2 =0
Example 2:

 b1  b2 
the opposite sides of the line:
3 x + 7 y + 15 =
0 (1)
(iii) If l1 and l2 are not related as in (i) and (ii), then there is no simple relation of the
Solution: above forms.
Here c = 15
For P (5, -8), 4.4.1 The Point of Intersection of two Straight Lines
3(5) + 7(-8) + 15 = -26 < 0
Let l1 : a1 x + b1 y + c1 =
(2)
0 (1)
and l2 : a2 x + b2 y + c2 =
But c = 15 >0
c and the expression (2) have opposite signs. Thus O (0, 0) and P (5, -8) are on the opposite 0 (2)
sides of (1). be two non-parallel lines. Then a1b2 - b1a2 ≠ 0
Let P( x1 , y1 ) be the point of intersection of l1 and l2 . Then
Note: To check whether a point P(x1 , y1) lies above or below the line a1 x1 + b1 y1 + c1 =
0 (3) Recall that:
ax + by + c = 0 a2 x1 + b2 y1 + c2 =0 (4) Two non-parallel lines
we make the co-eicient of y positive by multiplying the equation by (-1) if needed. Solving (3) and (4) simultaneously, we have intersect each other at
one and only one point.
= =
x1 y1 1
4.4 TWO AND THREE STRAIGHT LINES b1c2 - b2c1 a2c1 - a1c2 a1b2 - a2b1
b1c2 - b2c1 a2c1 - a1c2
=
=
a1b2 - a2b1 a1b2 - a2b1
For any two distinct lines l1 , l2 . x1 and y1
l1 : a1=
x + b1 y + c 0 and l2 : a2 x=+ b2 y + c 0 , one and only one of the
is the required point of intersection.
following holds: Recall that:
(i) l1  l2 (ii) l1 ⊥ l2 (iii) l1 and l2 are not related as (i) or (ii). Two non-parallel lines
intersect each other at Note: a1b2 - a2b1 ≠ 0, otherwise l1  l2 .
- m-1 =
=
a1 a2
The slopes of l1 and l2 are , m2 one and only one point.
b1 b2

version: 1.1 version: 1.1

32 33
1. Quadratic Equations eLearn.Punjab 1. Quadratic Equations eLearn.Punjab
4. Introduction to Analytic Geometry eLearn.Punjab 4. Introduction to Analytic Geometry eLearn.Punjab

Examples 1: Find the point of intersection of the lines An easier way to write the above equation is in the following determinant form:
5x + 7 y =
35 (i)
3x - 7 y =
a1 b1 c1
21 (ii) a2 b2 c2 = 0
a3 b3 c3
Solution: We note that the lines are not parallel and so they Remember that:
This is a necessary and suicient condition of concurrency of the given three lines.
must intersect at a point. Adding (i) and (ii), we have * If the lines are parallel,

exist ( a1b2 - a2b1 =


0)
then solution does not
Example 1: Check whether the following lines are concurrent or not. If concurrent, ind
8x = 56 or x=7
the point of concurrency.
3x - 4 y - 3 =
Setting this value of x into (1), we ind, y = 0. * Before solving equations 0 (1)
5 x + 12 y + 1 =
Thus (7, 0) is the point of intersection of the two lines. one should ensure that 0 (2)
lines are not parallel. 32 x + 4 y - 17 = 0 (3)

4.4.2 Condition of Concurrency of Three Straight Lines Solution. The determinant of the coeicients of the given equations is
-4 - 3 18 32
1 ,by R1 + 3R2
3 0
1 = 5
Three non-parallel lines
l1 : a1 x + b1 y + c1 = 5 12 12
- 17 117 208
(1)
0 and R3 + 17 R2
0
l2 : a2 x + b2 y + c2 =
0 (2) 32 4
l3 : a3 x + b3 y + c3 =
=
-1 - ( 208 × 18 - 117 × 32 ) =
=
0 (3) 18 32
0
117 208
a1 b1 c1 Thus the lines are concurrent.
are concurrent if a2 b2 c2 = 0 The point of intersection of any two lines is the required point
a3 b3 c3 of concurrency. From (1) and (2), we have

= =
x y 1
-4 + 36 -15 - 3 36 + 20
P ( x1 , y1 ) say. As l1  l2 , so their point of intersection ( x, y ) is
Proof: If the lines are concurrent then they have a common point of intersection

-18 -9  4 -9 
=x = and = = i.e.  , 
32 4
b1c2 - b2c1 a2c1 - a1c2  7 28 
y
=
=
56 7 56 28
a1b2 - a2b1 a1b2 - a2b1
x and y is the point of intersection.

This point also lies on (3), so


4.4.3 Equation of Lines through the point of
 bc -b c  a c -ac 
a3  1 2 2 1  + b3  2 1 1 2  + c3 =
intersection of two lines
 1 2 - 2 1   1 2 - 2 1 
0

a3 ( b1c2 - b2c1 ) + b3 ( a2c1 - a1c2 ) + c3 ( a1b2 - a2b1 ) =


a b a b a b a b
We can ind a family of lines through the point of intersection of two non parallel lines
or 0
l1 and l2 .
version: 1.1 version: 1.1

34 35
1. Quadratic Equations eLearn.Punjab 1. Quadratic Equations eLearn.Punjab
4. Introduction to Analytic Geometry eLearn.Punjab 4. Introduction to Analytic Geometry eLearn.Punjab

l1 : a1 x + b1 y + c1 =
3+ k -2
Do you remember?
- = or 9 + 3k = - 8 + 4k i.e., k = 17
Let 0 (1)
and l2 : a2 x + b2 y + c2 =
0 (2) An ininite number of
- 4 + 2k
Substituting k = 17 into (3), equation of the member of the family is
3
lines can pass through

a1 x + b1 y + c1 + h ( a2 x + b2 y + c2 ) =
For a non-zero real h, consider the equation
20 x + 30 y - 180 = 0 i.e., 2 x + 3 y - 18 =
a point
0 (3) 0
This, being a linear equation, represents a straight line. For diferent values of h, (3) (ii) Slope of 3 x - 4 y + 1 =0 (4)
represents diferent lines. Thus (3) is a family of lines.
. Since (3) is to be perpendicular to (4), we have - 3 + k × 3 =- 1
3
- 4 + 2k
is
or 9 + 3k =- 16 + 8k k =5
If ( x1 , y1 ) is any point lying on both (1) and (2), then it is their point of intersection. Since 4 4
or
Inserting this value of k into (3), we get 4 x + 3 y - 30 =
(x1 , y1) lies on both (1) and (2), we have
a1 x= + b1 y + c1 0=
+ and+ a2 x b2 y c2 0 0 which is required equation of
From the above two equations, we note that ( x1 , y1 ) also lies on (3). the line.

Thus (3) is the required family of lines through the point of intersection of (1) and (2).
Since h can assume an ininite number of values, (3) represents an ininite number of lines. Theorem: Altitudes of a triangle are concurrent.

Let the coordinates of the vertices of ∆ ABC be as


A particular line of the family (3) can be determined if one more condition is given.
Proof.
shown in the igure.
y - y3
Example 2: Find the family of lines through the point of intersection of the lines
3 x - 4 y - 10 = Then slope of BC = 2
x2 - x3
0 (1)
x + 2 y - 10 =
x2 - x3
0 (2)
Therefore slope of the altitude AD = -
y2 - y3
Find the member of the family which is

-2 Equation of the altitude AD is


(i) parallel to a line with slope
perpendicular to the line l : 3 x - 4 y + 1 =
3
x2 - x3
y - y1 =- ( x - x1 )
(ii) 0.
y2 - y3
(Point-slope form)

x (x2 - x3) + y (y2 - y3) - x1 (x2 - x3) - y1 (y2 - y3) = 0


Solution: (i) A family of lines through the point of intersection of equations (1) and (2) is
3 x - 4 y - 10 + k ( x + 2 y - 10) =
or (1)
0
(3 + k ) x + (- 4 + 2k ) y + (-10 - 10k ) =0
Equations of the altitudes BE and CF are respectively (by symmetry)
x ( x3 - x1 ) + y ( y3 - y1 ) - x2 ( x3 - x1 ) - y2 ( y3 - y1 ) =
or (3)
0 (2)
3+ k and x ( x1 - x2 ) + y ( y1 - y2 ) - x3 ( x1 - x2 ) - y3 ( y3 - y1 ) =
Slope m of (3) is given by: m = -
0 (3)
- 4 + 2k The three lines (1), (2) and (3) are concurrent if and only if
This is slope of any member of the family (3).
x2 - x3 y2 - y3 - x1 ( x2 - x3 ) - y1 ( y2 - y3 )
If (3) is parallel to the line with slope -
D= x3 - x1 y3 - y1 - x2 ( x3 - x1 ) - y2 ( y3 - y1 )
2
then
3 is zero
x1 - x2 y1 - y2 - x3 ( x1 - x2 ) - y3 ( y1 - y2 )
version: 1.1 version: 1.1

36 37
1. Quadratic Equations eLearn.Punjab 1. Quadratic Equations eLearn.Punjab
4. Introduction to Analytic Geometry eLearn.Punjab 4. Introduction to Analytic Geometry eLearn.Punjab

x2 - x3 y2 - y3 - ( y2 2 - y32 ) - ( x2 2 - x32 )
Adding 2nd and 3rd rows to the 1st row of the determihant, we have 1 1
2 2
x3 - x1 y3 - y1 - ( y32 - y12 ) - ( x32 - x12 )
1 1
0 0 0 = 0
x3 - x1 y3 - y1 - x2 ( x3 - x1 ) - y2 ( y3 - y1 ) =
2 2
x1 - x2 y1 - y2 - ( y12 - y2 2 ) - ( x12 - x2 2 )
0
x1 - x2 y1 - y2 - x3 ( x1 - x2 ) - y3 ( y1 - y2 )
1 1
2 2
Thus the altitudes of a triangle are concurrent.
Adding 2nd and 3rd rows to 1st row of the determinant, we have

Theorem: Right bisectors of a triangle are concurrent.


0 0 0

x3 - x1 y3 - y1 - ( y32 - y12 ) - ( x32 - x12 ) =


1 1
0
Proof. Let A ( x1 , y1 ) , B ( x2 , y2 ) and C ( x3 , y3 ) be the vertices
of ∆ABC
2 2
x1 - x2 y1 - y2 - ( y12 - y2 2 ) - ( x12 - x2 2 )
1 1
The midpoint D of BC has coordinates 2 2

 x2 + x3 y2 + y3 
Thus the right bisectors of a triangle are concurrent.
 
 2 2 
,

y - y3 x -x
, the slope of the right bisector DO of BC is - 2 3
Note: If equations of sides of the triangle are given, then intersection of any two lines
x2 - x3 y2 - y3
Since the slope of BC is 2 gives a vertex of the triangle.

Equation of the right bisector DO of BC is


4.4.4 Distance of a point from a line
y2 + y3 x2 - x3  x2 + x3 
y - - = - x  (Point-slope form)
2 y2 - y3  2  Theorem: The distance d from the point P( x1 , y1 ) to the line l
or x ( x2 - x3 ) + y ( y2 - y3 ) - ( y2 2 - y32 ) - ( x2 2 - x32 ) =
1 1
0 (1) l : ax + by + c =0 (1)

ax1 + by1 + c
2 2
is given by d =
a 2 + b2
By symmetry, equations of the other two right bisectors EO and FO are respectively:

x ( x3 - x1 ) + y ( y3 - y1 ) -( y32 - y12 ) - ( x32 - x12 ) =


1 1 Proof: Let l be non-vertical and
0 (2)
2 2 non-horizontal line.
and x ( x1 - x2 ) + y ( y1 - y2 ) - ( y12 - y2 2 ) - ( x12 - x2 2 ) =
1 1 From P, draw
PQR ⊥ Ox and PM ⊥ l .
0 (3)
2 2

The lines (1), (2) and (3) will be concurrent if and only if Let the ordinate of Q be y2 so that
coordinates of Q are ( x1 , y2 ) . Since Q lies on
l , we have ax1 + by2 + c =0

version: 1.1 version: 1.1

38 39
1. Quadratic Equations eLearn.Punjab 1. Quadratic Equations eLearn.Punjab
4. Introduction to Analytic Geometry eLearn.Punjab 4. Introduction to Analytic Geometry eLearn.Punjab

Solution: First ind any point on one of the lines, say l1 . If x = 1 Challenge!
-ax1 - c
or y2 = lies on l1 , then Check the answer by

From the igure it is clear that ∠MPQ = a = the


b
y = 3 and the point (1,3) lies on it. The distance d from (1, 3) taking
to l2 is (i) any other point on l1
inclination of l .
(ii) any point of l2 and
-a 2(1) - 5(3) + 6 2 - 15 + 6
== tan a = = =
7 inding its distance from l1
(-2) 2 + 52 4 + 25
Now slope of l d
b 29
Therefore, cos a =
b 7
a 2 + b2
The distance between the parallel lines is .

PM = d= PQ cos a= y1 - y2 cos a
29
Thus

-ax1 - c
= y1 -
b 4.4.6 Area of a Triangular Region Whose Vertices are Given
a 2 + b2
.
b
by1 + ax1 + c ax1 + by1 + c To ind the area of a triangular region whose vertices are:
=
=
b a 2 + b2 a 2 + b2
.b P ( x1 , y1 ) , Q( x2 , y2 ) and R ( x3 , y3 ) .

If l is horizontal, its equation is of the form y = - and the distance from P( x1 , y1 ) to l


c Draw perpendiculars PL , QN and RM on x -axis.
b Area of the triangular region PQR
is simply the diference of the y-values.
= Area of the trapezoidal region PLMR
 c by1 + c
∴ d= y1 -  -  =
+ Area of the trapezoidal region RMNQ
 b - Area of the trapezoidal region PLNQ .

( )( ) ( )( ) ( )( )
b
-c ax1 + c
==
Similarly, if the line is vertical and has equation: x then d 1
=
PL + RM LM +
1
RM + QN MN -
1
PL + QN LN
a a 2 2 2
= [( y1 + y3 )( x3 - x1 )+ ( y3+ y2 )( x2- x3 )- ( y1+ y2 )( x2- x1 )]
Note:
If the point P( x1 , y1 ) lies on l , then the distance d is zero, sinceP ( x1 , y1 ) satisies the 1
equation i.e., ax1 + by1 + c = 0 2
= ( x3 y1 + x3 y3 - x1 y1 + x1 y3 + x2 y3 + x2 y2 - x3 y3 - x2 y1 - x2 y2 + x1 y1 + x1 y2 )
1

= ( x3 y1 - x1 y3 + x2 y3 - x3 y2 - x2 y1 + x1 y2 )
2
4.4.5 Distance Between two Parallel Lines 1
Have you observed that:
2
Thus required area A is given by: x1 y1 1
∆ = x2 y2 1
The distance between two parallel lines is the distance from any point on one of the 1

=∆ [ x1 ( y2 - y3 ) + x2 ( y3 - y1 ) + x3 ( y1 - y2 )]
lines to the other line. 2
1
x3 y3 1
2
Example: Find the distance between the parallel lines
l :2 x - 5 y + 13 =0 and Corollary: If the points P,Q and R are collinear, then
l2 :2 x - 5 y + 6 =0 ∆ =0
version: 1.1 version: 1.1

40 41
1. Quadratic Equations eLearn.Punjab 1. Quadratic Equations eLearn.Punjab
4. Introduction to Analytic Geometry eLearn.Punjab 4. Introduction to Analytic Geometry eLearn.Punjab

Note: In numerical problems, if sign of the area is negative, then it is to be omitted. EXERCISE 4.3

Example 1: Find the area of the region bounded by the triangle with vertices (a,b + c ) , 1. Find the slope and inclination o f the line joining the points:
(a , b - c) and (-a , c). (3, -2) ; (2, 7)
Solution: Required area ∆ is
(i) (-2, 4) ; (5, 11) (ii) (iii) (4, 6) ; (4, 8)

a b+c 1
Sketch each line in the plane.

In the triangle A (8, 6) B (-4, 2), C (-2 , -6) , ind the slope of
=∆ a b-c 1
1 Trapezium: 2.

-a
2 A quadrilateral having two parallel and two (i) each side of the triangle
c 1
a b+c 1
non-parallel sides. (ii) each median of the triangle

- = -0 ,by R2
Area of trapezoidal region:
1 (iii) each altitude of the triangle.
0 2c R1
-a c
2 1
1 (sum of  sides) (distance between  sides) 3. By means of slopes, show that the following points lie on the same line:
(a) (-1, -3) ; (1, 5) ; (2, 9)
2
(b) (4 ,-5) ; (7, 5) ; (10, 15)
= [-2c(a + a )] , expanding by the second row (d) (a, 2b): (c, a + b); (2c - a, 2a)
1
(c) (-4, 6) ; (3, 8) ; (10, 10)
= -2ca Find k so that the line joining A (7, 3); B (k, -6) and the line joining C (-4, 5) ; D (-6, 4)
2

Thus ∆ = 2ca
4.
are (i) parallel (ii) perpendicular.

5. Using slopes, show that the triangle with its vertices A (6, 1), B (2, 7) and C (-6, -7) is a
Example 2: By considering the area of the region bounded by the triangle with vertices
right triangle.
A (1, 4), B (2, - 3) and C (3, - 10)
check whether the three points are collinear or not. 6. The three points A (7, -1), B (-2, 2) and C (1, 4) are consecutive vertices of a
parallelogram. Find the fourth vertex.
Solution: Area ∆ of the region bounded by the triangle ABC is
7. The points A (-1, 2), B (3, -1) and C (6, 3) are consecutive vertices
1 4 1 1 4 1 of a rhombus. Find the fourth vertex and show that the diagonals of the rhombus
=
∆ 2 - 3 1= 1 - 7 0 by R2 - R1 and R3 - R1
1 1 are perpendicular to each other.
3 - 10 1 3 - 14 0
2 2 8. Two pairs of points are given. Find whether the two lines determined by these points
are :

= [1(-14 + 14)] , expanding by third column


1 (i) parallel (ii) perpendicular (iii) none.
2 (a) (1, -2), (2, 4) and (4, 1), (-8, 2)
(b) (-3, 4 ), (6, 2) and (4, 5), (-2, -7)
=0
Thus the points are collinear.
9. Find an equation of
(a) the horizontal line through (7, -9)
(b) the vertical line through (-5, 3)

version: 1.1 version: 1.1

42 43
1. Quadratic Equations eLearn.Punjab 1. Quadratic Equations eLearn.Punjab
4. Introduction to Analytic Geometry eLearn.Punjab 4. Introduction to Analytic Geometry eLearn.Punjab

(c) the line bisecting the irst and third quadrants. 18. A house was purchased for Rs.1 million in 1980. It is worth Rs. 4 million in 1996.
(d) the line bisecting the second and fourth quadrants. Assuming that the value increased by the same amount each year, ind an equation
that gives the value of the house after t years of the date of purchase. What was its
10. Find an equation of the line
value in 1990?
(a) through A (-6, 5) having slope 7
through (8, -3) having slope 0
19. Plot the Celsius (C) and Fahrenheit (F) temperature scales on the horizontal axis
(b) and the vertical axis respectively. Draw the line joining the freezing point and the
(c) through (-8 , 5) having slope undeined boiling point of water. Find an equation giving F temperature in terms of C.
(d) through (-5, -3) and (9, -1)
(e) y-intercept: -7 and slope: -5 20. The average entry test score of engineering candidates was 592 in the year 1998

x-intercept: -3 and y-intercept: 4


while the score was 564 in 2002. Assuming that the relationship between time and
(f)
x-intercept: -9 and slope: -4
score is linear, ind the average score for 2006.
(g)

11. Find an equation of the perpendicular bisector of the segment joining the points 21. Convert each of the following equation into
A (3 ,5) and B (9, 8). (i) Slope intercept form (ii) two intercept form (iii) normal form
12. Find equations of the sides, altitudes and medians of the triangle whose vertices are (a) 2 x - 4 y + 11 =
0 (b) 4 x + 7 y - 2 =0 (c) 15 y - 8 x + 13 =
0
A (-3, 2), B (5, 4) and C (3, -8). Also ind the length of the perpendicular from (0, 0) to each line.
13. Find an equation of the line through (-4, -6) and perpendicular to a line having

-3
22. In each of the following check whether the two lines are
slope (i) parallel
2
Find an equation of the line through (11, -5) and parallel to a line with slope -24.
(ii) perpendicular
14.

The points A (-1, 2), B (6, 3) and C (2, -4) are vertices of a triangle.
(iii) neither parallel nor perpendicular
=
2x + y - 3 0 4=
x + 2y + 5 0
15.
(a) ;
3y = 2x + 5 3 x+ 2 y- 8= 0
Show that the line joining the midpoint D of AB and the midpoint E of AC is parallel
(b) ;
to BC and DE =
1 (c) 4 y=+ 2x - 1 0 ; x= - 2y - 7 0
=
4x - y + 2=
0- ;+ 12 x 3 y 1 0
BC .
2 (d)
12 x + 35=
y-7 0 ; 105 x - 36=
y + 11 0
16. A milkman can sell 560 litres of milk at Rs. 12.50 per litre and 700 litres of milk at Rs.
(e)
12.00 per litre. Assuming the graph of the sale price and the milk sold to be a straight
line, ind the number of litres of milk that the milkman can sell at Rs. 12.25 per litre.
23. Find the distance between the given parallel lines. Sketch the lines. Also ind an
17. The population of Pakistan to the nearest million was 60 million in 1961 and
equation of the parallel line lying midway between them.
3x =
- 4y + 3 0 3x -
= 4y + 7 0
95 million in 1981. Using t as the number of years after 1961, ind an equation of
(a) ;
12 x=
+ 5y - 6 0 12 x +=
5 y + 13 0
the line that gives the population in terms of t. Use this equation to ind the
(b) ;
(c) =
x + 2y=
-5 0+ ;
population in (a) 1947 (b) 1997.
2x 4 y 1
version: 1.1 version: 1.1

44 45
1. Quadratic Equations eLearn.Punjab 1. Quadratic Equations eLearn.Punjab
4. Introduction to Analytic Geometry eLearn.Punjab 4. Introduction to Analytic Geometry eLearn.Punjab

24. Find an equation of the line through (-4, 7) and parallel to the line 2 x - 7 y + 4 =.
0 Theorem: Let l1 and l2 be two non-vertical lines such that they are not perpendicular
25. Find an equation of the line through (5, -8) and perpendicular to the join of A (-15, -8), to each other. If m1 and m2 are the slopes of l1 and l2 respectively: the angle q from l1 to l2 is
B (10, 7). given by;
26. Find equations of two parallel lines perpendicular to 2 x - y + 3 = 0 such that the
m2 - m1
tan q =
product of the x-and y-intercepts of each is 3.
27. One vertex of a parallelogram is (1, 4); the diagonals intersect at (2, 1) and the sides 1 + m1m2

-1
have slopes 1 and . Find the other three vertices. Proof: From the igure, we have
7
a= a1 + q
q= a 2 - a1
28. Find whether the given point lies above or below the given line 2

(a) (5, 8) ; 2 x - 3 y + 6 = 0 or
(b) (-7, 6) ; 4 x + 3 y - 9 =0
29. Check whether the given points are on the same or opposite sides of the given line.
(a) (0, 0) and (-4, 7) ; 6 x - 7 y + 70 =
tan a 2 - tan a1 m2 - m1
∴ tan q = tan(a 2 - a1 ) = =
0
(b) (2, 3) and (-2, 3) ; 3 x - 5 y + 8 =
1 + tan a1 tan a 2 1 + m1m2
0
30. Find the distance from the point P(6, -1) to the line 6x - 4y + 9 = 0.

Corollary 1. l1  l2 if and only if m1 = m2


31. Find the area of the triangular region whose vertices are A (5, 3), B (-2, 2), C (4, 2).
32. The coordinates of three points are A(2, 3), B(-1, 1) and C(4, -5). By computing the
m2 - m1
⇔ = q=
area bounded by ABC check whether the points are collinear.
1 + m1m2
tan 0

4.5. ANGLE BETWEEN TWO LINES ⇔ m2 =


m1
Corollary 2. l1 ⊥ l2 iff 1 + m1m2 =
0
Let l1 and l2 be two intersecting lines, which meet at a point P. At the point P two
m2 - m1 p
⇔ tan q = ∞=
tan⇔ =+ =
supplementary angles are formed by the lines l1 and l2 .
Unless l1 ⊥ l2 one of the two angles is acute. The angle from l1 to l2 is the angle q 1 + m1m2
1 m1m2 0
2
through which l1 is rotated anti-clockwise about the point P so that it coincides with l2 These two results have already been stated in 4.3.1.
In the igure below q is angle of intersection of the two lines and it is measured from -7
l1 to l2 in counterclockwise direction, ψ is also angle of intersection but it is measured from
5
Example 1: Find the angle from the line with slope to the line with slope .
3 2
l2 to l1 .
-7
Solution: Here= = . If q is measure of the required angle, then
With this convention for angle of intersection, it is clear that the inclination of a line is 5
m2 , m1
the angle measured in the counterclockwise direction from the positive x-axis to the line, 2 3
and it tallies with the earlier deinition of the inclination of a line.
version: 1.1 version: 1.1

46 47
1. Quadratic Equations eLearn.Punjab 1. Quadratic Equations eLearn.Punjab
4. Introduction to Analytic Geometry eLearn.Punjab 4. Introduction to Analytic Geometry eLearn.Punjab

5  -7 
- 
4.5.1 Equation of a Straight Line in Matrix form
2  3 
tan q = = = -1
5  -7  -29
29
1+  
2 3 
It is easy to solve two or three simultaneous linear equations by elementary methods.
If the number of equations and variables become large, the solution of the equations by

Thus q = 135
ordinary method becomes very diicult. In such a case, given equations are written in matrix
form and solved.

Example 2: Find the angles of the triangle One Linear Equation:


A linear equation
l : ax + by + c =
whose vertices are
A (-5, 4), B (-2, -1), C (7, -5) 0 (1)

[ ax + by ] =
[ -c ]
in two variables x and y has its matrix form as:
Solution: Let the slopes of the sides AB, BC and CA

x 
[ a b]  = [ -c ]
be denoted by mc , ma , mb respectively. Then

4 + 1 -5 -5 + 1 -4 -5 - 4 -3
= = = = = =  y
or
-5 + 2 3 7+2 7+5
mc , ma , mb
9 4
or AX= C
Now angle A is measured from AB to AC.
-3 5 x 
mb - mc
+ A= [ a b] , X =   and C = [ -c ]
=
tan A == =  y
where
 -3  -5 
11
1 + mb mc
4 3 22.2
1 +   
or m A
27
 4  3 
A System of Two Linear Equations:
The angle B is measured from BC to BA
-5 4
+
A system of two linear equations
mc - ma -33
∴ tan B = = = or m B = 144.9 l1 : a1 x + b1 y + c =0
1 + mc ma  -5  -4 
3 9
1 +    
l : a2 x + b2 y + c = 0
47
 3  9 
(2)

The angle C is measured from CA to CB. in two variables x and y can be written in matrix form as:
-4 3
+  a1 x + b1 y   -c1 
ma - mb
∴ tan A = = = or m C = 12.9  a x + b y  =  -c 
 -4  -3 
11
1 + ma mb  2 2   2
9 4
1 +   
48
 9  4   a1 b1   x   -c1 
a   y  =  -c 
 2 2    2
or (3)
b
or AX=C

version: 1.1 version: 1.1

48 49
1. Quadratic Equations eLearn.Punjab 1. Quadratic Equations eLearn.Punjab
4. Introduction to Analytic Geometry eLearn.Punjab 4. Introduction to Analytic Geometry eLearn.Punjab

 a1 b1  x   -c1  3 4 - 7   x  0 
== 2 - 5 8   y = 0 
A 
 a2 b2   y
 
 -c 
 2     
where , X and C =

1 1 - 3 1  0 


Equations (2) have a solution if det A ≠ 0 .
Coeicient matrix of the system is
3 4 - 7 
 
A System of Three Linear Equations: 0 1 2
A= - A 0 7 14 =
by R1 - 3R3 -
 
2 5 8 and det
1 1 - 3 1 - 3 and R - 2 R
A system of three linear equations

l1 : a1 x + b1 y + c1 =
0 
1

 and det A = 1(14+14) = 28 ≠ 0


2 3

l2 : a2 x + b2 y + c2 =0
l3 : a3 x + b3 y + c3 =0 
(5) As A is non-singular, so the lines are not concurrent.

in two variables y and y takes the matrix form as Example 2: Find a system of linear equations corresponding to the matrix form
 a1 x + b1 y + c1  0 
a x + b y + c  = 0  1 5   x  0 
 2 2   3 1   y  = 0 
2

 a3 x + b3 y + c3  0      
2
5
 4 6  1  0 
(1)
7

 a1 c1   x  0 
a c2   y  = 0 
b1
Are the lines represented by the system concurrent?
or  2 b2
   
 a3 b3 c3  1  0  Solution: Multiplying the matrices on the L.H.S. of (1), we have
 x + 2 y + 5  0 
3 x + 5 y + 1  = 0 
If the matrix
 a1 b1 c1 
   
a c2  is singular, then the lines (5) are concurrent
(2)
 2 b2  4 x + 7 y + 6  0 
 a3 b3 c3  and so the system (5) has a unique solution. By using the deinition of equality of two matrices, we have from (2),
x + 2y + 5 = 0 
3x + 5 y + 1 =0

Example 1: Express the system
3x + 4 y - 7 =0 4x + 7 y + 6 =0 

2x - 5 y + 8 =0
x+ y -3= 0 
as the required system of equations. The coeicient matrix A of the system is such that
1 2 5 1 2 5
in matrix form and check whether the three lines are concurrent det A = 3 5 1 = 0 - 1 - 14 = 0
4 7 6 0 - 1 - 14
Solution. The matrix form of the system is
Thus the lines of the system are concurrent.
version: 1.1 version: 1.1

50 51
1. Quadratic Equations eLearn.Punjab 1. Quadratic Equations eLearn.Punjab
4. Introduction to Analytic Geometry eLearn.Punjab 4. Introduction to Analytic Geometry eLearn.Punjab

9. Find the coordinates of the vertices of the triangle formed by the lines
x - 2y - 6 = 3x - y + 3 = 2x + y - 4 =
EXERCISE 4.4
0; 0; 0
1. Find the point of intersection of the lines Also ind measures of the angles of the triangle.
(i) = x - 2 y=
+ 1 0- +and 2x y 2 0 10. Find the angle measured from the line l1 to the line l2 where
=
3 x + y + 12=
0+ and
-
l1 : Joining ( 2 , 7 ) and ( 7 , 10 )
(ii) x 2y 1 0
(iii) =
x + 4 y - 12 =
0 - and
+
l2 : Joining (1, 1) and ( -5, 3)
x 3y 3 0 (a)

l1 : Joining ( 3, - 1) and ( 5,7 )


2. Find an equation of the line through

l2 : Joining ( 2,4 ) and ( -8,2 )


(i) the point (2, -9) and the intersection of the lines
=
2 x + 5 y - 8=
0 - and
-
(b)
3x 4 y 6 0
(ii) the intersection of the lines
=
x - y -=
4 0 + + and
Also ind the acute angle in each case.
and
l1 : Joining (1, - 7 ) and ( 6, - 4 )
7x y 20 0
(a) parallel (b) perpendicular
to the line 6 x + y - 14 = l2 : Joining ( -1,2 ) and ( -6, - 1)
(c)
0

l1 : Joining ( -9, - 1) and ( 3, -5 )


(iii) through the intersection of the lines x + 2 y + 3 =,
0 3x + 4 y + 7 =0 and making

l2 :Joining ( 2,7 ) and ( -6, - 7 )


equal intercepts on the axes. (d)
3. Find an equation of the line through the intersection of
16x - 10y - 33 = 0 ; 12x - 14y - 29 = 0 and the intersection of 11. Find the interior angles of the triangle whose vertices are
x-y+4=0 ; x - 7y + 2 = 0 (a) A (-2, 11), B (-6, -3), (4, -9)
4. Find the condition that the lines y =m1 x + c1; y =m2 x + c2 and = y m3 x + c3 are (b) A (6, 1), B (2, 7), C(-6, -7)
concurrent. A (2, -5), B (-4, -3), (-1, 5)
Determine the value of p such that the lines 2x - 3y - 1 = 0,
(c)
B (-5, 4), C(4, -9)
5.
3x - y - 5 = 0 and 3x + 4y + 8 = 0 meet at a point.
(d) A (2, 8),

Show that the lines 4x - 3y - 8 = 0 , 3x - 4y - 6 = 0 and x - y - 2 = 0 are concurrent


12. Find the interior angles of the quadrilateral whose vertices are A (5, 2), B (-2, 3),
C (-3, -4) and D (4, -5)
6.
and the third-line bisects the angle formed by the irst two lines.
7. The vertices of a triangle are A (-2, 3), B (-4, 1) and C (3, 5). Find coordinates of the 13. Show that the points
(i) centroid (ii) orthocentre A (0, 0), B (2, 1), C (3, 3), D (1, 2) are the vertices of a rhombus.
(iii) circumcentre of the triangle Find its interior angles.
Are these three points collinear? 14. Find the area of the region bounded by the triangle whose sides are
8. Check whether the lines 7 x - y - 10 = 10 x + y - 14 = 3x + 2 y + 3 =
4x - 3y - 8 = 3x - 4 y - 6 = x- y-2=
0; 0; 0
0; 0; 0 15. The vertices of a triangle are A(-2, 3), B(-4, 1) and C(3, 5). Find the centre of the
are concurrent. If so, ind the point where they meet circumcircle of the triangle.

version: 1.1 version: 1.1

52 53
1. Quadratic Equations eLearn.Punjab 1. Quadratic Equations eLearn.Punjab
4. Introduction to Analytic Geometry eLearn.Punjab 4. Introduction to Analytic Geometry eLearn.Punjab

Let y = m1 x and y = m2 x be two lines passing through the origin. Their joint equation is:
16. Express the given system of equations in matrix form. Find in each case whether

( y - m1 x )( y - m2 x ) =
the lines are concurrent.
x + 3y - 2 = 2x - y + 4 = x - 11 y + 14 =
y 2 - ( m1 + m2 ) xy + m1m2 x
0
=
(a) 0; 0; 0
2x + 3y + 4 = x - 2y - 3 = 3x + y - 8 =
or 2
0 (5)
(b) 0; 0; 0
3x - 4 y - 2 = x + 2y - 4 = 3x - 2 y + 5 =
Equation (5) is a special type of a second degree homogeneous equation.
(c) 0; 0; 0.
17. Find a system of linear equations corresponding to the given matrix form. Check
whether the lines represented by the system are concurrent. 4.6.1 Homogeneous Equation

1 0 - 1  x  0  1 2   x  0  Let f ( x, y ) = 0 (1)
2 0 1   y  = 0  2 4 - 3  y  = 0 
1

         
(a) (b) be any equation in the variables x and y. Equation (1) is called a homogeneous equation
0 - 1 6  1  0  3 6 - 5  1  0  of degree n (a positive integer) if
f ( kx,ky ) = k n f ( x, y )
4.6 HOMOGENEOUS EQUATION OF THE for some real number k.
SECOND DEGREE IN TWO VARIABLES For example, in equation (5) above if we replace x and y by kx and ky respectively, we
have
k 2 y 2 - k 2 ( m1 + m2 ) xy + k 2 m1m2 x 2 =
We have already seen that if a graph is a straight line, then its equation is a linear

k 2  y 2 - ( m1 + m=
2 ) xy = k 2 f ( x, y ) 0
0
+ m1m2 x 2  0 i.e.,
equation in the variables x and y. Conversely, the graph of any linear equation in x and y is a
or
straight line.
Suppose we have two straight lines represented by Thus (5) is a homogeneous equation of degree 2.

a1 x + b1 y + c1 =
0 (1) ax 2 + 2hxy + by 2 =
0
a2 x + b2 y + c2 =
A general second degree homogeneous equation can be written as:
and 0 (2)
ax 2 + 2hxy + by 2 =
0

( a1x + b1 y + c1 )( a2 x + b2 y + c2 ) =
Multiplying equations (1) and (2), we have
provided a, h and b are not simultaneously zero.
0 (3)
It is a second degree equation in x and y.
Theorem: Every homogenous second degree equation
ax 2 + 2hxy + by 2 =
Equation (3) is called joint equation of the pair of lines (1) and (2). On the other hand,
given an equation of the second degree in x and y, say 0 (1)

ax 2 + 2hxy + by 2 + 2 gx + 2 fy + c =
represents a pair of lines through the origin. The lines are
real and distinct, if h 2 > ab
0 (4)
where a ≠ 0 , represents equations of a pair of lines if (4) can be resolved into two linear
(i) (ii) real and coincident, if h ab
factors. In this section, we shall study special joint equations of pairs of lines which pass (iii) imaginary, if h 2 < ab
through the origin.

version: 1.1 version: 1.1

54 55
1. Quadratic Equations eLearn.Punjab 1. Quadratic Equations eLearn.Punjab
4. Introduction to Analytic Geometry eLearn.Punjab 4. Introduction to Analytic Geometry eLearn.Punjab

Proof: Multiplying (1) by b and re-arranging the terms, we have 4.6.2 To ind measure of the angle between the lines
b 2 y 2 + 2bhxy + abx 2 =
0 represented by
b 2 y 2 + 2bhxy + h 2 x 2 - h 2 x 2 + abx 2 =

- x 2 ( h 2 - ab ) =
ax 2 + 2hxy + by 2 =
or 0

( by + hx )
0 (1)

(by + hx + x )( ) ( )
2
or 0 We have already seen that the lines represented by (1) are

h 2 - ab by + hx - x h 2 - ab = by + x h + h 2 - ab =

by + x ( h - h - ab ) =
or 0 0 (2)

( )
Thus (1) represents a pair of lines whose equations are: 2
and 0 (3)
by + x h + h 2 - ab =

by + x ( h - h - ab ) = ( ), ( )
0 (2) Now slopes of (2) and (3) are respectively given by:

- h + h 2 - ab - h - h 2 - ab
=
=
2
and 0 (3)
m1 and m2
Clearly, the lines (2) and (3) are b b
(i) real and distinct if h 2 > ab . (ii) real and coincident, if h 2 = ab . -2h
= = , m1 + m2
a
(iii) imaginary, if h 2 < ab .
Therefore and m1m2
b b
It is interesting to note that even in case the lines are imaginary, they intersect in a real If q is measure of the angle between the lines (2) and (3), then
point viz (0, 0) since this point lies on their joint equation (1).

m1 - m2 ( m1 + m2 ) - 4m1m2 -
2 h 2 - ab
4h 2 4a
tanq =
Example: Find an equation of each of the lines represented by
20 x 2 + 17 xy - 24 y 2 = = = =
2
2

1 + m1m2 1 + m1m2 a+b


b b
1+
0 a

The two lines are parallel, if q = 0 , so that tan q = 0 which implies h2 - ab =


b
Solution. The equation may be written as 0 , which

 y  y
is the condition for the lines to be coincident.
24   - 17   - 20 = If the lines are orthogonal, then q = 90 , so that tan q is not deined. This implies
2

x x
0

y 17 ± 289 + 1920 17 ± 47 4 -5
a + b = 0. Hence the condition for (1) to represent a pair of orthogonal (perpendicular)
⇒ = = = , lines is that sum of the coeicients of x2 and y2 is 0.
x 48 48 3 8
-5
⇒ y= y=
4 Example 1: Find measure of the angle between the lines represented by
x 2 - xy - 6 y 2 =
x and x
⇒ 4x - 3y = 5x + 8 y =
3 8 0
0 and 0

Solution. Here a- ==
1-, h= , b
1
6
2
version: 1.1 version: 1.1

56 57
1. Quadratic Equations eLearn.Punjab 1. Quadratic Equations eLearn.Punjab
4. Introduction to Analytic Geometry eLearn.Punjab 4. Introduction to Analytic Geometry eLearn.Punjab

If q is measure of the angle between the given lines, then 5. 6 x 2 - 19 xy + 15 y 2 =


0

+6
2 h - ab
1
tanq = = =-1 ⇒ q =135
2
x 2 + 2 xy sec a + y 2 =
2

a+b -5
4
6. 0
Acute angle between the lines =180° - q = 180° - 135° = 45°
7. Find a joint equation of the lines through the origin and perpendicular to
Example2: Find a joint equation of the straight lines through the origin perpendicular the lines:
to the lines represented by x 2 - 2 xy tan a - y 2 =
0
x 2 + xy - 6 y 2 =
0 (1)
8. Find a joint equation of the lines through the origin and perpendicular to the
lines:
( x - 2 y )( x + 3 y ) =
Solution: (1) may be written as
0 ax 2 + 2hxy + by 2 =
0
Thus the lines represented by (1) are
x - 2y =0 (2) 9. Find the area of the region bounded by:
and x + 3y =0 (3) 10 x 2 - xy=
- 21 y 2 0 and =
x + y +1 0
The line through (0, 0) and perpendicular to (2) is
y-=2 x or +y 2=x 0 (4)
Similarly, the line through (0, 0) and perpendicular to (3) is
= =y 3 x - or y 3x 0 (5)

( y + 2 x=
)( y - 3x )
Joint equation of the lines (4) and (5) is
0 or y2 -
= xy - 6 x 2 0

EXERCISE 4.5

Find the lines represented by each of the following and also ind measure of the,
angle between them (Problems 1-6):

1. 10 x 2 - 23 xy - 5 y 2 =
0

2. 3 x 2 + 7 xy + 2 y 2 =
0

3. 9 x 2 + 24 xy + 16 y 2 =
0

4. 2 x 2 + 3 xy - 5 y 2 =
0
version: 1.1 version: 1.1

58 59
version: 1.1

CHAPTER

5 Linear Inequalities and


Linear Programming

Animation 5.1: Feasible Solution Set


Source and credit: eLearn.Punjab
1. Quadratic Equations eLearn.Punjab 1. Quadratic Equations eLearn.Punjab
5. Linear Inequalities and Linear Programming eLearn.Punjab 5. Linear Inequalities and Linear Programming eLearn.Punjab

5.1 INTRODUCTION
Fig. 5.21
Many real life problems involve linear inequalities. Here we shall consider those We conclude that the solution set of an inequality consists of all solutions of the
problems (relating to trade, industry and agriculture etc.) which involve systems of linear inequality.
inequalities in two variables. Linear inequalities in such problems are used to prescribe
limitations or restrictions on allocation of available resources (material, capital, machine 5.2.1 Graphing of A Linear Inequality in Two Variables
capacities, labour hours, land etc.). In this chapter, our main goal will be to optimize
(maximize or minimize) a quantity under consideration subject to certain restrictions. Generally a linear inequality in two variables x and y can be one of the following forms:
The method under our discussion is called the linear programming method and it ax + by < c ; ax + by > c ; ax + by 7 c ; ax + by 8 c
involves solutions of certain linear inequalities. where a, b, c are constants and a, b are not both zero.
We know that the graph of linear equation of the form
5.2 LINEAR INEQUALITIES ax + by = c is a line which divides the plane into two disjoint regions as stated below:
(1) The set of ordered pairs (x, y) such that ax + by < c
Inequalities are expressed by the following four symbols; (2) The set of ordered pairs (x, y) such that ax + by > c
> (greater than); < (less than); 8 (greater than or equal to); 7 (less than or equal to) The regions (1) and (2) are called half planes and the line
For example (i) ax < b (ii) ax + b 8 c (iii) ax + by > c (iv) ax + by 7 c are ax + by = c is called the boundary of each half plane.
inequalities. Inequalities (i) and (ii) are in one variable while inequalities (iii) and (iv) are in Note that a vertical line divides the plane into left and right half planes while a non-
two variables. vertical line divides the plane into upper and lower half planes.
The following operations will not afect the order (or sense) of inequality while changing A solution of a linear inequality in x and y is an ordered pair of numbers which satisies
it to simpler equivalent form: the inequality.
(i) Adding or subtracting a constant to each side of it. For example, the ordered pair (1, 1) is a solution of the inequality x + 2y < 6 because
(ii) Multiplying or dividing each side of it by a positive constant. 1 + 2(1) = 3 < 6 which is true.
Note that the order (or sense) of an inequality is changed by multiplying or dividing its There are ininitely many ordered pairs that satisfy the inequality x + 2y < 6, so its graph
each side by a negative constant. will be a half plane.
Note that the linear equation ax + by = c is called “associated or corresponding
Now for revision we consider inequality, x <
3
(A) equation” of each of the above mentioned inequalities.
2
All real numbers <
3
are in the solution set of (A). Procedure for Graphing a linear Inequality in two Variables
2
 3
(i) The corresponding equation of the inequality is irst graphed by using ‘dashes’ if the
Thus the interval  - ∞,  or - ∞ < x <
3
 2
is the solution set of the inequality involves the symbols > or < and a solid line is drawn if the inequality involves
2
the symbols 8 or 7.
inequality (A) which is shown in the igure 5.21 (ii) A test point (not on the graph of the corresponding equation) is chosen which
determines that the half plane is on which side of the boundary line.

version: 1.1 version: 1.1

2 3
1. Quadratic Equations eLearn.Punjab 1. Quadratic Equations eLearn.Punjab
5. Linear Inequalities and Linear Programming eLearn.Punjab 5. Linear Inequalities and Linear Programming eLearn.Punjab

Example 1. Graph the inequality x + 2y < 6.


Solution. The associated equation of the inequality Note: 2 All points on the line (ii) and above the line (ii)
x + 2y < 6 (i) satisfy the inequality x + 2y 8 6 .... (v). This means that
is x + 2y = 6 (ii) the solution set of the inequality (v) consists of all points
above the line (ii) and all points on the lines (ii). The graph
The line (ii) intersects the x-axis and y-axis at (6, 0) and of the inequality (v) is partially shown as shaded region
(0. 3) respectively. As no point of the line (ii) is a solution in igure 5.22(d)
of the inequality (i), so the graph of the line (ii) is shown by Note: 3 that the graphs of
using dashes. We take O(0, 0) as a test point because it is x + 2y 7 6 and x + 2y 8 6 are closed half planes.
not on the line (ii).

Substituting x = 0, y = 0 in the expression x + 2y gives


0 - 2(0) = 0 < 6, so the point (0, 0) satisies the inequality (i).
Example 2. Graph the following linear inequalities in
xy-plane;
Any other point below the line (ii) satisies the (i) 2x 8 - 3 (ii) y 7 2
inequality (i), that is all points in the half plane containing Solution. The inequality (i) in xy-plane is considered as
the point (0, 0) satisfy the inequality (i). 2x + 0y 8 - 3 and its solution set consists of all point (x, y)
Thus the graph of the solution set of inequality (i) is the a
such that x, y d R and x ≥ -
region on the origin-side of the line (ii), that is, the region 3
below the line (ii). A portion of the open halfplane below 2
The corresponding equation of the inequality (i) is
2x = -3
the line (ii) is shown as shaded region in igure 5.22(a)
(1)
All points above the dashed line satisfy the
which a vertical line (parallel to the y-axis) and its
inequality x + 2y > 6 (iii)
graph is drawn in igure 5.23(a).
The graph of the inequality 2x > -3 is the open half
A portion of the open half plane above the line (ii) is
shown by shading in igure 5.22(b)
plane to the right of the line (1).
Thus the graph of 2x 8 -3 is the closed half-plane to
the right of the line (1).
Note: 1. The graph of the inequality x + 2y 7 6 ..(iv)
(ii) The associated equation of the inequality (ii) is
includes the graph of the line (ii),’ so the open half-plane
y=2 (2)
below the line (ii) including the graph of the line (ii) is the
which is a horizontal line (parallel to the x-axis) and its
graph of the inequality (iv). A portion of the graph of the
graph is shown in igure 5.23(b) Here the solution
inequality (iv) is shown by shading in igure 5.22(c)
set of the inequality y < 2 is the open half plane below
the boundary line y = 2. Thus the graph of y 7 2 consists
of the boundary line and the open half plane below it.
version: 1.1 version: 1.1

4 5
1. Quadratic Equations eLearn.Punjab 1. Quadratic Equations eLearn.Punjab
5. Linear Inequalities and Linear Programming eLearn.Punjab 5. Linear Inequalities and Linear Programming eLearn.Punjab

We draw the graph of the line 2x + y = 2 joining


the points (1, 0) and (0, 2). The point (0, 0) does not
satisfy the inequality 2x + y > 2 because 2(0) + 0 = 0
Note that the intersection of graphs of 2x 8 -3 and > 2. Thus the graph of the inequality 2x + y 8 2 is the
y 72 is partially shown in the adjoining igure 5.23(c). closed half-plane not on the origin-side of the line
2x + y = 2.

5.3 REGION BOUNDED BY 2 OR 3 Thus the closed half-plane is shown partially as a


SIMULTANEOUS INEQUALITIES shaded region in igure 5.31(b). The solution region of
the given system of inequalities is the intersection of
The graph of a system of inequalities consists of the set of all ordered pairs (x, y) in the the graphs indicated in igures 5.31(a) and 5.31(b) and is
xy-plane which simultaneously satisfy all the inequalities in the system. Find the graph of shown as shaded region in igure 5.31(c).
such a system, we draw the graph of each inequality in the system on the same coordinate The intersection point (2, - 2) can be found by
axes and then take intersection of all the graphs. The common region so obtained is called solving the equations x - 2y = 6 and 2x + y = 2.
the solution region for the system of inequalities.

Example 1: Graph the system of inequalities


x - 2y 7 6
2x + y 8 2

Solution. Note that the line x - 2y = 6 and 2x + y = 2 divide the


The graph of the line x - 2y = 6 is drawn by joining xy-plane into four region bounded by these lines. These
the point (6, 0) and (0, -3). The point (0,0) satisfy the four (bounded) regions are displayed in the adjoining
inequality x - 2y < 6 because 0 - 2(0) = 0 < 6. Thus the igure.
graph of x - 2y 7 6 is the upper half-plane including
the graph of the line x - 2y = 6. The closed half-plane is
partially shown by shading in igure 5.31(a).

version: 1.1 version: 1.1

6 7
1. Quadratic Equations eLearn.Punjab 1. Quadratic Equations eLearn.Punjab
5. Linear Inequalities and Linear Programming eLearn.Punjab 5. Linear Inequalities and Linear Programming eLearn.Punjab

Example 2. Graph the solution region for the following system of inequalities: Example 3. Graph the following systems of inequalities.
x - 2y 7 6, 2x + y > 2, x + 2y 8 1 0
(i) 2x + y 8 2 (ii) 2x + y 8 2 (iii) 2x + y 8 2
x + 2y 7 10 x + 2y 7 10 x + 2y 7 10
Solution: The graph of the inequalities x - 2y 7 6 and y80 x80 x 8 0, y 8 0
2x + y 8 2 have already drawn in igure 5.31(a) and
5.31(b) and their intersection is partially shown as a Solution:
shaded region in igure 5.31(c) of the example 1 Art (i) The corresponding equations of the inequalities
(5.3). Following the procedure of the example 1 of Art 2x + y 8 2 and x + 2y 7 10 are
(5.3) the graph of the inequality x + 2y 7 10 is shown 2x + y = 2 (I) and x + 2y = 10 (II)
partially in the igure 5.32(a). For the partial graph of 2x + y 8 2 see igure 5.31(b) of the example 1 and the graph of
the inequality x + 2y 7 10 is partially shown in igure 5.32(a) of the example 2.

The intersection of three graphs is the required The solution region of the inequalities
solution region which is the shaded triangular region 2x + y 8 2 and x + 2y 7 10 is the intersection of their
PQR (including its sides) shown partially in the igure individual graphs. The common region of the graphs
5.32(b). of inequalities is partially shown as a shaded region in
igure 5.33(a).
Now we deine a corner point of a solution region.

DEFINITION:
A point of a solution region where two of its boundary lines intersect, is called a
corner point or vertex of the solution region.
The graph of y 8 0 is the upper half plane
Such points play a useful role while solving linear programming problems. In example
including the graph of the corresponding line y = 0
2, the following three corner points are obtained by corresponding equations (of linear
(the x-axis) of the linear inequality y 8 0. The graph of
inequalities given in the example 2) in pairs.
y 8 0 is partially displayed in igure 5.33(b).
Corresponding lines of inequalities: Corner Points
x - 2y = 6, 2x + y = 2 P(2, -2)
x - 2y = 6, x + 2y = 10 Q(8, 1)
2x + y = 2, x + 2y = 10 R(-2, 6)
version: 1.1 version: 1.1

8 9
1. Quadratic Equations eLearn.Punjab 1. Quadratic Equations eLearn.Punjab
5. Linear Inequalities and Linear Programming eLearn.Punjab 5. Linear Inequalities and Linear Programming eLearn.Punjab

The solution region of the system of (iii) The graphs of the system of inequalities in (iii)
inequalities in (i) is the intersection of the graphs are drawn in the solution of (i) and (ii). The solution
shown in igure 5.33(a) and 5.33(b). This solution region in this case, is shown as shaded region ABCD
region is displayed in igure 5.33(c). in igure 5.34. (c).

(ii) See igure 5.33(a) for the graphs of the inequalities 2x + y 8 2 and x + 2y 7 10.
EXERCISE 5.1

1. Graph the solution set of each of the following linear inequality in xy-plane:
(i) 2x + y 7 6 (ii) 3x + 7y 8 21 (iii) 3x - 2y 8 6
The graph of x 8 0 consists of the open (iv) 5x - 4y 7 20 (v) 2x + 1 8 0 (vi) 3y - 4 7 0
half-plane to the right of the corresponding line 2. Indicate the solution set of the following systems of linear inequalities
x = 0 (y-axis) of the inequality x 8 0 and its graph. by shading:
See igure 5.34(a). (i) 2x - 3y 7 6 (ii) x + y 8 5 (iii) 3x + 7y 8 21
2x + 3y 7 12 -y + x 7 1 x-y72
(iv) 4x - 3y 7 12 (v) 3x + 7y 8 21

x ≥ -
3
y74
2
3. Indicate the solution region of the following systems of linear inequalities
Thus the solution region of the inequalities in by shading:
(ii) is partially shown in igure 5.34(b). This region (i) 2x - 3y 7 6 (ii) x + y 7 5 (iii) x + y 8 5
is the intersection of graphs in igure 5.33(a) and 2x + 3y 7 12 y - 2x 7 2 x-y81
5.34(a). y80 x80 y80
(iv) 3x + 7y 7 21 (v) 3x + 7y 7 21 (vi) 3x + 7y 7 21
x-y72 x-y72 2x - y 8 -3
x80 y80 x80

version: 1.1 version: 1.1

10 11
1. Quadratic Equations eLearn.Punjab 1. Quadratic Equations eLearn.Punjab
5. Linear Inequalities and Linear Programming eLearn.Punjab 5. Linear Inequalities and Linear Programming eLearn.Punjab

4. Graph the solution region of the following system of linear inequalities and It is the polygonal region ABCDE (including its sides)
ind the corner points in each case. as shown in the igure 5.51.
(i) 2x - 3y 7 6 (ii) x + y 7 5 (iii) 3x + 7y 7 21
2x + 3y 7 12 -2x + y 7 2 2x - y 7 -3 Such a region (which is restricted to the irst
x80 y80 y80 quadrant) is referred to as a feasible region for the set
(iv) 3x + 2y 8 6 (v) 5x + 7y 7 35 (vi) 5x + 7y 7 35 of given constraints. Each point of the feasible region
x + 3y 7 6 -x + 3y 7 3 x - 2y 7 2 is called a feasible solution of the system of linear
y80 x80 x80 inequalities (or for the set of a given constraints). A set
5. Graph the solution region of the following system of linear inequalities consisting of all the feasible solutions of the system of
by shading. linear inequalities is called a feasible solution set.
(i) 3x - 4y 7 12 (ii) 3x - 4y 7 12 (iii) 2x + y 7 4
3x + 2y 8 3 x + 2y 7 6 2x - 3y 8 12 Example 1. Graph the feasible region and ind the corner points for the following
x + 2y 7 9 x+y81 x + 2y 7 6 system of inequalities (or subject to the following constraints).
(iv) 2x + y 7 10 (v) 2x + 3y 7 18 (vi) 3x - 2y 8 3 x-y73
x+y77 2x + y 7 10 x + 4y 7 12 x + 2y 7 6 , x 8 0, y80
-2x + y 7 4 -2x + y 7 2 3x + y 7 12
Solution: The associated equations for the inequalities
5.4 PROBLEM CONSTRAINTS x - y 7 3 (i) and x + 2y 7 6 (ii)
are x - y = 3 (1) and x + 2y = 6 (2)
As the point (3, 0) and (0, -3) are on the line (1),
so the graph of x - y = 3 is drawn by joining the points
In the beginning we described that linear inequalities prescribe limitations and

(3, 0) and (0, -3) by solid line.


restrictions on allocation of available sources. While tackling a certain problem from every
day life each linear inequality concerning the problem is named as problem constraint.
The system of linear inequalities involved in the problem concerned are called problem Similarly line (2) is graphed by joining the points
constraints. The variables used in the system of linear inequalities relating to the problems (6, 0) and (0, 3) by solid line. For x = 0 and y = 0, we
of every day life are non-negative and are called non-negative constraints. These non- have;

0 - 0 = 073 and 0 + 2(0) = 076,


negative constraints play an important role for taking decision. So these variables are called
decision variables.

5.5 Feasible solution set so both the ciosed half-planes are on the origin
sides of the lines (1) and (2). The intersection of these
We see that solution region of the inequalities in example 2 of Art 5.3 is not within the closed half-planes is partially displayed as shaded
irst quadrant. If the nonnegative constraints x 8 0 and y 8 0 are included with the system of region in igure 5.52(a).
inequalities given in the example 2, then the solution region is restricted to the irst quadrant.

version: 1.1 version: 1.1

12 13
1. Quadratic Equations eLearn.Punjab 1. Quadratic Equations eLearn.Punjab
5. Linear Inequalities and Linear Programming eLearn.Punjab 5. Linear Inequalities and Linear Programming eLearn.Punjab

Example 2. A manufacturer wants to make two types of concrete. Each bag of Agrade
concrete contains 8 kilograms of gravel (small pebbles with coarse sand) and 4 kilograms of
cement while each bag of B-grade concrete contains 12 kilograms of gravel and two kilograms
For the graph of y 8 0, see igure 5.33(b) of cement. If there are 1920 kilograms of gravel and 480 kilograms of cement, then graph
of the example 3 of art 5.3. the feasible region under the given restrictions and ind corner points of the feasible region.

The intersection of graphs shown in Solution: Let x be the number of bags of A-grade concrete produced and y denote the
igures 5.52(a) and 5.33(b) is partially graphed number of bags of B-grade concrete produced, then 8x kilograms of gravel will be used
as shaded region in igure 5.52(b). for A-grade concrete and 12y kilograms of gravel will be required for B-grade concretes so
8x + 12y should not exceed 1920, that is,
8x + 12y 7 1920
Similarly, the linear constraint for cement will be
4x + 2y 7 480
Now we have to graph the feasible region for the
linear constraints
8x + 12y 7 1920
The graph of x 8 0 is drawn in igure
4x + 2y 7 480, x 8 0, y 8 0
5.34(a). The intersection of the graphs shown
in igures 5.52(a) and 5.34(a) is graphed in
Taking the one unit along x-axis and y-axis
igure 5.52(c).
equal to 40 we draw the graph of the feasible region
required.
The shaded region of igure 5.53(a) shows the
graph of 8x + 12y 7 1920 including the nonnegative
constraints x 8 0 and y 8 0

Finally the graph of the given system


of linear inequalities is displayed in igure In the igure 5.53(b), the graph of 4x + 2y 7 480
5.52(d) which is the feasible region for the including the non-negative constraints x 8 0 and y 8 0
given system of linear inequalities. The is displayed as shaded region.
points (0, 0), (3, 0), (4, 1) and (0, 3) are corner
points of the feasible region.

version: 1.1 version: 1.1

14 15
1. Quadratic Equations eLearn.Punjab 1. Quadratic Equations eLearn.Punjab
5. Linear Inequalities and Linear Programming eLearn.Punjab 5. Linear Inequalities and Linear Programming eLearn.Punjab

The intersection of these two graphs is shown as


The graph of 2x + y 8 2 is the closed half-plane
shaded region in igure 5.53(c), which is the feasible
not on the origin side of 2x + y = 2. The portion of
region for the given linear constraints.
the graph of the system 2x + y 8 2,
The point (0, 0), (120, 0), (60, 120) and (0, 160) are
x 8 0, y 8 0
the corner points of the feasible region.
is displayed as shaded region in igure 5.54(b).

Example 3. Graph the feasible regions subject to the following constraints.


(a) 2x - 3 y 7 6 (b) 2x - 3y 7 6 The graph of the system
2x + y 8 2 2x + y 8 2 2x - 3y 7 6, 2x + y 7 2,
x 8 0, y 8 0 x + 2y < 8, x 8 0, y 8 0 x 8 0, y 8 0

is the intersection of the graphs shown in


Solution: The graph of 2x - 3y 7 6 is the igures 5.54(a) and 5.54(b) and it is partially displayed
closed half-plane on the origin side of in igure 5.54(c) as shaded region.
2x - 3y = 6. The portion of the graph of
system 2x - 3y 7 6,
x 8 0, y 8 0
is shown as shaded region in igure 5.54(a).
(b) The graph of system x + 2y 7 8, x 8 0, y 8 0 is
a triangular region indicated in igure 5.45(d).
Thus the graph of the system
2x - 3y 7 6
2x + y 8 2
x + 2y 7 8 x 8 0, y 8 0

version: 1.1 version: 1.1

16 17
1. Quadratic Equations eLearn.Punjab 1. Quadratic Equations eLearn.Punjab
5. Linear Inequalities and Linear Programming eLearn.Punjab 5. Linear Inequalities and Linear Programming eLearn.Punjab

is the intersection of the graphs shown in igures 2. Graph the feasible region of the following system of linear inequalities and
5.54(c) and 5.54(d). It is the shaded region indicated ind the corner points in each case.
in the igure 5.54(e). (i) 2x + y 7 10 (ii) 2x + 3y 7 18 (iii) 2x + 3y 7 18
x + 4y 7 12 2x + y 7 10 x + 4y 7 12
Note: The corner points of feasible region x + 2y 7 10 x + 4y 7 12 3x + y 7 12
the set of constraints in (a) are (1, 0), (3, 0) and x 8 0, y 8 0 x80,y80 x 8 0, y 8 0
(0, 2) while the corner points of the feasible (iv) x + 2y 7 14 (v) x + 3y 7 15 (vi) 2x + y 7 20
region for the set of constraints in (b) are (1, 0), 3x + 4y 7 36 2x + y 7 12 8x+15y 7120
2x + y 7 10 4x + 3y 7 24 x + y 7 11
 36 10 
(3, 0),  ,  , (0, 4) and (0, 2)
 7 7
x 8 0, y 8 0 x 8 0, y 8 0 x 8 0, y 8 0

We see that the feasible solution regions in example 3(a) and 3(b) are of diferent types. 5.6 LINEAR PROGRAMMING
The feasible region in example 3(a) is unbounded as it cannot be enclosed in any circle how
large it may be while the feasible region in example 3(b) can easily be enclosed within a A function which is to be maximized or minimized is called an objective function.
circle, so it is bounded. If the line segment obtained by joining any two points of a region lies Note that there are ininitely many feasible solutions in the feasible region. The feasible
entirely within the region, then the region is called convex. solution which maximizes or minimizes the objective function is called the optimal solution.
The theorem of linear programming states that the maximum and minimum values of the
Both the feasible regions of example 3(a)
objective function occur at corner points of the feasible region.
and 3(b) are convex but the regions such
as shown in the adjoining igures are not Procedure for determining optimal solution:
convex. (i) Graph the solution set of linear inequality constraints to determine feasible region.
(ii) Find the corner points of the feasible region.
(iii) Evaluate the objective function at each corner point to ind the optimal solution.

EXERCISE 5.2 Example 1. Find the maximum and minimum values of


the function deined as:
f(x, y) = 2x + 3y subject to the constraints;
x-y72 2x - y 7 6, x 8 0
1. Graph the feasible region of the following system of linear inequalities and
ind the corner points in each case. x+y74
(i) 2x - 3y 7 6 (ii) x + y 7 5 (iii) x + y 7 5
2x + 3y 7 12 -2y + y 7 2 -2x + y 8 2 Solution. The graphs of x - y 7 2 is the closed half plane on
x 8 0, y 8 0 x 8 0, y 8 0 x80 the origin side of x - y = 2 and the graph of x + y 7 4 is the
(iv) 3x + 7y 7 21 (v) 3x + 2y > 6 (vi) 5x + 7y 7 35 closed half-plane not on the origin side of x + y = 4. The graph
x-y73 x+y74 x - 2y 7 4 of the system
x 8 0, y 8 0 x 8 0, y 8 0 x 8 0, y 8 0 x - y 7 2, x + y 8 4
version: 1.1 version: 1.1

18 19
1. Quadratic Equations eLearn.Punjab 1. Quadratic Equations eLearn.Punjab
5. Linear Inequalities and Linear Programming eLearn.Punjab 5. Linear Inequalities and Linear Programming eLearn.Punjab

including the non-negative constraints x 8 0 is Solution. The graphs of 2x - 3y 7 6, 2x + y 8 2, are displayed in the example 3 of
partially displayed as shaded region in the igure 5.61. Art. 5.5. Joining the points (6. 0) and (0, 4), we obtain the graph of the line 2x + 3y = 12. As
The graph of 2x - y 7 6 consists of the graph of the line 2(0) + 3(0) = 0 < 12, so the graph of 2x + 3y < 12 is the half plane below the line 2x + 3y = 12.
2x - y = 6 and the half plane on the origin side of the line Thus the graph of 2x + 3y 7 12 consists of the graph of the line 2x + 3y = 12 and the half plane
2x - y=6. A portion of the solution region of the given below the line 2x + 3y = 12. The solution region of 2x - 3y 7 6, 2x + y 8 2 and 2x + 3y 7 12 is the
system of inequalities is shaded in the igure 5.62. triangular region PQR shown in igure 5.63. The non-negative constraints x 8 0,
We see that feasible region is unbounded upwards y 80 indicated the irst quadrant. Thus the feasible region satisfying all the constrains is
and its corner points are A(0, 4), B(3, 1) and C(4, 2). shaded in the igure 5.63 and its corner points are (1, 0) (0, 2), (0, 4),

9 
Note that the point at which the lines x + y = 4 and
2x - y = 6 intersect is not a corner point of the feasible  , 1 and (3, 0).
2 
region.
It is obvious that the expression 2x + 3y does not We ind values of f at the corner points.
posses a maximum value in the feasible region because Corner
f(x, y) = 4x + 5y
its value can be made larger than any number by point
(1, 0) f (1, 0) = 4 x 1 + 5.0 = 4
increasing x and y. We calculate the values of f at the
(0, 2) f (0, 2) = 4 x 0 + 5.2 = 10
corner points to ind its minimum value:
(0, 4) f (0, 4) = 4 x 0 + 5.4 = 20
f (0, 4) = 2(0) + 3 x 4 = 12
(9/2, 1) f (9/2, 1) = 4 x 9/2 + 5.1 = 23
f (3, 1) = 2x3+3x1 = 6+3=9
(3, 0) f (3, 0) = 4 x 3 + 50 x 0 = 12
f (4, 2) = 2x4+3x2 = 8 + 6 = 14
From the above table, it follows that the minimum value of f is 4 at the corner point
Thus the minimum value of 2x + 3y is 9 at the corner point (3, 1).
9 
(1, 0) and maximum value of f is 23 at the corner point  , 1 . The values of f at the corner
Note: lf f(x , y) = 2x + 2y, then f (0 , 4) = 2 x 0 + 2 x 4 = 8, f (3, 1) = 2 x 3 + 2 x 1 = 6 + 2 = 8 2 
points are given below in tabular form.
and f(4, 2) = 2 x 4 + 2 x 2 = 8 + 4 =12. The minimum value of 2x + 2y is the same at two corner points

f(x, y) = 4x + 5y
(0, 4) and (3, 1).
Corner point
f (1, 0) = 4.1 + 6.0 = 4
We observe that the minimum value of 2x + 2y at each point of the line segment AB is
(1, 0)
8 as: f (0, 2) = 4.0 + 6.2 = 12
f(x, y) = 2x + 2(4 - x) (a x + y = 4 ⇒ y = 4 - x)
(0, 2)
(0, 4) f (0, 4) = 4.0 + 6.4 = 24
= 2x + 8 - 2x = 8 (9/2, 1) f (9/2, 1) = 4 . 9/2 + 6.1 = 24
(3, 0) f (3, 0) = 4 x 3 + 6.0 = 12
Example 2. Find the minimum and maximum values of f and f deined as: The minimum value of f is 4 at the point (1, 0) and maximum value of f is 24 at the
f(x, y) = 4x + 5y, f (x, y) = 4x + 6y
9 
corner points (0, 4) and  , 1 . As observed in the above example, it follows that the
2 
under the constraints
2x - 3y 7 6
function f has maximum value at all the points of the line segment between the points
2x + y 8 2 2x + 3y 7 12 x 8 0, y 8 0

version: 1.1 version: 1.1

20 21
1. Quadratic Equations eLearn.Punjab 1. Quadratic Equations eLearn.Punjab
5. Linear Inequalities and Linear Programming eLearn.Punjab 5. Linear Inequalities and Linear Programming eLearn.Punjab

Now the problem is to maximize the proit function P under the given constraints.
9 
(0, 4) and  , 1 .
Graphing the inequalities, we obtain the feasible region which is shaded in the igure 5.71.
2  Solving the equations x + y = 100 and 3x + 2y = 240 gives x = 240 - 2(x + y) = 240 - 200 = 40
and y = 100 - 40 = 60, that is; their point of intersection is (40, 60). The corner points of the
Note 1. Some times it may happen that each point of constraint line gives the optimal
feasible region are (0, 0),(0, 100), (40, 60) and (80, 0).
value of the objective function.
Now we ind the values of P at the corner points.
Corner point P(x, y) = 20x + 15y
Note 2. For diferent value of k, the equation 4x + 5y = k represents lines parallel to the
(0, 0) P(0, 0) = 2 0 x 0 + 15 x 0 = 0
line 4x + 5y = 0. For a certain admissible value of k, the intersection of 4x + 5y = k with the
(0, 100) P(0, 100) = 20 x 0 + 15 x 100 = 1500
feasible region gives feasible solutions for which the proit is k.
(40, 60) P(40, 60) = 20 x 40 + 15 x 60 = 1700
(80, 0) P(80, 0) = 20 x 80 + 15 x 0 = 1600
5.7 LINEAR PROGRAMMING PROBLEMS From the above table, it follows that the maximum proit is Rs. 1700 at the corner point
(40, 60). Thus the farmer will get the maximum proit if he cultivates 40 canals of corn and
Convert a linear programming problem to a mathematical form by using variables,
60 canals of wheat.
then follow the procedure given in Art 5.6.
Exam ple 2. A factory produces bicycles and motorcycles by using two machines A and
Example 1: A farmer possesses 100 canals of land and wants to grow corn and wheat.
B. Machine A has at most 120 hours available and machine B has a maximum of 144 hours
Cultivation of corn requires 3 hours per canal while cultivation of wheat requires 2 hours per
available. Manufacturing a bicycle requires 5 hours in machine A and 4 hours in machine B
canal. Working hours cannot exceed 240. If he gets a proit of Rs. 20 per canal for corn and
while manufacturing of a motorcycle requires 4 hours in machine A and 8 hours in machine
Rs. 15/- per canal for wheat, how many canals of each he should cultivate to maximize his
B. If he gets proit of Rs. 40 per bicycle and proit of Rs. 50 per motorcycle, how many bicycles
proit?
and motorcycles should be manufactured to get maximum proit?
Solution: Suppose that he cultivates x canals of corn
and y canals of wheat. Then constraints can be written Solution: Let the number of bicycles to be
as: manufactured be x and the number of motor cycles to
x + y 7 100 be manufactured be y.
3x + 2y 7 240
Then the time required to use machine A for x
Non-negative constraints are x 8 0, y 8 0. Let P(x, y) bicycles and y motorcycles is 5x + 4y (hours) and the time
be the proit function, then required to use machine B for x bicycles and y motorcycles
P(x, y) = 20x + 15y in 4x + 8y (hours). Thus the problem constraints are
5x + 4y 7 120
And 4x + 8y 7 144
⇒ 2x + 4y 7 72 .

version: 1.1 version: 1.1

22 23
1. Quadratic Equations eLearn.Punjab 1. Quadratic Equations eLearn.Punjab
5. Linear Inequalities and Linear Programming eLearn.Punjab 5. Linear Inequalities and Linear Programming eLearn.Punjab

5. Maximize the function deined as; f(x, y) = 2x + 3y subject to the constraints:


Since the numbers of articles to be produced cannot be negative, so x 8 0, y 8 0. 2x + y 7 8; x + 2y 7 14; x 8 0; y80
Let P(x, y) be the proit function, then P(x, y) = 40x + 50y. 6. Minimize z = 3x + y; subject to the constraints:
Now the problem is to maximize P subject to the constraints: 3x + 5y 8 15; x + 6y 8 9; x 8 0; y80
5x + 4y 7 120 7. Each unit of food X costs Rs. 25 and contains 2 units of protein and 4 units of iron
2x + 4y 7 72 ; x80,y80 while each unit of food Y costs Rs. 30 and contains 3 units of protein and 2 unit of iron.
Solving 5x + 4y = 120 and 2x + 4y = 72, gives 3x = 48 ⇒ x = 16 and Each animal must receive at least 12 units of protein and 16 units of iron each day.
4y = 72 - 2x = 72 - 32 = 40 ⇒ y = 10. How many units of each food should be fed to each animal at the smallest
Thus their point of intersection is (16, 10). Graphing the linear inequality constraints, possible cost?
the feasible region obtained is depicted in the igure 5.72 by shading. The corner points of 8. A dealer wishes to purchase a number of fans and sewing machines. He has only
the feasible region are (0, 0), (0, 18), (16, 10) and (24, 0). Rs. 5760 to invest and has space atmost for 20 items. A fan costs him Rs. 360 and
Now we ind the values of P at the comer points. a sewing machine costs Rs. 240. His expectation is that the can sell a fan at a proit
Corner point P(x, y) = 40x + 50y of Rs. 22 and a sewing machine at a proit of Rs. 18. Assuming that he can sell all the
(0, 0) P(0, 0) = 40 x 0 + 50 x 0 = 0 items that he can buy, how should he invest his money in order to maximize his proit?
(0, 18) P(0, 18) = 40 x 0 + 50 x 18 = 900 9. A machine can produce product A by using 2 units of chemical and 1 unit of a
(16, 10) P(16, 10) = 40 x 16 + 50 x 10 = 1140 compound or can produce product B by using 1 unit of chemical and 2 units of the
(24, 0) P(24, 0) = 40 x 24 + 50 x 0 = 960 compound. Only 800 units of chemical and 1000 units of the compound are available.
From the above table, it follows, that the maximum proit is Rs. 1140 at the corner The proits per unit of A and B are Rs. 30 and Rs. 20 respectively, maximize the
point (16, 10). Manufacturer gets the maximum proit if he manufactures 16 bicycles and 10 proit function.
motorcycles.

EXERCISE 5.3

1. Maximize f(x, y) = 2x + 5y
subject to the constraints
2y - x 7 8; x - y 7 4; x 0 8 0; y80
2. Maximize f(x , y) = x + 3y
subject to the constraints
2x + 5y 7 30; 5x + 4y 7 20; x 8 0; y80
3. Maximize z = 2x + 3y; subject to the constraints:
3x + 4y 7 12; 2x + y 7 4: 4x - y 7 4; x 8 0; y 8 0
4. Minimize z = 2x + y: subject to the constraints:
x + y 8 3; 7x + 5y 7 35; x 8 0; y80

version: 1.1 version: 1.1

24 25
version: 1.1

CHAPTER

6 Conic Sections

Animation 6.1: Conic Section


Source and credit: eLearn.Punjab
1. Quadratic Equations eLearn.Punjab 1. Quadratic Equations eLearn.Punjab
6. Conic Sections eLearn.Punjab 6. Conic Sections eLearn.Punjab

The theory of conics plays an important role in modern space mechanics, occeangraphy
6.1 INTRODUCTION and many other branches of science and technology.
We irst study the properties of a Circle. Other conics will be taken up later.
Conic sections or simply conics, are the curves obtained by cutting a (double)
right circular cone by a plane. Let RS be a line through the centre C of a given circle and
6.1.1 Equation of a Circle
perpendicular to its plane. Let A be a ixed point on RS. All lines through A and points on the
circle generate a right circular cone. The lines are called rulings or generators of the cone.
The set of all points in the plane that are equally distant from a ixed point is called a
The surface generated consists of two parts, called nappes, meeting at the ixed point A,
circle. The ixed point is called the centre of the circle and the distance from the center of
called the vertex or apex of the cone. The line RS is called axis of the cone.
the circle to any point on the circle is called the radius of the circle.
If the cone is cut by a plane perpendicular to the axis of the cone, then the section is a
If C(h,k) is centre of a circle, r its radius and P(x, y) any point on the circle, then the circle,
circle.

{ }
denoted S(C ; r) in set notation is

S ( C ; r ) P ( x, y ): CP
=
= r
By the distance formula, we get

CP = ( x - h) + ( y - k ) = r
2 2

( x - h) + ( y - k ) =
2 2
or r2 (1)
is an equation of the circle in standard form.
If the centre of the circle is the origin, then (1) reduces to
x2 + y2 = r2 (2)
If r = 0, the circle is called a point circle which consists
of the centre only.
The size of the circle depends on how near the plane is to the vertex of the cone. If the Let P(x, y) be any point on the circle (2) and let the
plane passes through the vertex A, the intersection is just a single point or a point circle. If inclination of OP be q as shown in the igure. It is clear that

x = r cos q 
the cutting plane is slightly tilted and cuts only one nappe of the cone, the resulting section

y = r sinq 
is an ellipse. If the intersecting plane is parallel to a generator of the cone, but intersects
(3)
its one nappe only , the curve of intersection is a parabola. If the cutting plane is parallel

The point P(r cosq, r sin q) lies on (2) for all values of
to the axis of the cone and intersects both of its nappes, then the curve of intersection is a

q. Equations (3) are called parametric equations of the


hyperbola.
The Greek mathematicians Apollonius’ (260-200 B.C.) and Pappus (early fourth
circle (2).
century) discovered many intersecting properties of the conic sections. They used the
methods of Euclidean geometry to study conics. We shall not study conics from the point
Example 1: Write an equation of the circle with centre (-3, 5) and radius 7.
of view stated above, but rather approach them with the more powerful tools of analytic
Solution: Required equation is
geometry.

version: 1.1 version: 1.1

2 3
1. Quadratic Equations eLearn.Punjab 1. Quadratic Equations eLearn.Punjab
6. Conic Sections eLearn.Punjab 6. Conic Sections eLearn.Punjab

(x + 3)2 + (y - 5)2 = 72
 -12 -18 
Thus centre of the circle =( - g ,- f ) =
x2 + y2 + 6x - 10y - 15 = 0

or
 5 5 
,

Radius of the circle= g 2 + f 2 - c= + -2


6.1.1 General Form of an Equation of a Circle 144 324
25 25

= =
Theorem: The equation 418 418
x2 + y2 +2gx + 2fy + c = 0 (1) 25 5
represents a circle g, f and c being constants.
Equation (1) can be written as:
(x2 + 2gx + g2) + (y2 + 2fy + f2) = g2+ f2 - c
6.1.2 Equations of Circles Determined by Given Conditions

(
 x - ( - g )  +  y - ( - f )=
 )
g + f -c
2 2 2 2
2 The general equation of a circle x2 + y2 + 2gx + 2fy + c = 0 contains three independent
or
which is standard form of an equation of a circle with centre (-g, - f) and radius
constants g, f and c, which can be found if the equation satisies three given conditions. We
discuss diferent cases in the following paragraphs.
g2 + f 2 - c .
The equation (1) is called general form of an equation of a circle. 1. A Circle Passing Through Three Non-collincar Points.
If three non-collinear points, through which a circle passes, are known, then we can
Note: ind the three independent constants f, g and c occurring in the general equation of a circle.
1. (1) is a second degree equation in which coeicient of each of x2 and y2 is 1.
2. (1) contains no term involving the product xy. Example 3: Find an equation of the circle which passes through the points A(5,10), B(6,9)
and C(-2,3).
Thus a second degree equation in which coeicients of x2 and y2 are equal and there is
no product term xy represents a circle. Solution: Suppose equation of the required circle is
If three non-collinear points through which a circle passes are known, then we can ind x2 + y2 + 2gx + 2fy + c = 0 (1)
the three constants f, g and c in (1). Since the three given points lie on the circle, they all satisfy (1). Substituting the three
points into (1), we get
Example 2: Show that the equation: 25 + 100 + 10g + 20f + c = 0
5x2 + 5y2 + 24x + 36y + 10 = 0 ⇒ 10g + 20f + c + 125 = 0 (2)
represents a circle. Also ind its centre and radius. 36 + 81 +12g + 18f + c + 117 = 0
Solution: The given equation can be written as: ⇒ 12g + 18f + c + 117 = 0 (3)
4 + 9 - 4g + 6f + c = 0
x2 + y 2 +
x+ y+2= -4g + 6f + c + 13 = 0
24 36
0 (4)
5 5
which is an equation of a circle in the general form. Here Now we solve the equations (2), (3) and (4).

= = =
Subtracting (3) from (2), we have
-2g + 2f + 8 = 0
12 18
g ,f ,c 2
5 5
version: 1.1 version: 1.1

4 5
1. Quadratic Equations eLearn.Punjab 1. Quadratic Equations eLearn.Punjab
6. Conic Sections eLearn.Punjab 6. Conic Sections eLearn.Punjab

or g-f-4=0 (5) 3. A circle passing through two points and equation of tangent at one of these
Subtracting (4) from (2), we have. points is known.
14g + 14f + 112 = 0 (6)
or g+f+8=0 Example 5: Find an equation of the circle passing through the point (-2, -5) and touching
From (5) and (6), we have, the line 3x + 4y - 24 = 0 at the point (4, 3).
f = -6 and g = -2.
Inserting the values of f and g into (2), we get c = 15 Solution: Let the circle be
Thus equation of the circle is: x2 + y2 - 4x - 12y + 15 = 0 x2 + y2 + 2gx + 2fy + c = 0 (1)
The points (-2, -5 ) and (4, 3) lie on it. Therefore
2. A circle passing through two points and having its centre on a given line. -4g - 10f + c + 29 = 0 (2)
8g + 6f + c + 25 = 0 (3)
Example 4: Find an equation of the circle having the join of A (x1, y1) and B (x2, y2) as a The line
diameter. 3x + 4y - 24 = 0 (4)
Touches the circle at (4, 3).
Solution: Since AB is a diameter of the circle, its A line through (4, 3) and perpendicular to (4) is

y - 3= ( x - 4) or 4 x - 3 y - 7= 0
midpoint is the centre of the circle. The radius of the
4
circle is known and standard form of an equation of the
3
This line being a normal through (4, 3) passes through the centre (-g, -f) of the
circle may be easily written. However, a more elegant
procedure is to make use of the plane geometry. If
circle (1). Therefore
-4 g + 3f - 7 = 0
P(x, y) is any point on the circle, then m∠APB = 900
(5)
From (2) - (3), we get
Thus the lines AP and BP are perpendicular to each

-12g - 16f + 4 = 0
other.

y - y1 y - y2 3g + 4f - 1 = 0
=
=
or (6)
x - x1 x - x2 Solving (5) and (6), we have g = -1, f = 1. Inserting these values of g and f into (3),
Slope of AP and Slope of BP

we ind c = -23. Equation of the required circle is


x2 + y2 - 2x + 2y - 23 = 0
By the condition of perpendicularity of two lines, we get

y - y1 y - y2
× - =
x - x1 x - x2
1
4. A circle passing through two points and touching a given line.

(x - x1) (x - x2) + (y - y1)(y - y2) = 0


Find an equation of the circle passing through the points A(1, 2) and B(1, -2)
or
Example 6:
This is required equation of the circle.
and touching the line x + 2y + 5 = 0.

Solution: Let O(h, k) be the centre of the required circle. Then


version: 1.1 version: 1.1

6 7
1. Quadratic Equations eLearn.Punjab 1. Quadratic Equations eLearn.Punjab
6. Conic Sections eLearn.Punjab 6. Conic Sections eLearn.Punjab

= OB
= radius of the circle.
2. Find the centre and radius of the circle with the given equation
OA (a) x2 + y2 +12x - 10y = 0
i.e., (h - 1)2 + (k - 2)2 = (h - 1)2 + (k + 2)2 (b) 5x2 + 5y2 + 14x + 12y - 10 = 0
(c) x2 + y2 - 6x + 4y + 13 = 0
(d) 4x2 + 4y2 - 8x +12y - 25 = 0
or 8k = 0 i.e., k = 0

Hence OA = OB

( h - 1)
3. Write an equation of the circle that passes through the given points
= +4 (a) A(4, 5), B(-4, -3 ), C(8, -3)
2

Now length of perpendicular from (h, k) i.e., (h, 0) to the line (b) A(-7, 7), B(5, -1), C(10, 0)
(c) A(a, 0), B(0, b), C(0, 0)
(d) A(5, 6), B(-3, 2), C(3, -4)
x + 2y + 5 = 0 equals the radius of the circle and is given by

h+5 4. In each of the following, ind an equation of the circle passing through
5 (a) A(3, -1), B(0, 1) and having centre at 4x - 3y - 3 = 0
(b) A(-3, 1) with radius 2 and centre at 2x - 3y + 3 = 0
h+5
= OA = ( h - 1) +4 A(5,1) and tangent to the line 2x - y - 10 = 0 at B(3, -4)
2
Therefore, (c)
(d) A(1, 4), B(-1, 8) and tangent to the line x + 3y - 3 = 0
5

( h + 5) =( h - 1) + 4 or 4h 2 - 20h =0
5. Find an equation of a circle of radius a and lying in the second quadrant such that it
h =0 ,5
2
2
is tangent to both the axes.
Show that the lines 3x - 2y = 0 and 2x + 3y - 13 = 0 are tangents to the circle
or i.e.,
5 6.
Thus centres of the two circles are at (0, 0) and (5, 0). x2 + y2 + 6x - 4y = 0
Radius of the irst circle = 5 ; Radius of the second circle = 20 7. Show that the circles
Equations of the circles are x2 + y2 + 2x - 2y - 7 = 0 and x2 + y2 - 6x + 4y + 9 = 0 touch externally.
(x - 5)2 + y2 = 20 8. Show that the circles
x2 + y2 + 2x - 8 = 0 and x2 + y2 - 6x + 6y - 46 = 0 touch internally.
x2 + y2 = 5 and
i.e., x2 + y 2 = 5 and x2 + y2 - 10x + 5 = 0
9. Find equations of the circles of radius 2 and tangent to the line
EXERCISE 6.1 x - y - 4 = 0 at A(1, -3).

1. In each of the following, ind an equation of the circle with 6.2 TANGENTS AND NORMALS
(a) centre at (5, -2) and radius 4

( )
A tangent to a curve is a line that touches the curve without cutting through it.
(b) centre at 2 ,-3 3 and radius 2 2 We know that for any curve whose equation is given by y = f(x) or f(x, y) = 0, the derivative
ends of a diameter at (-3, 2) and (5, -6).
dy
(c) is slope of the tangent at any point P(x, y) to the curve. The equation of the tangent to
dx
the curve can easily be written by the pointslope formula. The normal to the curve at P is
the line through P perpendicular to the tangent to the curve at P. This method can be very

version: 1.1 version: 1.1

8 9
1. Quadratic Equations eLearn.Punjab 1. Quadratic Equations eLearn.Punjab
6. Conic Sections eLearn.Punjab 6. Conic Sections eLearn.Punjab

conveniently employed to ind equations of tangent and normal to the circle Theorem: The point P(x1, y1) lies outside, on or inside the circle
x2 + y2 + 2gx + 2fy + c = 0 at the point P(x1, y1). x2 + y2 + 2gx + 2fy + c = 0 according as

>
Here f(x, y) = x2 + y2 +2gx + 2fy + c = 0 (1)
x12 + y12 + 2 gx1 + 2 fy1 + c =0
Diferentiating (1) w.r.t. x, we get <

x+g
2x + 2 y + 2g + 2 f =
0 or - =
dy dy dy Proof. Radius r of the given circle is
y+ f
r= g 2 + f 2 - c.
dx dx dx
dy  x +g
 = - 1
y1 + f
The point P(x1, y1) lies outside, on or inside the circle, according as:
dx  ( x1 ,y1 )
= Slope of the tangent at (x1 , y1 )
>
m CP =  r

}
<
Equation of the Tangent at P is given by

x1 + g >
y - y1 =
- ( x - x1 ) ( x1 + g ) 2 + ( y1 + f ) 2 =< g2 + f 2 - c

}
y1 + f
(Point-slope form) i.e., according as:

>
y ( y1 + f ) - y12 - y1 f =
- x ( x1 + g ) + x12 + x1 g x12 + 2 gx1 + g 2 + y12 + f 2 + 2 fy1 <
= g2 + f 2 - c

}
or or according as :

or xx1 + yy1 + gx + fy = x12 + y12 + gx1 + fy1 or according as : >


x12 + y12 + 2 gx1 + 2 fy1 +c <
= 0.

or xx1 + yy1 + gx + fy + gx1 + fy1 + c = x12 + y12 + gx1 + fy1 + gx1 + fy1 + c
Example 1: Determine whether the point P(-5, 6) lies outside, on or inside the circle:
(adding gx1 + fy1 + c to both sides) x2 + y2 + 4x - 6y - 12 = 0
or xx1 + yy1 + g(x + x1) + f(y + y1) + c = 0
since (x1, y1) lies on (1) and so Solution: Putting x = -5 and y = 6 in the left hand member of the equation of the circle,
x12 + y12 + 2 gx1 + 2 fy1 + c =0 we get
25 + 36 - 20 - 36 - 1 2 = -7 < 0
Thus xx1 + yy1 + g(x + x1) + f(y + y1) + c = 0 , is the required equation of the tangent.
Thus the point P(-5, 6) lies inside the circle.
To ind an equation of the normal at P, we note that slope of the normal is
y1 + f
Theorem: The line y = mx + c intersects the circle x2 + y2 = a2

x1 + g
(negative reciprocal of slope of the tangent) in at the most two points.
Proof: It is known from plane geometry that a line can meet a
Equation of the normal at P(x1, y1) is circle in at the most two points.
y1 + f
y=- y1 (x - x1 )
To prove it analytically, we note that the coordinates of the
x1 + g points where the line
(y - y1)(x1 + g) = (x - x1)(y1 + f), is an equation of the normal at (x1, y1).
y = mx + c (1)
or
intersects the circle
x 2 + y2 = a2 (2)
are the simultaneous solutions of the equations (1) and (2).

version: 1.1 version: 1.1

10 11
1. Quadratic Equations eLearn.Punjab 1. Quadratic Equations eLearn.Punjab
6. Conic Sections eLearn.Punjab 6. Conic Sections eLearn.Punjab

Substituting the value of y from equation (1) into equation (2), we get
8 9
Thus the points of intersection are P(2,1) and Q  , 
x2 + (mx + c)2 = a2
or x2(1 + m2) + 2mcx + c2 - a2 = 0 (3) 5 5
Length of the chord intercepted
This being quadratic in x, gives two values of x say x1 and x2. Thus the line intersects
8  9 
= PQ =  - 2  +  - 1 = + =
2 2
the circle in at the most two points. For nature of the points we examine the discriminant 4 16 2
of (3). 5  5  25 25 5
The discriminant of (3) is (2mc)2 - 4(1 + m2) (c2 - a2)
= 4m2 c2 - 4(1 + m2)(c2 - a2)
Theorem: Two tangents can be drawn to a circle from any point P(x1, y1). The tangents

= 4m2 c2 - 4m2 c2 - 4(c2 - a2 - a2m2)


are real and distinct, coincident or imaginary according as the point lies outside, on or inside
the circle.
= 4 [- c2 + a2(1 + m2)]
These points are
a2(1 + m2) - c2 > 0
Proof: Let an equation of the circle be x2 + y2 = a2
(i) Real and distinct, if
(ii) Real and coincident if a2(1 + m2) - c2 = 0
We have already seen that the line

(iii) Imaginary if a2(1 + m2) - c2 < 0 y =mx + a 1 + m 2


Condition that the line may be a tangent to the circle. is a tangent to the given circle for all values of m. If it passes through the point
The line (1) is tangent to the circle (2) if it meets the circle in one point. P(x1, y1),then

c2 = a2(1 + m2) or ±c = a+ 1 m 2 y1 = mx1 + a 1 + m 2


(y1 - mx1)2 = a2(1 + m2)
i.e., if
is the condition for (1) to be a tangent to (2). or
or m 2 ( x12 - a 2 ) - 2mx1 y1 + y12 - a 2 =
0
Example 2: Find the co-ordinates of the points of intersection of the line 2x + y = 5 and This being quadratic in m, gives two values of m and so there are two tangents from
the circle x + y + 2x - 9 = 0. Also ind the length of the intercepted chord.
2 2
P(x1, y1) to the circle. These tangents are real and distinct, coincident or imaginary according

}
as the roots of (2) are real and distinct, coincident or imaginary

>
i.e., according as x12 y12 - ( x12 - a 2 )( y12 - a 2 ) <
=
Solution: From 2x + y = 5, we have

} }
y = (5 - 2x). 0
Inserting this value of y into the equation of the circle, we get
x12 a 2 + y1= > 0
a + a4 < x12 += > 0
y12 - a 2 <
x2 + (5 - 2x)2 + 2x - 9 = 0
2 2
or or

or 5x2 - 18x + 16 = 0 i.e., according as the point P(x1, y1) lies outside, on or inside the circle x2 + y2 - a2 = 0

18 ± 324 - 320 18 ± 2
⇒ x= = =
8 Example 3: Write equations of two tangents from (2, 3) to the circle x2 + y2 = 9.
2,
10 10 5
When x = 2, y = 5 - 4 = 1 Solution. Any tangent to the circle is
y =mx + 3 1 + m 2
When x = , y =5 - =
8 16 9
5 5 5 If it passes through (2, 3), then

version: 1.1 version: 1.1

12 13
1. Quadratic Equations eLearn.Punjab 1. Quadratic Equations eLearn.Punjab
6. Conic Sections eLearn.Punjab 6. Conic Sections eLearn.Punjab

3 = 2m + 3 1 + m
Example 5: Find a joint equation to the pair of tangents drawn from (5, 0) to the circle:
2
(1)
(3 - 2m)2 = 9(1 + m2)
x2 + y2 = 9 (1)
or
or 9 - 12m + 4m2 = 9 + 9m2 Solution: Let P(h,k) be any point on either of the two tangents drawn from A(5,0) to the
-12
5m 2 += =
given circle (1). Equation of PA is

k -0
or 12m 0 i.e., m 0,
y=
-0 ( x - 5) or kx - (h - 5) y =
- 5k 0
5
h-5
Inserting these values of m into (1), we have equations of the tangents from (2,3) to (2)
the circle as : Since (2) is tangent to the circle (1), the perpendicular distance of (2) from the centre of the

For m = 0 : y = 0. x + 3 1 + 0
circle equals the radius of the circle.

or y = 3 -5k
=3
-12 -12 -12 k + (h - 5)
i.e.,
m= : y= x + 3 1+ = x+
2 2
144 39
25k 2= 9[k 2 + (h - 5) 2 ] or 16k 2 - 9(h - 5) 2 = 0
For
5 5 25 5 5 or
5 y + 12 x - 39 = Thus (h,k) lies on
9(x - 5)2 - 16y2 = 0
or 0.
(3)
Example 4: Write equations of the tangents to the circle But (h,k) is any point of either of the two tangents.
x + y - 4x + 6y + 9 = 0
2 2
(1) Hence (3) is the joint equations of the two tangents.
at the points on the circle whose ordinate is -2.
6.2.1 Length of the tangent to a circle
Solution: Substituting y = -2 into (1), we get (Tangential Distance)
x2 - 4x + 1 = 0

4 ± 16 - 4
Let P(x1, y1) be a point outside the circle
or =
x = ±2 3 x2 + y2 +2gx + 2fy + c = 0 (1)
2
The points on the circle with ordinate -2 are
We know that two real and distinct tangents can be drawn to the circle from an external

(2 + 3, -2),(2 - 3, -2)
point P. If the points of contact of these tangents with the circle are S and T, then each of
the length PS and PT is called length of the tangent or tangential distance from P to the
Equations of the tangents to (1) at these points are circle (1).
(2 + 3) x - 2 y - 2( x + 2 + 3) + 3( y - 2) + 9 =0 The centre of the circle has coordinates
and (2 - 3) x - 2 y - 2( x + 2 - 3) + 3( y - 2) + 9 =0 (-g, -f). Join PO and OT. From the right triangle OPT

3 x + y - 2 3 - 1 =0
we have,
i.e.,
and - 3 x + y + 2 3 - 1 =0 length of the tangent ==
PT OP 2 - OT 2

= ( x1 + g ) 2 + ( y1 + f ) 2 - ( g 2 + f 2 - c)

version: 1.1 version: 1.1

14 15
1. Quadratic Equations eLearn.Punjab 1. Quadratic Equations eLearn.Punjab
6. Conic Sections eLearn.Punjab 6. Conic Sections eLearn.Punjab

For m =-2 : -2 x - y =0 or 2 x + y =
= x12 + y12 + 2 gx1 + 2 fy1 + c
0
(2)

For= : x - y += 0 or x - 2 y = +5 0
It is easy to see that length of the second tangent PS also equals (2). 1 1 5
m
2 2 2
Example 6: Find the length of the tangent from the point P(-5, 10) to the circle Tangential distance = 1 + 4 - 4 + 4= 5
5x2 + 5y2 + 14x + 12y - 10 = 0
Example 8: Tangents are drawn from (-3,4) to the circle x2 + y2 = 21. Find an equation
Solution: Equation of the given circle in standard form is of the line joining the points of contact (The line is called the chord of contact).

x2 + y 2 + x+ y-2=
14 12
0 (2) Solution: Let the points of contact of the two tangents be P(x1, y1) and Q(x2, y2)
5 5
An equation of the tangent at P is
Square of the length of the tangent from P(-5,10) to the circle (1) is obtained by
substituting -5 for x and 10 for y in the left hand member of (1)
xx1 + yy1 = 21 (1)
An equation of the tangent at Q is
∴ Required length = (-5) 2 + (10) 2 - 14 + 24 - 2 =133 xx2 + yy2 = 21 (2)
Since (1) and (2) pass through (-3 ,4 ), so
-3x1 +4y1 = 21 (3)
and -3x2 + 4y2 =21
Example 7: Write equations of the tangent lines to the circle x2 + y2 + 4x + 2y = 0
drawn from P(-1,2). Also ind the tangential distance. (4)
(3) and (4) show that both the points P(x1, y1) , Q(x2, y2) lie on -3x + 4y = 21 and so it is
Solution: An equation of the line through P(-1,2) having slope m is the required equation of the chord of contact.
y - 2 = m(x + 1) or mx - y + m + 2 = 0. (1)
Centre of the circle is C(-2,-l). EXERCISE 6.2
Radius = 4 + 1 = 5
1. Write down equations of the tangent and normal to the circle
x2 + y2 = 25 at (4 , 3) and at (5 cos q, 5 sin q)
If (1) is tangent to the circle, then its distance from the centre of the circle equals the
radius of the circle. Therefore (i)

-2m + 1 + m + 2  10 
3 x 2 + 3 y 2 + 5 x - 13 y + 2 =0 at 1, 
= 5  3
(ii)
m2 + 1
2. Write down equations of the tangent and normal to the circle
4x2 + 4y2 - 16x + 24y - 117 = 0
or (-m + 3)2 = 5(m2 +1)
4m2 + 6m - 4 = 0 or 2m2+ 3m - 2 = 0
at the points on the circle whose abscissa is -4.
or

-3 ± 9 + 16 -3 ± 5
m= = = - 2,
1 3. Check the position of the point (5 , 6) with respect to the circle
4 4 2 (i) x2 + y2 = 81 (ii) 2x2 + 2y2 + 12x - 8y + 1 = 0
Equations of the tangents are from equation (1) 4. Find the length of the tangent drawn from the point (-5 , 4) to the circle
5x2 + 5y2 - 10x + 15y - 131 = 0
version: 1.1 version: 1.1

16 17
1. Quadratic Equations eLearn.Punjab 1. Quadratic Equations eLearn.Punjab
6. Conic Sections eLearn.Punjab 6. Conic Sections eLearn.Punjab

5. Find the length of the chord cut of from the line 2x + 3y = 13 by the circle

x=
+ 2 x a2 or x 2 (=
x12 + y12 ) a 2 x12
x2 + y2 = 26 2y12 2
6. Find the coordinates of the points of intersection of the line x + 2y = 6 with the circle: x1
x2 + y2 - 2x - 2y - 39 = 0 or a 2 x 2 = a 2 x12 ( x12 + y12 =
x2 )
x = ± x1
7. Find equations of the tangents to the circle x2 + y2 = 2
parallel to the line x - 2y + 1 = 0
i.e.,
(i)
 y1 
= x x= then y y1=
 y .x 
(ii) perpendicular to the line 3x + 2y = 6
 x1 
If 1,
8. Find equations of the tangents drawn from

Similarly when x = -x1, y = -y1


(i) (0 , 5) to x2 + y2 = 16
then
Thus B has coordinates (-x1 , -y1).
(ii) (-1 ,2 ) to x2 + y2 + 4x + 2y = 0
(iii) (-7, -2 ) to (x + 1)2 + (y - 2)2 = 26
Also ind the points of contact Length of diameter AB = ( x1 + x1 ) 2 + ( y1 + y1 ) 2
9. Find an equation of the chord of contact of the tangents drawn from (4 , 5) to the circle = 4( x12 + y12 )= = 2a
2x2 + 2y2 - 8x + 12y + 21 = 0
4a 2

Theorem 2: Perpendicular dropped from the centre of a circle on a chord bisects the
6.3 ANALYTIC PROOFS OF IMPORTANT chord.
PROPERTIES OF A CIRCLE
Proof: Let x2 + y2 = a2 be a circle, in which AB is a chord with
A line segment whose end points lie on a circle is called a chord of the circle. A diameter end points A(x1 , y1), B(x2 , y2) on the circle and OM is perpendicular
of a circle is a chord containing the centre of the circle. from the centre to the chord. We need to show that OM bisects
the chord AB.

y2 - y1
Theorem: Length of a diameter of the circle x2 + y2 = a2 is 2a.
Slop of AB =
Proof: Let AOB be a diameter of the circle x2 - x1

-( x2 - x1 ) x1 - x2
x2 + y2 = a2 (1)
Slop of perpendicular= = = m (say)
y2 - y1 y2 - y1
O(0,0) is center of (1). to AB
Let the coordinates of A be (x1, y1).
Equation of AOB is So equation of OM with slope m and point O(0,0) on it, is given by

( x1 - x2 )
y= =
y-0 ( x - 0)
( y2 - y1 )
y1 (point - slope form)
x (2)
x1
 x -x 
y = 1 2 x
 y2 - y1 
Substituting the value of y from (2) into (1), we have or (1)

version: 1.1 version: 1.1

18 19
1. Quadratic Equations eLearn.Punjab 1. Quadratic Equations eLearn.Punjab
6. Conic Sections eLearn.Punjab 6. Conic Sections eLearn.Punjab

(1) is the equation of the perpendicular OM from centre to the chord. We will show that it Theorem 3:
bisects the chord i.e., intersection of OM and AB is the midpoint of AB. The perpendicular bisector of any chord of a circle passes through the centre of the
Equation of AB is circle.

y1 - y2
y=
- y1 ( x - x1 )
x1 - x2
(2) Proof: Let x2 + y2 = a2 be a circle and A(x1 , y1),
B(x2 , y2) be the end points of a chord of this circle. Let M be the mid point of AB, i.e.

 x + x y + y2 
The foot of the perpendicular OM is the point of intersection of (1) and (2). Inserting the
M 1 2, 1 
 2 2 
value of y from (1) into (2), we have

x1 - x2 y1 - y2
- =
x - y1 ( x - x1 ) y2 - y1
y1 - y2 x1 - x2 The slop of AB =
x2 - x1
 y - y2 x1 - x2  x1 ( y1 - y2 )
x 1 + = - y1
 x1 - x2 y1 - y2  x1 - x2
or The slope of perpendicular bisector of AB is

x  y1 + y2 - 2 y1 y2 + x1 + x2 - 2 x1 x2  x y - x y  x -x 
= 2 1 1 2 - 2 1 
2 2 2 2

( x1 - x2 ) ( y1 - y2 ) x1 - x2  y2 - y1 
or

x(2a - 2 x1 x2 - 2 y1 y2 ) =x2 y1 - x1 y1 y2 - x2 y1 y2 + x1 y2 So, equation of perpendicular bisector in point-slope form, is


2 2 2
or
2 x(a - x1 x2 - y1 y2 )= x2 (a - x1 ) - y1 y2 ( x1 + x2 ) + x1 (a - x2 )
y +y  x2 - x1   x +x 
2 2 2 2 2
or
= a ( x1 + x2 ) - x1 x2 ( x1 + x2 ) - y1 y2 ( x1 + x2 ) y - 1- 2 =  -  x 1 2 
 y2 - y1   2 
2
(1)
= ( x1 + x2 ) (a - x1 x2 - y1 y2 )
2
2
We check whether the centre (0,0) of the circle lies on (1) or not
(The points (x1 , y1) and (x2 , y2) lie on the circle)

x +x y1 + y2 -( x2 - x1 )  x1 + x2 
x= 1 2 0- = 0 - 
or ( y2 - y1 )  2 
( x1 + x2 )
2
x +x  y + y2 
2
x = 1 2 into (1) , we get - 1  ( y2 - y1 ) = ( x2 - x1 )
 2 
Putting or
2 2
( x1 - x2 ) ( x1 + x2 ) x1 - x2 - ( y2 - y1 ) = x2 - x1 or x1 + y1 = x2 + y2
==
2 2 2 2 2 2 2 2 2 2

y2 - y1 2( y2 - y1 )
or
a2 = a2
y .
2 or which is true
 x12 + y12 = 
y -y ( y2 - y1 )( y2 + y1 )  2 
2
a Hence the perpendicular bisector of any chord passes through the centre of the circle.
= =  x2 + y2 = 
2 2

2( y2 - y1 ) 2( y2 - y1 )
2 1

 2
or y 2
a
2

y + y2 ⇒ x1 - x2 = y1 - y2 
Theorem 4:
y= 1
2 2 2

or The line joining the centre of a circle to the midpoint of a chord is perpendicular to the
2
 x1 + x2 y1 + y2 
chord.

  is the point of intersection of OM and AB which is the midpoint of AB.


 2 2 
So, ,
Proof: Let A(x1 , y1) , B(x2 , y2) be the end points of any chord the circle x2 + y2 = a2. O(0, 0)
version: 1.1 version: 1.1

20 21
1. Quadratic Equations eLearn.Punjab 1. Quadratic Equations eLearn.Punjab
6. Conic Sections eLearn.Punjab 6. Conic Sections eLearn.Punjab

 x + x y + y2   y + y2  x +x  y1 + y2 + x1 + x2 + 2 x1 x2 + 2 y1 y2
is centre of the circle and M  1 2 , 1  is the midpoint of ∴ OM=  1 - 0  +  1 2 - 0= 
2 2 2 2 2 2

 2 2   2   2 
2

4
( x + y1 ) + ( x2 + y2 ) + 2 x1 x2 + 2 y1 y2
= 1
AB. Join the centre O with the mid point M. We need to show 2 2 2 2

that OM is perpendicular to AB i.e., product of slopes of AB and


OM is -1.
4
a + a + 2 x1 x2 + 2 y1 y2
=
2 2

(  A and B lie on the circle.)


y2 + y1
-0
4
y2 - y1 y2 + y1 2a + 2 x1 x2 + 2 y1 y2
= m= =of = = OM =
2

x2 - x1 x2 + x1 x2 + x1
2 2

-0
Slope of AB 1 ; Slope OM m2
4
a + x1 x2 + y1 y2
=
2 2

(1)
y2 - y1 y2 + y1 y2 - y1
2
∴= .= a + x3 x4 + y3 y4
2 2

x2 - x1 x2 + x1 x22 - x12 Similarly ON =


2
m1m2 (1) 2

2 (2)
We know that AB = CD
2 2
As A and B lie on the circle, so ( chords are congruent)
=
x1 + y1 a and =
x2 + y2 a or ( x2 - x1 ) + ( y2 - y1 ) = ( x4 - x3 ) + ( y4 - y3 )
2 2 2 2 2 2 2 2 2 2

Their subtraction gives x1 - x2 + y1 - y2 = or x2 + x1 + y2 + y1 - 2 x1 x2 - 2 y1 y2 = x4 + x3 - 2 x3 x4 + y4 + y3 - 2 y3 y4


2 2 2 2 2 2 2 2 2 2 2 2
0
or y2 - y1 = x1 - x2 =
-( x2 - x1 ) or a + a - 2 x1 x2 - 2 y1 y2 = a + a - 2 x3 x4 - 2 y3 y4 ( x1 + y1 = a etc)
2 2 2 2 2 2
(2) 2 2 2 2 2 2 2

Putting this value in (1), we get or 2a - 2 x1 x2 - 2 y1 y2 =2a - 2 x3 x4 - 2 y3 y4


2 2

or x1 x2 + y1 y2 = x3 x4 + y3 y4
Challenge!
( x2 - x1 )
(3)
- m1-m2 = =
2 2
State and prove the
( x2 - x1 ) OM = ON
1 2 2
2 2
or converse of this Theorem.
So OM is perpendicular to AB.
Theorem 6: Show that measure of the central angle of a
Theorem 5: Congruent chords of a circle are equidistant from the centre. minor arc is double the measure of the angle subtended in the
corresponding major arc.
Proof: Let x2 + y2 = a2 be the circle in which AB and CD are
two congruent chords i.e., AB = CD and the coordinates of Proof: Let the circle be x2 + y2 = a2.
A, B, C and D be as in the igure. Also let OM and ON be the A(a cosq1 , a sinq1) and B(a cosq2 , a sinq2) be end points of a
perpendicular distances of the chords from the centre (0, 0) minor arc AB. Let P (a cosq , a sinq) be a point on the major arc.
of the circle. Central angle subtended by the minor arc AB is ∠ AOB = q2 - q1.

We need to show m∠APB = (q 2 - q1 )


We know from Theorem 2 that M and N are the midpoints
1
of AB and CD respectively.
2

version: 1.1 version: 1.1

22 23
1. Quadratic Equations eLearn.Punjab 1. Quadratic Equations eLearn.Punjab
6. Conic Sections eLearn.Punjab 6. Conic Sections eLearn.Punjab

y - y2
q + q1 q - q1 m1m2 = 12
2 2

a (sin q - sin q1 ) x1 - x2
=
==
2

a ( cosq - cosq1 )
2cos sin (1)
q + q1 q - q1
2 2
-2sin
m1 slope of AP
sin Since A( x1 , y1 ) and P ( x2 , y2 ) lie on the circle, we have

 q + q1   p q + q1  x1 + y1 = a ⇒ x1 = a - y1 
2 2
= + - = 
2 2 2 2 2 2

cot  tan  
 2  2 2  x2 + y2 = a ⇒ x2 = a - y2 
2 2 2 2 2 2
(2)

Similarly, (by symmetry) 2 2


Substituting the values of x1 and x2 from (2) into (1), we get
p q + q2 
=m2 = + tan   y1 - y2 y1 - y2
2  m1m2 = 2 = = -1
2 2 2 2
slope of BP
(a - y1 ) - (a - y2 ) -( y1 - y2 )
2
 p q + q2   p q + q1 
2 2 2 2 2

tan  +  - tan  +  Thus AP ⊥ BP and so m∠APB =


m2 - m1 2 2  2 2 

tan ∠APB
= =
90
1 + m1m2  p q + q1   p q + q2 
1 + tan  +  .tan  + 
2 2  2 2 
 p q + q 2 p q + q1  q -q 
= tan  + - - =
Theorem 8: The tangent to a circle at any point of the circle is perpendicular to the
 tan  2 1 
2 2   2 
2 2 radial segment at that point.

Hence m∠APB = (q 2 - q1 )
1
Proof: Let PT be the tangent to the circle x2 + y2 = a2 at any point P(x1 , y1) lying on it.
We have to show that the radial segment OP ⊥ PT.
2

Theorem 7: An angle in a semi-circle is a right angle. Diferentiating x2 + y2 = a2, we have

2 x + 2 y. =
0 ⇒- =
dy dy x
Proof: Let x2 + y2 = a2 be a circle, with centre O. Let AOB be any diameter of the circle and dx dx y
- x1
Slope of the tangent at= =
P(x2 , y2) be any point on the circle.
dy
We have to show that m∠APB= 900. P
dx P y1
y1 - 0 y1
Slope of = =
Suppose the coordinates of A are (x1 , y1).

x1 - 0 x1
Then B has coordinates OP
(-x1 , -y1). (Theorem 1)
- x1 y1
. = -1
y1 - y2
Product of slopes of OP and PT =
= = m1 , say
y1 x1
x1 - x2 Thus OP ⊥ PT.
Slope of AP

y1 + y2
= = m2 , say
x1 + x2
Slope of BP Challenge!
State and prove the
converse of this Theorem.
version: 1.1 version: 1.1

24 25
1. Quadratic Equations eLearn.Punjab 1. Quadratic Equations eLearn.Punjab
6. Conic Sections eLearn.Punjab 6. Conic Sections eLearn.Punjab

Theorem 9: The perpendicular at the outer end of a radial segment is tangent to the
Suppose PM denotes the distance of a point P(x, y) from the line L. The set of all points
circle.
P in the plane such that

= e. (a positive constant)
PF
Proof: Let PT be the perpendicular to the outer end of the radial segment OP of the circle
PM
x2 + y2 = a2. We have to show that PT is tangent to the circle at P. Suppose the coordinates of
is called a conic section.
P are (x1 , y1).
(i) If e = 1, then the conic is a parabola.
Since PT is perpendicular to OP so
-1 -1 - x1
= = =
(ii) If 0 < e < 1, then the conic is an ellipse.
Slope of PT (iii) If e > 1, then the conic is a hyperbola.
slope of OP y1 y1
The ixed line L is called a directrix and the ixed point F is called a focus of the conic.
- x1
x1
Equation of PT is y - = ( x - x1 )
The number e is called the eccentricity of the conic.
y1
y1
or yy1 - y =- xx1 + x1 6.4 PARABOLA
2 2

or yy1 + xx1 = y1 + x1 = a (  P lies on the circle)


2 2 2

or yy1 + xx1 - a =
2
We have already stated that a conic section is a parabola if e = 1.
0
We shall irst derive an equation of a parabola in the standard form and study its
Distance of PT from O (centre of the circle)
y1 (0) + x1 (0) - a
important properties.

= = = =
2 2
a a
2
If we take the focus of the parabola as F (a, 0), a > 0 and its directrix as line L whose
x +y equation is x = -a, then its equation becomes very simple.
a (radius of the circle)
2 2
a
2 a
Let P(x, y) be a point on the parabola. So, by deinition
Thus PT is tangent to the circle at P(x1 , y1).

=
= 1. or
PF
PF PM
EXERCISE 6.3 PM

1. Prove that normal lines of a circle pass through the centre of the circle. Now PM = x + a (1)
2. Prove that the straight line drawn from the centre of a circle perpendicular to a
tangent passes through the point of tangency. and PF = ( x - a ) + ( y - 0)
2 2

3. Prove that the mid point of the hypotenuse of a right triangle is the circumcentre of Substituting into (1), we get

( x - a ) + y =x + a
the triangle. 2 2

( x - a) + y = ( x + a)
4. Prove that the perpendicular dropped from a point of a circle on a diameter is a mean
or 2 2 2
proportional between the segments into which it divides the diameter.

y = ( x + a ) - ( x - a ) = 4ax or y = 4ax
2 2 2 2
or (2)
In the following pages we shall study the remaining three conics. which is standard equation of the parabola.
Let L be a ixed line in a plane and F be a ixed point not on the line L.
version: 1.1 version: 1.1

26 27
1. Quadratic Equations eLearn.Punjab 1. Quadratic Equations eLearn.Punjab
6. Conic Sections eLearn.Punjab 6. Conic Sections eLearn.Punjab

Deinitions with either a = 0 or b = 0 but not both zero, represents a parabola. The equation can be
(i) The line through the focus and perpendicular to the directrix is called axis of the analyzed by completing the square.
parabola. In case of (2), the axis is y = 0.
(ii) The point where the axis meets the parabola is called vertex of the parabola. Clearly 6.4.2 Other Standard parabolas
the equation (2) has vertex A(0,0). The line through A and perpendicular to the axis
of the parabola has equation x = 0. It meets the parabola at coincident points and so There are other choices for the focus and directrix which also give standard equations
it is a tangent to the curve at A. of parabolas.
(i) If the focus lies on the y-axis with coordinates F(0,a) and directrix of the parabola is
y = -a, then equation of the parabola is
(iii) A line joining two distinct points on a parabola is called a chord of the parabola.
A chord passing through the focus of a parabola is called a focal chord of the
parabola. The focal chord perpendicular to the axis of the parabola (1) is called x2 = 4ay (3)
The equation can be derived by diinition.
(ii) If the focus is F(0, -a) and directrix is the line y = a, then equation of the parabola is
latusrectum of the parabola. It has an equation x = a and it intersects the curve at

x2 = -4ay
the points where
y = 4a y = ± 2a (4)
2 2
or
Thus coordinates of the end points L and L’ of the latusrectum are Opening of the parabola is upward in case of (3) and downward in case of (4). Both the
L ( a, 2a ) and L′(a, -2a ). curves are symmetric with respect to the y-axis.
The length of the latusrectum is LL′ = 4a. The graphs of (3) and (4) are shown below.
(iv) The point (at2 , 2at) lies on the parabola y2 = 4ax for any real t.
x = at2 , y = 2at
are called parametric equations of the parabola y2 = 4ax.

6.4.1 General Form of an Equation of a Parabola.

Let F(h,k) be the focus and the line lx + my + n =0 be the directrix of a parabola. An
equation of the parabola can be derived by the deinition of the parabola . Let P(x , y) be a
point on the parabola. Length of the perpendicular PM from P(x , y) to the directix is given by; (iii) If the focus of the parabola is F(-a, 0), and its directrix is the line x = a, then equation
of the parabola is
lx + my + n y2 = -4ax
PM =
l +m
( lx + my + n )
2 2

By definition, (x - h) + (y - k ) =
2 The curve is symmetric with respect to the x-axis

l +m
2 2
2 2 and lies in the second and third quadrants only. Opening of
the parabola is to the left as shown in the igure
is an equation of the required parabola.
A second degree equation of the form
ax2 + by2 + 2gx + 2fy + c = 0
version: 1.1 version: 1.1

28 29
1. Quadratic Equations eLearn.Punjab 1. Quadratic Equations eLearn.Punjab
6. Conic Sections eLearn.Punjab 6. Conic Sections eLearn.Punjab

6.4.3 Graph of the Parabola Here 4a = 16 or a = 4.


The focus of the parabola lies on the y-axis and its
y2 = 4ax opening is downward. Coordinates of the focus = (0, -4).
We note that corresponding to each positive value of x there are two equal and opposite Equation of its axis is x = 0
values of y. Thus the curve is symmetric with respect to the x-axis. Length of the latusrectum is 16 and y = 0 is tangent to
The curve passes through the origin and x = 0 is tangent the parabola at its vertex. The shape of the curve is as shown
to the curve at (0,0). If x is negative, then y2 is negative in the igure.
and so y is imaginary. Thus no portion of the curve lies
Example 2. Find an equation of the parabola whose focus is F (-3, 4) and directrix is
3x - 4y + 5 = 0.
on the left of the y-axis. As x increases, y also increases
numerically so that the curve extends to ininity and
lies in the irst and fourth quadrants. Opening of the
Solution: Let P(x , y) be a point on the parabola. Lentgh of the perpendicular PM from
P(x , y) to the directrix 3x - 4y + 5 = 0 is
parabola is to the right of y-axis.
Sketching graphs of other standard parabolas is
similar and is left as an exercise.
3x - 4 y + 5
3 + (-4)
PM
Summary of Standard Parabolas 2 2

=
=
Sr.No. 1 2 3 4
x = -4ay
By deinition, PF PM
2 2
2 2 2 2 or PF PM
Equation y = 4ax y =-4ax x = 4ay
(0, -a) (3 x - 4 y + 5)
( x + 3) + ( y - 4) =
Focus (a, 0) (-a, 0) (0, a)
x = -a y = -a
2
2 2
Directrix x=a y=a or

or 25(x + 6x + 9 + y - 8y + 16) = 9x2 + 16y2 + 25 - 24xy + 30x - 40y


25
Vertex (0,0) (0,0) (0,0) (0,0) 2 2

16x2 +24xy + 9y2 + 120x - 160y +600 = 0


Axis y=0 y=0 x=0 x=0
x = -a y = -a
or
Latusrectum x=a y=a
is an equation of the required parabola.

Example 3. Analyze the parabola


Graph x2 - 4x - 3y + 13 = 0
and sketch its graph.

Solution. The given equation may be written as


x2 - 4x + 4 = 3y - 9
Analyze the parabola x = -16y and draw its graph.
(1)
(x - 2)2 = 3(y - 3)
2
Example 1: or
Let x-2=X , y-3=Y (2)
Solution. We compare the given equation
with x2 = -4ay
The equation (2) becomes X2 = 3Y (3)

version: 1.1 version: 1.1

30 31
1. Quadratic Equations eLearn.Punjab 1. Quadratic Equations eLearn.Punjab
6. Conic Sections eLearn.Punjab 6. Conic Sections eLearn.Punjab

-3
which is a parabola whose focus lies on X = 0 and whose directix is Y =
Since y can take up only non-negative values, PF is minimum when y = 0. Thus P
4 coincides with A so that of all points on the parabola, its vertex A is closest to the focus.
Thus coordinates of the focus of (3) are
-3
= X 0= ,Y Example 4. A comet has a parabolic orbit with the sun at the focus. When the comet is
4 100 million km from the sun, the line joining the sun and the comet makes an angle of 600
i.e.,==
x - 2 0 - and with the axis of the parabola. How close will the comet get to the sun?
3
y 3
4
=
x 2,=
15 Solution. Let the sun S be the origin . If the vertex of the parabola has coordinates (-a,0)
or y
4 then directrix of the parabola is
Thus coordinates of the focus of the parabola x = -2a, (a >0)

 15 
if the comet is at P(x, y), then
(1) are  2,  by deinition PS = PM
 4
Axis of (3) is X = 0 or x - 2 = 0 is the axis of (1) .
i,e., x2 + y2 = (x + 2a)2
or y2 = 4ax + 4a2 is orbit of the comet
Veitex of (3) has coordinates
= x +y
2 2
X = 0, Y = 0 Now PS
or x - 2 = 0, y - 3 = 0 = x + 2a = 100,000,000
i.e., x = 2, y = 3 are coordinates of the vertex of (1). The comet is closest to the sun when it is at A.
Equation of the directrix of (3) is Now x = PS cos 600
-3 -3 PS x + 2a
= i.e. =
y -3 = = =
9
Y or y is an equation of the directrix of (1). x
4 4 4
x + 2a 2 x + 2a
2 2
= or 2= =
2- 2=
Magnitude of the latusrectum of the parabola (3) and also of (1) is 3.
or ,( x a a)
The graph of (1) can easily be sketched and is as shown in the above igure. x 1 2a

=2
100,000,000
Theorem: The point of a parabola which is closest to the focus is the vertex of the or

a = 25,000,000
2a
parabola. or
Proof: Let the parabola be Thus the comet is closest to the sun when it is 25,000,000 km from the sun.
x2 = 4ay , a > 0
with focus at F(0, a) and P(x, y) be any point on the Relecting Property of the parabola.
parabola. A frequently used property of a parabola is its relecting property. If a light source is

PF = x + ( y - a)
2 2 placed at the focus of a parabolic relecting surface then a light ray travelling from F to a point
P on the parabola will be relected in the direction PR parallel to the axis of the parabola.
= 4ay + ( y - a ) The designs of searchlights, relecting telescopes and microwave antenas are based
2

= y+a on relecting property of the parabola.


version: 1.1 version: 1.1

32 33
1. Quadratic Equations eLearn.Punjab 1. Quadratic Equations eLearn.Punjab
6. Conic Sections eLearn.Punjab 6. Conic Sections eLearn.Punjab

2. Write an equation of the parabola with given elements.


Another application of the parabola is in a (i) Focus (-3, 1) ; directrix x = 3
Suspension bridge. The main cables are of parabolic shape. (ii) Focus (2, 5) ; directrix y = 1
The total weight of the bridge is uniformly distributed (iii) Focus (-3, 1) ; directrix x - 2y - 3 = 0
along its length if the shape of the cables is parabolic. (iv) Focus (1, 2) ; vertex (3, 2)
Cables in any other shape will not carry the weight evenly. (v) Focus (-1, 0) ; vertex (-1, 2)
(vi) Directrix x = -2 ; Focus (2, 2)
(vii) Directrix y = 3 ; vertex (2, 2)
Example 6. A suspension bridge with weight uniformly distributed along the length has (viii) Directrix y = 1, length of latusrectum is 8. Opens downward.
two towers of 100 m height above the road surface and are 400 m apart. The cables are (ix) Axis y = 0, through (2, 1) and (11, -2)
parabolic in shape and are tangent to road surface at the centre of the bridge. Find the (x) Axis parallal to y-axis, the points (0, 3), (3, 4) and (4, 11) lie on the graph.
height of the cables at a point 100 m from the centre.
3. Find an equation of the parabola having its focus at the origon and directrix, parallel
to the (i) x-axis (ii) y-axis.
Solution. The parabola formed by the P cables
has A(0, 0) as vertex and focus on the y-axis. 4. Show that an equation of the parabola with focus at (acosa, asina) and directrix
An equation of this parabola is x2 = 4ay. x cos a + ysina + a = 0 is
The point Q(200,100) lies on the parabola and (xsina - ycosa)2 = 4a(xcosa + ysina)
so
5. Show that the ordinate at any point P of the parabola is a mean proportional
(200)2 = 4a x 100
between the length of the latus rectum and the abscissa of P.
or a = 100
Thus an equation of the parabola is 6. A comet has a parabolic orbit with the earth at the focus. When the comet is 150,000
x2 = 400y. (1) km from the earth, the line joining the comet and the earth makes an angle of 300
To ind the height of the cables when x = 100, we have from (1) with the axis of the parabola. How close will the comet come to the earth?
(100)2 = 400y
7. Find an equation of the parabola formed by the cables of a suspension bridge
or y = 25
whose span is a m and the vertical height of the supporting towers is b m.
Thus required height = 25 m
8. A parabolic arch has a 100 m base and height 25 m. Find the height of the arch at
EXERCISE 6.4 the point 30 m from the centre of the base.

9. Show that tangent at any point P of a parabola makes equal angles with the line
1. Find the focus, vertex and directrix of the parabola. Sketch its graph.
(ii) x2 = -16y
PF and the line through P parallel to the axis of the parabola, F being focus.
(i) y2 = 8x (iii) x2 = 5y
(iv) y2 = -12x (v) x2 = 4 (y - 1) (vi) y2 = -8(x - 3)
(These angles are called respectively angle of incidence and angle of relection).

(vii) (x - 1)2 = 8(y + 2) (viii) y = 6x2 - 1


(ix) x + 8 - y2 + 2y = 0 (x) x2 - 4x - 8y + 4 = 0
version: 1.1 version: 1.1

34 35
1. Quadratic Equations eLearn.Punjab 1. Quadratic Equations eLearn.Punjab
6. Conic Sections eLearn.Punjab 6. Conic Sections eLearn.Punjab

6.5 ELLIPSE AND ITS ELEMENTS


x (1 - e ) + y = a (1 - e ). where =
2 2 2 2 2 c
or a
e

+ 2 =
We have already stated that a conic section is an ellipse if e < 1. x
2
y
2

a (1 - e )
Let 0 < e < 1 and F be a ixed point and L be a ixed line not containing F. Let P(x, y) be or 2 2
1 (1)
a
a point in the plane and PM be the perpendicular distance of P from L. If we write b2 = a2 (1 - e2), then (1) takes the form
The set of all points P such that
+ 2 =
2 2
x y
=e
1 (2)
PF a
2
b
PM
which is an equation of the ellipse in the standard form.
is called an ellipse.
Moreover, eccertricity of the ellipse is e = .
The number e is eccentricity of the ellipse, F a focus and L a directrix. c

We have b2 = a2 (1 - e2)
a
6.5.1 Standard Form of an Ellipse (i) From the relation b2 = a2 (1 - e2), we note that b < a

= a , the focus F has coordinates (-ae, 0) and equation of the


-c
c
Let F(-c, 0) be the focus and line x =
(ii) Since we set

-a
be the directix of an ellipse with eccentricity e, e
directrix is x =
2
e
.
(0 < e < 1). Let P(x, y) be any point on the ellipse and suppose that PM is the perpendicular e
distance of P from the directrix. Then
If we take the point (ae, 0) as focus and the line x =
a
(iii) as directrix, it can be
PM = x +
c e
e
2 seen easily that we again obtain equation (2). Thus the ellipse (2) has two foci

The condition PF = e PM takes the analytic form


(-ae, 0) and (ae, 0) and two directrices x = ± .
a
e

(iv) The point (acosq, bsinq) lies on (2) for all real q. x = acosq, y = bsinq are
 c
( x + c) + y = e  x + 2 
2 called parametric equations of the ellipse (2).
 e 
2 2 2
(v) If in (2), b = a then it becomes
x 2 + y2 = a2
which is a circle. In this case b2 = a2(1 - e2) = a2 and so e = 0. Thus circle is a special case
of an ellipse with eccenctricty 0 and foci tending to the centre.
Deinitions: Let F ’ and F be two foci of the ellipse

x + 2cx + c + y = e x + 2cx + 2 - e +) y= - e )
2 2
c c
+ 2 =
2 2 2 2 2 2 2 2 2
or or x (1 2
(1 x
2
y
2
e e 2
1 (1)
a b
version: 1.1 version: 1.1

36 37
1. Quadratic Equations eLearn.Punjab 1. Quadratic Equations eLearn.Punjab
6. Conic Sections eLearn.Punjab 6. Conic Sections eLearn.Punjab

6.5.2 Graph of an Ellipse

Let an equation of the ellipse be

+ =
x2 y 2
1
a 2 b2
Since only even powers of both x and y occur in (1), the curve is symmetric with respect
to both the axes.
From (1), we note that

≤ 1 and 2 ≤ 1
x2 y2
a2 b
x ≤ a and y 2 ≤ b 2
(i) The midpoint C of FF ’ is called the centre of the ellipse. In case of (1) coordinates of
2 2
i.e.,
-a ≤ x ≤ a -b≤ y ≤b
C are (0,0).
(ii) The intersection of (1) with the line joining the foci are obtained by setting y = 0 or and
into (1). These are the points A’(-a, 0) and A(a, 0). The points A and A’ are called Thus all points of the ellipse lie on or within the rectangle (2). The curve meets the
vertices of the ellipse. x-axis at A(-a, 0) and A’ (a, 0) and it meets the y-axis at B(0,-b), B’ (0, b). The graph of the ellipse
(iii) The line segment AA’ = 2a is called the major axis of the ellipse. The line through can easily be drawn as shown in the following igure.
the centre of (1) and perpendicular to themajor axis has its equation as x = 0. It
meets (1) at points B’ (0, b) and B (0,-b). The line segment BB’=2b is called the
minor axis of the ellipse and B’, B are some-times called thecovertices of the
ellipse. Since b2 = a2(1 - e2) and e < 1, the length of the major axis is greater than
the length of the minor axis. (See igure)
(iv) Foci of an ellipse always lie on the major axis.
(v) Each of the focal chords LFL‘ and NF'N' perpendicular to the major axis of an
ellipse is called a latusrectum of the ellipse. Thus there are two laterarecta of an
2
2b
ellipse. It is an easy exercise to ind that length of each latusrectum is
a
{See problem 5}.

(vi) If the foci lie on the y-axis with coordinates (0,-ae) and (0,ae), then equation of the The graph of the ellipse
ellipse is
+ = 1, a > b
x2 y 2
+ 2 = 1. a > b.
2 2
x y
2 b2 a 2
b a
can be sketched as in the case of (1). Its shape is shown in above igure (ii).
The reader is urged to derive this equation.
version: 1.1 version: 1.1

38 39
1. Quadratic Equations eLearn.Punjab 1. Quadratic Equations eLearn.Punjab
6. Conic Sections eLearn.Punjab 6. Conic Sections eLearn.Punjab

Summary of standard Ellipses Example 1. Find an equation of the ellipse having centre at (0,0), focus at (0,-3) and one
vertex at (0,4). Sketch its graph.
+ = 1, a > b + = 1, a > b
x2 y 2 x2 y 2
Solution. The second vertex has coordinates (0, -4).
Equation
a 2 b2 b2 a 2
c2 = a2 - b2 c2 = a2 - b2 Length of the semi-major axis is
Foci (±c, 0) (0, ±c) a=4
x= ± y= ±
c c Also c=3
From b2 = a2 - c2,
Directrices
e2 e2 we have
Major axis y=0 x=0 b2 = 16 - 9 = 7
Vertices (±a, 0) (0, ±a) b = 7 which is length of
Convertices (0, ±b) (±b, 0) the semi-minor axis.
Centre (0, 0) (0, 0) Since the foci lie on the y-axis;
e= <1 e= <1
c c equation of the ellipse is
Eccentricity
a a
+ =
y 2 x2
1
16 7
The graph is as shown above.

Example 2. Analyze the equation


4x2 + 9y2 = 36
and sketch its graph.
Graph
Solution: The given equation may be written as

+ =
x2 y 2
1
9 4
which is standard form of an ellipse.
Semi-major axis a = 3
Semi-minor axis b = 2
From b2 = a2 - c2 , we have
Note: In each ellipse

c2 = b2 - a2 = 9 - 4 = 5
Length of major axis = 2a, Length of minor axis = 2b

2b 2 or c= ± 5
, Foci lie on the major axis
F (- 5,0), F ′( 5,0); A(-3,0), A′(3,0)
Length of Latusrectum =
a Foci: Vertices:

version: 1.1 version: 1.1

40 41
1. Quadratic Equations eLearn.Punjab 1. Quadratic Equations eLearn.Punjab
6. Conic Sections eLearn.Punjab 6. Conic Sections eLearn.Punjab

B(0, -2), B′(0,2) ; =


c 5 Vertices of (2) are
X =0, Y =±3 =y =-
1 3±
Covertices: Eccentricity = .
a 3 i.e., x 1,

=
or (1,-4) and (1,2)
Directrices: x =
± 2= ± =
±
c 5 9 2b 2 4
; Length of latusrectum = are the vertices of (1).
e 5 5 a 3
9 Covertices of (2) are
The graph is as shown above. X = ± 2, Y = 0
i.e., x - 1 = ±2, y + 1 = 0
Example 3. Show that the equation (-1, -1) (3, -1)
9x2 - 18x + 4y2 + 8y - 23 = 0
or and
(1) are the covertices of (1).
represents an ellipse. Find its elements and sketch its graph. The graph of (1) is as shown.

Solution: We complete the squares in (1) and it becomes


(9x2 - 18x + 9) + (4y2 + 8y + 4) - 36 = 0
Example 4. An arch in the form of half an ellipse is 40 m wide and 15 m high at the

9(x - 1)2 + 4(y + l)2 = 36


centre. Find the height of the arch at a distance of 10 m from its centre.
or

( x - 1) 2 ( y + 1) 2
Solution: Let the x-axis be along the base of the arch and the y-axis pass through its centre.
or + =
1 (2) An equation of the ellipse representing the arch is
4 9
If we set x - 1 = X, y + 1 = Y into (2), it becomes x2
+
y2
=
1 (1)
202 152
+ =
X2 Y2
1 (3) Let the height of an arch at a distance of 10 m from the centre be y. Then the points
22 32
which is an ellipse with major axis along X = 0 i.e., along the line, x - l = 0
(10, y) lies on (1)
For x = 10, we have
(i.e. a line parallel to the y-axis)
1  3
Semi-major axis = 3, Semi-minor axis = 2
= - =  ,
2
y2
4  2 
1
c = 9 - 4 = 5 , Eccentricity =
152

so that y =
. 15 3
Centre of (2) is X = 0, Y = 0
x - 1, y = -1 i.e., (1, -1) is centre of (1)
2
or 15 3
The foci of (2) are Required height = m.
2
X = 0, Y = ± 5
i.e., x - 1 =0, y + 1 =± 5
i.e., (1, -1 + 5) and (1, - 1 - 5) are foci of (1).

version: 1.1 version: 1.1

42 43
1. Quadratic Equations eLearn.Punjab 1. Quadratic Equations eLearn.Punjab
6. Conic Sections eLearn.Punjab 6. Conic Sections eLearn.Punjab

EXERCISE 6.5 6. The major axis of an ellipse in standard form lies along the x-axis and has length
4 2 . The distance between the foci equals the length of the minor axis. Write an
1. Find an equation of the ellipse with given data and sketch its graph: equation of the ellipse.
(i) Foci (±3,0) and minor axis of length 10 7. An astroid has elliptic orbit with the sun at one focus. Its distance from the sun
(ii) Foci (0,-1) and (0,-5) and major axis of length 6. ranges from 17 million miles to 183 million miles. Write an equation of the orbit
(iii) Foci (-3 3,0) and vertices (±6,0) of the astroid.
(iv) Vertices (-1,1), (5,1); foci (4,1) and (0,1) 8. An arch in the shape of a semi-ellipse is 90m wide at the base and 30m high at the
centre. At what distance from the centre is the arch 20 2 m high?
3 
Foci (± 5,0) and passing through the point  , 3 
2 
(v) 9. The moon orbits the earth in an elliptic path with earth at one focus. The major and
minor axes of the orbit are 768,806 km and 767,746 km respectively. Find the
3 greatest and least distances (in Astronomy called the apogee and perigee
(vi) Vertices (0, ±5), eccentricity .
5
(vii) Centre (0,0), focus (0, -3), vertex (0,4)
respectively) of the moon from the earth.

(viii) Centre (2, 2), major axis parallel to y-axis and of length 8 units, minor axis
6.6 HYPERBOLA AND ITS ELEMENTS
parallel to x-axis and of length 6 units.
(ix) Centre (0, 0), symmetric with respect to both the axes and passing through
We have already stated that a conic section is a hyperbola if e > 1. Let e > 1 and F be
the points (2, 3) and (6, 1).
a ixed point and L be a line not containing F. Also let P(x, y) be a point in the plane and
(x) Centre (0, 0), major axis horizontal, the points (3, 1), (4, 0) lie on the graph.
PM be the perpendicular distance of P from L.
2. Find the centre, foci, eccentricity, vertices and directrices of the
The set of all points P(x, y) such that
ellipse, whose equation is given:

= e >1
(i) x2 + 4y2 = 16 (ii) 9x2 + y2 = 18 PF
(2 x - 1) ( y + 2)
(1)
+ =
2 2 PM
(iii) 25x2 + 9y2 = 225 (iv) 1

(v) x2 + 16x + 4y2 - 16y + 76 = 0


4 16 is called a hyperbola.

(vi) 25x2 + 4y2 - 250x - 16y + 541 = 0


F and L are respectively focus and directrix of the hyperbola e is the eccentricity.

3. Let a be a positive number and 0 < c < a. Let F(-c, 0) and F ’(c, 0) be two given points. 6.6.1 Standard Equation of Hyperbola
Prove that the locus of points P(x, y) such that
PF + PF ′ = Let F(c, 0) be the focus with c > 0 and x =
c
2a , is an ellipse. be the directrix of the hyperbola.
e2
4. Use problem 3 to ind equation of the ellipse as locus of points P(x, y) such that the
Also let P(x, y) be a point on the hyperbola, then by deinition
sum of the distances from P to the points (0, 0) and (1, 1) is 2.

=e
5. Prove that the lactusrectum of the ellipse. PF

+ =
PM
x2 y 2 2b 2
1 is
a 2 b2 a
version: 1.1 version: 1.1

44 45
1. Quadratic Equations eLearn.Punjab 1. Quadratic Equations eLearn.Punjab
6. Conic Sections eLearn.Punjab 6. Conic Sections eLearn.Punjab

meets the x-axis at points with y = 0 and x = ±a. The points A(-a, 0 and A’(a, 0) are called
 c
i.e. (x - c) + y = e  x - 2  x-2 2cx
+ c+2 y= 2 e2 x-2 2cx
+
2
c2 vertices of the hyperbola. The line segament AA’ = 2a is called the transverse (or focal)
 e 
2 2 2
or
e2 axis of the hyperbola (3). The equation (3) does not meet the y-axis in real points. However
 1  c2 2 the line segment joining the points B(0, -b) and B’(0, b) is called the conjugate axis of
x (e - 1) - y = c 1 - 2 = 2 (e - 1)
 e  e
2 2 2 2
or (2) the hyperbola. The midpoint (0,0) of AA’ is called the centre of the hyperbola.
In case of hyperbola (3), we have
Let us set a = , so that (2) becomes
c
b2 = a2(e2 - 1) = c2 - a2. The eccentricity e=
>1
c
e
a
x (e - 1) - y =
- a (e - 1) 0 =
- or
x2 y2
a 2 (e 2 - 1)
2 2 2 2 2
1 so that, unlike the ellipse, we may have b > a or b < a or b = a
a2
The point (a sec q, b tan q) lies on the hyperbola 2 - 2 = 1 for all real values of q.
x2 y 2
- =
x2 y 2 (ii)
The equations x = a sec q, y = b tan q are called parametric equations of the hyperbola.
or 1 (3) a b
a 2 b2
where b2 = a2(e2 - 1) = c2 - a2 a c = ae
Since y =
± x - a 2 , when | x | ,so that x 2 a-2 →,
(3) is standard equation of the hyperbola. b 2
(iii) x 2 we have
a
It is clear that the curve is symmetric with respect to both the axes.
y±= x - =
If we take the point (-c, 0) as focus b x2 y 2
i.e. 0 (2)
-c
a a 2 b2
and the line x = as directrix, then it The lines (2) do not meet the curve but distance of any point on the curve from any of
e2
is easy to see that the set of all points the two lines approaches zero. Such lines are called asymptotes of a curve. Joint equation
P(x, y) such that of the asymptotes of (3) is obtained by writing 0 instead of 1 on the right hand side of the
PF = e PM standard form (3). Asymptotes are very helpful in graphing a hyperbola.
is hyperbola with (3) as its equation. The ellipse and hyperbola are called central conics because each has a centre of
Thus a hyperbola has two foci and two symmetry.
directrices.
If the foci lie on the y-axis, then roles of x and y are interchanged in (3) and the equation 6.6.2 Graph of the hyperbola

- =
of the hyperbola becomes x2 y 2
1 (1)

- =
y 2 x2 a 2 b2
1.
a 2 b2 The curve is symmetric with respect to both the axes. We rewrite (1) as

= - y a2 )=
Deinition: The hyperbola y 2 x2 2 b2 2
1 or (x
b2 a 2 a2
- = or ± y = - x a
x2 y 2 b 2 2
1 (1) (2)
a 2 b2 a
version: 1.1 version: 1.1

46 47
1. Quadratic Equations eLearn.Punjab 1. Quadratic Equations eLearn.Punjab
6. Conic Sections eLearn.Punjab 6. Conic Sections eLearn.Punjab

If x < a , then y is imaginary so that no portion of the curve lies between -a < x < a. For Centre (0, 0) (0, 0)

x ≥ a, = x - a2 ≤ x
b 2 b
y
a a
so that points on the curve lie below the corresponding points on the line y = x in
b Graph
a
irst quadrant.

-b
y-= -x 2 a≥2 x if ≥x a
b

-b
a a
and in this case the points on the curve lie above the line y =
Example 1. Find an equation of the hyperbola whose foci are (±4, 0) and vertices (±2, 0).
x in fourth quadrant.
a Sketch its graph.
If x 7 a, then by similar arguments,

= x - a 2 lies below the corresponding point


b 2 Solution: The centre of the hyperbola is the origin
y
and the transverse axis is along the x-axis. Here
-b
a
on y = x in second quadrant. c = 4 and a = 2 so that b2 = c2 - a2 = 16 - 4 = 12.

-b 2
a Therefore, the equation is
= x - a 2 , then points on the curve lie
- =
If y x2 y 2
a 1.
above the correspondent point on y = x in
b 4 12
a The graph of the curve is as shown.
third quadrant. Thus there are two branches of
the curve. Moreover, from (2) we see that as x → ∞, y → ∞ so that the two branches extend Example 2. Discuss and sketch the graph of the equation
to ininity 25x2 - 16y2 = 400 (1)
Solution: The given equation is

= -= 1 - or
Summary of Standard Hyperbolas x2 y 2 x2 y2
- = - =
1
x2 y 2 y 2 x2 16 25 42 52
Equation 1 1
a 2 b2 a 2 b2 which is an equation of the hyperbola with
Foci (±c, 0) (0, ±c) transverse axis along the x-axis.

x= ± y= ±
c c Here a = 4, b = 5
b2 = c2 - a2 , we have
Directrices
e2 e2 From
Transverse axis y=0 x=0 c 2 = 34 or c = ± 34
Vertices (±a, 0) (0, ±a) Foci of the hyperbola are: (± 34,0)
e= >1 e= >1
c c Vertices: (±4, 0)
Eccentricity
a a Ends of the conjugate axes are the points (0, ±5)
version: 1.1 version: 1.1

48 49
1. Quadratic Equations eLearn.Punjab 1. Quadratic Equations eLearn.Punjab
6. Conic Sections eLearn.Punjab 6. Conic Sections eLearn.Punjab

We write x - 1 = X, y + 1 = Y in (2), to have

Eccentricity: e= =
- 2 =
c 34
X2 Y2
a 4 1 (3)
12
so that it is a hyperbola with centre at X = 0, Y = 0 i.e., the centre of (1) is (1, -1).
2
± y± = =
b 5
The curve is below the lines x x x
a 4 The transverse axis of (3) is Y = 0 i.e., y + 1 = 0 is the transverse axis of (1). Vertices of
which are its asymptotes. The sketch of the curve is as shown. (3) are: X = ±1, y = 0
i.e. x - 1 = ±1, y + 1 = 0 or (0, -1) and (2, -1)
Example 3. Find the eccentricity, the coordinates of the vertices and foci of the Here a = 1 and b = 2 so that, we have c = a 2 + b2 = 5
asymptotes of the hyperbola

- = Eccentricity e= =
y 2 x2 c
1 (1) 5
16 49 a
±X ==
Also sketch its graph.
Foci of (3) are: 5 ,Y 0

Solution. The transverse axis of (1) lies along the y-axis. Coordinates of the vertices are i.e., x=
1 5 and ±y 1 =-
(0,±4). i.e., (1 + 5, -1) and (1 - 5, -1)
Here a = 4, b = 7 so that from c2 = a2 + b2, we get are foci of (1).
c2 = 16 + 49 or c = 65
Foci are: (0, ± 65)
Ends of the conjugate axis are (±7, 0)
Equations of the directrices of (3) are: X =
± 2=± =
±
c 5 1

Eccentricity= =
c 65 e 5 5

x -1 = ± x =1+ x =1-
a 4 1 1 1
x = ±7, y = ±4 or or and
5 5 5
The graph of the curve is as shown. The sketch of the curve is as shown.

Example 4. Discuss and sketch the graph of the equation EXERCISE 6.6
4x2 - 8x - y2 - 2y - 1 = 0 (1)
1. Find an equation of the hyperbola with the given data. Sketch the graph of each.
Solution: Completing the squares in x and y in the given equation, we have (i) Centre (0, 0), focus (6, 0), vertex (4, 0)
4(x2 - 2x +1) - (y2 + 2y +1) = 4 (ii) Foci (±5, 0), vertex (3, 0)
4(x - 1)2 - (y + 1)2 = 4
Foci (2 ± 5 2, -7) , length of the transverse axis 10.
or
(iii)
( x - 1) 2 (y + 1) 2
- =
(iv) Foci (0, ±6), e = 2.
or 1 (2)
12 22 (v) Foci (0, ±9), directrices y = ±4
version: 1.1 version: 1.1

50 51
1. Quadratic Equations eLearn.Punjab 1. Quadratic Equations eLearn.Punjab
6. Conic Sections eLearn.Punjab 6. Conic Sections eLearn.Punjab

(vi) Centre (2, 2), horizontal transverse axis of length 6 and eccentricity e = 2 conic at some point on the conic, we shall irst ind the slope of the tangent at the given
(vii) Vertices (2, ±3), (0, 5) lies on the curve.
(viii) Foci (5, -2), (5,4) and one vertex (5, 3)
dy
point by calculating from the equation of the conic at that point and then using the

point - slope form of a line, it will be quite simple to write an equation of the tangent.
dx
2. Find the centre, foci, eccentricity, vertices and equations of directrices of each of
the following: Since the normal to a curve at a point on the curve is perpendicular to the tangent through

x -y =9 - =
x2 y 2 the point of tangency, its equation can be easily written.
2 2
(i) (ii) 1
4 9

- = - x2 =
y 2 x2 Example 1. Find equations of the tangent and normal to
y2
(iii) 1 (iv) 1 (i) y2 = 4ax (1)
16 9 4
( x - 1) 2 ( y - 1) 2 ( y + 2) 2 ( x - 2) 2
(v) - = 1 (vi) - = 1 (ii) +
x2 y 2
=1 (2)
2 9 9 16
9 x 2 - 12 x - y 2 - 2 y + 2 = (viii) 4 y 2 + 12 y - x 2 + 4 x + 1 =
a 2 b2

- =
(vii) 0 0 x2 y 2
(ix) x 2 - y 2 + 8 x - 2 y - 10 = (x) 9 x 2 - y 2 - 36 x - 6 y + 18 =
(iii) 1 (3)
0 0 a 2 b2
3. Let 0 < a < c and F ’ (-c, 0), F(c, 0) be two ixed points. Show that the set of points at the point (x1, y1).
P(x, y) such that

±′ =
PF - PF - =
x2 y2 Solution: (i). Diferentiating (1) w.r.t. x, we get
a2 c2 - a2
2a, is the hyperbola 1
=
=
dy dy
(F, F ’ are foci of the hyperbola) 2y 4a or 2a
Using Problem 3, ind an equation of the hyperbola with foci (-5, -5) and (5, 5),
dx dx
dy 
=
4.
vertices (-3 2, -3 2) and (3 2,3 2) . 
2a
dx  ( x1 , y1 ) y1
Slope of the tangent at (x1, y1)
5. For any point on a hyperbola the diference of its distances from the points (2, 2) and
(10, 2) is 6. Find an equation of the hyperbola. Equation of the tangent to (1) at (x1, y1) is
Two listening posts hear the sound of an enemy gun. The diference in time is one
y - y1 = ( x - x1 ) or yy1 - y12 = 2ax - 2ax1 or yy1 - 2ax = y12 - 2ax1
6. 2a
second. If the listening posts are 1400 feet apart, write an equation of the hyperbola y1
Adding -2ax, to both sides of the above equation, we obtain
passing through the position of the enemy gum. (Sound travels at 1080 ft/sec).

yy1 = 2a ( x + x1 ) = y12 - 4ax1


= on y 2 4ax, so y=
1 - 4ax1
6.7 TANGENTS AND NORMALS 2
Since ( x1 , y1 ) lies 0
Thus equation of the required tangent is
We have already seen in the geometrical interpretation of the derivative
yy1 = 2a(x + x1).

- y1
dy
Slope of the normal =
of a curve y = f(x) or f(x, y) = 0 that represents the slope of the tangent line to
dx (negative reciprocal of slope of the tangent)
the curve at the point (x, y). In order to ind an equation of the tangent to a given 2a

version: 1.1 version: 1.1

52 53
1. Quadratic Equations eLearn.Punjab 1. Quadratic Equations eLearn.Punjab
6. Conic Sections eLearn.Punjab 6. Conic Sections eLearn.Punjab

Equation of the normal is

- y1 + 2 =1 and + =a 2 + b 2 , respectively.
a 2 x b2 y
y -= ( x - x1 )
xx1 yy1
y1 a 2
b x1 y1
2a

+ =
x2 y 2 Remarks
(ii) 1
a 2 b2 An equation of the tangent at the point (x1, y1) of any conic can be written by making
Diferentiating the above equation, w.r.t. x, we have
replacements in the equation of the conic as under:

+ =
0 - or =
2 x 2 y dy dy b2 x Replace x2 by xx1
a 2 b 2 dx dx a2 y y2 by yy1
dy  -b 2 x1
= ( x + x1 )
dx  ( x1 , y1 ) a 2 y1
or 1
x by
2
( y + y1 )
Equation of the tangent to (2), at (x1, y1) is 1
y by
-b 2 x1
2
y=
- y1 ( x - x1 )
a 2 y1
yy1 y12 - xx1 x12
Example 1. Write equations of the tangent and normal to the parabola x2 = 16y at the
- = 2 + 2 + = +
xx1 yy1 x 2 y12 point whose abscissa is 8.
or or
b2 b2 a a a 2 b2 a 2 a 2

Since (x1 , y1 ) lie on (2) so, 2 + 2 =


x12 y12 Solution: Since x = 8 lies on the parabola, substituting this value of x into the given equation,
1
a b we ind
+ =
xx1 yy1 64 = 16y or y=4
Hence an equation of the tangent to (2) at (x1 , y1 ) is 1
a 2 b2 Thus we have to ind equations of tangent and normal at (8, 4).
a 2 y1 Slope of the tangent to the parabola at (8, 4) is 1. An equation of the tangent the
Slope of the normal at (x1 , y1 ) is 2 .
b x1 parabola at (8, 4) is
Equation of the normal at (x1, y1) is y-4=x-8
or x-y-4=0
y -=
y1
a 2 y1
( x - x1 ) Slope of the normal at (8, 4) is -1. Therefore, equation of the normal at the given
b 2 x1
b 2 x1 y - b 2 x1 y1 = a 2 y1 x - a 2 x1 y1 or a 2 y1 x - b 2 x1 y = x1 y1 (a 2 - b 2 )
point is
or y - 4 = -(x - 8)
Dividing both sides of the above equation by x1 y1, we get or x + y - 12 = 0

- =a 2 - b 2 , as an equation of the normal. Write equations of the tangent and normal to the conic x + y =
a 2 x b2 y 2 2
Example 2. 1 at the
x1 y1 8 9
(iii) Proceeding as in (ii), it is easy to see that equations of the tangent and normal point  8 ,1 .
to (3) at (x1, y1) are 3 
version: 1.1 version: 1.1

54 55
1. Quadratic Equations eLearn.Punjab 1. Quadratic Equations eLearn.Punjab
6. Conic Sections eLearn.Punjab 6. Conic Sections eLearn.Punjab

Solution: The given equation is ordinates of the points of intersection. Thus the line (1) cuts the parabola (2) in two points.
9x2 +8y2 - 72 = 0 (1) In order that (1) is a tangent to (2), the points of intersection of a line and the parabola must
Diferentiating (1) w.r.t. x, we have be conicident. In this case, the roots of (3) should be real and equal.

8 
This means that the discriminant of (3) is zero. Thus
This is slope of the tangent to (1) at  ,1 . 4(mc - 2a)2 - 4m2c2 = 0 i.e., -4mca + 4a2 = 0
3 

c=
Equation of the tangent at this point is a
or , is. the required condition for (1) to be a tangent to (2). Hence
 8
y - 1 =-3 . x -  =-3 x + 8 or 3x + y - 9 =0 .
m

 3 =
y mx + , is a tangent to y2 = 4ax for all nonzero values of m.
a
m

8 
(ii) To determine the points of intersection of
The normal at  ,1 has the slope .
1
3 
y = mx + c (1)
3
+ =
x2 y 2
Equation of the normal is and 1 (2)
a 2 b2
1 8
y -1 =  x -  or 3y - 3 = x - or 3 x - 9 y + 1 = 0
8 we solve (1) and (2) simultaneously. Putting the value of y from
3 3 3 (1) into (2), we have

x 2 (mx + c) 2
+ =
1
Theorem: To show that a straight line cuts a conic, in general, in two points and to ind the a2 b2
condition that the line be a tangent to the conic. or (a2m2 + b2)x2 + 2mca2 x + a2c2 - a2b2 = 0 (3)
Let a line y = mx + c cut the conics which is a quardratic in x and it gives the abscissas of the two points where (1) and (2)
intersect. The corresponding values of y are obtained by setting the values of x
+ = - =
x2 y 2 x2 y 2 obtained from (3) into (1). Thus (1) and (2) intersect in two points. Now (1) is a
(i) y = 4ax (ii) 1 (iii) 1
a 2 b2 a 2 b2 tangent to (2) if the point of intersection is a single point.
This requires (3) to have equal roots. Hence (1) is a tangent to (2) if
(2mca2)2 - 4(a2m2 + b2)(a2c2 - a2b2) = 0
We shall discuss each case separately.

m2c2a2 - (a2m2 + b2)(c2 - b2) = 0


(i) The points of intersection of
i.e.,
m2c2a2 - a2m2c2 + a2m2b2 - b2c2 + b4 = 0
y = mx + c (1)
and y2 = 4ax (2) or
are obtained by solving (1) and (2) simultaneously for x and y. Inserting the value of or c2 = a2m2 + b2
y from (1) into (2), we get or ± c= +a 2 m 2 b 2
(mx + c)2 = 4ax Putting the value of c into (1), we have
m2x2 + (2mc - 4a)x + c2 = 0
y=
±mx a 2+m 2
or (3)
b2
which being a quadratic in x gives two values of x. These values are the x coordinates of
the common points of (1) and (2). Setting these values in (1), we obtain the corresponding which are tangents to (2) for all non-zero values of m.
version: 1.1 version: 1.1

56 57
1. Quadratic Equations eLearn.Punjab 1. Quadratic Equations eLearn.Punjab
6. Conic Sections eLearn.Punjab 6. Conic Sections eLearn.Punjab

(iii) We replace b2 by -b2 in (ii) and the line y = mx + c is a tangent to Example 5. Find equations of the tangents to the ellipse

- 2 =
1 if± c =-a 2 m 2 b 2 + =
x2 y 2 x2 y 2
2
1 (1)
a b 128 18
which are parallel to the line 3x + 8y + 1 = 0. Also ind the points of contact.
Thus y =±mx a -m are tangents to the hyperbola: 2 - 2 =
x2 y 2
-3
2 2 2
b 1 for all non-zero
a b Solution: The slope of the required tangents is . Equations of the tangents are
values of m.
8

Find an equation of the tangent to the parabola y2 = -6x which is parallel -3  3 -3


y= x ± 128. -  + 18 = x±6
2
Example 4.
to the line 2x + y + 1 = 0. Also ind the point of tangency. 8  8 8
Thus the two tangents are
Solution: Slope of the required tangent is m = -2 3x + 8y + 48 = 0 (2)
In the parabola y2 = -6x (1) and 3x + 8y - 48 = 0 (3)

-6 -3
We solve (1) and (3) simultaneously to ind the point of contact. Inserting the value of
=
a= y from (3) into (1), we get
4 2
 -3 
x + 6 x + 36 - x
Equation of the tangent is

2

+  = 1 or
9 2 9

y=
mx + = -2 x + + 64 2 = 1
2 2
a 3 x 8 x
128 18 128 18
i.e., 8x + 4y - 3 = 0
m 4
+= +2- = 1 - or+
(2) x2 x2 x x2 x
or 1 0
-8 x + 3
Inserting the value of y from (2) viz y =
128 128 4 64 4
x  -3
 =- 1 0 i.e.,
= x 8 = + so
into (1), we have 2

8 
4 or and x 6 3

 -8 x + 3 
8

  = -6 x
2
Thus (8, 3) is the point of tangency of (3).
 4  It can be seen in a similar manner that point of contact of (2) is (-8, -3).
or 64x - 48x + 9 = -96x
2
or 64x2 +48x + 9 = 0

-3
Example 6. Show that the product of the distances from the foci to any tangent to the
or (8x + 3)2 = 0 i.e., x= hyperbola
8
Putting this value of x into (2), we get
- =
x2 y 2
 -3 
-8   + 3
1 (1)
a 2 b2
=
=y  8  3 is constant.
4 2
 -3 3 
The point of tangency is  ,  .
Solution: The line
 8 2 y=
+mx a 2-m 2 b 2 (2)
version: 1.1 version: 1.1

58 59
1. Quadratic Equations eLearn.Punjab 1. Quadratic Equations eLearn.Punjab
6. Conic Sections eLearn.Punjab 6. Conic Sections eLearn.Punjab

is a tangent to (1). Example 7. Find the points of intersection of the ellipse


Foci of (1) are F(-c, 0) and F ’(c, 0).
=
= + 1 - (1)
Distance of F(-c, 0) from (2) is x2 y2 x2 y2
and the hyperbola 1 (2)
43 43 7 14
-cm + a 2 m 2 - b 2
3 4
d1 =
Also sketch the graph of the two conics.
1 + m2
Solution: The two equations may be written as
(1) and 2x2 - y2 =14
Distance of F ‘(c, 0) from (2) is
3x2 + 4y2 = 43 (2)
cm + a 2 m 2 - b 2 Multiplying (2) by 4 and adding the result to (1), we get
d2 =
1 + m2
11x2 = 99 or x = ±3
Setting x = 3 in to (2), we have 18 - y2 = 14 or y = ±2
a 2m2 - b2 - c 2m2 a 2m2 - c 2 + a 2 - c 2m2 Thus (3, 2) and (3, -2) are two points of intersection
d1 × d 2 = = as b 2 = c 2 - a 2
1+ m 2
1+ m 2
of the two conics
a2 - c2 Putting x = -3 into (2), we get
= c2 - a2 y = ±2
Therefore (-3, 2) and (-3, -2) are also points of
since c>a
= c2
which is constant. intersection of (1) and (2). The four points of intersection
are as shown in the igure.
Intersection of Two Coincs
Suppose we are given two conics Example 8. Find the points of intersection of the conics
y = 1 + x2 (1)
-
x2 y 2
=
1 (1) and y = 1 + 4x - x2 (2)
a 2 b2 Also draw the graph of the conics.
and y2 = 4ax (2)
To ind the points common to both (1) and (2), we need to solve (1) and (2) Solution. From (1), we have
simultaneously. It is known from algebra, that the simultaneous solution set of two x±=- y 1
equations of the second degree consists of four points. Thus two conics will always intersect Inserting these values of x into (2), we get
in four points. These points may be all real and distinct, two real and two imaginary or all y = 1 ± 4 y - 1 - ( y - 1)
2 y - 2 =±4 y - 1 (y - 1) 2 =4(y - 1)
imaginary. Two or more points may also coincide. Two conics are said to touch each other if
or or
(y - 1) (y - 1 - 4) = 0
they intersect in two or more coincident points.
or
Therefore, y = 1,5
When y = 1, x = 0
When y = 5, x = ±2
version: 1.1 version: 1.1

60 61
1. Quadratic Equations eLearn.Punjab 1. Quadratic Equations eLearn.Punjab
6. Conic Sections eLearn.Punjab 6. Conic Sections eLearn.Punjab

But (-2,5) does not satisfy (2). 2. Write equation of the tangent to the given conic at the indicated point
Thus (0,1) and (2,5) are the points of (i) 3x2 = -16y at the points whose ordinate is -3.
intersections of (1) and (2). y = 1 + x2 is a parabola (ii) 3x2 - 7y2 = 20 at the points where y = -1.
with vertex at (0,1) and opening upward, (iii) 3x2 - 7y2 + 2x - y - 48 = 0 at the point where x = 4.
y = 1 + 4x - x2 may be written as y - 5 = -(x - 2)2 which 3. Find equations of the tangents to each of the following through the given point:
is a parabola with vertex. (2,5) and opening downward (i) x2 + y2 = 25 through (7 ,-1)
(ii) y2 = 12x through (1, 4)
(iii) x2 - 2y2 = 2 through (1, -2)
Example 9. Find equations of the common tangents to the two conics 4. Find equations of the normals to the parabola y2 = 8x which are parallel to the line
2x + 3y = 10.
= += 1 + and
x2 y 2 x2 y2
+ y2 =
1
16 25 25 9 x2
5. Find equations of the tangents to the ellipse 1 which are parallel to the line
2x - 4y + 5 = 0.
4

Find equations of the tangents to the conic 9x2 - 4y2 = 36 parallel to 5x - 2y + 7 = 0.


Solution. The tangents with slope m, to the two conics are respectively given by
y= ±mx 16+m 2 25 y=
±mx 25+m 2
6.
and 9
7. Find equations of the common tangents to the given conics
For a tangent to be common, we must have
(i) x2 = 80y and x2 + y2 = 81
16m2 + 25 = 25m2 + 9
(ii) y2 =16x and x2 = 2y

m= ±
4 8. Find the points of intersection of the given conics
or 9m = 16 or
3
+ = - =
Using these values of m, equations of the four common tangents are: x2 y 2 x2 y 2
(i) 1 and 1
x2 - y2 = 1
18 8 3 3
y± =± x
4 (ii) x2 + y2 = 8 and
3x2 - 4y2 = 12 3y2 - 2x2 = 7
481
3 (iii) and
(iv) 3x2 + 5y2 = 60 and 9x2 + y2 = 124
EXERCISE 6.7 (v) 4x2 + y2 = 16 and x2 + y2 + y + 8 = 0

1. Find equations of the tangent and normal to each of the following at the indicated
6.8 TRANSLATION AND ROTATION OF AXES
point:
(i) y2 = 4ax at (a t2, 2a t)
Translation of Axes

+ = (a cos q, b sin q)
x2 y 2 In order to facilitate the investigation of properties of a curve
(ii) 1 at with a given equation, it is sometimes necessary to shift the origin
a 2 b2

- = (a sec q, b tan q)
x2 y 2 O(0, 0) to some other point O’ (h, k). The axes O‘X , O’Y drawn through O’ remain parallel to
(iii) 1 at the original axes Ox and Oy. The process is called translation of axes.
a 2 b2

version: 1.1 version: 1.1

62 63
1. Quadratic Equations eLearn.Punjab 1. Quadratic Equations eLearn.Punjab
6. Conic Sections eLearn.Punjab 6. Conic Sections eLearn.Punjab

We have already obtained in Chapter 4 Example 2: By transforming the equation


formulas showing relationships between the x2 + 4y2 - 4x + 8y + 4 = 0 (1)
two sets of coordinates of a point referred to referred to a new origin and axes remaining parallel to the original axes, the irst degree
the two sets of coordinate axes. terms are removed. Find the coordinates of the new origin and the transformed equation.
Recall that if a point P has coordinates (x,
y) referred to the xy-system and has coordinates Solution. Let the coordinates of the new origin be (h, k). Equations of transformation are
(X, Y) referred to the translated axes O’X, O’Y x=X+h , y=Y+k
through O’(h, k) , then Substituting these values of x, y into (1), we get
(X + h)2 + 4(Y + k)2 - 4(X + h) + 8(Y + k) + 4 = 0
or X2 + 4Y2 + X(2h - 4) + Y(8k + 8) + h2 + 4k2 - 4h + 8k + 4 = 0 (2)
(h, k) is to be so chosen that irst degree terms are removed from the transformed
=
x X + h

y= Y + k 
equation.
Therefore, 2h - 4 = 0 and 8k + 8 = 0 giving h = 2 and k = -1. New origin is O‘ (2, -1).
(1)

These are called equations of transformation. Putting h = 2, k = -1 into (2), the transformed equation is X2 + 4Y2 - 4 = 0.
From (1), we have
Rotation of Axes
X= x - h  To ind equations for a rotation of axes about the origin through an angle q(0 < q < 900).

Y= y - k 
(2)
(origin remaining unaltered).
Let the axes be rotated about the origin
through an angle q. The new axes OX, OY are as
(1) and (2) will be used to transform an equation in one system into the other system.
The axes Ox and Oy are referred to as the original (or old) axes and O‘X, O’Y are called
shown in the igure.
the translated axes (or new axes).
Let P be any point in the plane with

Example 1: Transform the equation x2 + 6x - 8y + 17 = 0


coordinates P(x, y) referred to the xy-system and
(1)
P(X, Y) referred to the XY-system. In either system
referred to O‘(-3, 1) as origin, axes remaining parallel to the old axes.
the distance r between P and O is the same.
Draw PM ⊥ Ox and PQ ⊥ OX. Let a be the
Solution. Equations of transformation are
x=X-3
inclination of OP with OX. From the igure, we have

X = OQ = r cos a, Y = QP = r sin a
y=Y+1
(1)
and x = r cos(q + a), y = r sin (q + a)
Substituting these values of x, y into (1), we have
(X - 3)2 + 6(X - 3 ) - 8 (Y + 1) + 17 = 0
X2 - 6X + 9 + 6X - 18 - 8Y - 8 + 17 = 0 = x -r cosq cos a r sin q sin a 

or
or X2 - 8Y = 0 is the required transformed equation. =
or
y r sin q cos a + r cosq sin a 
(2)

version: 1.1 version: 1.1

64 65
1. Quadratic Equations eLearn.Punjab 1. Quadratic Equations eLearn.Punjab
6. Conic Sections eLearn.Punjab 6. Conic Sections eLearn.Punjab

Substituting the values of r cos a, r sin a from (1) into (2), we get Since this equation is to be free from the product term XY, the coeicient of XY
is zero, i.e. -10sinq cosq +2 3(cos 2q - sin 2q )+14sinq cosq = 0
=x X cosq - Y sin q 
2sin 2q +2 3cos 2q = 0
=y X sin q + y cosq 
or

-2 3
as the required equations of transformation for a rotation of axes through an angle q. or tan 2q = = tan 1200 or q = 600
2
Example 3: Find an equation of 5x2 - 6xy + 5y2 - 8 = 0 with Thus axes be rotated through an angle of 600 so that XY term is removed from
respect to new axes obtained by rotation of axes about the origin through an angle of 1350. the transformed equation.
Setting q = 600 into (1), the transformed equation is (after simpliication)
Solution. Here q = 135. Equations of transformation are 8X2 + 4Y2 - 16 = 0 or 2X2 + Y2 - 4 = 0

-X -1
x = X cos1350 - Y sin1350 = - = (X + Y)
Y
EXERCISE 6.8
2 2 2
x = X sin1350 + Y cos1350 = - = (X -Y)
X Y 1
1. Find an equation of each of the following with respect to new parallel axes obtained
2 2 2
Substituting these expressions for x, y into the given equation, we have by shifting the origin to the indicated point:
(i) x2 + 16y - 16 = 0, O’ (0, 1)
 X +Y   X +Y X -Y   X -Y  (ii) 4x2 + y2 + 16x - 10y + 37
5 -  - 6 -  + 5  -8 =
2 2
= 0, O’ (2, 5)
 2   2   2  (iii) 9x2 + 4x2 + 18x - 16y - 11 = 0, O’ (-1, 2)
. 0
2
(iv) x2 - y2 + 4x + 8y - 11 = 0, O’ (-2, 4)
( X + 2 XY + Y 2 ) + 3( X 2 - Y 2 ) + ( X 2 - 2 XY + Y 2 ) - 8 = (v) 9x2 - 4y2 + 36x + 8y - 4
5 2 5
or 0 = 0, O’ (2, 1)

8X2 + 2Y2 - 8 = 0
2 2
2. Find coordinates of the new origin (axes remaining parallel) so that irst degree
or or 4X2 + Y2 = 4
terms are removed from the transformed equation of each of the following. Also ind
is the required transformed equation.
the transformed equation:
(i) 3x2 - 2y2 + 24x + 12y + 24 = 0
(ii) 25x2 + 9y2 + 50x - 36y - 164 = 0
Example 4: Find the angle through which the axes be rotated about the origin so that
the product term XY is removed from the transformed equation of 5x 2 + 2 3 xy + 7x 2 - 16 = 0 .
(iii) x2 - y2 - 6x + 2y + 7 = 0
Also ind the transformed equation.
3. In each of the following, ind an equation referred to the new axes obtained by

Solution. Let the axes be rotated through an angle q. Equations of transformation are
rotation of axes about the origin through the given angle:
q = 450
x = X cos q - Y sin q y = X sin q + Y cos q
(i) xy = 1,
(ii) 7x2 - 8xy + y2 - 9 = 0, q = arctan 2
;
Substituting into the given equation, we get
5(Xcosq - Ysinq ) 2 +2 3(Xcosq - Ysinq )(Xsinq +Ycosq ) 9 x 2 + 12 xy + 4 y 2 - x -= q arctan
y 0, =
2
(iii)
+ 7(X sin q + y cos q)2 - 16 = 0
3
x 2 - 2 xy + y 2 - 2 2 x - 2 2 y + 2 = 0, q = 45°
(1)
(iv)
version: 1.1 version: 1.1

66 67
1. Quadratic Equations eLearn.Punjab 1. Quadratic Equations eLearn.Punjab
6. Conic Sections eLearn.Punjab 6. Conic Sections eLearn.Punjab

4. Find measure of the angle through which the axes be rotated so that the product we can write (1) as
term XY is removed from the transformed equation. Also ind the transformed equation:
2x2 + 6xy + 10y2 - 11 = 0 xy + 4x - 3y - 10 = 0  2 G G2   2 F F 2  G2 F 2
(i) A x + x + ′
 - B  y - ′ y + ′2  = - - C = M (say)
4 B  4 A 4 B′
(ii)
(iii) 5x - 6xy + 5y - 8  4 A2  
or
2 2
=0 A B
 G 
′  F 
A x +  -  -  =
2 2

 2A   2 B′ 
or B y M
6.9 THE GENERAL EQUATION OF
AX 2 - B′Y 2 =M , where+X = =
-, Y
2 B′
G F
SECOND DEGREE or x y
2A

( ) ( )
- =
Standard equations of conic sections, namely circle, parabola, ellipse and hyperbola X2 Y2
M B′
have already been studied in the previous sections. Now we shall take up the general equation or 2 2
1
M A
of second degree viz.
Ax2 + By2 + Gx + Fy + C = 0 (1) and this is standard equation of a hyperbola in XY-coordinates system.
(iv) If A = 0 or B = 0 (both cannot be zero since in that case the equation (1)
reduces to a linear equation). Assume A ≠ 0 and B = 0.
The nature of the curve represented by (1) can be determined by examining the
coeicients A, B in the above equation. The following cases arise:
(i) If A = B ≠ 0, equation (1) may be written as The equation (1) becomes Ax2 + Gx + Fy + C = 0

 G2 
A( x 2 + y 2 ) + Gx =
+ Fy + C 0 or x2 + y 2 + x += y+ A x2 + x +  =- - +
G2
G F C G
 4 A2 
0 or Fy C
A A A A 4A
 G F   G   C G2 
which represents a circle with centre at  - ,- + - . A x +
 and radius  = -F  y + - 
G2 F 2 C 2

 2A 2A   2A   F 4 AF 
4 A2 4 A A or

(ii) If A ≠ B and both are of the same sign, then we have or AX =2


- FY , where X =+x
G
, Y =+
y
C
-
G2
(Ax2 + Gx) + (By2 + Fy) + C = 0 2A F 4 AF
which is standard equation of a parabola in XY-coordinates system.
 2 G G2   2 F F 2  G2 F 2
A x + x + 2 
+ B y + y + 2 
= + -C
We summarize these results as under:
   
or Let an equation of second degree be of the form Ax2 + By2 + Gx + Fy + C = 0.
A 4 A B 4 B 4 A 4 B
 G   F  G2 F 2
A x +  + B y +  = + -C
It represents:
a circle if A = B ≠ 0
2 2

 2A   2B  4 A 4B
or (2) (i)
(ii) an ellipse if A ≠ B and both are of the same sign
If we write X+= x += (iii) a hyperbola if A ≠ B and both are of opposite signs
G F
,Y y , then (2) can be written as
4A 2B
(iv) a parabola if either A = 0 or B = 0.
AX + BY = + - C = K (say) or + =
2 2G2 F 2 X2 Y2
1
4 A 4B ( K A )2 ( K B )2
6.9.1 Classiication of Conics by the Discriminant
which is standard equation of an ellipse in XY-coordinate system.
(iii) If A ≠ B and both have opposite signs (say A is positive and B is negative), The most general equation of the second degree
version: 1.1 version: 1.1

68 69
1. Quadratic Equations eLearn.Punjab 1. Quadratic Equations eLearn.Punjab
6. Conic Sections eLearn.Punjab 6. Conic Sections eLearn.Punjab

Discuss the conic 7 x 2 - 6 3 xy + 13 y 2 - 16 =


ax2 + 2hxy + by2 + 2gx + 2fy + c = 0 (1)
represents a conic. The quantity h - ab is called the discriminant of (1). Nature of the conic
2
Example 1: 0 (1)
and ind its elements.
can be determined by the discriminant as follows. (1) represents:
(i) an ellipse or a circle if h2 - ab < 0
(ii) a parabola if h2 - ab = 0
Solution. In order to remove the term involving xy, the angle through which axes be

(iii) a hyperbola if h2 - ab > 0


rotated is given by

-6 3
= 2q = 3 °or q =30
The equation (1) can be transformed to the form
7 - 13
tan
AX2 + BY2 + 2GX + 2Fy + C = 0 (2)
if the axes are rotated about the origin through an angle q, (0 < q < 90°) where q is given by Equations of transformation are
3X - Y 
= x X cos30° - Y sin
= 30° 
tan 2q =

2h
a -b
X + 3Y 
2 (2)
= y X sin 30° + Y cos30
= ° 

If a = b or a = 0 = b, then the axes are to be rotated through an angle of 450.
2
Equations of transformation (as already found) are Substituting these expressions in to the equation (1), we get

= x X cosq - Y sin q 
 3X - Y   3 X - Y  X + 3Y   X + 3Y 
= y X sin q + Y cosq  7  - 6 3   + 13   =
2 2
(3)
      
16
2 2 2 2
Substitution of these values of x, y into (1) will result in an equation of the form (2) in
which product term XY will be missing. Nature of the conic (2) has already been discussed in which simpliies to
the last article.
=
4 X + 16
=Y 16 + or
X2 Y2
Solving equations (3) for X, Y we ind 2 2
1 (3)
4 1
=X x cosq + y sin q 
This is an ellipse.

-Y =x+sin q y cosq 
(4) Solving equations (2) for X and Y, (or as already found in (4) of 7.7.1, we have

3x + y -x + 3y
=
=
These equations will be useful in numerical problems.
X , Y
2 2
Note: Under certain conditions equation (1) may not represent any conic. In such a case
Centre of the ellipse (3) is X = 0, Y = 0
i.e., = 3 x + y= 0 - + and
we say (1) represents a degenerate conic.
One such degenerate conic is a pair of straight lines represented by (1) if x 3y 0
giving x = 0, y = 0. Thus centre of (1) is (0, 0)
Length of the major axis = 4, length of minor axis = 2
a h g
f = 0.
Vertices of (3) are: X = ±2, Y = 0
h b
3x + y -x + 3y
= ± =
g f c
i.e., 2 and 0
2 2
The proofs of the above observations are beyond our scope and are omitted.
version: 1.1 version: 1.1

70 71
1. Quadratic Equations eLearn.Punjab 1. Quadratic Equations eLearn.Punjab
6. Conic Sections eLearn.Punjab 6. Conic Sections eLearn.Punjab

Solving these equations for x, y, we have X = 0, Y = 0


( 3 , 1),(- 3, -1) , as vertices of (1). x+ y -x + y
=
=
3x + y -x + 3y
i.e., 0, and 0
Ends of the minor axis are X = 0 and Y = ± 0 and = = ±1 . Solving these
2 2
1 .i.e., or x = 0, y = 0 is the centre of (1)
2 2
Equation of the focal axis:
1 - 3  1 3
Y = 0 i.e. y = x.
y = -x.
equations, we get  ,  and  - , 
Equation of the conjugate axis: X = 0 i.e.
2 2   2 2  Eccentricity = 2
as ends of the minor axis of (1). Foci of (3): X =
2 2 . 2 ±Y 0 =
Equation of the major axis: Y = 0, i.e., -x + 3y =0 or x + y =± 4 2
Equation of the minor axis: X = 0, i.e., 3 x+ y=
0 and -x + y = 0
Solving the above equations for x, y, we have the foci of (1) as (2 2, 2 2) and (-2 2, - 2 2)
± (3): X =
2=
Example 2: Analyze the conic xy = 4 and write its elements.
Vertices of 2, Y 0

x+ y
Solution: Equation of the conic is =± 2 2 and -x +y =
xy - 4 = 0
i.e., 0
(1) 2
Here a = 0 = b, so we rotate the axes through an angle of 450. Equations of Solving these equations, we have the foci of (1) as
(2 2, 2 2) and (-2 2, - 2 2)
X -Y 
transformation are
= x X cos 45° - Y sin
= 45° ± (3): X =
2=
2 
Vertices of 2, Y 0
 x+ y
X +Y  = ±2 2 - x + y =0
(2)
= y X sin 45° - Y cos
= 45°
2 
and
2
Substituting into (1), we have Solving these equations, we have
 X - Y  X + Y  (2, 2) , (-2, -2) as vertices of (1).
  -4 =
 2  2  Asymptotes of the hyperbola (3) are given by X2 - Y2 = 0
0

or X2 - Y2 = 8 or X-Y=0 and X+Y=0

x+ y -x + y x+ y -x + y
-
X2 Y2
=1 (3) i.e., - = 0 and - =0
8 8 2 2 2 2
which is a hyperbola. i.e., x = 0 and y = 0 are equations of the asymptotes of (1).
Solving equations (2) for X, Y, we have

x+ y -x + y
Example 3: By a rotation of axes, eliminate the xy-term in the equation
=
= X , Y 9x2 + 12xy + 4y2 + 2x - 3y = 0 (1)
2 2
Identify the conic and ind its elements.
Centre of the hyperbola (3) is
version: 1.1 version: 1.1

72 73
1. Quadratic Equations eLearn.Punjab 1. Quadratic Equations eLearn.Punjab
6. Conic Sections eLearn.Punjab 6. Conic Sections eLearn.Punjab

Solution: Here a = 9, b = 4, 2h = 12. The angle q through which axes be rotated to given by
13 X= - 13 =
q
1
tan 2= =
2
12 12 or Y 0 or X2 Y
9-4
13
5 which is a parabola.
2 tan q
= 3x + 2 y -2 x + 3 y
1 - tan q Solving equation (2) for X, Y, we have X = ,Y =
12
or 2

5 tan q 6 - 6tan2 q
5
13 13
or = Elements of the parabola are:
or 6 tan2 q + 5 tan q - 6 = 0
= X 0,=
1
-5 ± 25 + 144 -5 ± 13 2 -3
Focus: Y
tan q = =
=
4 13
3x + 2 y -2 x + 3 y
=
=
,
12 12 3 2 1
i.e., 0 and
Since q lies in the irst quadrant, tan q = - is not admissible.
2 13 13 4 13
3
 1 3 
tan q =⇒ q
sin= cos=q Solving these equations, we have- x == , y - i.e., Focus
= , 
2 2 3 1 3
 52 
,
3 13 13 26 52 26

Y = 0 i.e., 3x + 2y = 0 and -2x + 3y = 0


Equations of transformation become
Vertex:

X=0,
x = X cosq - Y sin q = X- Y
3 2 i.e., x = 0, y = 0 i.e., (0, 0)

13 13 Axis:
3 
X=0 i.e., 3x + 2y = 0
y = X sin q + Y cosq =
(2)
X+
13 
2
x-intercept = - ,
Y
13 2 3
y -intercept = .
Substituting these expressions for x and y into (1), we get 9 4
Show that 2x2 - xy + 5x - 2y + 2 = 0 represents a pair of lines. Also ind an
(3 X - 2Y ) 2 + (3 X - 2Y )(3 X + 3Y ) + (2 X + 3Y ) 2
9 12 4 Example 4:
( 13) 2
13 13 equation of each line.

+ (3 X - 2Y ) - (2 X + 3Y ) = Solution: Here a = 2, b = 0,- h =-= , g =


2 3 1 5
0 , f 1, c = 2.
13 13 2 2

(9 X 2 - 12 XY + 9Y 2 ) + (6 X 2 + 5 XY - 6Y 2 ) 2 -
9 12 1 5
or
13 13 a h g 2 2

+ (4 X 2 + 12 XY - 9Y 2 ) - 13Y = h b -f = - 0
4 1
0 1
13 2
 81 72 16  2  108 60 48  -1 2
g f c
 + +  X +- + +  XY
5
 13 13 13   13 13 13 
or 2

 36 72 36  2 1 5  5
+ - +  Y - 13Y = =  -1 +  + 1 -2 + 
 13 13 13  2 2  4
0

version: 1.1 version: 1.1

74 75
1. Quadratic Equations eLearn.Punjab 1. Quadratic Equations eLearn.Punjab
6. Conic Sections eLearn.Punjab 6. Conic Sections eLearn.Punjab

= ax12 + 2hx1 y1 + gx1 + by12 + fy1


= - =0
3 3
4 4 Adding gx1 + fy1 + c to both sides of the above equation and regrouping the terms,
The given equation represents a degenerate conic which is a pair of lines. The given we have
equation is axx1 + h(xy1 + yx1) + byy1 + g(x + x1) + f(y + y1) + c
2x2 + x(5 - y) + (-2y + 2) = 0 = = ax12 + 2hx1 y1 + by12 + 2 gx1 + 2 fy1 + c

y - 5 ± ( y - 5) 2 - 8(-2 y + 2)
=0
or x= since the point (x1, y1) lies on (1).
4
y - 5 ± y 2 - 10 y + 25 + 16 y - 16
Hence an equation of the tangent to (1) at (x1, y1) is
= axx1 + h(xy1 + yx1) + byy1 + g(x + x1) + f(y + y1) + c = 0

y - 5 ± ( y + 3)
4
= Note: An equation of the tangent to the general equation of the second degree at the

2y - 2
4
point (x1, y1) may be obtained by replacing
= , -2 x2 by xx1
4
y2 by yy1
Equations of the lines are 2x - y + 1 = 0 and x + 2 = 0.
2xy by xy1 + yx1
2x by x + x1
Tangent
2y by y + y1
Find an equation of the tangent to the conic
in the equation of the conic.
ax2 + 2hxy + by2 + 2gx + 2fy + c = 0 (1)

Find an equation of the tangent to the conic x2 - xy + y2 - 2 = 0 at the point


at the point (x1, y1)
Diferentiating (1) w.r.t. x, we have Example 5:
whose ordinate is 2.
2ax + 2hy + 2hx + 2by + 2 g + 2 f =
dy dy dy
0
Solution: Putting y = 2 into the given equation, we have
dx dx dx
ax + hy + g
= - x2 - 2 x =
dy
hx + by + f
or 0
x( x - = =
dx
dy  ax + hy1 + g
= - 1
2) 0 x 0, 2
or 
dx  ( x1 , y1 ) hx1 + by1 + f The two points on the conic are (0, 2) and ( 2, 2) .
Equation of the tangent at (x1, y1) is Tangent at (0, 2) is

ax + hy1 + g 0.x - ( x. 2 + 0.y) + 2 y - 2 =


y - y1 =
- 1
1
hx1 + by1 + f
( x1 , y1 ) 0
2
x - 2y + 2 2 =
( x - x1 )(ax1 + hy1 + g ) + ( y - y1 )(hx1 + by1 + f ) =
or 0
or 0
axx1 + hxy1 + gx + + hx1 y + by1 y + fy
Tangent at ( 2, 2) is
or
version: 1.1 version: 1.1

76 77
1. Quadratic Equations eLearn.Punjab
6. Conic Sections eLearn.Punjab

2 x - ( 2 x + 2 y) + 2 y - 2 =
1
0
2
or 2 x+ 2 y-4=0

EXERCISE 6.9

1. By a rotation of axes, eliminate the xy-term in each of the following equations.


Identify the conic and ind its elements:
(i) 4x2 - 4xy + y2 - 6 = 0
(ii) x2 - 2xy + y2 - 8x - 8y = 0
(iii) x 2 + 2 xy + y 2 + 2 2 - 2 2 y + 2 =0
(iv) x + xy + y - 4 = 0
2 2

(v) 7 x 2 - 6 3 xy + 13 y 2 - 16 =
0
(vi) 4x2 - 4xy + 7y2 + 12x + 6y - 9 = 0
(vii) xy - 4x - 2y = 0
(viii) x2 + 4xy - 2y2 - 6 = 0
(ix) x2 - 4xy - 2y2 + 10x + 4y = 0

2. Show that (i) 10xy + 8x - 15y - 12 = 0 and


(ii) 6x2 + xy - y2 - 21x - 8y + 9 = 0
each represents a pair of straight lines and ind an equation of each line.

3. Find an equation of the tangent to each of the given conics at the indicated point.
(i) 3x2 - 7y2 + 2x - y - 48 = 0 at (4, 1)
(ii) x2 + 5xy - 4y2 + 4 = 0 at y = -1
(iii) x2 + 4xy - 3y2 - 5x - 9y + 6 = 0 at x = 3.

version: 1.1

78
version: 1.1

CHAPTER

7 Vectors

Animation 7.1: Cross Product of Vectors


Source and credit: eLearn.Punjab
1. Quadratic Equations eLearn.Punjab 1. Quadratic Equations eLearn.Punjab
7. Vectors eLearn.Punjab 7. Vectors eLearn.Punjab


7.1 INTRODUCTION (iii) If terminal point B of a vector AB coincides with its initial point A, then magnitude

AB = 0 and AB = 0 , which is called zero or null vector.
In physics, mathematics and engineering, we encounter with two important quantities,
(iv) Two vectors are said to be negative of each other if they have same magnitude but
known as “Scalars and Vectors”.
opposite direction.
BA = - AB = - v
A scalar quantity, or simply a scalar, is one that possesses only magnitude. It can   
If AB = v , then
be speciied by a number alongwith unit. In Physics, the quantities like mass, time, density,
and BA = - AB
 
temperature, length, volume, speed and work are examples of scalars.
A vector quantity, or simply a vector, is one that possesses
both magnitude and direction. In Physics, the quantities like displacement, velocity, 7.1.2 Multiplication of Vector by a Scalar
acceleration, weight, force, momentum, electric and magnetic ields are examples of vectors.
In this section, we introduce vectors and their fundamental operations we begin with We use the word scalar to mean a real number. Multiplication of a vector v by a scalar
a geometric interpretation of vector in the plane and in space. ‘k’ is a vector whose magnitude is k times that of v. It is denoted by kv .
(i) If k is +ve, then v and kv are in the same direction.
(ii) If k is -ve, then v and kv are in the opposite direction

(a) Equal vectors



Two vectors AB and are said to be equal, if
they have the same magnitude and same direction

i.e., AB = CD
 

7.1.1 Geometric Interpretation of vector (b) Parallel vectors


 Two vectors are parallel if and only if they are non-zero
Geometrically, a vector is represented by a directed line segment AB with A its initial scalar multiple of each other, (see igure).
point and B its terminal point. It is often found convenient to denote a vector by an arrow

and is written either as AB or as a boldface symbol like v or in underlined form v.
 7.1.3 Addition and Subtraction of Two Vectors
(i) The magnitude or length or norm of a vector AB or v, is its absolute value and is

written as AB or simply AB or v . Addition of two vectors is explained by the following two laws:
(ii) A unit vector is deined as a vector whose magnitude is unity. Unit vector of vector
(i) Triangle Law of Addition
v is written as v̂ (read as v hat) and is deined by vˆ =
v
v

version: 1.1 version: 1.1

2 3
1. Quadratic Equations eLearn.Punjab 1. Quadratic Equations eLearn.Punjab
7. Vectors eLearn.Punjab 7. Vectors eLearn.Punjab

If two vectors u and v are represented by 7.1.4 Position Vector


the two sides AB and BC of a triangle such
that the terminal point of u coincide with The vector, whose initial point is the origin O and whose
the initial point of v, then the third side AC of terminal point is P, is called the position vector of the point P
the triangle gives vector sum u + v, that is 
and is written as OP .
AB + BC= AC ⇒ u = + v AC
   
The position vectors of the points A and B relative to the
= by OA a=
 
origin O are deined and OB b respectively.
(ii) Parallelogram Law of Addition In the igure, by triangle law of addition,
OA + AB =
If two vectors u and v are represented by two adjacent   
OB
a + AB =
sides AB and AC of a parallelogram as shown in the 
b
igure, then diagonal AD give the sum or resultant
⇒ -=

  AB b a
of AB and AC , that is
AD = AB + AC =+
  
u v
7.1.5 Vectors in a Plane
Note: This law was used by Aristotle to describe the combined action of two forces.
Let R be the set of real numbers. The Cartesian plane is deined to be the R2 = {(x, y) : x,
(b) Subtraction of two vectors y d R}.
 
The diference of two vectors AB and AC is deined by An element (x, y) d R2 represents a point P(x, y) which
AB - AC= AB + (- AC )
   
is uniquely determined by its coordinate x and y. Given a
u - v = u + ( -v ) vector u in the plane, there exists a unique point

P(x, y) in the plane such that the vector OP is equal to u
(see igure). So we can use rectangular coordinates (x, y) for P to
associate a unique ordered pair [x, y] to vector u.
We deine addition and scalar multiplication in R2 by:

vectors u [ x, y ] and v [ x′, y′] , we have


In igure, this diference is interpreted as the main diagonal of the parallelogram with
sides AB and - AC . We can also interpret the same vector diference as the third side For any two=
=
 

u + v = [ x, y ] + [ x′, y′] = [ x + x′, y + y′]


(i) Addition:
 
of a triangle with sides AB and AC . In this second interpretation, the vector diference
AB - AC = CB points the terminal point of the vector from which we are subtracting the For u = [x, y] and a d R, we have
  
(ii) Scalar Multiplication:
second vector. au = a[x, y] = [ax, ay]
Deinition: The set of all ordered pairs [x, y] of real numbers, together with the rules of
addition and scalar multiplication, is called the set of vectors in R2.

version: 1.1 version: 1.1

4 5
1. Quadratic Equations eLearn.Punjab 1. Quadratic Equations eLearn.Punjab
7. Vectors eLearn.Punjab 7. Vectors eLearn.Punjab

For the vector u = [x, y], x and y are called the components of u. 7.1.6 Properties of Magnitude of a Vector
Note: The vector [x, y] is an ordered pair of numbers, not a point (x, y) in the plane.
Let v be a vector in the plane or in space and let c be a real number, then
(a) Negative of a Vector (i) v ≥ 0, and v =
0 if and only if v = 0
In scalar multiplication (ii), if a = -1 and u = [x, y] then (ii) cv = c v
au = (-1) [x, y] = [-x, -y]
which is denoted by -u and is called the additive inverse of u or negative vector of u. Proof: (i) We write vector v in component form as v = [x, y], then
v= x 2 + y 2 ≥ 0 for all x and y.
(b) Diference of two Vectors
We deine u - v as u + (-v) Further v = x 2 + y 2 = 0 if and only if x = 0, y = 0

=
= u [ x, y ] and v [ x' , y′], then
In this case v = [0,0] = 0

( cx ) + ( cy )=
If
u - v = u + (-v) cv= cx, cy= c 2 x 2 + y=
2 2

[ x, y ] + [ - x′ - y′]= [ x - x′, y - y′]


2
(ii) c v
=
7.1.7 Another notation for representing vectors in plane
(c) Zero Vector
Clearly u + (-u) = [x, y] + [-x, -y] = [x - x, y - y] = [0,0] = 0. We introduce two special vectors,
0 = [0,0] is called the Zero (Null) vector. = =
i [1,0], j [0,1] in R 2

(d) Equal Vectors As magnitude of i = 12 + 02 = 1


Two vectors u = [x, y] and v = [x ’, y ’] of R2 are said to be equal if and only if they have the magnitude of j= 02 + 12 = 1
same components. That is,
[x, y] = [x ’, y ’] if and only if x = x ’ and y = y ‘ So i and j are called unit vectors along x-axis, and along y-axis respectively. Using the
and we write u = v deinition of addition and scalar multiplication, the vector [x, y] can be written as
= u [ x=
, y] [ x,0] + [0, y ]
= x[1,0] + y[0,1]
(e) Position Vector

= xi + y j
For any point P(x, y) in R2, a vector u = [x, y] is represented by a directed line segment

OP , whose initial point is at origin. Such vectors are called position vectors because
Thus each vector [x, y] in R2 can be uniquely represented by
xi + y j .
they provide a unique correspondence between the points (positions) and vectors.

u [ x, y ] and v [ x′, y′] is written as


(f) Magnitude of a Vector In terms of unit vector i and j , the sum u + v of two vectors
For any vector u = [x, y] in R2, we deine the magnitude or norm =
=
or length of the vector as of the point P(x, y) from the origin O u+v= [ x + x′, y + y′]
∴ Magnitude of OP = = = + = ( x + x′ ) i + ( y + y′ ) j
 
OP u x2 y 2
version: 1.1 version: 1.1

6 7
1. Quadratic Equations eLearn.Punjab 1. Quadratic Equations eLearn.Punjab
7. Vectors eLearn.Punjab 7. Vectors eLearn.Punjab

7.1.8 A unit vector in the direction of another given vector. Example 3: Find a unit vector in the direction of the vector
(i) v= 2i + 6 j (ii) v =[-2,4]
A vector u is called a unit vector, if u = 1
Now we ind a unit vector u in the direction of any other given vector v. Solution: (i) v= 2i + 6 j
We can do by the use of property (ii) of magnitude of vector, as follows: v= ( 2) + (6) = 4 + 36 =
2 2
40

∴ A unit vector in the direction of v =


= + i = + i
= =
v 2 6 1 3
1 1 j j

[ -2, 4] =-2i + 4 j
v 1 v 40 40 10 10
v=
v v
(ii)
∴ the vector v =
( -2 ) + ( 4 ) = 4 + 16 = 20
1
v=
v is the required unit vector 2 2
v
-2 -1
∴ A unit vector in the direction of v = + i = j + i
It points in the same direction as v, because it is a positive scalar multiple of v. v 4 2
j
v 20 20 5 5
Example 1:
Example 4: If ABCD is a parallelogram such that the points A, B and C are respectively
For v = [1, -3] and w = [2,5] (-2, -3), (1,4) and (0, -5). Find the coordinates of D.
(i) v + w = [1, -3] + [2,5] = [1 + 2, -3 + 5] = [3,2]
(ii) 4v + 2w = [4, -12] + [4,10] = [8,-2] Solution: Suppose the coordinates of D are (x, y)
(iii) v - w = [1, -3]- [2,5] = [l - 2, -3 -5] = [-1,-8] As ABCD is a parallelogram
(iv) v - v = [l -1, -3 + 3] = [0,0] = 0 ∴ AB = DC and AB  DC
v= (1) 2 + (-3) 2 = 1 + 9= ⇒ AB =
 
(v) 10 DC
∴ (1 + 2)i + (4 + 3) j =(0 - x)i + (-5 - y ) j
Example 2: Find the unit vector in the same direction as the vector v = [3, -4]. ⇒ 3i + 7 j =- xi + (-5 - y ) j

v = [3, -4] = 3i - 4 j
Equating horizontal and vertical components, we have
Solution: -x = 3 ⇒ x = -3
v= 32 + (-4)=
2
25= 5 and -5 - y = 7 ⇒ y = -12

= = [3, -4]
Hence coordinates of D are (-3, 12).
1 1
Now u v (u is unit vector in the direction of v)
v 5
 3 -4 
7.1.9 The Ratio Formula
= , 
5 5 
Let A and B be two points whose position vectors (p.v.) are a and b respectively. If a
 3   -4 
Veriication: u =   +  = + = 1
2 2
9 16 point P divides AB in the ratio p : q, then the position vector of P is given by
5  5  25 25

version: 1.1 version: 1.1

8 9
1. Quadratic Equations eLearn.Punjab 1. Quadratic Equations eLearn.Punjab
7. Vectors eLearn.Punjab 7. Vectors eLearn.Punjab

qa + pb Now 2OC= a + b

r=
p+q ⇒ OC + OC = OA + OB
   

⇒ OC - OA = OB - OC
Proof: Given a and b are position vectors of the points A and B respectively. Let r be    

⇒ OC + AO = OB + CO
the position vector of the point P which divides the line segment AB in the ratio p : q. That is    
m AP : mPB = p : q
⇒ AO + OC = CO + OB
   

= ∴ AC =
m AP p  

( ) ( )
So CB
Thus m AC = mCB
mPB q  
⇒ q m AP = ⇒
( ) ( )
p mPB
C is equidistant from A and B, but A, B, C are collinear.
Thus q AP = p PB
 
Hence C is the mid point of AB.
⇒ q (r - a ) = p (b - r )
⇒ qr - qa = pb - pr Example 6: Use vectors, to prove that the diagonals of a parallelogram bisect each
⇒ pr + qr = qa + pb
other.

⇒ r ( p + q ) = qa + pb
qa + pb
Solution: Let the vertices of the parallelogram be A, B, C and D (see igure)
⇒ r= = AB + AD , the vector from A to the mid point of diagonal AC is
   
q+ p
Since AC

Corollary: If P is the mid point of AB, then p : q = 1 : 1 = v (


AB + AD
1  
)
a+b
2
∴ positive vector of P = = = AB - AD , the vector from A to the mid point of diagonal DB is
   
r Since DB

( )
2
w= AD + AB - AD
 1  
2
= AD + AB - AD
7.1.10 Vector Geometry  1  1 

( )
2 2
= AB + AD
Let us now use the concepts of vectors discussed so far in proving Geometrical 1 
 
Theorems. A few examples are being solved here to illustrate the method. 2
=v
Since v = w , these mid points of the diagonals AC and DB are the same.
 
Example 5: If a and b be the p.vs of A and B respectively w.r.t. origin O, and C be a point

a+b
on AB such that OC =
Thus the diagonals of a parallelogram bisect each other.
, then show that C is the mid-point of AB.
2

= a = b and OC
= ( a + b)
   1
Solution: OA , OB
2
version: 1.1 version: 1.1

10 11
1. Quadratic Equations eLearn.Punjab 1. Quadratic Equations eLearn.Punjab
7. Vectors eLearn.Punjab 7. Vectors eLearn.Punjab

EXERCISE 7.1 (i) Point C with position vector 2i - 3 j and point D with position vector 3i + 2 j in
the ratio 4 : 3
Write the vector PQ in the form xi + y j . (ii) Point E with position vector 5 j and point F with position vector 4i + j in ratio 2 : 5

1.
(i) P(2,3), Q(6, -2) (ii) P(0,5), Q(-1, -6) 13. Prove that the line segment joining the mid points of two sides of a triangle is
2. Find the magnitude of the vector u: parallel to the third side and half as long.
(i) u= 2i - 7 j (ii) u= i + j (iii) u = [3, - 4] 14. Prove that the line segments joining the mid points of the sides of a quadrilateral
3. If u =2i - 7 j , v =i - 6 j and w =-i + j . Find the following vectors: taken in order form a parallelogram.

u+v-w 2u - 3v + 4w u+ v+ w
1 1 1
(i) (ii) (iii)
2 2 2
7.2 INTRODUCTION OF VECTOR IN SPACE
Find the sum of the vectors AB and CD , given the four points A(1, -1), B(2 ,0 ),
 
4. In space, a rectangular coordinate system is constructed
C(-1, 3) and D(-2, 2). using three mutually orthogonal (perpendicular) axes, which
Find the vector from the point A to the origin where AB= 4i - 2 j and B is the point

5. have orgin as their common point of intersection. When
(-2, 5). sketching igures, we follow the convention that the positive
6. Find a unit vector in the direction of the vector given below: x-axis points towards the reader, the positive y-axis to the
right and the positive z-axis points upwards.
v= 2i - j = i+ -v =- i
1 3 3 1
(i) (ii) v j (iii) j
2 2 2 2

7. If A, B and C are respectively the points (2, -4), (4, 0) and (1, 6). Use vector method
to ind the coordinates of the point D if: These axis are also labeled in accordance with the right
(i) ABCD is a parallelogram (ii) ADBC is a parallelogram hand rule. If ingers of the right hand, pointing in the direction
8. If B, C and D are respectively (4, 1), (-2, 3) and (-8, 0). Use vector method to ind of positive x-axis, are curled toward the positive y-axis,
the coordinates of the point: then the thumb will point in the direction of positive z-axis,
(i) A if ABCD is a parallelogram. (ii) E if AEBD is a parallelogram. perpendicular to the xy-plane. The broken lines in the igure
If O is the origin and OP = AB , ind the point P when A and B are (-3, 7) and (1, 0)
  represent the negative axes.
9.
respectively.
10. Use vectors, to show that ABCD is a parallelogram, when the points A, B, C and D
are respectively (0, 0), (a, 0), (b, c) and (b - a, c).
A point P in space has three coordinates, one along

If AB = CD , ind the coordinates of the point A when points B, C, D are (1, 2), (-2, 5),
  x-axis, the second along y-axis and the third along z-axis. If the
11.
distances along x-axis, y-axis and z-axis respectively are a, b,
(4, 11) respectively.
and c, then the point P is written with a unique triple of real
12. Find the position vectors of the point of division of the line segments joining the
numbers as P = (a, b, c) (see igure).
following pair of points, in the given ratio:

version: 1.1 version: 1.1

12 13
1. Quadratic Equations eLearn.Punjab 1. Quadratic Equations eLearn.Punjab
7. Vectors eLearn.Punjab 7. Vectors eLearn.Punjab

7.2.1 Concept of a vector in space f) Magnitude of a vector: We deine the magnitude or norm or length of a vector u
in space by the distance of the point P(x, y, z) from the origin O.

∴ OP = u = x 2 + y 2 + z 2

The set R3 = {(x, y, z) : x, y, z d R} is called the
3-dimensional space. An element (x, y, z) of R3 represents
a point P(x, y, z), which is uniquely determined by its Example 1: For the vectors, v = [2,1,3] and w = [-1,4,0], we have the following
coordinates x, y and z. Given a vector u in space, there (i) v + w = [2 - 1, 1 + 4, 3 + 0]= [l,5,3]
exists a unique point P(x, y, z) in space such that the
 (ii) v - w = [2 + 1,1 - 4, 3 - 0]= [3, -3, 3]
vector OP is equal to u (see igure). (iii) 2w =2[-1, 4, 0] = [-2, 8, 0]
v - 2w = [ 2 + 2,1 - 8,3 - 0] = [ 4, -7,3] = ( 4) + ( -7 ) + ( 3) = 16 + 49 + 9 =
Now each element (x, y, z ) d P3 is associated to 2 2 2
(iv) 74
a unique ordered triple [x, y, z], which represents the

vector u = OP = [x, y, z].
We deine addition and scalar multiplication in R3 7.2.2 Properties of Vectors

Addition: For any two vectors u = [x, y, z] and v = [ x′, y′, z′] , we have
by:

[ x, y, z ] + [ x′, y′, z′] = [ x + x′, y + y′, z + z′]


(i) Vectors, both in the plane and in space, have the following properties:
u+v= Let u, v and w be vectors in the plane or in space and let a, b d R, then they have the

For u = [x, y, z] and a d R, we have


following properties
(ii) Scalar Multiplication:
au =a[x, y, z] = [ax, ay, az]
(i) u+v=v+u (Commutative Property)
(ii) (u + v) + w = u + (v + w) (Associative Property)
(iii) u + (-1)u = u - u = 0
Deinition: The set of all ordered triples [x, y, z] of real numbers, together with the rules
(Inverse for vector addition)
of addition and scalar multiplication, is called the set of vectors in R3.
(iv) a(v + w)=av + aw (Distributive Property)
For the vector u = [x, y, z], x, y and z are called the components of u.
(v) a(bu) = (ab)u (Scalar Multiplication)
The deinition of vectors in R3 states that vector addition and scalar multiplication are
Proof: Each statement is proved by writing the vector/vectors in component form in

The negative of the vector u =[ x; y, z ] as - u =- [ x, - y , - z ]


to be carried out for vectors in space just as for vectors in the plane. So we deine in R3:
( 1) u =-
R2 / R3 and using the properties of real numbers. We give the proofs of properties (i) and (ii)

v [ x′, y′, z′] and w [ x′′, y′′, z′′]


a)
==
as follows.
diference
v - w = v + ( - w ) = [ x′ - x′′, y′ - y′′, z′ - z′′]
b) The of two vectors as (i) Since for any two real numbers a and b
a + b = b + a, it follows, that
for any two vectors u = [x, y] and v = [ x′, y′] in R2, we have
v [ x′, y′, z′] and w [ x′′, y′′, z′′] by v w if and only
c) The zero vector as 0 = [0,0,0]
d) Equality of two = = =
vectors v [ x, y ] + [ x′ + y′]
u +=
=x′ x= ′′, y′ y′′ and=z′ z′′ . =[ x + x′, y + y′]
e) Position Vector =[ x′ + x, y′ + y ]
For any point P(x, y, z) in R3, a vector u = [x, y, z] is represented by a directed line
 = [ x′, y′] + [ x, y ]
segment OP , whose initial point is at origin. Such vectors are called position vectors =v+u
in R3. So addition of vectors in R2 is commutative
version: 1.1 version: 1.1

14 15
1. Quadratic Equations eLearn.Punjab 1. Quadratic Equations eLearn.Punjab
7. Vectors eLearn.Punjab 7. Vectors eLearn.Punjab

(ii) Since for any three real numbers a, b, c, 7.2.4 Distance Between two Points in Space
(a + b) + c = a + (b + c) , it follows that
=
for any three vectors, u [ x= , y ], v [ x′, y′] and w [ x′′, y′′] in R 2 , we have
=  
(u + v) + w = [ x + x′, y + y′] + [ x′′, y′′]
P1 ( x1 , y1 , z1 ) and P2 ( x2 , y2 , z2 )
If OP1 and OP2 are the position vectors of the points

=[( x + x′) + x′′,( y + y′) + y′′]


=[ x + ( x′ + x′′), y + ( y′ + y′′)]


= [ x, y ] + [ x′ + x′′, y′ + y′′]
The vector PP1 2 , is given by

1 2 = OP2 - OP1 = [ x2 - x1 , y2 - y1 , z2 - z1 ]
  
=u + (v + w)
PP

∴ Distance between P1 and P2 = PP



So addition of vectors in R2 is associative 1 2

= ( x2 - x1 ) 2 + ( y2 - y1 ) 2 + ( z2 - z1 ) 2
The proofs of the other parts are left as an exercise for the students.

7.2.3 Another notation for representing vectors in space This is called distance formula between two points P1 and P2 in R3,

As in plane, similarly we introduce three special Example 2: If u = 2i + 3 j + k , v = 4i + 6 j + 2k and w = -6i - 9 j - 3k , then


vectors (a) Find
= =
i [1,0,0], =
j [0,1,0] and k [0,0,1] in R 3 . (i) u + 2v (ii) u-v-w

As magnitude of i = 12 + 02 + 02 = 1
(b) Show that u, v, and w are parallel to each other.

magnitude of j = 02 + 12 + 02= 1
Solution: (a)
(i) u + 2v = 2i + 3 j + k + 2(4i + 6 j + 2k )
= 2i + 3 j + k + 8i + 12 j + 4k
=10i + 15 j + 5k
and magnitude of k = 02 + 02 + 12 = 1 So i, j and k are called unit vectors along
(ii) u - v - w
= (2i + 3 j + k ) - (4i + 6 j + 2k ) - (-6i - 9 j - 3k )
= (2 - 4 + 6)i + (3 - 6 + 9) j + (1 - 2 + 3)k
x-axis, along y-axis and along z-axis respectively. Using the deinition of addition and scalar

= 4i + 6 j + 2k
multiplication, the vector [x, y, z] can be written as
u = [ x, y , z ] = [ x,0,0] + [0, y,0] + [0,0, z ]
= x[1,0,0] + y[0,1,0] + z[0,0,,1] (b) v = 4i + 6 j + 2k = 2(2i + 3 j + k )

=xi + y j + zk ∴ v= 2u

Thus each vector [x, y, z] in R3 can be uniquely represented by xi + y j + zk .
u and v are parallel vectors, and have same direction
Again w=-6i - 9 j - 3k
= -3(2i + 3 j + k )
u [ x, y, z ] and v [ x′, y′, z′] is written as
In terms of unit vector i , j and k , , the sum u + v of two vectors
=
=
∴ w= -3u
u+v= [ x + x′, y + y′, z + z′] ⇒
= ( x + x′ ) i + ( y + y′ ) j + ( z + z′ ) k
u and w are parallel vectors and have opposite direction.
Hence u, v and w are parallel to each other.
version: 1.1 version: 1.1

16 17
1. Quadratic Equations eLearn.Punjab 1. Quadratic Equations eLearn.Punjab
7. Vectors eLearn.Punjab 7. Vectors eLearn.Punjab

7.2.5 Direction Angles and Direction Cosines of a Vector EXERCISE 7.2

Let A = (2, 5), B = (-1,1) and C = (2, -6). Find


Let r = OP =xi + y j + zk be a non-zero vector, let a, b and
 1.
2AB - CB 2CB - 2CA
    
g denote the angles formed between r and the unit coordinate (i) AB (ii) (iii)
2. Let u =i + 2 j - k , v =3i - 2 j + 2k , w =5i - j + 3k . Find the indicated vector or number.
u + 2v + w (ii) u - 3w (iii) 3v + w
vectors i, j and k respectively.
such that (i)
0 ≤ a ≤ p , 0 ≤ b ≤ p , and 0 ≤ g ≤ p , 3. Find the magnitude of the vector v and write the direction cosines of v.
(i) the angles a , b , g are called the direction angles and (i) v = 2i + 3 j + 4k (ii) v =i - j - k (iii) v= 4i - 5 j
(ii) the numbers cos a, cos b and cos g are called direction 4. Find a, so that a i + (a + 1) j + 2k =
3.
Find a unit vector in the direction of v =+
i 2j-k.
cosines of the vector r.
5.
6. If a = 3i - j - 4k , b =-2i - 4 j - 3k and c =+
i 2j-k.
Find a unit vector parallel to 3a - 2b + 4c .
Important Result:
Prove that cos2 a + cos2 b + cos2 g = 1
7. Find a vector whose
(i) magnitude is 4 and is parallel to 2i - 3 j + 6k

r =[ x, y, z ] =xi + y j + zk
Solution:
(ii) magnitude is 2 and is parallel to -i + j + k
If u = 2i + 3 j + 4k , , v =-i + 3 j - k and w =+
i 6 j + zk represent the sides of a triangle.
Let

∴ r= x +y +z =
8.
2 2 2
r Find the value of z.
The position vectors of the points A, B, C and D are 2i - j + k , 3i + j ,
x y z
=  , ,  is the unit vector in the direction of the vector r = OP .
 9.
2i + 4 j - 2k -i - 2 j + k
r 
r r r
then
r and respectively. Show that AB is parallel

It can be visualized that the triangle OAP is a right triangle with ∠A = 900.
to CD .
10. We say that two vectors v and w in space are parallel if there is a scalar c such that
Therefore in right triangle OAP,
v = cw. The vectors point in the same direction if c > 0, and the vectors point in the

= a
=
OA x opposite direction if c < 0
Find two vectors of length 2 parallel to the vector v =2i - 4 j + 4k .
cos , similarly
OP r (a)
= cos b =
y
, cos g
z (b) Find the constant a so that the vectors v =-
i 3 j + 4k and w = ai + 9 j - 12k are
r r parallel.
The numbers cos a = , , cos b = and cos g = Find a vector of length 5 in the direction opposite that of v =-
i 2 j + 3k .
x y z
are called (c)
Find a and b so that the vectors 3i - j + 4k and ai + b j - 2k are parallel.
r r r
 (d)
the direction cosines of OP .

x2 y 2 z 2 x2 + y 2 + z 2 r 2
∴ cos a + cos b + cos g = 2 + 2 + 2 =
2 2 2
= 2 =1
r r r r2 r
version: 1.1 version: 1.1

18 19
1. Quadratic Equations eLearn.Punjab 1. Quadratic Equations eLearn.Punjab
7. Vectors eLearn.Punjab 7. Vectors eLearn.Punjab

11. Find the direction cosines for the given vector: 7.3.1 Deductions of the Important Results
(i) v = 3i - j + 2k (ii) 6i - 2 j + k
(iii) =
=

PQ , where P (2, 1, 5) and Q (1, 3, 1) . By Applying the deinition of dot product to unit vectors i, j , k , we have,
=
= =
=
12. Which of the following triples can be the direction angles of a single vector:
(a) i.i i i cos 0 1 (b) i. j i j cos 90 0
=
= =
=
(i) 450, 450, 600 (ii) 300, 450, 600 (iii) 450, 600, 600
j. j j j cos 0 1 j.k j k cos 90 0
7.3 THE SCALAR PRODUCT OF TWO VECTORS =
=k .k k k cos 0 1 =
=k .i k i cos 90 0
(c) u.v = u v cosq
We shall now consider products of two vectors that originated in the study of Physics = v u cos( - q )
= v u cosq
and Engineering. The concept of angle between two vectors is expressed in terms of a scalar
⇒ u.v =
product of two vectors.
v.u

Deinition 1:
Let two non-zero vectors u and v, in the plane or in space, have same initial point. The ∴ Dot product of two vectors is commutative.
dot product of u and v, written as u.v, is deined by
u.v = u v cosq

7.3.2 Perpendicular (Orthogonal) Vectors

Deinition: Two non-zero vectors u and v are perpendicular if and only if u.v = 0.

p p
Since angle between u and v is and cos =0

p
2 2
where q is the angle between u and v and 0767p so u.v = u v cos
2

Deinition 2:
(a) If+ u = a1 i b+1 j =
u.v = 0
and v a2 i b2 j .
are two non-zero vectors in the plane. The dot product u.v is deined by Note: As 0 . b = 0, for every vector b. So the zero vector is regarded to be perpendicular
u.v = a1a2 +b1b2
(b) If u = a1 i + b1 j + c1 k and v = a2 i + b2 j + c2 k .
to every vector.

are two non-zero vectors in space. The dot product u.v is deined by 7.3.3 Properties of Dot Product
u.v = a1a2 + b1b2 + c1c2
Let u, v and w be vectors and let c be a real number, then
u.v = 0 ⇒ u = 0 or v = 0
Note: The dot product is also referred to the scalar product or the inner product.
(i)
version: 1.1 version: 1.1

20 21
1. Quadratic Equations eLearn.Punjab 1. Quadratic Equations eLearn.Punjab
7. Vectors eLearn.Punjab 7. Vectors eLearn.Punjab

(ii) u.v = v.u (commutative property) By law of cosines,


(iii) u . (v + w) = u.v + u.w (distributive property) v - w = v + w - 2 v w cosq
v [ x1 , y1 ] and w [ x2 , y2 ], then
2 2 2
(1)
=
=
(iv) (c u ).v = c (u.v), (c is scalar)

[ x1 - x2 , y1 - y2 ]
if
v-w=
The proofs of the properties are left as an exercise for the students.

So equation (1) becomes:


7.3.4 Analytical Expression of Dot Product u.v
(Dot product of vectors in their components form)
x1 - x2 + y1 - y2 = x12 + y12 + x2 2 + y2 2 - 2 v w cosq
2 2

Let u = a1 i + b1 j + c1 k and v = a2 i + b2 j + c2 k 2 v w cosq


- 2 x1 x2 - 2 y-1 y2 =
⇒ x1 x2 += = q v.w
be two non-zero vectors.
y1 y2 v w cos
From distributive Law we can write:

∴ u.v = (a1 i + b1 j + c1 k ).(a2 i + b2 j + c2 k )


If u =3i - j - 2k and v =i + 2 j - k , then
= a1a2 (i.i ) + a1b2 (i. j ) + a1c2 (i.k )
Example 2:
u.v = (-3)(1) + (-1)(2) + (-2)(-1) = 3
 =
i.i j=
. j k=
+b1a2 ( j.i ) + b1b2 ( j. j ) + b1c2 ( j.k )
.k 1
i.=
j j.=
k k= If u =2i - 4 j + 5k and v =-4i - 3 j - 4k , then
+c1a2 (k .i ) + c1b2 (k . j ) + c1c2 (k .k )
.i 0 Example 3:
=
u.v (2)(4) + (-4)(-3) + (5)(=
-4) 0
⇒ u.v = a1a2 + b1b2 + c1c2 ⇒ u and v are perpendicular
Hence the dot product of two vectors is the sum of the product of their corresponding
components. 7.3.5 Angle between two vectors
Equivalence of two deinitions of dot product of two vectors has been proved in
the following example. The angle between two vectors u and v is determined from the deinition of dot
product, that is
= ≤q ,
(a) u.v u ≤v cos where 0 q p
Example 1: (i) If v = [x1, y2] and w = [x2, y2] are two vectors in the plane, then
v=
.w x1 x2 + y1 y2
∴ cosq =
u.v
(ii) If v and w are two non-zero vectors in the plane, then u v
v.w = v w cosq (b) u = a1 i + b1 j + c1 k and v = a2 i + b2 j + c2 k , then
where q is the angle between v and w and 0 7 q 7 p. u.v = a1a2 + b1b2 + c1c2
u= a12 + b12 + c12 and v = a2 2 + b2 2 + c2 2
Proof: Let v and w determine the sides of a triangle then the third side, opposite to the
angle q, has length v - w (by triangle law of addition of vectors) cosq =
u.v

u v

version: 1.1 version: 1.1

22 23
1. Quadratic Equations eLearn.Punjab 1. Quadratic Equations eLearn.Punjab
7. Vectors eLearn.Punjab 7. Vectors eLearn.Punjab

⇒ (2i + a j + 5k ) . (3i + j + a k ) =
a1a2 + b1b2 + c1c2
0
∴ cosq = ⇒ 6 + a + 5a =
a12 + b12 + c12 a2 2 + b2 2 + c2 2 ∴ -a =
0
1

Corollaries:
If q = 0 or p, the vectors u and v are collinear.
Example 6:
Show that the vectors 2i - j + k , i - 3 j - 5k and 3i - 4 j - 4k form the sides of a right
(i)

p
(ii) If q = , cosq =0 ⇒ u.v =0. triangle.
2 Solution:
Let AB =2i - j + k and BC =i - 3 j - 5k
The vectors u and v are perpendicular or orthogonal.  

Now AB + BC = (2i - j + k ) + (i - 3 j - 5k )
 
Example 4: Find the angle between the vectors
u =2i - j + k and v =-i + j =3i - 4 j - 4k =AC

(third side)
∴ AB , BC and AC form a triangle ABC.
  
Solution: u . v= (2i - j + k ) . (-i + j + 0k )
=(2)(-1) + (-1)(1) + (1)(0) =-3 Further we prove that rABC is a right triangle

∴ u = 2i - j + k = (2) 2 + (-1) 2 + (1) 2 = AB . BC = (2i - j + k ).(i - 3 j - 5k )


 

= (2)(1) + (-1)(-3) + (1)(-5)


6

v = -i + j + 0k = (-1) 2 + (1) 2 + (0) 2 = 2


= 2+3-5
and
=0
cosq =
u.v
∴ AB ⊥ BC
Now  
u.v
-3 Hence rABC is a right triangle.
⇒ - q ==
3
cos

5p
6 2 2
∴q=
7.3.6 Projection of one Vector upon another Vector:
6
In many physical applications, it is required to know

Find a scalar a so that the vectors


“how much” of a vector is applied along a given direction.
Example 5:
2i + a j + 5k and 3i + j + a k are perpendicular.
For this purpose we ind the projection of one vector
along the other vector.
= Let OA u=
 
and OB v
Let q be the angle between them, such that
Solution:
u = 2i + a j + 5k and v = 3i + j + a k
0 7 q 7 p.
Let
It is given that u and v are perpendicular
∴ u .v= 0

version: 1.1 version: 1.1

24 25
1. Quadratic Equations eLearn.Punjab 1. Quadratic Equations eLearn.Punjab
7. Vectors eLearn.Punjab 7. Vectors eLearn.Punjab

Draw BM ⊥ OA . Then OM is called the projection of v along u.


Solution: Let the vectors a, b and c be along the sides BC, CA and AB of the triangle ABC as
shown in the igure.

= cosq , that is,
OM a+b+c=0
⇒ a = -(b + c)
Now
OB
= = cosq v cosq Now a.a = (b + c).(b + c)

OM OB (1)
= b.b + b.c + c.b + c.c
By deinition, cosq =
u.v

(2)
u v a2 = b2 + 2b.c + c2 ( b.c c.b)
From (1) and (2), OM = v .
u.v ⇒ a2 = b2 + c2 + 2bc.cos(p - A)
u v ∴ a2 = b2 + c2 - 2bc cos A

∴ Projection of v along u =
u.v (ii) a + b + c = 0
u ⇒ a = -b - c

Similarly, projection of u along v =


Take dot product with a
a.a = -a.b - a.c
u.v

= - ab cos(p - C) - ac cos(p - B)
v

a2 = ab cos C +ac CosB


⇒ a = b cos C + c CosB
Example 7: Show that the components of a vector are the projections of that vector
along i , j and k respectively.
Prove that: cos(a - b) = cos a cos b + sin a sin b
Solution: Let v = ai + b j + ck , then
Example 9:

Let OA and OB be the unit vectors in the xy-plane making angles a and b
 

Projection of v along i = = (ai + b j + ck ).i = a


Solution:
v.i
with the positive x-axis.
So that ∠AOB = a - b
i

Projection of v along j = = (ai + b j + ck ). j = b Now= OA cos a i + sin a j


v. j 

and=OB cos b i + sin b j


j 
Projection of v along k = = (ai + b j + ck ).k = c
v.k
k
∴ OA.OB =(cos a i + sin a j ).(cos b i + sin b j )
 
Hence components a, b and c of vector v = ai + b j + ck are projections of vector v along

⇒ OA OB cos(a=
- b ) cos a cos b + sin a sin b
i, j and k respectively.  

∴ cos(a=
- b ) cos a cos b + sin a sin b ( )
∴ OA = OB = 1
 
Example 8: Prove that in any triangle ABC
(i) a2 = b2 + c2 - 2bc cos A (Cosine Law)
(ii) a = b cosC + c cosB (Projection Law)
version: 1.1 version: 1.1

26 27
1. Quadratic Equations eLearn.Punjab 1. Quadratic Equations eLearn.Punjab
7. Vectors eLearn.Punjab 7. Vectors eLearn.Punjab

EXERCISE 7.3 u×v =( u v sin q ) nˆ


1. Find the cosine of the angle q between u and v: where q is the angle between the vectors, such that 0 7 q 7 p and n̂ is a “unit vector
u = 3i + j - k , v = 2i - j + k (ii) u =i - 3 j + 4k , v =4i - j + 3k
perpendicular to the plane of u and v with direction given by the right hand rule.

[ 3,=
5] , v [ 6, - 2] [ 2, 3, 1=
] , v [ 2, 4, 1]
(i)
(iii) - u = (iv) u -=
2. Calculate the projection of a along b and projection of b along a when:
(i) a-=i k+=
, b j k (ii) a =3i + j - k , b =-2i - j + k
3. Find a real number a so that the vectors u and v are perpendicular.
(i) u =2a i + j - k , v =i + a j + 4k
(ii) u= a i + 2a j + 3k , v= i + a j + 3k
4. Find the number z so that the triangle with vertices A(1, -1, 0), B(-2, 2, 1) and C(0, 2, z)
is a right triangle with right angle at C.
5. If v is a vector for which
=v.i 0,=
v. j 0,=
v.k 0, find v. .
6. (i) Show that the vectors 3i - 2 j + k , i - 3 j + 5k and 2i + j - 4k form a right angle. Right hand rule
(ii) Show that the set of points P = (1,3,2), Q = (4,1,4) and P = (6,5,5) form a right triangle. (i) If the ingers of the right hand point along the vector u and then curl towards the
7. Show that mid point of hypotenuse a right triangle is equidistant from its vertices. vector v, then the thumb will give the direction of n̂ which is u x v. It is shown in the igure (a).
8. Prove that perpendicular bisectors of the sides of a triangle are concurrent. (ii) In igure (b), the right hand rule shows the direction of v x u.
9. Prove that the altitudes of a triangle are concurrent.
10. Prove that the angle in a semi circle is a right angle. 7.4.1 Derivation of useful results of cross products
11. Prove that cos(a + b) = cos a cos b - sin a sin b (a) By applying the deinition of cross product to unit vectors i, j and k , we have:
12. Prove that in any triangle ABC. (a)=i × i i i=
sin 0 nˆ 0
=j× j j =
(i) b = c cos A + a cos C (ii) c = a cos B + b cos A
(iii) b2 = c2 + a2 - 2ca cos B (iv) c2 = a2 + b2 - 2ab cos C.
j sin 0 nˆ 0
=
k × k k k=
sin 0 nˆ 0
7.4 THE CROSS PRODUCT OR VECTOR =
(b) i × j i j=
sin 90 k k
PRODUCT OF TWO VECTORS =j×k j k=
sin 90 i i
=k ×i k =
i sin 90 j j
The vector product of two vectors is widely used in Physics, particularly, Mechanics and
Electricity. It Is only deined for vectors in space. (c) u × v =u v sin q nˆ =v u sin(-q ) nˆ =- v u sin q nˆ
Let u and v be two non-zero vectors. The cross or vector product of u and v, written as ⇒ u × v =-v × u
u x v, is deined by (d)=
u × u u u=
sin 0 nˆ 0

version: 1.1 version: 1.1

28 29
1. Quadratic Equations eLearn.Punjab 1. Quadratic Equations eLearn.Punjab
7. Vectors eLearn.Punjab 7. Vectors eLearn.Punjab

i j k
a1 b1 c1 = (b1c2 - c1b2 )i - (a1c2 - c1a2 ) j + (a1b2 - b1a2 )k
a2 b2 c2
Note: The cross product of i, j and k are written in the cyclic pattern. The
given igure is helpful in remembering this pattern. The terms on R.H.S of equation (i) are the same as the terms in the expansion of the above
determinant
i j k
Hence u × v =a1 b1 c1 (ii)
7.4.2 Properties of Cross product a2 b2 c2

which is known as determinant formula for u x v.


The cross product possesses the following properties:
(i) u=
×v 0 = =
if u 0 or v 0
u × v =-v × u
Note: The expression on R.H.S. of equation (ii) is not an actual determinant, since its entries
(ii) are not all scalars. It is simply a way of remembering the complicated expression on R.H.S.
(iii) u × (v + w) = u × v + u × w (Distributive property)
u × (kv)= (ku ) × v= k (u × v) ,
of equation (i).
(iv) k is scalar
(v) u×u = 0 7.4.4 Parallel Vectors
The proofs of these properties are left as an exercise for
the students. If u and v are parallel vectors, ( q =

0 sin=
0 0) , then
u × v =u v sin q nˆ
u=× v 0 or v= ×u 0
7.4.3 Analytical Expression of u x v
And if u × v =
(Determinant formula for u x v)
0 . then
sin q 0=
either= or u 0 =
Let u = a1 i + b1 j + c1 k and v = a2 i + b2 j + c2 k , then
or v 0
If sin q = 0 ⇒ q = 0 or 180 , which shows that the vectors u and v are parallel.
u × v= (a1 i + b1 j + c1 k ) × (a2 i + b2 j + c2 k )
(i)
If u = 0 or v = 0, then since the zero vector has no speciic direction, we adopt the
= a1a2 (i × i ) + a1b2 (i × j ) + a1c2 (i × k )
(ii)
(by distributive property) convention that the zero vector is parallel to every vector.
+b1a2 ( j × i ) + b1b2 ( j × j ) + b1c2 ( j × k ) ∴ i × j =k =- j × i
+c1a2 (k × i ) + c1b2 (k × j ) + c1c2 (k × k ) i×i = j× j = k ×k = 0
Note: Zero vector is both parallel and perpendicular to every vector. This apparent
contradiction will cause no trouble, since the angle between two vectors is never applied
= a1b2 k - a1c2 j - b1a2 k + b1c2 i + c1a2 j - c1b2 i when one of them is zero vector.

⇒ u ×=
v (b1c2 - c1b2 )i - (a1c2 - c1a2 ) j + (a1b2 - b1a2 )k (i)
Example 1: Find a vector perpendicular to each of the vectors
a = 2i + j + k and b = 4i + 2 j - k
The expansion of 3 x 3 determinant

version: 1.1 version: 1.1

30 31
1. Quadratic Equations eLearn.Punjab 1. Quadratic Equations eLearn.Punjab
7. Vectors eLearn.Punjab 7. Vectors eLearn.Punjab

Solution: A vector perpendicular to both the vectors a and b is a x b So that ∠AOB = a +b


Now= OA cos a i + sin a j
i j k 

∴ a × b =2 -1 1 =-i + 6 j + 8k
and OB = cos(- b )i + sin(- b ) j

2 -1
= cos b i - sin b j
4

= (cos b i - sin b j ) × (cos a i + sin a j )


Veriication:
∴ OB × OA
 
a.a × b= (2i + j + k ).(-i + 6 j + 8k ) = (2)(-1) + (-1)(6) + (1)(8)= 0
b.a × b= (4i + 2 j - k ).(-i + 6 j + 8k ) = (4)(-1) + (2)(6) + (-1)(8)= 0 i j k
OB OA sin(a + b =
)k cos b - sin b
and

 
0
cos a sin a
Hence a x b is perpendicular to both the vectors a and b.
0
Example 2: If a = 4i + 3 j + k and b = 2i - j + 2k .. Find a unit vector perpendicular to ⇒ sin(a +=b )k (sin a cos b + cos a sin b )k
both a and b. Also ind the sine of the angle between the vectors a and b. ∴ sin(a=+ b) sin a cos b + cos a sin b

i j k Example 4: In any triangle ABC, prove that


a×b = 4 1 = 7i - 6 j - 10k
= =
Solution: 3 a b c
2 -1 2 (Law of Sines)
sin A sin B sin C
and a=
×b (7) 2 + (-6) 2 + (10)
= 2
185 Proof: Suppose vectors a, b and c are along the sides BC, CA and AB respectively of the
a×b

triangle ABC.
a×b ∴ a+b+c =
A unit vector nˆ perpendicular to a and b =
0
⇒ b + c =-a
= (7i - 6 j - 10k )
1 (i)
Take cross product with c
a= (4) 2 + (3) 2 + (1) 2 = b×c + c×c = - a×c
185
Now 26
= (2) 2 + (-1) 2 + (2)
= 2 b×c = c×a (∴ c × c = 0)
⇒ b×c = c×a
b 3
If q is the angle between a and b, then a × b =a b sin q
sin(p - A) - c a sin(p
b c = B)
a×b
⇒ sin q = =
185
a × b 3 26
⇒ bc sin A = ca sin B ⇒ b sin A = a sin B
Example 3: Prove that sin(a + b) = sin a cos b +cos a sin b ∴
a
=
b
(ii)
sin A sin B
Let OA and OB be unit vectors in the xy-plane making angles a and -b with the
  similarly by taking cross product of (i) with b, we have
Proof:
positive x-axis respectively
version: 1.1 version: 1.1

32 33
1. Quadratic Equations eLearn.Punjab 1. Quadratic Equations eLearn.Punjab
7. Vectors eLearn.Punjab 7. Vectors eLearn.Punjab

=
a c
(iii)
sin A sin C i j k
AB × AC = 1 - 2 = (2 + 4)i - (1 - 4) j + (2 + 4)k = 6i + 3 j + 6k
 
= =
a b c Now 2
-2
From (ii) and (iii), we get
sin A sin B sin C 2 1
 
7.4.5 Area of Parallelogram The area of the parallelogram with adjacent sides AB and AC is given by

AB × AC = 6i + 3 j + 6k = 36 + 9 + 36 = 81 = 9
 
If u and v are two non-zero vectors and q is the angle between
∴ Area of triangle = AB × AC = 6i + 3 j + 6k =
u and v, then u and v represent the lengths of the adjacent sides of a parallelogram, (see 1   1 9
2 2 2
igure)
AB × AC 1
 
A unit vector ⊥ to the plane ABC =  = (6i + 3 j + 6k= (2i + j + 2k )
1
AB × AC 9
We know that: )
3
Area of parallelogram = base x height
= (base) (h) = u v sin q
∴ Area of parallelogram = u × v Example 6: Find area of the parallelogram whose vertices are P(0, 0, 0), Q(-1, 2, 4),
R(2, -1, 4) and S(1, 1, 8).

Solution: Area of parallelogram = u × v


7.4.6 Area of Triangle
where u and v are two adjacent sides of the parallelogram
From igure it is clear that
PQ =(-1 - 0)i + (-2 - 0) j + (4 - 0)k =-i + 2 j + 4k


Area of triangle = (Area of parallelogram)


1
PR = (2 - 0)i + (-1 - 0) j + (4 - 0)k = 2i - j + 4k

2 and
∴ Area of triangle = ×
1
u v i j k
PQ × PR = -1 4 = (8 + 4)i - (-4 - 8) j + (1 - 4)k
2  
where u and v are vectors along two adjacent sides of the triangle. Now 2
2 -1 4

∴ Area of parallelogram = PQ × PR = 12i + 12 j - 3k


Example 5: Find the area of the triangle with vertices  
A(1, -1, 1), B(2, 1, -1) and C(-1, 1, 2)
Be careful!:

= 144 + 144 + 9
Not all pairs of vertices give a
Also ind a unit vector perpendicular to the plane ABC. 
side e.g. PS is not a side, it is
= 297 diagonal since PQ + PR =
  
Solution: AB = (2 - 1)i + (1 + 1) j + (-1 - 1)k = i + 2 j - 2k
 PS

AC =(-1 - 1)i + (1 + 1) j + (2 - 1)k =-2i + 2 j + k



Example7: If u = 2i - j + k and v = 4i + 2 j - k , ind by determinant formula
(i) uxu (ii) uxv (iii) v x u
version: 1.1 version: 1.1

34 35
1. Quadratic Equations eLearn.Punjab 1. Quadratic Equations eLearn.Punjab
7. Vectors eLearn.Punjab 7. Vectors eLearn.Punjab

Solution: u = 2i - j + k and v= 4i+ 2 j- k


5. Which vectors, if any, are perpendicular or parallel
(i) u =5i - j + k ; v =j - 5k ; w =-15i + 3 j - 3k
By determinant formula
p p
i j k (ii) u =i + 2 j - k ; v =-i + j + k ; w =- i - p j + k
= =
u×u 2 -∴
2 2
(i) 1 1 0 ( Two rows are same)
2 -1 1 6. Prove that: a x (b + c) + b x (c + a) + c x (a + b) = 0
7. If a + b + c = 0, then prove that a x b = b x c = c x a
Prove that: sin(a - b) = sin a cos b + cos a sin b.
i j k
u × v =2 1 =(1 -2)i -( -2 -4) j +(4 +4)k =-i +6 j +8k
8.
(ii) -1
9. If a x b = 0 and a.b = 0, what conclusion can be drawn about a or b?
4 2 -1

i j k 7.5 SCALAR TRIPLE PRODUCT OF VECTORS


(iii) v×u = 4 2 -1 = (2 - 1)i - (4 + 2) j + (- 4 - 4)k = i - 6 j - 8k
2 -1 1 There are two types of triple product of vectors:
(a) Scalar Triple Product: (u × v).w or u.(v × w)
(b) Vector Triple product: u × (v × w)
EXERCISE 7.4
In this section we shall study the scalar triple product only
1. Compute the cross product a x b and b x a. Check your answer by showing that each Deinition
Let u = a1 i + b1 j + c1 k , v = a2 i + b2 j + c2 k and w = a3 i + b3 j + c3 k
a and b is perpendicular to a x b and b x a.
(i) a =2i + j - k , b =i - j + k (ii) + a =
i- j=
, b i j
(iii) a =3i - 2 j + k , b =+ (iv) a =-4i + j - 2k , b =2i + j + k
be three vectors
i j The scalar triple product of vectors u, v and w is deined by
2. Find a unit vector perpendicular to the plane containing a and b. Also ind sine of the u.(v x w) or v.(w x u) or w.(u x v)
angle between them. The scalar triple product u.(v x w) is written as
(i) a = 2i - 6 j - 3k , b = 4i + 3 j - k (ii) a =-i - j - k , b =2i - 3 j + 4k u.(v x w) = [u v w]
(iii) a =2i - 2 j + 4k , b =-i + j - 2k (iv) + a = i- j= , b i j
3. Find the area of the triangle, determined by the point P, Q and R. 7.5.1 Analytical Expression of u.(v x w)

u = a1 i + b1 j + c1 k , v = a2 i + b2 j + c2 k and w = a3 i + b3 j + c3 k
(i) P(0, 0, 0) ; Q(2, 3, 2) ; R (-1, 1, 4)
(ii) P(1, -1, -1) ; Q(2, 0, -1) ; R (0, 2, 1) Let
4. ind the area of parallelogram, whose vertices are:
i j k
v×w =
(i) A(0, 0, 0) ; B (1, 2, 3) ; C (2, -1, 1) ; D (3, 1, 4)
(ii) A(1, 2, -1) ; B (4, 2, - 3) ; C (6, - 5, 2) ; D (9, - 5, 0) Now a2 b2 c2
(iii) A(-1, 1, 1) ; B (-1, 2, 2) ; C (-3, 4, - 5) ; D(-3, 5, - 4) a3 b3 c3

version: 1.1 version: 1.1

36 37
1. Quadratic Equations eLearn.Punjab 1. Quadratic Equations eLearn.Punjab
7. Vectors eLearn.Punjab 7. Vectors eLearn.Punjab

⇒ v × w= (b2c3 - b3c2 )i - (a2c3 - a3c2 ) j + (a2b3 - a3b2 )k Note: (i) The value of the triple scalar product depends upon the cycle order of the

∴ u.(v × w
=) a1 (b2c3 - b3c2 ) - b1 (a2c3 - a3c2 ) + c1 (a2b3 - a3b2 )
vectors, but is independent of the position of the dot and cross. So the dot and cross, may
be interchanged without altering the value i.e;
(ii) (u × v) . w= u . (v × w)= [u v w]
(v × w) . =u v . ( w × u=
a1 b1 c1
⇒ u.(v × w) = ) [v w u ]
( w × u ) .=
v w . (u × v=
a2 b2 c2
a3 b3 c3 ) [ w u v]
which is called the determinant formula for scalar triple product of u, v and w in (iii) The value of the product changes if the order is non-cyclic.
component form. (iv) u.v.w and u x (v.w) are meaningless.
a1 b1 c1
Now u.(v × w) =
a2 b2 c2 7.5.2 The Volume of the Parallelepiped
a3 b3 c3
The triple scalar product (u x v).w
a2 b2 c2
= - a1
represents the volume of the parallelepiped
b1 c1 Interchanging R1 and R2
having u, v and w as its conterminous edges.
a3 b3 c3
As it is seen from the formula that:
u v w cosq
(u × v).w =×
a2 b2 c2
= a3
Hence (i) u × v = area of the parallelogram
b3 c3 Interchanging R2 and R3
a1 b1 c1
∴ u.(v × w) = v.( w × u )
with two adjacent sides, u and v.
(ii) w cosq = height of the parallelepiped
u v w cosq =
(u × v).w =×
a2 b2 c2
v.( w × u ) =
(Area of parallelogram)(height)
Now a3 b3 c3
= Volume of the parallelepiped
a1 b1 c1
Similarly, by taking the base plane formed by v and w, we have
a3 b3 c3
= - a2
The volume of the parallelepiped = (v x w).u
b2 c2 Interchanging R1 and R2 And by taking the base plane formed by w and u, we have
a1 b1 c1 The volume of the parallelepiped = (w x u).v
a3 b3 c3 So, we have: (u x v).w = (v x w).u = (w x u).v
= a1 b1 c1 Interchanging R2 and R3
a2 b2 c2 7.5.3 The Volume of the Tetrahedron:
∴ v.( w × u )= w.(u × v)
u.(v × w) =v.( w × u )= w.(u × v)
Volume of the tetrahedron ABCD
Hence

= (∆ABC ) (height of D above the place ABC )


1
3

version: 1.1 version: 1.1

38 39
1. Quadratic Equations eLearn.Punjab 1. Quadratic Equations eLearn.Punjab
7. Vectors eLearn.Punjab 7. Vectors eLearn.Punjab

2 -4 5
= . u × v ( h)  AB AC AD = 2 - 3 6 = 2(3 + 6) + 4(-2 - 0) + 5(-2 - 0)
1 1   
 
0 -1 -1
3 2
= (Area of parallelogram with AB and AC as adjacent sides) (h)
1
6 = 18 - 8 - 10 = 0
= (V olume of the parallelepiped with u , v, w as edges)
1 As the volume is zero, so the points A, B, C and D are coplaner.

Thus Volume = (u × v).w = [u v w]


6
1 1
Example 3: Find the volume of the tetrahedron whose vertices are
6 6
A(2, 1, 8), B(3, 2, 9) , C(2, 1, 4) and D(3, 3, 0)
Solution: AB =(3 - 2)i + (2 - 1) j + (9 - 8)k =i + j + k
Properties of triple scalar Product: 
1. If u, v and w are coplanar, then the volume of the parallelepiped so formed is zero
AC = (2 - 2)i + (1 - 1) j + (4 - 8)k = 0i - 0 j - 4k

i.e; the vectors u, v, w are coplanar ⇔ (u × v).w =
0
AD =(3 - 2)i + (3 - 1) j + (0 - 8)k =i + 2 j - 8k

2. If any two vectors of triple scalar product are equal, then its value is zero i.e;

∴ Volume of the tetrahedron =  AB AC AD 


[u u w] = [u v v] = 0
1   
6

0 - 4 = [ 4(2 - 1) ] = =
Example 1: Find the volume of the parallelepiped determined by
u =i + 2 j - k , v =i - j + 3k , w =i - 7 j - 4k
1 1 1
= 0
1 1 4 2

2 -8
6 6 6 3
1
1 2 -1
Solution: V olume of the parallelepiped = u.v × w = 1 - 2
Example 4: Find the value of a, so that a i + j , i + j + 3k and 2i + j - 2k are coplaner.
3
1 -7 -4
⇒ Volume = 1 (8 + 21) - 2(-4 - 3) -1 (-7 + 2) Solution: Let u = a i + j , v =i + j + 3k and w =2i + j - 2k
= 29+ 14 + 5 = 48 Triple scalar product
a
[ u v w] =
1 0
1 3 = a (-2 - 3) - 1(-2 - 6) + 0(1 - 2)
Example 2: Prove that four points
A(-3, 5, -4), B(-1, 1, 1), C(-1, 2, 2) and D(-3, 4, -5) are coplaner.
1
2 1 -2
Solution: AB =-( 1 + 3)i + (1 - 5) j + (1 + 4)k =2i - 4 j + 5k

= -5a + 8
AC =-( 1 + 3)i + (2 - 5) j + (2 + 4)k =2i - 3 j + 6k


AD = (3 - 3)i + (4 - 5) j + (-5 + 4)k = 0i - j - k = - j - k


 The vectors will be coplaner if -5a + 8= 0 ⇒ a=
8
5
  
Volume of the parallelepiped formed by AB , AC and AD is

version: 1.1 version: 1.1

40 41
1. Quadratic Equations eLearn.Punjab 1. Quadratic Equations eLearn.Punjab
7. Vectors eLearn.Punjab 7. Vectors eLearn.Punjab

Example 5: Prove that the points whose position vectors are A(-6i + 3 j + 2k ) ,
In igure, a constant force F acting on a body, displaces it from A to B.
B (3i - 2 j + 4k ) , C (5i + 7 j + 3k ) , D(-13i + 17 j - k ) are coplaner. ∴ Work done = (component of F along AB) (displacement)
== ( F cosq )( AB) F . AB

Solution: Let O be the origin.
∴ OA = -6i + 3 j + 2k ; OB = 3i - 2 j + 4k
 

∴ OC =+ 5i 7 j + 3k ; OD- = + 13i - 17 j k
 
Example 6: Find the work done by a constant force F= 2i + 4 j ,, if its points of application
∴ AB= OB - OA= (3i - 2 j + 4k ) - (-6i + 3 j + 2k )
  
to a body moves it from A(1, 1) to B(4, 6).
∴ =9i - 5 j + 2k (Assume that F is measured in Newton and d in meters.)
AC= OC - OA= (5i + 7 j + 3k ) - (-6i + 3 j + 2k )
  
Solution: The constant force F= 2i + 4 j ,
∴ = 11i + 4 j + k
The displacement of the body = d = AB

AD = OD - OA = (-13i + 17 j - k ) - (-6i + 3 j + 2k )
  
= (4 - 1)i + (6 - 1) j = 3i + 5 j
∴ =-7i + 14 j - 3k
-5 ∴ work done = F . d
+
= (2i 4 j )+. (3i 5 j )
9 2
AB.( AC × AD) = 11
  

=(2)(3) + (4)(5) =26 nt. m


Now 4 1
-7 14 -3
= 9(-12 - 14) + 5(-33 + 7) + 2(154 + 28)
-= - + =0
The constant forces 2i + 5 j + 6k and - i + 2 j + k act on a body, which is
234 130 364
∴ AB, AC , AD are coplaner
   Example 7:
⇒ The points A, B, C and D are coplaner. displaced from position P(4,-3,-2) to Q(6,1,-3). Find the total work done.

7.5.4 Application of Vectors in Physics Solution: Total force = (2i + 5 j + 6k ) + (-i + 2 j + k )


and Engineering ⇒ F =+
i 3 j + 5k
The displacement of the body = PQ = (6 - 4)i + (1 + 3) j + (-3 + 2)k


⇒ d =2i + 4 j - k
(a) Work done.


If a constant force F, applied to a body, acts at an
angle q to the direction of motion, then the work done
work done = F . d
=(i + 3 j + 5k ) . (2i + 4 j - k )
by F is deined to be the product of the component of
F in the direction of the displacement and the distance =2 + 12 - 5 =9 nt. m
that the body moves.

version: 1.1 version: 1.1

42 43
1. Quadratic Equations eLearn.Punjab 1. Quadratic Equations eLearn.Punjab
7. Vectors eLearn.Punjab 7. Vectors eLearn.Punjab

2. Verify that
(b)
Moment of Force a . b × c= b . c × a= c . a×b
if a = 3i - j + 5k , b = 4i + 3 j - 2k , and c = 2i + 5 j + k

Let a force F ( PQ) act at a point P as shown in the igure,
then moment of F about O. 3. Prove that the vectors i - 2 j + 3k , - 2i + 3 j - 4k and i - 3 j + 5k are coplanar
= product of force F and perpendicular ON . nˆ 4. Find the constant a such that the vectors are coplanar.
=
= ( PQ)(ON )(nˆ ) ( PQ)(OP )sin q . nˆ (i) i- j+k , i - 2 j - 3k and 3i - a j + 5k .
= OP × PQ =r × F i - 2 a j - k , i - j + 2k ai - j + k
 
(ii) and

(iv) [i i k ]
5. (a) Find the value of:
2i × 2 j.k 3 j.k × i  k i j 
(b) Prove that u.( v × w ) + v.( w × u ) + w.( u × v=) 3 u.( v × w )
(i) (ii) (iii)

Example 8: Find the moment about the point M(-2 , 4, -6) of the force represented by
AB , where coordinates of points A and B are (1, 2, -3) and (3, -4, 2) respectively.
 6. Find volume of the Tetrahedron with the vertices
(i) (0, 1, 2), (3, 2, 1), (1, 2, 1) and (5, 5, 6)
(ii) (2, 1, 4) and (3, 3, 10) .
AB = (3 - 1)i + (-4 - 2) j + (2 + 3)k = 2i - 6 j + 5k
 (2, 1, 8), (3, 2, 9),
Solution: 7. Find the work done, if the point at which the constant force F = 4i + 3 j + 5k is applied
MA = (1 + 2)i + (2 - 4) j + (-3 + 6)k = 3i - 2 j + 3k to an object, moves from P1 (3,1, -2) to P2 (2,4,6) .


Moment of AB about ( - 2, 4, - 6) = r × F = MA × AB A particle, acted by constant forces 4i + j - 3k and 3i - j - k , is displaced from


  
8.
A(1, 2, 3) to B(5, 4, 1). Find the work done.
A particle is displaced from the point A(5, -5, -7) to the point B(6, 2, -2) under the
i j k
= 3 -2
9.
action of constant forces deined by 10i - j + 11k , 4i + 5 j + 9k and -2i + j - 9k . Show that
3
2 -6 5
= (-10 + 18)i - (15 - 6) j + (-18 + 4)k
the total work done by the forces is 102 units.
A force of magnitude 6 units acting parallel to 2i - 2 j + k displaces, the point of
=8i - 9 j - 14k
10.
application from (1, 2, 3) to (5, 3, 7). Find the work done.
Magnitude of the moment = (8) 2 + (-9) 2 + (-14) 2 = 341 11. A force F =3i + 2 j - 4k is applied at the point (1, -1, 2). Find the moment of the force
about the point (2, -1, 3).
EXERCISE 7.5 12. A force F= 4i - 3k , passes through the point A(2,-2,5). Find the moment of F about
the point B(1,-3,1).
1. Find the volume of the parallelepiped for which the given vectors are three edges. 13. Give a force F = 2i + j - 3k acting at a point A(1, -2, 1). Find the moment of F about the
(i) u =3i + 2k ; v =
i +2 j +k ; w =-j +4k point B(2, 0, -2).
(ii) u =i - 4 j - k ; v =i - j - 2k ; w =2i - 3 j + k 14. Find the moment about A(1, 1, 1) of each of the concurrent forces i - 2 j , 3i + 2 j - k ,
(iii) u =i - 2 j - 3k ; v =2i - j - k ; w=j+k 5 j + 2k , where P(2,0,1) is their point of concurrency.
15. A force F =7i + 4 j - 3k is applied at P(1,-2,3). Find its moment about the point Q(2,1,1).
version: 1.1 version: 1.1

44 45

Вам также может понравиться